YDS KPDS ÜDS Paragraflar

September 17, 2017 | Author: aramayapar | Category: Mercury (Element), Oceanography, Economics, Global Warming, Petroleum
Share Embed Donate


Short Description

YDS 2013 PARAGRAF, KPDS 1992 - 2012 PARAGRAFLAR, ÜDS 2000 - 2012 PARAGRAFLAR (FEN SOSYAL SAĞLIK -2004 2005- EKSİK)...

Description

KPDS 1992 İLKBAHAR *** When it was formed many million years ago the earth was a liquid. It is still cooling and many miles below the hard crust is still hot. However, in some places the heat is closer to the surface. These places are associated with volcanic activity or hot sulphur springs. By drilling deep into the earth’s crust we can reach rocks that are much warmer than those at the surface. Pumping water down into contact with these rocks and extracting the steam so produced is a source of energy that can be used to produce electricity. It is called geothermal energy. *** The Japanese have a special way of making decisions. They call it the consensus system. This is how it works. When a firm is thinking of taking a certain action, it encourages workers at all levels to discuss the proposal and give their opinions. The purpose is to reach consensus (general agreement). As soon as everyone agrees on the right course of action, the decision is taken. Because of this method, a group of workers, rather than a person, is responsible for company policies. One advantage of this is that decisions come from a mixture of experience from the top, the middle and the bottom of an enterprise. Another advantage is that junior staff frequently suggest ideas for change. A disadvantage, perhaps, is that decision-making can be slow. *** Rabies is a very frightening disease because once symptoms develop it is always fatal. The disease is caused by a virus and it affects many species of animals, particularly dogs, jackals, foxes and bats. In Britain no indigenous case of human rabies has been reported since 1902 but it is widespread among animals in most parts of the world. Unfortunately, in the last 30 years the disease has been spreading across Europe from the East, especially in foxes, and has now reached Northern France. For this reason strict animal quarantine laws are in force in Britain and it is rightly regarded as a serious offence to attempt to evade them. *** When we turn to the problem of fishing, we see that through a UN Convention on the Law of the Sea, the world’s nations have indicated that they recognize the risks of over-fishing. Nations can now declare 200-mile exclusive economic zones and exclusive fishing zones and control the catch at a level that is sustainable. Developing nations seem to be beginning to benefit from the new fisheries regime which offers the promise of allowing them to manage fishing resources for optimum, that is long-term, benefits. *** The exact number of people who died in the cyclone that struck Bangladesh last year will probably never be known. Winds reaching 145 miles per hour hammered the country’s low-lying south-eastern coast for nine hours, at one point driving a wall of water roughly 20 feet high across the area – one of the most densely populated places in the world. It was the strongest storm ever recorded in the region. The official news agency reported that 125.000 victims had been confirmed dead, but it was believed that the toll was actually much higher. *** Written communication is the basis of much communication in business. This includes letters, reports, memoranda, notices, telex, and fax messages. Although written communication is a slower form of communication than verbal or oral, it provides a record of what is being discussed so that disagreements are avoided and accuracy can be checked; it will also be more detailed than other forms of communication, with the possibility of technical points being explained and interpreted.

KPDS 1992 SONBAHAR *** Nigeria is heavily dependent on the export of crude oil to finance industrial development. 90% of Nigeria's exports by value are crude oil. At current production rates, known reserves are only sufficient until the end of the century. Industrialisation was boosted after I973 following the fourfold increase in oil prices. In the early 1980s prices fell, and Nigeria lost important income. Oil production peaked in 1974 when output reached 112 million tonnes. *** Real depression cannot be as easily overcome as some people often suppose. It usually passes with time - but the time can seem endless. Activities giving companionship and a new interest can help. But for the sufferer to talk, again and again, about the causes of the depression helps most. People with depression need to be listened to and encouraged to find their own solutions, not made to feel yet more inadequate by good advice. They may need professional counselling as well as the support of family and friends. *** Many art museums and galleries and many individuals in the world faced financial problems in 1975 as the effects of world recession deepened. On the surface, things seemed to continue as before, with important exhibitions in major museums attracting large crowds. But smaller galleries and the artists whose work was shown by their resourceful proprietors fared less well, and over the long term it is the work of young artists that determines the course of art for the future. *** Computers should never have acquired the exalted status they now have. Fascinating and invaluable as they are, even the most advanced have less brain power than a three-year-old. They do, however, score on single-mindedness. The three-year-old uses his brain not only to think but also to do tasks like seeing, hearing and running about, which need incredibly rapid and sophisticated electro-mechanical interactions - we too run on electricity. But the computer just sits there and sends spacecraft to the moon or reorganises the world banking system, which is very much easier. That's why man’s dream of robot servants is still a long way off. *** The dramatic growth of the world’s population in the twentieth century has been on a scale without parallel in human history. Most of this growth has occurred since 1950 and is known as the population ‘explosion'. Between 1950 and 1980 the world population increased from 2,5 to over 4 billion, and by the end of the century this figure will have risen to at least 6 billion. Growth of this size cannot continue indefinitely. Recent forecasts suggest that the total population will level-out at between 10 and 15 billion in the mid twenty-first century. Already there are encouraging signs that the rate of increase in many less developed countries is beginning to slow down. *** Many substances, whether man-made or natural, can cause harm to man or the environment. Some of these reach the environment in waste streams; however, emission limits and environmental quality standards can, in some instances, reduce the amounts released. But some other substances cannot be controlled in this way because they are released, not in industrial waste streams, but through the use or disposal of products which contain them. In many cases these substances pose little or no threat if the product containing them is used and disposed of properly. The right way to deal with them is usually through controls over their supply, use and disposal.

KPDS 1993 İLKBAHAR

KPDS 1993 SONBAHAR

*** The practical advantages of prefabrication are twofold: It is quicker and it does away with uncertainty. Speed in building is important in these days because of the high cost of land: the time during which such an expensive commodity is out of use must be reduced to a minimum. And, partly or wholly prefabricated methods of construction save time on the job because parts are prepared in the factory beforehand. Prefabrication does away with uncertainty because it means that the whole building is made of standard parts the behaviour of which is known and has been tested.

*** There are twelve and a half acres of land for each man, woman, and child in the world today. However, only three and a half acres of this land can be cultivated. If the population of the world reaches six billion by the year 2000, there will be only one and a half acres for each person. Man just increases his production of food. One scientist has said that the world could support ten billion people if better agricultural methods were used everywhere. The supply of food can also be increased by the control of plant diseases, and by the irrigation of desert lands. By using these ways and others, man can feed himself and his fellow men.

*** Computers can store vast amounts of information in a very small space and are used by the banks to keep accounts, print out statements and control transactions. They are also used by the police to keep personal records, fingerprints and other details. In the rapidly developing field of robotics computers are now being used to control manual operations done by mechanics. These, too, are taking over work, previously done by people in the manufacture of cars, in weaving and in other industries. Computers play an important role in controlling artificial satellites, decoding information and communications generally. They are used to predict the weather with increasing accuracy.

*** Though there has always been a certain amount of concern about pollution since the start of the Industrial Revolution, this was largely an interest of relatively limited numbers of concerned people. But during the 1960s there was a great upsurge of anxiety which was reflected internationally by the calling of the Stockholm UN Conference. In response to the rising public pressures, action has been taken mainly in the industrial countries, even though sometimes reluctantly and with many warnings from industry and governments about the costs. However, in many areas there has been a gratifying improvement.

*** Looking ahead from the present position where food production has kept ahead of population growth globally, but has fallen per capita in 55 (mainly African) countries, it would seem that these trends will continue. About 30 countries - most of them African - can expect serious problems unless they reduce population growth and give higher priority to agriculture and conservation. Though a warmer, wetter earth with high CO2 levels is likely to be capable of producing more food, the amounts will still be inadequate for many poorer countries. In many cases, the population projections are greater than the entire local land resources can support. *** Psychology is literally the study of mind (or soul) but its areas has broadened somewhat in the last century as we have learned that one cannot consider the mind as totally isolated from the body, and it now includes the study of human personality and behaviour. It is important to realise that psychologists are first and foremost trained as scientist rather than as medical experts and do not necessarily take much interest in abnormalities of the brain and mental processes. *** Aid to underdeveloped countries takes many forms and it is given for many reasons. Underdeveloped countries need aid to provide finance for development projects; to provide foreign exchange with which imports for development purpose can be bought; and to provide the trained manpower and technical knowledge they lack. The motives of the donor are not always humanitarian. “Aid” can take a military form; it can be used to support an incompetent or unjust government. Nor is aid always beneficial to the recipient country. It may be wasted on ill-conceived or prestige projects, or cause the government simply to relax on its own efforts. *** Both as a profession and a science, economics lost considerable prestige during the recession of 1974-75. The crisis that seized the western industrialised countries including Japan was of a character not to be found in economics textbooks. Rate of inflation exceeding 10% a year coupled with declining production and high levels of unemployment. Hitherto, peacetime inflation had been associated with high employment and an overactive economy, while high rate of unemployment went with the recession or depression. The next combination was apply called stagflation.

*** For nearly a decade now, the manufacturing of automobiles has been undergoing radical changes. The principle cause is the introduction of new production and management techniques, originally engineered by Toyota and subsequently applied by other Japanese car manufacturers. This is called “lean” production. It implies no less a revolution in the process of car manufacturing than the mass production Henry Ford introduced at the beginning of the century. In fact, lean production combines the advantages of craft and mass production. A vital feature of this production technique is that it achieves its highest efficiency, quality and flexibility when all activities - form design to assembly - occur in the same area. *** Much in medicine which is now taken for granted was undreamed of even as recently as 50 years ago. Progress in diagnosis, in preventive medicine and in treatment, both medical and surgical, has been so rapid as to be almost breathtaking. Today a doctor retiring from active practice will among other things have seen smallpox completely eradicated, tuberculosis become curable, coronary artery disease relievable surgically. One aspect of medicine still resistant to progress in understanding is the effect of mind on body. Many of us still like to think that our bodies are just something we have got into, like cars, that ill health is simply something that has “gone wrong” in one system or another and that therefore provided we find the appropriate expert to correct the fault, we will recover. *** National income is a measure of the total income accruing to the residents in a country in return for services rendered. It therefore consists of the sum of wages, salaries, profits and rents. But not all these income accrues to persons; for instance, companies do not distribute all their profits to shareholders and some nationalised industries earn profits. This is part of national income but not of personal income. On the other hand, some personal incomes are not payments for services rendered. Such incomes are called transfer incomes to emphasise that their payment does not add to the national income, but only transfers income from one agent to another. Included in this category are retirement pensions, family allowances, and student grants. *** According to the United Nations sources, world population in mid-1985 was, 5,290 million, an increase of 90 million in one year. More than half of the total live in Asia (56,9 per cent). Different countries are at different stages in a demographic transition from the stability provided by a combination of high birth rate and high death rate to that provided by a combination of low birth rate and low death rate. Their recent population history and current trend of growth, the age-structure of their population, and consequently their population potential for the near future are all widely different. Most rapid growth is in Africa with rates of over 3 per cent in some countries. In most European countries the rate is less than 1 per cent.

KPDS 1994 İLKBAHAR

KPDS 1994 SONBAHAR

*** Ever since Nobel prize-winner Linus Pauling first advocated vitamin C as a common-cold war weapon more than 20 years ago, researchers have been busy trying to verify that claim. But so far, they've found little evidence that vitamin C prevents colds - in fact, there are more studies that say it doesn't. But there is evidence that it can keep coughing and sneezing to a minimum, and that low levels of vitamin C in the body may be related to bronchitis.

*** Society is, regrettably, less interested in the individual than in the position which he occupies. One almost never asks a person “Who are you?”, but one onstantly asks “What do you do?”. For one reason or another, people are assorted into various categories which determine the roles they are to play in society. This assorting process, called social differentiation, goes on in all societies. Women as a class have a status distinct from that of men, and children have a status unlike that of adults.

*** Since early times it has been assumed that the actions of animals are unconscious. Behaviour, in this view, stems almost exclusively from instinct. If animals behave in ways that seem pretty clever, they do so without thinking about it. Animals may know things, the argument goes, but they don't know that they know. Or do they know? Recent research reports suggest a startling depth of intelligence among animals. Although no one can yet 'prove' the existence of animal consciousness, the data offered make a compelling case for at least considering it. *** The first universities developed in Europe in the 12th century. By 1600 Western Europe boasted 108 institutions of higher learning, many of which had obtained special privileges from existing regimes because of their close association with the Church. In most European countries, universities were designed primarily for the sons of nobility and gentry. Scholarly standards were low, and scholarship was irrelevant for most professions. Education for earning a livelihood in, say, medicine or law could be acquired after college by serving as an apprentice. *** The effects of sleep loss are subject to a number of popular misconceptions. The belief that everyone must sleep 8 hours a night is a myth. According to the results of a recent survey on the subject, dults average about 7 to 7 1 / 2 hours of sleep per night, and many individuals function effectively with 5 to 6 ours of sleep. In fact, 20 percent of the population (slightly more in men) sleep less than 6 hours per night. Another significant fact is that sleep time decreases with age. *** Until recently, many archaeologists took the view that civilized communities first arose in Egypt, though only a very short time before a similar development In Mesopotamia: a more recent opinion is now that the earliest advances may have taken place in Mesopotamia. Whichever view is followed, it is necessary to bear in mind that geographical conditions in both regions were not identical, and it can in fact be stated that in Mesopotamia environmental factors were not as wholly favourable as In the valley of the Nile. *** Dates and periods are necessary to the study and discussion of history, for all historical phenomena are conditioned by time and are produced by the sequence of events. Periods especially, are retrospective conceptions that we form about past events; they are useful to focus discussion, but very often they lead historical thought astray. Thus, while it is certainly useful to speak of the Middle Ages and of the Victorian Age, those two abstract ideas have deluded many scholars and millions of newspaper readers into supposing that during certain decades called the Middle Ages, and again during certain decades called Age of Victoria, everyone thought or acted more or less in the same way- till at last Victoria died or the Middle Ages came to an end. But in fact there was no such sameness.

*** Mercury has a number of interesting properties and a variety of industrial uses. It expands at a constant rate through the range of temperatures at which it is a liquid. Because of this property and because it does not cling to glass, mercury is often used in thermometers. At ordinary temperatures it evaporates very slowly and can thus be left in an open container for long periods of time. For this reason it is used in one type of barometer. Mercury is a good electrical conductor and is used in sealed electrical switches. An electric current passing through mercury vapour causes it to give off light, hence its use in certain kinds of lamps. *** Protoplasm, which is the fundamental basis of life, is constantly undergoing physical and chemical change. Life, therefore, is the resultant of these constantly occurring changes. There are two great groups into which living things may be classed: plants and animals. Both the plant and the animal kingdoms are very extensive. It is customary, therefore, to regard the science of life under two comprehensive heads, namely, botany which is the study of plants, and zoology which is the study of animals. Both subjects are subdivided into various specialised sections. ***Many observers, including policy makers, mistakenlyassume that economic competition between nationsmust result in winners and losers. It has beendemonstrated, however, that international tradeincreases the wealth of all participants, even thosewith lower productivity than their trading partners. Thereal issue, it appears, is the way internationalcompetition affects all kinds of goods that any countryproduces. It seems that the proper role of governmentis to ensure that its people are prepared to compete inthose industries in which they could or should have anadvantage. ***Those who visit the Mediterranean are invariably impressed with its unity. Everywhere it is the same, for the shades of difference here is less important than the resemblances. Yet this unity is the result of aggressive contrast; sea and mountain, sea and desert, sea and ocean! In these respects the Mediterranean is very different from either central Europe, or high tablelands of Asia, the Syrian and Saharan deserts, or even the Atlantic Ocean. *** It is the opinion of most archaeologists that civilisation first developed in the Middle East, where, of all the regions in the world, natural conditions offered the greatest assistance to man in his changeover from a life of nomadic wandering as a hunter to settled occupation of the soil. The regular rise of the three larger rivers, Nile Euphrates, and the Tigris; annual renewal of soil fertility by the deposition of a layer of silt; and the generally warm climate, favourable both to the growth of a rich plant-life, and to the activities of man himself, were all special inducements to the adoption of a way of life based on agriculture.

KPDS 1995 İLKBAHAR

KPDS 1995 SONBAHAR

*** In Britain, the Queen is a constitutional monarch. In law she is the head of the executive, an integral part of the legislature, head of the judiciary, commander-inchief of the armed forces and temporal head of the Church of England. In practice, the Queen's role is purely formal: she reigns, but she does not rule. In all important respects she acts only on the advice of her ministers. However, she still plays an important role symbolically as Head of State and Head of the Commonwealth.

*** In one century of strenuous research a vast amount of source material about Michalengelo has been collected, reviewed, edited and annotated including letters, poems, contracts, receipts and biographies. Biographical and artistic data have been checked and rechecked, sometimes corroborating and sometimes correcting our previous ideas, and an abundance of new facts has been revealed. Long lost works have been rediscovered and every single known piece has been studied in its formal and functional aspects. The artist’s character, his daily habits, his working methods, his personal attitudes and his artistic and political opinions have been traced as well as the peculiarities of the people with whom he had contact. Thus modern history of art has formed an image of Michalengelo that is much nearer to truth than those presented by his first biographers.

*** Of all the environmental problems facing us today, global warming is likely to have the most devastating effects. In order to combat these, the emission of harmful gases must be reduced; for this purpose, the rainforests, which absorb carbon dioxide in vast quantities must be protected. Global warming will place a premium on energy efficiency, for controlling global warming inescapably means reducing the burning of fossil fuels. The two industries that are most obviously going to be affected are the power suppliers and the vehicle manufacturers, but since energy is consumed by almost everything we manufacture, design or do, the effects will be felt everywhere. *** The 1970s were a period of marked economic recession in the West. The effects were widespread, even the publishing sector was badly hit. Inflation continued to push up the costs of paper and printing, increasing the price of books generally and reducing the amount of money available for the publication of new and experimental work. There was a growing sense, in the world of literature no less than in other spheres of production, that this crisis must involve changes that would be neither simple nor temporary. *** Built by the disconsolate Emperor Shah Jahan in memory of his wife, the Taj Mahal mausoleum has survived the rise and fall of many empires and it attracted looters, too; over the years they carried away the silver doors from its gates, the precious stones from its marble wall: and the gold from its graves. But those were small threats compared with the modern danger of pollution. Emissions from the coal-fired steel foundries thermal power stations, cars and an oil refinery in the industrial belt around Agra are corroding and yellowing the Taj Mahal's white marble. *** Usually educational processes involve not only learning, but teaching as well. There is, however, no logical connection in this case. Education can go on without any teaching. W e can say it was a 'real education’ for someone to take a boat out on his own, implying that he learnt something desirable without anybody having been there to teach him the lesson. There are many forms of learning that go on without teaching and ‘educative' learning does not mean that the learning must take place in a teaching situation. It may be argued that most things are learnt more rapidly and more reliably in a classroom situation. But even so, learning is not dependent upon teaching. *** Since health care accounts for nearly one seventh of the American national economy, any attempt to reform it enters a minefield of explosive issues. President Clinton's health-care plan, many experts say, might shake up the health-insurance industry so violently that it could shrink from about 500 to as few as to 10 companies; costing thousands of jobs. Businesses complain that new health-coverage contributions would be disastrous; there is even talk that up to a million jobs will be lost as a consequence. Though the American Medical Association has so far sent signals that it would endorse the plan, the physicians themselves and the taxpayers are extremely worried about it.

*** The fact that the brain is divided into a left and a right half is not a new discovery. Once the skull is removed the division is obvious to the naked eye and it is a common feature of brains throughout the animal kingdom. What is interesting about this division in man is that each half seems to have developed specialised functions, the left side appearing to be better at some tasks and the right side better at others. The most obvious difference in functioning is that the left side of the brain receives sensations from and controls the right side of the body and vice versa. The reasons for this are still unclear. Despite a number of interesting theories there is no obvious advantage in such a crossover. *** For years the theory of higher education in the United States operated something like this: men went to university to get rich, and women went to university to marry rich men. During the 60s, as a result of the fact that this theory lost much of its popularity, as the nation began to recognize the folly of relegating women to a secondary role, women soon joined men in what once were male pursuits. This rebellious decade pushed women toward independence, showed them their potential and compelled them to take charge of their lives. Many women took this opportunity. Since then famine autonomy has been the rule not the exception at least among university women. ***The collection of foreign intelligence, which is the pursuit of a special kind of information, is an indispensable service for any government having even the most elementary international associations. Nations must devise a strategy to provide for both their security and well-being. History teaches us that responsibility cannot be met without knowledge of the political, economic and military capabilities and intentions of other nations. Indeed advance knowledge of these matters, or its absence, could well settle the fate of a great nation especially in an era when a single nation or consortium of nations is capable of smashing another society in a single stroke or of controlling it under the threat of poised catastrophe. The well-being of any great nation will depend on decisions taken by others, which must be foreseen, correctly analysed and countered. *** A subject in which there has been a lot of interest recently is the acquisition of language. ‘Normal’ children - that is, those who have not had a particularly rich early environment - usually begin talking after the first year of their life. By eighteen months they have a vocabulary of about half a dozen words; at two years a vocabulary of more than a hundred words. The traditional view has been that during the first year of life babies are not mature enough to learn languages. Talking, however, is only the outer manifestation of the development of the language. Long before he first utters a meaningful word a baby can be observed responding to the language of the others. *** Economic dynamics have decisively shifted from the national economy. From now on, any country and also any business, especially a large one that wants to prosper will have to accept that it is the world economy that leads and that domestic economic policies will succeed only if they strengthen or at least do not impair the country’s international competitive position. This may be the most important - it surely is the most striking - feature of the changed world economy.

KPDS 1996 İLKBAHAR *** Today, the United States is in the grip of a second Industrial revolution. While the first, stretching from the 1870s to the 1970s, shifted the main sector of the American economy from agriculture to industry, the new revolution is shifting the economy away from traditional "smokestack" manufacturing industries to those based upon information, services and new technologies. It took the country decades to accommodate the cultural and social changes resulting from the first industrial revolution and it would be rashly optimistic to assume that Americans will not face serious stresses in coming to terms with the changes that are transforming the workplace today. *** A great many books have been written on computers, computer programming languages, particularly Fortran. To produce another book on Fortran, even the newest Fortran IV, probably seems unreasonable to most, and it is with mild trepidation that, I, the author, embark on this project. However, several good reasons can be stated for doing just that. Most computer professionals will agree that the field of computer and information science has quickly become a valid discipline for academia and that rapid changes are occurring in computer programming languages. Both of these facts demand that a new direction be taken in presenting the subject. *** Until the late l9th century most American museums and art academies considered watercolor an amateur pursuit or a preliminary to serious work in oils. Many American watercolorists saw the medium as a holiday diversion, using portable paint boxes and a free style to make what they called "snapshots" of their travels. In contrast, a few recognised the exceptional capacity of watercolours as a medium to provide clear and luminous colours in works that would evoke the ever changing nature of lakes and rivers they knew so well, and ultimately vie for supremacy with oil paintings in major art collections. *** "Human rights" is a fairly new name for what were formerly called "the rights of man”. It was Eleanor Roosevelt in 1940s who promoted the use of the expression "human rights" when she discovered, through her work in the United Nations, that the rights of men were not understood in some parts of the world to include the rights of women. The "rights of man” at an earlier date had itself replaced the original term "natural rights", in part, perhaps, because the concept of natural law, with which the concept of natural rights was logically connected, had become a subject of controversy. *** After 1933 the Western World realised that it was living in another age of absolutism, or rather, in an age of totalitarian dictatorship far worse than the worst of the old absolute kings; such regimes could be seen to be enforcing a "law" that was the command hardly of a "sovereign" but of a cruel and genocidal despot. It was ordinary people who protested: "This cannot be law. Law, if it is to deserve the name of law, must respect at least some basic rights to which every human being is entitled simply because he is human." *** The shopping centre emerged in the early 1900s in the suburbs that encircled American cities. Suburbs of that time tended to be chiefly residential and to depend on the traditional city centres for shopping.The first suburban commercial centres had three identifiable features; they consisted of a number of stores built and leased by a single developer; they were usually situated at an important intersection, and they provided plenty of free, offstreet parking. These "shopping villages" resembled small-town shopping districts, both in their architecture which was carefully traditional, and in their layout, which integrated them into the surrounding neighbourhood. The stores faced the street and the parking lots were usually in the rear.

KPDS 1996 SONBAHAR *** Certain features of the motorway undoubtedly ease the strain of driving. Gradients and bends are so controlled as to obviate the necessity of sharp braking and the absence of traffic approaching from the other direction removes one of the commonest sources of accidents. Many dangers remain, however, made more terrible by the high speeds of vehicles. A collision at seventy miles an hour is almost inevitably in its results. A mechanical defect in the car or a puncture can lead to loss of control and catastrophe. The car should be completely roadworthy and tyre pressures and treads need to be checked at regular intervals. *** The Antarctic is the most remote continent in the world and the last to be discovered, but nevertheless constitutes about one tenth of the world’s land surface. So far it has escaped the worst of man’s destructive ingenuity but today it is threatened by man’s insatiable appetite for natural resources, and seems to be in danger of losing its pristine environment which serves as the perfect natural laboratory for scientists to pursue knowledge for its own sake. *** Inflation is process of steadily rising prices, resulting in a diminishing of the purchasing power of a given nominal sum of money. In other words, you can buy fewer goods for 1 pound in December than you could in January of the same year. One type of inflation is known as demand-pull inflation. This occurs under conditions of full employment, when demand exceeds supply of goods; that is to say, when people want to buy more goods than are available. The process of demand-pull inflation operates as follows. An increased demand for goods leads to an increased demand for labour, resulting in higher wages and salaries. This has the effect of increasing costs of production and thus causes increased prices. However, as wages and salaries are higher, the increased demand for goods continues, and so the cycle goes on. *** In the business world today, many companies are showing less interest in the theoretical potential of their staff and more in what they are actually doing at the time. As an alternative or supplement to judging academic credentials, many firms have developed “assessmentcentres” in which employees handle simulated business problems, in a setting as close to real life as possible, to demonstrate their competence or indicate the need for training. Candidates for administrative jobs, for example, might work their way through a sample in-box. Bosses find those promoted because of their assessment-centre scores to be competent and the candidates feel the system is fair. In fact, the systems can be working well and giving satisfaction. *** In Eminent Victorians Lytton Strachey portrays four dominating personalities of the nineteenth century. He is, noticeably, free of undue reverence for the great; indeed his satirical view of life enables him to discover in them many flaws which were discreetly overlooked by previous historians. Perhaps his portrayal of General Gordon is the most controversial of all. Certainly he was a gifted and gallant soldier, but was he also an unbalanced mystic and a self opinionated eccentric? His portrait of Dr Arnold is also disturbing. Was he a wise and foreseeing educationalist and headmaster or try sternly imposing his will on the students in care? The questions thus raised are intensely provocative and make reading stimulating. *** If the key to good nutrition is consuming a variety of foods, then vegetables can truly stand as the cornerstone of a health diet. Of all foods, they offer the most diversity. There are literally hundreds of varieties available to us, and because of careful plant breeding, today’s vegetable harvest is continually being expanded and improved. In addition, vegetables are replete with nutrients. They supply nearly all of the vitamins and minerals required for good health, many of them – especially starchy vegetables like potatoes and winter squash - contain complex carbohydrates, which furnish us with energy. Most also provide dietary fiber, and a few, such as lima beans and potatoes, can contribute significantly to our protein intake. At the same time, vegetables contain no cholesterol, have little or no fat, and are low in calories. In nutritional parlance, vegetables are “nutrient dense” – that is, their store of nutrients is relatively high for the number of calories they supply.

KPDS 1997 İLKBAHAR

*** The unfavourable effects of cigarette smoking on the heart have frequently been described, but the exact basis for these effects has not been clarified. Some investigators believe nicotine to be culprit and there has been some experimental work in animals indicating that large doses of nicotine in conjunction with cholesterol feeding and vitamin D could produce a disease of the arteries resembling that seen in humans. An alternative explanation has been offered by other scientists who have pointed to the possible role of carbon monoxide being inhaled with the cigarette smoking.

*** Tigers grow to lengths of ten feet or more and can be bigger than the largest lion. They have immense strength. They clutch their prey to them, holding on with their claws, and depend on the crushing bite of their powerful jaws to end the struggle. They swim very well and can often be seen splashing about in water on very hot days, since they apparently suffer from heat. When the air is chilly, however, they avoid wet or damp vegetation. They can climb, but do not approach the leopard’s ability in this. They can negotiate treacherous rocky areas but generally prefer to stay on level ground. They are not as well equipped with senses as one might expect. They apparently depend on their hearing while hunting. Their eyesight is not particularly good, they seem unable to spot prey until it moves.

*** Agriculture remains the most crucial area for development, here it seems that the most intractable problems of resistance to change exist. One may argue that scientific training in agriculture by itself is unlikely to have any marked impact on agricultural output. Any attempt at vocational training in agriculture presupposes that a meaningful structure of incentive exists for the individual farmer to increase his output, improve his techniques, and expand his range of activities. Without such incentives and opportunities, agricultural education can have little impact.

*** Scientists have long sought ways to define and measure human intelligence. And while theories of intelligence have grown more sophisticated since the 1800s when some believed mental abilities were determined by the size of a person’s head, researchers still do not agree about certain fundamental principles of human thought. They, therefore, continue to debate such basic questions as whether heredity or the environment is more important in forming intelligence.

*** Some decades ago there was hardly such a subject as the economics of education. Today it is one of the most rapidly growing branches of economics. Together with health economics, it makes up the core of the economics of human resources, a field of inquiry which in the last few years has been silently revolutionising such traditional subjects as growth economics, labour economics, international trade, and public finance. Consequently, the economics of education with its concept of human investment has rapidly transformed large areas of orthodox economics.

*** The novelist E. L. Doctorow is best known for his mixing fiction with historical fact, by placing his stories within the framework of public events. In fact, by integrating the front-page news of the 20th century America with the lives of his characters, Doctorow gives readers the ‘feel’ of an era, combining the unusual and the commonplace. His latest novel ‘World’s Fair’ shows how the events of the turbulent 1930s helped mold the sensibilities of his young protagonist.

KPDS 1997 SONBAHAR

*** In 1964 the United Nations Conference on Trade and Development was held. For the first time the poorer nations of the world came together to act as a pressure group on trading matters. The Conference made the following recommendations. The Developing countries should be given free access to world markets for their manufactures and semi manufactures by the elimination of quotas and tariffs. International commodity agreements should be made for each major primary commodity in world trade to stabilise commodity prices. Moreover, compensation schemes, whereby the underdeveloped countries are compensated for the declining prices of their primary products, were recommended for consideration. The Conference also resolved that the developed countries should aim to provide at least 1 per cent of their national income as aid for the underdeveloped countries.

*** In 1945, following the Second World War, the allies that is, the United States, the Soviet Union, and Britain drew up and signed the Potsdam Agreement. The main points of this agreement were that militarism and Hitlerism should be destroyed; that industrial power should be so reduced that Germany would never again be in a position to wage aggressive war; that surplus equipment should be destroyed or transferred to replace wrecked plants in allied territories; that Germany should be treated as an economic whole, and that local self-government should be restored on democratic lines as rapidly as was consistent with military security.

*** In earlier centuries it was thought that a great continent must exist in the southern hemisphere, around the South Pole, to balance the known land masses in the north. Its real extent was better understood in the 18th century, particularly when Captain Cook sailed for the first time south of the Antarctic Circle and reached the edge of the icepack. A portion of the ice-covered continent was first sighted by Edward Bransfield in 1820. Explorers of several other nations also sighted portions of the coast-line in other quarters and wrote detailed accounts of their observations. However, in the light of these accounts, the first extensive exploration was made by Captain James Clarke Ross in 1841 when a great part of the Antarctic was discovered.

*** The police are a regular force established for the preservation of law and order and the prevention and detection of crime. The powers they have vary from country to country and with the type of government; the more civilised and democratic the state is, the less police intervention there is. England, compared with other countries, was slow to develop a police force, and it was not until 1829 that Sir Robert Peel’s Metropolitan Police Act established a regular force for the metropolis. Later legislation established county and borough forces maintained by local police authorities throughout England and Wales.

*** Oceanography is the scientific study of the world’s oceans which cover over 70 percent of the earth’s surface. The beginnings of modern oceanography go back to the 1870s when, for the first time, wide ranging scientific observations and studies of the oceans were undertaken by British. Since then, oceanography has developed into a highly technical and interdisciplinary science which is now divided into several fields of study. These are biological oceanography, which deals with the study of the marine organisms and marine ecology, chemical oceanography, which is concerned with the composition of sea water, and physical oceanography, which studies ocean currents, tides, waves, and the role played by the oceans in climate and weather. Geological oceanography is also another branch of oceanography and is mainly concerned with the formation, composition and evaluation of the ocean basins. Oceanographic knowledge is essential to allow exploitation of the enormous food, mineral and energy resources of the oceans wi

*** The International Bank for Reconstruction and Development (IBRD) known as “the International Bank” or as “the World Bank” is an agency of the United Nations established in 1945. It has the primary function of making loans available to assist developing countries. Usually, loans are made to finance specific projects of investment in underdeveloped countries; and the Bank will normally make a loan only if it is satisfied that the investment will yield a revenue sufficient to enable the payment of interest on the loan, and repayment of the sum lent. In 1983 the Bank made loans to the value of $3.300 million. Thus a sizeable amount of lending is channelled through the Bank, but it is clear that some projects of great value to underdeveloped countries cannot be financed in this way, because they would not yield returns quickly enough or large enough to meet the Bank’s requirements for interest and repayment.

KPDS 1998 İLKBAHAR

*** Paper has been known in one form or another from very early times. The papyrus reeds of the Nile swamps served the ancient Egyptians for sheets upon which to inscribe their records. The Chinese and Japanese, centuries later, were using something more akin to modern paper in substance, an Asiatic paper-mulberry, yielding a smooth fibrous material, being utilised. With the spread of learning in Western Europe the necessity of a readier medium made itself felt, and paper began to be manufactured from pulped rags and other substances. Other papermaking staples were later introduced, such as linen cotton and wood-pulp. The chief raw material in the world paper industry now is wood-pulp, the main exporters being the timber-growing countries of Canada, Sweden and Finland.

*** When there has been a serious disaster such as an earthquake or flooding, various relief efforts are rapidly put into effect. However, experience has shown that it is usually impractical to attempt mass immunisation immediately following a disaster and that, when attempted, it detracts from the overall relief effort without producing a discernible benefit. Effective immunisation requires prior planning good systems of communication and transport and access to the population at risk. These requirements cannot be met in the immediate postdisaster period. Efforts to achieve mass vaccination in the relief phase also drain whatever limited manpower, communication facilities, and transportation exist.

*** The great expansion in energy demand over recent years has been met to a large extent by petroleum oil. The total world reserves of petroleum oil are still uncertain since large parts of the world are still not fully prospected. The cutback in oil production and the rise in the price of Middle Eastern oil following the 1973 Arab-Israeli war unleashed a worldwide energy crisis which affected the economies of consumer countries. One result of this crisis has been that Britain has increased its North Sea oil production and become the fifth largest oil producing country in the world.

*** Universities are institutions of higher education whose principal objects are the increase of knowledge over a wide field through original thought and research and its extensions by the teaching of students. Such societies existed in the ancient world, notably in Greece and India, but the origin of the University as we know it today lies in medieval Europe, the word “universitas” being a contraction of the Latin term for corporations of teachers and students organised for the promotion of higher learning. The earliest bodies to become recognised under this description were at Bologna and Paris in the first half of the 12th century. Oxford was founded by an early migration of scholars from Paris, and Cambridge began with a further migration from Oxford. Other universities sprang up all over Europe from the 14 th century onwards.

*** In 1903 the United States signed a treaty with Panama, which gave the United States rights in perpetuity ever a 16 km wide strip of land extending across the narrowest part of Panama for the purpose of building and running a canal. The canal built, now known as the Panama Canal, connects the Atlantic and the Pacific Oceans and is just over 80 km long. Its depth varies from 12 to 26 meters. It is constructed above sea-level, with locks and has been available for commercial shipping since 3 August 1914. An agreement was reached in 1978 for the waterway to be turned over Panama by the end of the century.

*** Romanticism is a term for a movement in the arts, that is, in music, painting, sculpture or literature, which seeks to give expression to the artist’s feelings about his subject rather than to be concerned with form and reality. The romantic view is that art is nature seen through a temperament; the realist view, on the other hand, is that art is a slice of life. In painting Delacroix (1789-1863) is the romantic artist par excellence with his uncontrolled expression of the passions and love of the exotic. In literature the Romantic movement reached its finest form in the works of Goethe, Schiller and Heine; in the poetry of Byron, Keats, Wordsworth, Shelly and Blake; and in the writings of Victor Hugo. Since Romanticism is partly a matter of temperament in the artist just as Classicism is, it may be found at all times and places, although whether or not it becomes predominant depends on contemporary taste.

KPDS 1998 SONBAHAR

*** Eliminating poverty is largely a matter of helping children born into poverty to rise out of it. Once families escape from poverty they do not fall back into it. Middle-class children rarely end up poor. The primary reason poor children do not escape from poverty is that they do not acquire basic mental skills. They cannot read, write, calculate or articulate. Lacking these skills, they cannot get or keep a well-paid job. The best mechanism for breaking this vicious circle is to provide the poor with better educational opportunities. Since children born into poor homes do not acquire the skills they need from their parents, they must be taught these skills in school.

*** Alcohol, nicotine and caffeine are psychoactive drugs that are freely available in our society. Their wide spread use shows that they provide a common solution to the problems of vast numbers of individuals. The extent and the nature of their use is not , however, uniform but varies with the particular sub-culture involved. To take alcohol, for example, there are wide differences between the drinking habits and rituals of merchant, seamen and businessmen, between Italians and Jews. Each sub-group in society will have a conception of what the permissible and desirable effects of alcohol are, how much it is necessary to drink to achieve this desired state; what is normal and what is deviant drinking behaviour.

*** Not just in substance but in manner too, Robin Trevelyan, who is the Prime Minister’s new righthand man, is a politician in the old style. He avoids the flourish which characterises modern politicians. His speeches are at best unemotional, at worst dull. He is all but incapable of inspiring an audience. His face is inexpressive, solid almost. He evades making promises and is completely lacking in vision. He is a politician whose talent has never been to inspire the mob.

*** In the early 1970s, there was a great deal of optimism about improving women’s position, ending male privilege and doing away with gender divisions and even gender difference. Equal opportunities legislation was enacted in many countries, and the voice of the women’s movement was heard criticising discrimination between the sexes in every sphere of working life. Now it is clear that legislation can make only a marginal difference to entrenched patterns of job segregation and inequality. The voices of feminism, too, are varied; some demand equality with men while others pursue the revaluation of women’s skills and ‘womanly’ virtues.

*** Work is central in British culture. When someone asks one ‘What do you do?’, they really mean ‘What work do you do?’. When a woman is asked ‘Do you work?’, what is meant is ‘Are you doing a paid job?’. Yet many people without a paid job work at other kinds of productive activities. Women, notably, perform an unpaid ‘double shift’ in the home as housekeepers and mothers. To confine the term ‘work’ to paid employment, therefore, restricts it far too narrowly. There are many other kinds of work, some of which can take more time and energy than we put into our paid employment from the voluntary working in the garden to repairs to the house or the car. In other cultures, work is not as highly valued as this; some people value leisure more, and work only as much as they need in order to provide basic necessities.

*** All of us are born, all of us will die; but there is infinite variety in the nature and circumstances of these two events themselves and in what happens to our bodies and our minds in between. Some individuals, for example, are born without difficulty and grow uninterruptedly during childhood and adolescence, suffering at worst only minor infectious diseases and accidents. As adults, they reproduce their kind. They age gradually until, in extreme old age, they die peacefully without pain or discomfort. This is an idealised picture of how we would like things to be, rather than the reality that most people experience. Death comes to many of us, not when we are old, but during or before birth, in infancy, in adolescence, in early adulthood or in middle age.

KPDS 1999 İLKBAHAR

*** The Amazon is the largest river in the world. It carries about a quarter of the world's running water and is the second longest after the Nile. Much of it is brown, brackish, piranha-infested and bitterly cold. Ranging from narrow tributaries and raging rapids to stretches of prodigious width and calm, the river's banks can take half a day to reach. In parts, it can drop up to 40 metres in less than a kilometre Furthermore, it runs through deep canyons and steep gorges that have been carved out by its turbulent waters.

*** Most poetry anthologies are assembled by poets. This is not necessarily a good thing. They are in fact assembled for many different reasons. Some resemble star charts, trying to define the scope of the new and show us what direction poetry is heading. Others turn their gaze on the past, seeking to define poetries of earlier centuries or to identify influential currents of thinking and feeling. Yet other anthologies strive to present enduring images of the beautiful for the reader's pleasure, as if poems were bunches of flowers.

*** The great window-dresser Gene Moore seems to have been self-taught. As a young man his main idea was to get away from Birmingham, Alabama, then a town of steel and pollution. It was, he said, the wrong place to be born in for anyone with dreams. He dreamt of being a concert pianist and then of being a painter. But he decided that he did not play very well, and presently gave up painting. In New York in the 1930s he got various casual jobs. One was with a store that decided he had flair and put him in its display department, and that was the start of his career. He worked for a number of shops promoting their wares, and built a reputation for innovative ideas.

*** Everybody needs vitamins and minerals to remain healthy. The questions are, which ones, how much and when? And the answer is surprisingly simple: take XXX. Actually, the Department of Health has recognised 18 essential vitamins and mineral that we need on a daily basis. The daily amount required of these vitamins and minerals is termed the Recommended Daily Allowance (RDA). XXX meet this requirement and more. As a new vitamin complex, it contains these 18 essential vitamins and minerals, plus a total of no less than 31 other micro nutrients, including the complete antioxidant group and folic acid. There is no more complete a multimineral multivitamin on the market. So, because you don’t always eat as you should, it makes sense to take XXX.

*** Pollution is no respecter of national boundaries today. But environmental scientists can still be surprised by the distances that large quantities of industrial pollutants can sometimes be carried by winds. For instance, a group of chemists at the University of Washington in Seattle have been involved in a case study of such pollutants which reached the West Coast of America all the way from Asia. They are keen to understand how such an event could take place and to what extent it could have been forecast. In fact, back in March 1997, pollutants such as carbon monoxide from Asia had been spotted as far across the Pacific Ocean as Hawaii. Thus, it seems increasingly likely that the West Coast of America is particularly exposed to pollution from Asia.

*** In Japan, there is a government investment and loan programme, known as zaito. Unlike normal government spending, zaito relies not on tax revenues but on people's savings. These are drawn from the publicly-owned postal savings system, which by law must place all deposits with zaito, and from the postal life-insurance schemes and various pension funds. The finance ministry, which has run zaito for more than 100 years, then lends the money out. During the second World War, zaito financed Japan’s military buildup. Afterwards, it paid for reconstruction and helped to channel lowcost funds into such strategic industries as steel and car-making. More recently, it has turned to “social” investments, such as infrastructure projects and housing.

KPDS 1999 SONBAHAR

*** Edison, one of the pioneers of modern technology, lacked formal education. His understanding of literature, art, history and philosophy was superficial. Also, despite the fact that he had invented the phonograph and founded a recording company, his musical taste was abominable. He is, therefore, sometimes regarded with disdain by academic scientists, who often forget that his ingenuity, inquiring spirit and tireless efforts contributed significantly to the development of modern technology.

*** Edger Lawrence Doctorow is 55, and on almost anyone’s list he is counted among the leading serious American novelists today. Although he is not simply a writer of comedies, his books sell widely, and three have been made into movies. Readers, some critics excepted, have come to relish the blending of the fact and fiction that marks his odd scrutiny of the American past. In his recently published book, World’s Fair, he turns his historically inventive method on himself drawing heavily on material taken from his 1930s boyhood.

*** Under increasing social pressure in the late nineteenth century, some universities opened their doors to a small number of women. More significant, however, was the founding of many women's colleges, frequently run by women. These colleges strove over the years to maintain a curriculum equivalent to that of the largely male universities. Therefore, many leaders of the women's college movement saw themselves as social reformers. Although women entered universities in large numbers in the first half of the twentieth century, their participation was limited by their professional objectives: teaching, social work, nursing, home economics and the like were "women's fields".

*** William Saroyan’s parents and relatives were Armenian immigrants who settled in the farming area around Fresno, California. Saroyan left school at fifteen and went to work, doing odd jobs. During this time he read widely and began writing in his distinctive natural style. By the late 1930s his many short stories, novels and plays had established him as a writer. Many of his stories have grown out of his experiences in the Armenian community around Fresno.

*** Atmosphere is the gaseous envelope of the earth, and consists of a mixture of gases and water vapour. The variability of the latter is meteorologically of great importance. The ozone layer, which absorbs solar ultraviolet radiation, especially lethal to plant life, lies between 12 and 50 kilometres above the earth. The lower level of the atmosphere, up to a height of about 12 kilometres, is known as the troposphere, and it is in this region that nearly all weather phenomena occur. This is the region of most interest to the forecaster studying temperature, humidity, wind-speed and the movement of air masses.

*** For many years after Mt. Everest had been shown to be the highest mountain in the world, political conditions in Nepal, lying south of the summit, and in Tibet to the North, prevented mountaineers from attending an ascent. At last in 1921 the Tibetan authorities gave permission and the first expedition organised, as were all subsequent expeditions by international joint committee, was sent out. This was primarily a reconnaissance. Besides mapping the Northern flank, it found a practicable route up to the mountain. By 1939, six further expeditions had climbed on the northern face. Some were hampered by bad weather, others by problems previously little known, such as the effect of high altitudes on the human body and spirit. Nevertheless, notable climbs were accomplished, though the summit was never reached.

KPDS 2000 İLKBAHAR

*** Heat-waves, if the temperature is high enough, above 40°0 for instance, lead to wilting, and even death in plant, because of structural damage to essential proteins. The problem is that plants react by closing their pores when, due to a serious heat-wave, they are subjected to water stress, so shutting down on transpiration and conserving water. Just as the body would overheat dangerously if it shut its pores to prevent sweating, so, in a plant, the shutting of the pores will cause permanent damage, if not death. Temperatures above -5°0 can damage most plants if lasting for half an hour or more. High soil temperatures will also damage roots and prevent nutrient uptake.

***People in other European countries have been wondering for some time why and how Norway has stayed out of the European Union. Austria, Finland, Sweden joined in 1994, almost without any public debate, just a few months after their governments had proposed the joining. By then, the Norwegians had been debating the issue for 33 years, ever since their government had started the drive towards unionisation. One reason for the success of Norwegian resistance is that in both 1952 and 1967, when the Norwegian government sent off applications for joining the EEC, President De Gaulle of France rejected the proposals. He feared that the inclusion of Norway, as of England, would complicate and slow down EEC integration.

*** A conspicuous feature of cities in many countries, in particular those of Western Europe, is that buildings and streets devastated during the war are, once peace is reinstated, rebuilt in exactly the same manner as they existed before. Enormous efforts are taken to recreate the environment with total fidelity. This reflects the extent to which ordinary people value the traditions and culture of the past. In Japanese cities, however, one sees little evidence of such respect for tradition. Tokyo presents an extreme example: it is quite common these days for the appearance of a street or quarter to change almost beyond recognition every year. In provincial cities as well, one often finds that an absence of several years has rendered a city almost unrecognisable.

*** In its full force the Gulf Stream, which begins in the Gulf of Mexico, carries warm water to a depth of up to 100 meters at rates of up to 8 kilometres an hour, and penetrates right up into the Arctic Circle to the north of Scandinavia, bearing with it a climate that makes life just about tolerable, even in the thick of the winter. The energy it carries in the form of heat is equivalent to 100 times the entire use of energy in human societies across the world or put another way, more than 27,000 times Britain's electricity generating capacity. In terms of temperature the Gulf Stream heats the surface over a wide area by at least 5°C. Were the-Gulf Stream to fail, temperatures over northern Europe would fall by more than 10 centigrade degree during the winter months. Northern Europe would have a climate comparable to that of Siberia: just how it would support its current population is difficult to imagine.

***Modern education is almost exclusively focused on preparing children for an urban future, as consumers in a global “free” market. This makes a return to any sort of rural existence almost an impossibility for those tutored by the Western education system in the 21st century. The fact is that, for all the fashionable talk about cultural diversity, schools, colleges and universities today prepare their graduates poorly for anything other than a uniform urban existence. We educate the young from country to city alike, to be urban with urban appetites, skills, minds, dependencies and expectations. And as globalised, future will overwhelmingly mean an urban future, our graduates of tomorrow will be trained, above all, to keep the wheels of the global economy running, with all the implications that has for nature and society.

*** Within a short time after the outbreak of the Second World War, Britain was without imports of many vital pharmaceuticals that had formerly come from Japan, Germany and the Far East. As a result, the first wartime government set up systematic research into the cultivation and medical use of herbs, By 1940, women's voluntary organisations had been drawn into a national campaign to gather wild herbs, Up and down the country, County Herb Committees were organised to oversee the gathering, drying, distillation and distribution of the medicinal herbs. Lay people were given brief locally-based training in how to recognise herbs, store and dry them. Farmers were given subsidies to farm certain naturally hardto-find herbs. By 1943, every county had its herb committee and during the five years of the Second World War, over 750 tons of dried herbs were gathered and turned into medicines.

KPDS 2000 SONBAHAR

*** Restorative justice does not ask 'how do we punish?', but instead asks 'how do we get people to take responsibility for what they have done?'. Paying a fine, or even going to prison are easy options for some people. They are all ways that offenders can avoid taking responsibility, because in this way they never have to face the human reality of what they have done. Prisons have been called "universities for criminals". Young people go in for unpaid fines, often for victimless crimes, and they come out with a degree in burglary or worse. I am not saying that the answer is to tear down all prisons. Far from it. There are people who are dangerous to society, who the community will want to keep locked up. Prison can also be part of a sentencing package under restorative justice. But the vast majority of people in prison are not violent, and do not need to be there. What they do need is to be brought face to face with the human reality of the harm they have caused, and they must be given an opportunity to rectify.

*** In theory, the multimedia age should be killing off bookshops. Who still has time to read books, what with surfing the Internet, viewing scores of new digital television channels, and putting in ever-longer hours at work? And presumably those few people who do still read books will be buying them on the Internet. After all, Amazon, a bookseller, is the most cited example of a successful online retailer. So much for the theory. What about the practice? This week the largest bookshop in Britain opened up in the old Simpson's of Piccadilly in London. With 265,000 titles and 1.5 million books, the new branch of Waterstone's stretches over seven floors. A department store, which once sold everything from sushi to plus-fours, is now devoted entirely to one product - books. The new Waterstone's is almost next door to Hatchards, a mere fivestorey bookshop, with a wellestablished clientele, and two smaller bookshops. It is also less than a mile from Borders, another huge bookstore in Oxford Street.

*** In the coming weeks, wine makers north of the equator will oversee the harvesting and fermenting of the first vintage of the millennium. But long before the finished product reaches the shelves - before it even makes it out of the barrel, in some cases samples will be offered to exporters and distributors. A select group of wine critics will also be given a taste. Most will record their impressions in the extravagant prose that wine journalists unfortunately love to use. Others will go one step further and assign numerical grades. These days a high score is more effective than mere praise. it can make a comparatively unknown wine into a highly desirable one that everyone is seeking to buy.

*** The seventeenth-century scientist Francis Bacon was the first to insist that science be methodically separated from values so as to make it truly 'neutral', or objective. In reality, he did nothing of the sort. His 'scientific knowledge', instead of being value-free, set out explicitly and purposefully to give humanity power over nature. 'Truth and utility are perfectly identical,' he wrote in his “Novum Organum”, and 'that which is most useful in practice is most correct in theory'. In effect, he merely replaced the old 'subjective' values of 'good' and 'evil' with the values of 'useful' and 'useless', or more precisely 'of contributing or not contributing to man's domination over or transformation of the natural world'. There were to be no limits to this transformation. His goal was explicitly stated. It was to 'achieve all things achievable'. At least he was honest enough to admit the fact. Modern science has followed Bacon's lead exactly, but does not admit it.

*** One never finishes learning about art. There are always new things to discover. Great works of art seem to look different each time one stands before them. They seem to be as inexhaustible and unpredictable as real human beings. It is an exciting world of its own with its own strange laws and its own adventures. Nobody should think he knows all about it, for nobody does. Nothing, perhaps, is more important than just this: that to enjoy these works we must have a fresh mind, one which is ready to catch every hint and to respond to every hidden harmony: a mind, most of all that has not been dulled by the terminology and ready-made phrases of art appreciation. it is infinitely better not to know anything about art than to have the kind of half-knowledge which makes for snobbishness. The danger is very real.

*** If a greater proportion of the food people eat were to be locally produced, this would be of great benefit to the farmer. A mix of local, regional, national, and international production would still be available; indeed, the goal would not be to put an end to the international trade in food, but to avoid transporting food thousands of miles when it could instead be produced next door. Such a shift would help revitalise rural economies ruined by the global economy. Less money would go into the hands of corporate middlemen, and far more would remain in the hands of farmers, This would especially be the case with the direct marketing of food via farmers' markets and farm stands and other forms of community supported agriculture. If farmers were not impelled to specialise their production in a few global commodities, the trend towards ever larger and more highly mechanised farms would slow down. Moreover, since small farms use a proportionally higher amount of human labour than mechanised inputs, a return to smaller farms would help bring back some of the 700.000 farm jobs the UK has lost during the last halfcentury of agricultural progress.

KPDS 2001 İLKBAHAR

*** In the case of shallow tunnels or in urban areas it is often possible. by means of carefully sited boreholes, to gain an idea as to the nature of the ground and water conditions. Under high mountains boring becomes expensive so reliance has to be placed upon geological interpretations. As strata can vary so much, surprises are often met with and techniques sometimes have to change in a single tunnel. In the Severn railway tunnel (4 mls 628 yd long, completed in 1886) great quantities of water were unexpectedly encountered and are still being pumped out.

*** The ideal of a family life shared by all in 19th century England survived into the early 20th century, until home life was seriously dislocated in 1914 by World War I, which was a war on the largest scale the world had ever known. But since the last decade of the 19th century new developments and inventions had been rapidly affecting the home life of an increasing number of people. Town and country were knit more closely together by easier railway travel, cheap and efficient postal services, the popularity of the bicycle, the development of the petrol engine and the cheap popular newspaper; such things as these helped to break down social formalities and to place women again on a more equal footing with men.

*** As with all revolutions, the causes of the American Revolution which separated the original thirteen American colonies from Great Britain were social, economic and political and so inextricably interwoven that it is difficult to appraise them. First there was the distance from Great Britain and the environment of a new country which, whether they willed it or not, had gradually over a period of 150 years turned Englishmen into Americans. The older stock was largely English but the bulk of them, as a contemporary historian commented. "knew little of the mother country, having only heard of her as a distant kingdom, the rulers of which had in the preceding century persecuted and banished their ancestors to the woods of America". With each generation and with each move westward old contacts were broken. Furthermore large groups of colonists had come from Germany, Ireland and other parts of Europe and had no ties with England and, in the case of the Irish, no affection.

*** Most people take it for granted prices will always nice and understandably so. A 60-year-old American has seen them go up by more than 1.000 % in his life time. Yet prolonged inflation is a comparatively recent phenomenon. Until about 60 years ago prices in general were as likely to fall as to rise. On the of the First World War, for example, prices in Britain, over all, were almost exactly the same as they had been at the time of London in 1666. Now the world may be reverting to that earlier normality. The prices of many things have fallen over the past 12 months or so. Not only computers and video players, but a wide range of goods- from cars and clothes to coffee and petrol - are in many countries, cheaper than they were a year ago.

*** Translation renders knowledge mobile. The task of the scientific translator, no less than the literary translator, has been to create new texts, to multiply sources into new languages, and thereby to produce new "originals". Over time, translation itself has built a great scientific library, ever more enriched, and accessible. Although we may think of scientific translation as literal, mechanical work, this has never been the case. The reasons for this are complex, but have much to do with the lack of exact one to-one correspondence among languages. Translating science always involves interpretation, the remaking of an original. If it did not, machine translation would have long ago rendered the scientific translator extinct.

*** Water of doubtful purity for drinking can be rendered safe by boiling and then can be cooled in water bags or in earthenware containers, which must be protected from dust and flies when boiling is not possible, drinking water can in many areas be adequately sterilised by chlorination; one tablet of halazone is added to one litre of water and allowed to stand for 30 minutes. Water containing suspended matter should be filtered first. There is, however the danger of a particularly serious infectious disease in many regions of Africa, the Middle and Far East and South America. In these regions the water of rivers, lakes and canal may be infected, and the disease is acquired when the water comes in contact with the skin.

KPDS 2001 SONBAHAR

*** The great expansion in energy demand over recent years has been met to a large extent by petroleum oil. The total world reserves of petroleum oil are still uncertain since large parts of world are still not fully prospected. The cutback in oil production and the rise in the price of Middle Eastern oil following the 1973 Arab-Israel war unleashed a worldwide energy crisis. The result has been that Britain has increased its north sea oil production and has become the fifth largest oil producing country in the world.

*** Each year thousands of people are diagnosed with congestive heart failure - a condition in which a weakened heart can't pump much blood as the body needs. Drugs like beta-blockers help stabilize many patients in the earliest stages of the disease. But there aren't a lot of options for folks in the later stages. Heart transplants are one solution, but they are short in supply. It is such good news to hear that another type of mechanical pump, called a left ventricular assist device, may be a viable alternative. Instead of replacing the heart entirely, the device attaches to the organ's left main chamber, boosting its output. The device is twice as likely as drugs to keep patients alive after one year.

*** The continent of South America looks as if it managed to escape the attentions of the British Empire. However, this was more because the British didn't need to exert formal control over the countries and peoples of this continent. The Monroe doctrine imposed by America, served British interests quite well enough. The doctrine made it clear that the United States would not tolerate foreign meddling in the Americas. This policy meant that Britain could get all the benefits of trade and investment in South America with very little of the administrative costs. In addition, it could be reasonably safe in the knowledge that other European states wouldn't be able to steal the markets through annexation. Britain had very strong commercial links with South America, especially with Argentina. In many ways, the influence and power that Britain could hold over the policies of the individual South American states meant that they could almost be termed as being part of Britain's informal empire. The islands in the region that were formally annexed were done so mostly out of strategic naval considerations.

*** Hong Kong, with a population of about 6.8 million at mid-2000, is a small but dynamic city which has earned an international reputation as a leading commercial and financial center as well as a highly efficient port. Subsequent historical and political events led to the development of the manufacturing industry. Hong Kong has also seen a rapid expansion of its services sector in the past two decades, contributing over 85 per cent of Hong Kong's Gross Domestic Product (GDP) in recent years. Hong Kong has a two-tier system of representative government. At the central level, the Legislative Council legislates, approves taxation and public expenditure and raises questions on the work of the Government. According to the Basic Law, the Legislative Council is to be constituted by election.

*** Failing to discover any account of the purposes, for which nature is arranged as it is, and finding the explanations actually offered by the philosophers to be suspect and non illuminating, Socrates abandoned all his effort to find out why things are as they are by examining nature itself. He turned instead to the examination of "logos" that is, statements, arguments, or in general, words-as a way of discovering something true. The distinctive feature of Socratic inquiries is that they took as their immediate object not some phenomenon in the natural world but some person and his ideas. Socrates hoped that by methodologically and repeatedly examining someone's ideas he might ultimately lead him to the discovery and establishment of the truth.

*** College students who eat all their meals at one of the college or university regulated food services are likely to be offered a wellbalanced diet. The offering of well-balanced meals does not ensure, however, that the students make wise selections. In addition many students eat their meals in restaurants or other public eating-places where they may not be offered foods that provide all the nutrients needed by the body. College students, generally speaking, are well fed; few of them show extreme deficiency symptoms. Many students, on the other hand, are operating at a level below their achievement potential because of the insufficiencies in their diet.

KPDS 2002 İLKBAHAR

*** Does advertising encourage waste by persuading consumers to buy goods that they do not need? In reply to this, it has been pointed out that all the consumer really needs, is a bare minimum of clothing, food and shelter, and that one of the distinguishing marks of any civilized community is that it lives well above the minimum subsistence level. Most advertising is designed to influence the consumer's spending power. In western countries, advertising has played a great part in bringing labour saving equipment, and so a degree of leisure, and even luxury, to millions. Advertising that encourages the public to want more is also claimed to act as an incentive making people want to earn more in order to buy the goods advertised, and therefore making them work harder. For this reason advertising has been defended as having an essential part to play in the move towards higher standards of living. The defenders of advertising also point out that it is not solely concerned with encouraging the public to spend. Banks, insurance companies and building societies are amongst the commercial advertisers who encourage saving.

*** Though Italy's national boundaries have altered relatively little since unification in the 1860s, national identity is qualified by sharp internal differentiation. Economic and occupational structures, standards of living, political loyalties, cultural traditions and even language vary substantially between parts of the country. Only since the 1970s has there existed a comprehensive system of regional government with financial and legislative authority. However, the division of powers between central and regional governments is imprecise, and in practice the latter depend on substantial resources from the former. In the absence of clear and effective rules, relations between the regions and the central government are determined by a process of political bargaining. In this process, political alliances and personal linkages play a vital role. In this respect, the Italian system may be defined as a kind of federalism.

*** Sir Philip Sidney was a 16th-century English poet and critic. His Defence of Poesyis the only major work of literary criticism in sixteenth-century England, a period during which Italy and France produced large numbers of critical treatises, heavily influenced by Aristotle's Poetics. By contrast, Sidney's text is highly eclectic, drawing together aesthetic principles from several traditions and emphasizing especially those principles that are of primary importance to the Elizabethans: ideal imitation, moral teaching and decorum. Looking back to Aristotle, Sidney defines poetry as an imitation of nature, but links that imitation to his view of the poet as maker. The poet imitates not the real nature we see but rather he imitates an ideal nature. Sidney also makes large claims for the didactic role of poetry, following Horace's idea that poetry teaches by delighting.

*** Although the idea of the skyscraper is modern, the inclination to build upward is not. The Great Pyramids, with their broad bases, reached heights unapproached for the next four millennia. But even the great Gothic cathedrals, crafted of bulky stone into an aesthetic of lightness and slenderness are dwarfed by the steel and reinforced concrete structures of the 20th century. It was modern building materials that made the true skyscraper structurally possible, but it was the mechanical device of the elevator that made the skyscraper truly practical. Ironically, it is also the elevator that has had so much to do with limiting the height of most tall buildings to about 70 or 80 stories. Above that, elevator shafts occupy more than 25 percent of the volume of a tall building, and so the economics of renting out space argues against investing in greater height.

*** Land cleared of trees is exposed to erosion, which can be severe in deforested areas having slopes greater than 15 to 17 percent. If land is not disturbed any further and new growth becomes established, erosion may gradually subside. If, however, vegetation on the cutover land is continually removed by man or livestock, erosion will intensify, and environmental problems can be severe. When a forest is removed from a slope, the rate of water runoff is increased two to tenfold or more, depending on the degree of clearing, slope, and rainfall. All too often this leads to flooding of agricultural land in the lowlands. In Pakistan, for example, almost 2 million hectares of standing crops on the lowlands were destroyed by floodwater in 1973, and about 10,000 villages were wiped out. Since valuable soil is lost in floods, the quantity of the arable lands decreases. Alluvial silt deposited elsewhere is rarely usable enough to compensate for such losses.

KPDS 2002 SONBAHAR

*** In modern times, it was perhaps the "gentleman scientists" of the nineteenth century who came closest to a genuinely objective form of scientific research. These privileged amateurs enjoyed a financial independence which most scientists today cannot have, and which enabled them to satisfy their scientific curiosity without the need to please patrons. With the growth of scientific research after World War II, science has become an expensive occupation. Many scientists today look back upon the 1960s as a golden age of modern-day science, when research was mainly funded by the taxpayer, and scientific enquiry was seen by governments to be part of the public good, and worth paying for. Today, the situation is very different. "Academic freedom" is now often little more than an illusion for most scientists working at universities or in publiclyfunded research institutes. Moreover, science is now largely dominated by the interests of the industrial world, and hence, hardly deserves the name "science".

*** Some people believe that meat consumption contributes to famine and depletes the Earth's natural resources. Indeed, it is often argued that cows and sheep require pasturage that could be better used to grow grain for starving millions in poor countries. Additionally, claims are made that raising livestock requires more water than raising plant foods. But both these arguments are illogical. As for the pasturage argument, this ignores the fact that a large portion of the Earth's dry land is unsuited to cultivation. For instance, desert and mountainous areas are not suitable for cultivation, but are suitable for animal grazing. However, modern commercial farming methods prefer to raise animals in an enclosed space feeding them on grains and soybeans. Unfortunately the bulk of commercial livestock is not range-fed but stall-fed. Stall-fed animals do not ingest grasses and shrubs (like they should), but are fed an unnatural array of grains and soybeans - which could be eaten by humans. The argument here, then, is not that eating meat depletes the Earth's resources, but that commercial farming methods do. Such methods subject livestock to deplorable living conditions where infections, antibiotics, and synthetic hormones are common. These all lead to an unhealthy animal and, by extension, to an unhealthy food product.

*** The chief triumph of this book is its depiction of Wellington. He is not simply the famous British general who defeated Napoleon at Waterloo. He remains a great general but he is also shown to have had feet of clay inside his splendid boots. For example, the writer dwells on Wellington's vanity and his unattractive lack of generosity in sharing the credit for his victories. This is a splendid book. Never less than interesting, but always trenchant. It redefines Wellington without diminishing his achievements and ends by reminding us that it was Napoleon who so forcefully articulated a wish that there should be "a European code of laws, a European judiciary ... one people in Europe". The ogre's dream is coming true.

*** Fast-food is such a pervasive part of American life that it has become synonymous with American culture. Fastfood was born in America and it has now swolleninto a $106-billion industry. America exports fast-food worldwide and its attendant corporate culture, has probably been more influential and done more to destroy local food economies and cultural diversity than any government propaganda programme could hope to accomplish. No corner of the earth is safe from its presence and no aspect of life is unaffected. Fast-food is now found in shopping malls, airports, hospitals, gas stations, stadiums, on trains, and increasingly, in schools. There are 23,000 restaurants in one chain alone, and another 2,000 are being opened every year. Its effect has been the same on the millions of people it feeds daily and on the people it employs. Fast-food culture has changed how we work, from its assemblyline kitchens filled with robotic frying machines to the trite phrases spoken to customers by its poorly paid parttime workforce. In the United States, more than 57 per cent of the population eat meals away from home on any given day and they spend more money on fastfood than they do on higher education, personal computers, or even on new cars.

*** Even though there have been truly significant advances in modern medicine, health problems still abound and cause untold misery. Although heart disease and cancer were rare at the beginning of the 20th century, today these two diseases strike with increasing frequency, in spite of billions of dollars in research to combat them, and in spite of tremendous advances in diagnostic and surgical techniques. In America, one person in three suffers from allergies, one in ten has ulcers and one in five is mentally ill. Every year, a quarter of a million infants are born with a birth defect and undergo expensive surgery, or are hidden away in institutions. Other degenerative diseases such as arthritis, multiple sclerosis, diabetes, and chronic fatigue afflict a significant majority of Americans. Further learning disabilities make life miserable for seven million young people and their parents. These diseases were extremely rare only a generation or two ago. Today, chronic illness afflicts nearly half of all Americans and causes three out of four deaths in the United States.

KPDS 2003 İLKBAHAR

*** It may be that golf originated in Holland but certainly Scotland fostered the game and is famous for it. In fact, in 1457 the Scottish Parliament, disturbed because football and golf had lured young Scots from the more soldierly exercise of archery, passed an ordinance that banned football and golf. James I and Charles I of the royal line of Stuarts were golf enthusiasts, whereby the game came to be known as "the royal and ancient game of golf". The golf balls used in the early games were leather-covered and stuffed with feathers. Clubs of all kinds were fashioned by hand to suit individual-players. The great step in spreading the game came with the change from the feather ball to the present-day ball introduced in about 1850. In 1860, formal competitions began with the establishment of an annual tournament for the British Open championship. There are records of "golf clubs" in the United States as far back as colonial days. However, it remained a rather sedate and almost aristocratic pastime until a 20-year-old Francis Ouimet of Boston defeated two great British professionals, Harry Vardon and Ted Ray, in the United States Open championship at Brookline, Mass., in 1913. This feat put the game and Francis Ouimet on the front pages of the newspapers and stirred a wave of enthuslasm for the sport

*** The US National Institute of Standards and Technology (NITS) will soon be testing a controversial theory about the collapse of the World Trade Center towers. According to an analysis by a leading fire-safety expert, had the fire-proofing insulation on the towers' steel structures been thicker, the towers would have survived longer and might even have remained standing after they were hit by the hijacked planes. The work is being seized on by lawyers representing victims' families and insurance companies. If confirmed, it could also lead to changes in building codes. NIST is responsible for drawing up the final report on the towers' collapses and recommending if any changes are needed. It is widely accepted that the collapses were caused by the failure of the buildings' steel structure as it was weakened by the heat of the fires.

*** The economic news from Europe was particularly disappointing in the second half of 2002. Moreover, recent surveys from the region imply little prospect of improvement in the near future. Perhaps the most worrying aspect has been the sharp decline in conditions in Germany-the area's largest and most important economy. Domestic demand in Germany is very weak and, with the global economy also struggling, Germany's manufacturers have not been able to export their way out of trouble as they have done in the past. With the economy in such a weak state, it is no surprise then that European stock markets have followed the US stock markets' downturn over the past 6 months. While individual share pries may be lower and market valuations look attractive, the economy does not. Recovery seems some way off and strong equity performance from Europe's markets seems unlikely in 2003.

*** Scientist who study earth's moon have two big regrets about the six Apollo missions that landed a dozen astronauts on the lunar surface between 1969 and 1972. The biggest regret, of course, is that the emissions ended so abruptly, with so much of the moon still unexplored. But researchers also lament that the great triumph of Apollo led to a popular misconception: because astronauts have visited the moon, there is no compelling reason to go back. In the 1990s, however, two probes that orbited the moon raised new questions about Earth's airless satellite. One stunning discovery was strong evidence of water ice in the perpetually shadowed areas near the moon's poles. Because scientists believe that comets deposited water and organic compounds on both Earth and its moon, well-preserved ice at the lunar poles could yield clues to the origins of life.

*** The long-expected decline in the dollar is now well under way. For years economists have predicted that America's huge currentaccount deficit would eventually cause its currency to plunge. So far the dollar's slide has been fairly gradual: it is down by 13% in tradeweighted terms over the past year, though it has dropped by almost twice as much against the euro since its 2001 peak. As the decline seemed to pick up speed this week, John Snow, George Bush's Treasury Secretary, declared that he favours a "strong dollar policy". That was surely the wrong answer, even leaving aside the debatable issue of whether cabinet secretaries can influence the level of exchange rates. A weaker, not a stronger dollar, is what the world needs now-so long as policymakers elsewhere respond appropriately. America promoted a strong dollar throughout the 1990s, when inflation was still thought to be the main enemy. Today it makes less sense. Even after its recent slide, the dollar seems overvalued. Moreover, with ample space capacity in America, deflation looks a bigger risk than inflation.

KPDS 2003 SONBAHAR

*** During the past few decades four East Asian economies - South Korea, Taiwan, Singapore and Hong Kong - have achieved the fastest rates of economic growth the world has ever seen. In 1962 Taiwan stood between Zaire and the Congo on the global ranking of income per head: by 1986 its neighbors were Greece and Malta. In 1962 South Korea was poorer than Sudan: by 1986 it was richer than Argentina. Today the four "dragons" account for 10 per cent of manufactured exports worldwide, not far short of America's 12 per cent. Understanding this miracle is the most urgent task in development economics. But most economists are content to cite the dragons as proof of their favorite theories - whatever those theories may be. Free marketers point to the dragons' reliance on private enterprise, markets and relatively undistorted trade regimes. Interventionists point with equal assurance to clever bureaucracies, non-market allocation of resources and highly distorted trade regimes.

*** Although women have made huge strides in catching up with men in the workplace, a gender gap still persists both in wages and levels of advancement. Commonly cited explanations for this gap range from charges of sex discrimination to claims that women are more sensitive than men to work versus family conflicts and thus less inclined to make sacrificesfor their careers. Now, however, two new studies suggest that another factor may be at work: a deeply ingrained difference in the way men and women react to competition that manifests itself even at an early age. Apparently, females tend to be far less responsive to competition than males a tendency with important implications for women and business. It may hurt women in highly competitive labor markets, for example, and hamper efficient job placement - especially for positions in which competitiveness is not a useful trait.

*** The Sahara desert takes up most of Egypt's land, so overcrowding is a huge problem. Sixty-two million people live squeezed together into the six million fertile acres along the Nile delta and narrow river valley - just five per cent of the total area of Egypt. Between 12 and 15 million people live in Cairo alone. Until recently, it was impractical and dangerous to even consider moving into the southern desert, where temperatures regularly rise above 50 C and water is scarce and can only be reached using carefully placed irrigation wells. But in the last 20 years a "New Valley" has slowly been taking shape. Towns with industrial centres, tourist areas and spacious apartment blocks are being constructed, factories are springing up. The main development making this possible is the construction of the vast Sheikh Zayed canal, also known as the Toshka canal. Named for Sheikh Zayed al Nahya, president of the United Arab Emirates, which is financially backing the project, the canal is part of the irrigation scheme dreamed up by the Egyptian government to make it possible for people to move away from the traffic, pollution and bustle of Cairo. If a "second Nile" cuts through the desert and water is distributed to surrounding land, people and crops can thrive there as they do around the existing Nile. The area isbecoming known as the New Valley.

*** When Lyndon Johnson assumed the presidency, after the assassination of John F. Kennedy, in November of 1963, he knew that in order to accrue political capital he would initially need to champion goals and policies that Kennedy had already been pursuing. Not long before his death Kennedy had scrawled the word "poverty" on a piece of paper and circled it multiple times; this note fell into the hands of his brother Robert and became a symbolic justification for Johnson's declaration of the War on Poverty, early in 1964. Similarly, many of the things that Johnson pushed through Congress in his first two years as President, can readily be seen as extensions of the avowed policies of the Kennedy Administration. The details might have been different, but historians generally agree that if Kennedy had lived out his first term and won a second, America would have witnessed something similar to the early years of Johnson's Great Society. On foreign policy, too, Johnson at first strove consciously to follow his predecessor. And some historians have argued that in this realm as well, Johnson indeed pursued a course that Kennedy had already introduced. If Kennedy had lived, according to this line of thinking, he would have continued a policy of antagonism towards Cuba and steady escalation of US involvement in Vietnam. Johnson certainly believed that this was what Kennedy intended to do.

*** The space shuttle and its rockets are huge - some 4.5 million pounds at lift-off. About 85 per cent of that weight is fuel. Since it is designed to work in a vacuum, the shuttle must carry not only fuel but the oxygen to burn it. Because this is an inefficient way to go, NASA engineers have recently tested an engine that gets some of its oxygen on the run. This should reduce takeoff weights by half. A spacecraft equipped with this engine would take off like a rocket. But within minutes, incoming air would begin to supplement liquid oxygen. Once the spacecraft reaches a speed of 1,500 miles per hour - twice the speed of sound - the liquid oxygen would shut off completely and the engine would burn fuel mixed with air. Consequently the craft would accelerate to about ten times the speed of sound. When the air got too thin for the engine to breathe, the ship would shift back to rocket mode to punch its way into space.

KPDS 2004 İLKBAHAR

*** Behavioural biologist Jane Atkinson and her colleagues have been studying the subtleties of how crows steal food from one another. Atkinson had been watching the birds at the beach as they fed on fish, clams and other small animals in the intertidal zone. She noticed that if a crow had found a particularly large meal that couldn't be eaten in a single gulp, another crow would often come by and try to steal the food away. Food theft is fairly common in the bird world, so the crows' thievery wasn't unexpected. What really intrigued Atkinson was that the birds employed two different tactics to take the food. in some instances, the thieving bird would take an aggressive approach - typically involving some chasing or physical contact, such as pecking in other exchanges, however, the thief would use a more passive method: merely approaching the other bird secretively and steallng the food without any commotion at all. What the team wanted to know was: how did these tactics fit into the group foraging practices of the crows?

*** In many ways, Hollywood seems to exemplify the most joyless aspects of capitalism. The "industry", as it insists upon calling itself, packages artistic ideas and images as commodities and then values those commodities accordmg to how they "penetrate" markets. The system's worrying inefliciency, of course, is that studios never know what the public at large will want to buy. So films are tested in front of preview audiences, revised according to the audience's suggestions, tested again, and then marketed with a vigour directly proportionate to the test scores. There are two problems with this approach. The first is that the test-sample size is minimal but can determine a film's fate. The second is that by the time the test audience sees a film it's too late to change it very much anyway, particularly when twenty, fifty or a hundred million dollars has already been spent.

*** Reading presents a real paradox to neurobiologists. It was only invented a few thousand years ago, so there really has not been enough time for our brain to evolve speciallzed ways to do it. How do brain circuits produced by millions of years of evolution in a world without written words adapt to the specific challenges of reading? We know we have to learn the skill but how does our brain learn to read? in the social sciences, the majority of researchers do not see a problem. There is a widespread view that the brain is a completely adaptable organ, capable of absorbing any form of culture. Yet recent findings from brain imaging studies and neurophysiology throw new light on the organization of the reading circuits in the brain. The findings contradict this simplistic model of a brain that merely absorbs everything from its cultural environment. And they suggest that the architecture of our brain is limited by strong genetic constraints though it seerns that it has still some degree of flexibility.

*** Throughout history, eyewitnesses have reported orange glows, fireballs or flashes in the days before and during an earthquake. it was in 1968, however, that the first photographs of "earthquake lights" were taken during a series of earthquakes in Japan. Some showed red streaks across the sky. Others looked like a low blue dawn from a distance. in 1999, floating bails of light in the sky were broadcast on Turkish television, reportedly filmed the night before the devastating earthquake of 7.4 on the Richter scale that killed many thousand people in the Marmara region of Turkey. Mysterious or not, repeated sightings of earthquake lights confirm their existence. it has to be said that earthquake lights are a fairly wellknown phenomenon, but we don't know what they mean, or what causes them. Seismologists have struggled far years to find a reliable earthquake predictor. Could the lights hold the key?

*** Much has been said and written about the declining numbers of and disappointing lack of diversity among American college students majoring in engineering. Among the factors cited to explain this phenomenon are the lack of exposure of high school students to the very idea of engineering and the fact that many have insufficient mathematics and science background to gain entrance to engineering school, even if they do identify the profession as a possible career. This is unfortunate, for the ideas of engineering should be integrated into the curricula not only of high schools but also of middle and primary schools. Our children are being done a disservice by not being exposed properly throughout their education to engineering activities identifted as such. Arter all, even pre-school children have the prerequisites in their play for appreciating exactly what engineering is: design. Indeed, design is everywhere around them throughout their school day, even in their before-school and after-school activities. it need only be pointed out to them that they are designing something, and therefore being engirteers of sorts, in virtually everything that they do.

KPDS 2004 SONBAHAR

*** Why are people prejudiced? Not surprisingly, theories of prejudice have tended to focus on the more extreme forms of prejudice, in particular when there is aggression and violence. At the turn of the last century, it was popular to consider prejudice to be an innate and instinctive reaction to certain categories of person (e.g. certain races) much as animals would react in instinctive ways to one another. This approach is no longer popular, as it doesn't stand up well to scientific scrutiny. However there may be an innate component to prejudice. There is some evidence that higher animals, including humans, have an inherent fear of the unfamilier and unusual, which might set the mould for negative attitudes towards groups that are considered different in certain ways. There is also evidence for a mere exposure effect, in which, people's attitudes towards various stimuli (e.g. other people) ,improve as a direct function of repeated exposure or familiarity with the stimulus provided that initial reactions to the stimuli are not negative. Another perspective rests on the belief that prejudices are learned. Indeed, it has been argued that hatred and suspicion of certain groups are learned early in life before the child even knows anything about the target group and that this provides an emotional framework that colours all subsequent information about an experience with the group.

*** By the early 19th century, the eminent French zoologist Georges Cuvier believed he had found rock solid evidence for the Biblical great flood. While studying the geological strata around Paris, Cuvier found that fossils of sea creatures in one ancient layer of chalk were overlaid by those of land creatures. Then just as abruptly, the layer above contained sea creatures again, with the top layer showing evidence of a vast and rapid inundation around present day Paris. Cuvier regarded these sudden changes in the fossil record as evidence for sudden catastrophes which devastated life on Earth, of which the great flood was just the most recent example. Cuvier's discoveries, published in 1812 won support from a large number of eminent scientists such as the geologist Sir James Hall. However there were a few who were deeply sceptical, pointing out that the evidence of a global flood was far from conclusive. Most sceptical of all were the followers of the Scottish geologist James Hutton. In 1795 he had published a two volume text based on the view that the slow steady processes that shape our planet today, such as erosion, were also crucially important in the distant past.

*** No child is too young to play and therefore to engage in engineering, even though it is of a primitive kind. We all did so as children ourselves when we devised our own toys and games and sometimes even imaginary friends to enjoy them with us. The idea of playfulness is embedded in engineering through the concepts of invention and design. Not that engineering is trivial; rather, the heart of the activity is to give imagination its freedom to dream and turn those dreams into reality. Children do experience the essence of engineering in their earliest activities, yet there is seldom any recognition that this is the case. They may hear the word "engineer" only in connection with railroad locomotives and have no idea that their playful activity could become a lifelong profession. Engineers themselves are understandably reluctant to equate their professional activity with mere child's play. After all, they studied long and hard to master complicated knowledge of atoms and molekules, stresses and strains, heat and power, current and voltages, bits and bytes. They use computers for serious modelling and calculation, not for fun and games. They design and build real towers and bridges that test the limits of reliability and safety, not the ones that totter and fall down with little consequence.

*** Europe and Japan do not use fuel economy standards to any significant degree, but instead rely principally on high taxes to reduce gas consumption. Their average tax is more than $200 per gallon, while in the US, federal gas taxes are only 18 EURO per gallon and average state taxes 22 EURO per gallon. Higher prices at the pump rusulting from higher taxes increase consumer demand for cars with better fuel economy. They also encouarege consumers to reduce their driving. Research shows that federal taxes on gasoline would have to increase by a bit less than 50 euro per gallon to cut gasoline consumptin in the US. Although a 50 EURO incerase is a lot compared with the present average total tax of 40 EURO, it would raise retail gas prices to only a little more than $2 per gallon, tax included. This is far below prices in Europe and Japan. Even if federal taxes on gas were doubled, US retail gas prices would still be much below those in other developed nations.

*** Throughout his working life, Shakespeare worked as an actor in the midst of a troupe. We know little about his first years in London. For a few years between 1585 and 1592 his name disappears altogether from the public records, and the most likely reason for this is that, for at least some of this time, he was working for one of the city's acting companies. As a junior member, he would not be listed among the troupe's principal players. In the late 1580s theatrical activity in London was largely concentrated in Shoreditch and Southwark districts of London. Shakespeara could have lived anywhere, but Shoreditch, which would have been cheap and convenient, is a likely candidate for a young actor. In his early career Shakespeare may have moved from troupe to troupe in order to survive. Whatever the case, working conditions must have been similar. Sundays, religious holidays and disasters aside, a company would perform a different play each afternoon of the week, though some plays would be repeated in the weeks ahead. An actor usually had to keep at least 30 parts in his memory and a leading player such as Alleyn or Burbage must have kept in mind nearly 500 lines a week.

KPDS 2005 İLKBAHAR

*** For two decades after World War II, mass production reigned supreme. Mass production techniques pushed companies into standardized products, long product life cycles, and rigid manufacturing, emphasizing efficiency and low cost over flexibility. Special orders cost more. But today's consumers are very choosy. They want quality, value and products specially tailored to their needs, but always at the lowest possible price. For now mass customization has come to the fore. Mass customization uses information technology to produce and deliver products and services designed to fit the specifications of individual customers. Companies can customize products in quantities as small as one with the same speed and low cost as mass production methods. Mass customization systems use information taken from the customer to control the flow of goods.

*** Before the Polish-born French-American mathematician Benoit Mandelbrot made his mark on the world, scientists liked to forget about the imperfections and irregularities of nature. The study of perfect squares, triangles and planes had dominated their field for over 2,000 years, since the Greek geometer Euclid wrote maths' oldest treatise "Elements" and provided us with the tools to measure these flawlessly smooth shapes. Early question about how to measure the real shape of a tree, a coastline or anything with a rough edge could not be tackled by Euclidean geometry and had therefore been ignored. But Mandelbrot changed all this when he invented fractal geometry, which enables us to measure roughness. "My whole career has been one long, ardent pursuit of the concept of roughness", he says. "The roughness of clusters in the physics of disorder, of turbulent flows, of exotic noises, of chaotic dynamical systems, of the distribution of galaxies, of coastlines, of stock-price charts and of mathematical constructions."

***Recent activity in several US church communities has seemed almost unbelievable: churchgoers have gathered around huge fires and cheered as they cast Harry Potter books into the flames. They fear that the incredibly popular series about a school for young wizards is spurring children and adolescents toward a life of witchcraft and onto the dangerous path toward Satanism. For these congregations, J.K. Rowling's books are none other than the work of the devil herself. To most people, however, the Harry Potter books and films are merely compelling adventure stories, not a threat to children's psyches. But what has been forgotten in the excitement of "Pottermania" is that boys and girls have been fascinated by magic and sorcery for generations. Surveys about magical practices among adolescents vary widely, but some indicate that as many as 44 per cent have shown some slight, passing interest in it. Although satanically motivated violence occasionally makes headlines, research shows that less than 5 per cent of young people take part in more extensive witchcraft, and very few end up in the kind of organized devil worship that can lead to such acts as ritual murder.

*** Everybody experiences tensions. Anxiety and tension are essential functions of living, just as hunger and thirst are. They are our selfprotecting reactions when we are confronted by threats to our safety, well-being, happiness or self-esteem. So, while an occasional encountering of anxiety and tension may be disagreeable, it is quite normal, and it need not to be a cause for concern. The time to become watchful is when emotional upsets come frequently, shake us severely and fail to disappear after a while. *** Therapists have to be very careful before they make a diagnosis of delusional disorder. A great many complaints are founded on fact. It is possible that a patient is really being harassed at work, that her husband is deceiving her, or that her business partner is cheating her. Indeed, therapists must be careful not to mislabel facts as delusions, a trap known as "the Martha Mitchell effect". Martha Mitchell was the wife of former US attorney general John Mitchell. In October 1972, he was accused of having ordered the break-in at the Democratic campaign headquarters in the Watergate Hotel in Washington, D.C. Mrs Mitchell repeatedly told the press that her husband was being made a scapegoat to protect the real culprit - President Richard M. Nixon. The White House spread disinformation about Mrs Mitchell, saying she had a drinking problem and implying that her statements were delusional. When the scandal was ultimately unravelled, Mrs Mitchell's statements were proved true and she was shown to be utterly sane and with no drinking problem.

KPDS 2005 SONBAHAR

*** We can only guess when Shakespeare wrote his plays. He may have had his own writing "season" perhaps in the quieter winter months, but he never stopped acting probably taking two or three minor parts instead of a major one. He seems to have chosen for himself the more static and undemanding roles in his plays, such as old Adam in As You Like It and the Ghost in Hamlet. His audiences included many habitual playgoers, and many must have known Shakespeare and he must have known them. We can imagine, as a recent biographer has said, "that there might have been a complex, subtle communicative exchange when he appeared in one of his own plays". In spring 1613, he purchased his first property in London. He was renting it out by 1616, but may oriçinally have entertained other intentions for the property. It would certainly have been a handy place to stay, being near the Globe, which was his theatre. Perhaps the destruction of the Globe in 1613, which probably prompted him to sell his share in the theatre company, altered his plans for it. He may not have given up acting, but his writing career was over by the end of that year. In 1614, he returned to his hometown, Stratford-upon-Avon, and died there in 1616.

*** Family-owned companies are bad for business, a new study argues - at least when they dominate a large portion of a country's economy. Outside the United States and Britain most major corporations are in the hands of a few wealthy families, rather than, as in the US and Britain, being owned by a wide network of shareholders. The power of these small families often extends far beyond the companies they own directly, thanks to a system of "control pyramids" in which they exercise indirect control over a large number of smaller companies. This concentration of corporate power doesn't merely leave a high percentage of wealth in the hands of billionaires it also retards growth, diminishes efficiency, and limits economic freedom. Moreover, "a tiny elite that cannot be sacked," as the study puts it, is likely to pursue "economic entrenchment", in which property rights and financial openness are restricted to protect a few families' economic and political prerogatives or rights.

*** One of the greatest natural catastrophes the world will ever see could be little more than a decade away. The film Super volcano traces the evolution of an enormous volcanic eruption, one that not only wipes out several states of America but that threatens the entire planet. But is such an eruption really possible? Well, super volcanoes certainly aren't fiction. They're a normal part of the way the Earth works and occur perhaps every 50,000 years. Every statistic associated with a super-eruption is always wildly overexaggerated. Molten magma is blasted out at a rate 140 times greater than the flow of water over the Victoria Falls. Ash and gas are thrown more than 50km upwards to the edge of space before falling over one percent of the Earth's surface. Enough ash would pile up on the ground to bury Britain under a blanket 4m thick. Further, devastating winds carrying burning gas and red hot ash would scour the land surface over an area of 10,000 square kilometers. Worst of all, a super-eruption is foIIowed by a dramatic fall in global temperatures, leading to years and years of bitter cold known as a volcanic winter.

*** We should care about dying languages for the same reason that we care when a species of animal or plant dies. It reduces the diversity of our planet. In the case of language, we are talking about intellectual and cultural diversity, not biological diversity, but the issues are the same. As a result of decades of environmental publicity and activism, most people have come to accept that biodiversity is a good thing. But linguistic diversity has not enjoyed the same publicity. Diversity occupies a central place in evolutionary theory because it enables a species to survive in different environments. Increasing uniformity holds dangers for the long-term survival of a species. The strongest ecosystems are those which are most diverse. It has often been said that our success in colonizing the planet can be accounted for by our ability to develop diverse cultures which suit different environments.

*** The discovery of an ancient tomb in modern China is so commonplace that it often annoys as much as excites, because it can delay construction for months or even years. So when archaeologists were called in last May to check structures discovered during the expansion of a bone meal factory in a southern suburb of Beijing, they weren't expecting to find anything of great interest. To the archaeologists' surprise, the structures were the remains of two traditional domed tombs, each over a thousand years old. One was flooded and badly damaged, but the other contained beautifully-preserved wall frescoes from the 10th century. “It's only recently that the Chinese have been publishing artifacts from ancient tombs, and it's unusual to see them in the Western press,” says Dr Jessica Rawson, Professor of Oriental Art and Archaeology at Oxford University.

KPDS 2006 İLKBAHAR

*** Engineering is akin to writing or painting in that it is a creative endeavor that begins in the mind’s eye and proceeds into new frontiers of thought and action, where it does not so much find as make new things. Just as the poet starts with a blank sheet of paper and the artist with a blank canvas, so the engineer today begins with a blank computer screen. Until the outlines of a design are set down, however tentatively, there can be no appeal to science or to critical analysis to judge or test the design. Scientific, rhetorical or aesthetic principles may be called on to inspire, refine and finish a design, but creative things do not come of applying the principles alone. Without the sketch of a thing or a diagram of a process, scientific facts and laws are of little use to engineers. Science may be the theater, but engineering is the action on the stage.

*** Just as every teenager thinks he is brighter than his parents, every decade considers itself superior to the one that came before. Over the past few months, we of the 2000 decade have made it quite clear that we are morally heads above those who lived in the 1990s. We’ve done it first by establishing a reigning cliché for that period. Just as the 1960s are known for student unrest, the 1980s for Reagan, Thatcher and the Yuppies, the 1990s will henceforth be known as the second Gilded Age. They will be known as the age when the real problems in the world were ignored while the illusions of the dotcom types were celebrated. It was the age of effortless abundance, cell phones on every ear, stock markets that only went up and Mercedes sport utility vehicles. Never before had business leaders enjoyed so much prestige, and never before had capitalism had fewer mortal enemies. Bill Gates couldn’t be on enough business-magazine covers; tycoons like him felt free to assume the role of global sages, writing books with such weighty titles as “The Road Ahead.

*** BBC4, a comparatively new TV channel, has a character of its own. From the start it aimed to be “a place to think”, and it was always designed as something “that the commercial market would never do”, says Roly Keating, its controller and formerly head of arts at the BBC. Its first week’s schedule indeed verged on a parody of non-commercial TV, with township opera from South Africa and a performance by a Senegalese singer in a London church hall. A top-rated show will typically draw some 50,000 viewers – almost negligible in television terms. Yet that narrow appeal makes BBC4 a model of what a publicly financed broadcaster ought to do. It has roamed into territory where its ratings-driven sister channel, BBC1, seldom dares to tread. Despite a tiny 35m budget, it boasts an intelligent prime-time talk show and a world news programme so internationally minded that its London provenance is barely visible. BBC4 may wear its gravity a little too heavily at times, but it supplies a variety and thoughtfulness unavailable on prime time BBC1. The more the other BBC channels chase the ratings, and the more that BBC4 refuses to be dictated to by them, the more the channel looks like a model for what BBC television could look like.

*** The natives of the Lewis Island know wind – sometimes too well. Every winter the Atlantic gales come blasting across the northern tip of Scotland’s Outer Hebrides. The wind hardly slows down even after striking land; in the island’s marshy interior, gusts regularly exceed 160kph. Everyone stays indoors but the sheep. Tourists arrive in summer, lured by mild temperatures and unspoiled countryside; even so, there’s rarely a calm day. “The weather here is changeable”, says Nigel Scott, spokesman for the local government. “But the wind is constant”. The brutal climate could finally be Lewis’s salvation. The place has been growing poorer and more desolate for generations, as young people seek sunnier prospects elsewhere. But now the energy industry has discovered the storm-swept island. The multinationals AMEC and British Energy are talking about plans to erect some 300 outsize wind turbines across a few thousand hectares of moorland. If the 500 million-pound project goes through, the array will be Europe’s largest wind farm, capable of churning out roughly 1 per cent of Britain’s total electrical needs – and generating some badly needed jobs and cash for the people of Lewis.

*** In this century, the wealth and success of nations will depend like never before on the ability to produce and use knowledge. Universities have long been instrumental in generating knowledge and ideas. But in an increasingly globalized world, and in the face of rapid scientific change, they will need to think about a set of new challenges and how best to prepare their students for the coming decades. Universities will need to teach a new kind of literacy, in which global awareness will play an important role. They also need to deal with the dilemmas posed by the accelerating pace of change brought on by scientific and technological advances. We are on the brink of once-in-human-history progress in combating disease through the application of modern science. Doctors will have at their disposal blood tests that will tell you with substantial predictive power how long you will live and from what diseases you are likely to suffer. The Internet and the application of information technology may well represent the most profound change in the way knowledge is disseminated since the printing press. We are close to understanding the first second of the history of the cosmos.

KPDS 2006 SONBAHAR

*** Unlike the older forms of occultism, such as magic and astrology, organized occultism is a modern phenomenon. Few of the various organized occult movements have existed for more than 150 years; some were formed as a belated countermovement to the Enlightenment, when people began to follow rational schools of thought. Today’s occult views are based on the idea that there are events within nature, as well as within one’s spiritual life, which seem mysterious and cannot be explained by science. Examples include extrasensory perceptions such as telepathy and telekinesis, and haunted places or people. Believers maintain that these phenomena stem from unknown powers that can often be accessed only by some people with special abilities.

*** The assumption that a person’s attitudes determine his or her behaviour is deeply ingrained in Western thinking, and in many instances the assumption holds. However, research has shown that the relationship between attitudes and behaviour is complex. A classic study conducted during the 1930s was the first to question the link. A white professor travelled across the US with a young Chinese couple. At that time, there was quite strong prejudice against Asians, and there were no laws against racial discrimination. The three travellers stopped at over 200 hotels, motels and restaurants, and were served at all the restaurants and all but one of the hotels and motels without problem. Later, a letter was sent to all of the establishments visited,asking them whether or not they would accept a Chinese couple as guests. Of the 128 replies received, 92 per cent said they would not. In other words, these proprietors expressed attitudes that were much more prejudiced than their behaviour.

*** When you stay as a guest in someone’s house, you give up your anonymity. This becomes quite a challenge if you are the kind of person who cherishes independence. However, when you and your host are on the same wavelength, you can have a trip more special than money can buy. Some years ago when I went to Auckland, New Zealand, for the first time, my hosts were a couple, about my age, whom I had met while travelling in Europe. They had a full programme lined up for me. They drove me around and showed me their favourite hot springs and also the beach where a popular TV series had once been filmed. At mealtimes, they introduced me to their favourite restaurants, where I sampled cheeses from south New Zealand that don’tget exported, and fruits grown locally. Normally such a tight schedule would make me nervous, but I found myself happily relinquishing control to my hosts, who truly understood the pleasures of their native country and enjoyed sharing them. I couldn’t have encountered this New Zealand on my own.

*** In Finland now, everything is all right. Fifteen years after one of the worst recessions any European country has seen, triggered by the collapse of the Soviet Union, the Finns feel very content. Their small country of a population of 5 million is the first in the World Economic Forum’s list of the world’s most competitive countries, and the second in its business-competitiveness index. It is also the first in the OECD’s world ranking of educational performance and has the second-highest share of research-and-development spending in the European Union. Moreover, the country is reversing its demographic decline and, hence, its fertility rate is one of the highest in Europe. Perhaps best of all the Finns are facing globalization without paranoia. Theirs is one of the few European countries to have succeeded in businesses in which international prices are falling because of global competition and technological change. In most of Europe public opinion and even business élites seem gloomily resigned to being overwhelmed by India and China. Finland suggests that this fate is not inevitable.

*** Until the giant American energy company Enron collapsed, and its director Kenneth Lay was imprisoned, his life had been a model of the American dream of rising from rags to riches on the strength of merit and hard work. His beginnings were socially and financially very modest. He was born in Tyrone, Missouri, in 1942, as the son of a preacher who was also a part-time salesman. He helped his father make ends meet by cutting grass and delivering papers. His start in the energy industry seemed similarly modest. After obtaining a doctoral degree in economics from the University of Houston, he got his start in the booming Texan oil industry. In 1985 he merged Houston Natural Gas with InterNorth of Nebraska inorder to form Enron. As Enron became stronger, Mr Lay turned increasingly to politics and was one of the biggest donors to the Bush-Cheney campaign. After Mr Bush entered the White House, Mr Lay had hopes of a seat in the cabinet, perhaps as energy secretary or even at the Treasury. However, for reasons that remain unclear, Mr Bush overlooked him, so his professional life ended in frustration.

KPDS 2007 İLKBAHAR

*** There seems no question but that the clock dial, which has existed in its present form since the seventeenth century and in earlier forms since ancient times, is on its way out. More and more common are the digital clocks that mark off the hours, minutes, and seconds in ever-changing numbers. This certainly appears to be an advance in technology. You will no longer have to interpret the meaning of “the big hand on the eleven and the little hand on the five.” Your digital clock will tell you at once that it is 4:55. And yet there will be a loss in the conversion of dial to digital, and no one seems to be worrying about it. Actually, when something turns, it can turn in just one of two ways, clockwise or counter-clockwise, and we all know which is which. Clockwise is the normal turning direction of the hands of a clock, and counter-clockwise is the opposite of that. Since we all stare at clocks (dial clocks, that is), we have no trouble following directions or descriptions that include those words. But if dial clocks disappear, so will the meaning of those words for anyone who has never stared at anything but digitals.

*** When Mozambique and South Africa ended their internal conflicts in the early 1990s, they enacted widescale amnesties, and in both countries the rule of law quickly improved. In each of them, political leaders opted to move past the violence and injustices of the past and to focus on the tasks of social and political reconstruction. As part of that reconstruction, each country became a multiparty democracy in which the accountability of leaders and other key norms of the rule of law could finally take root. The restoration of public security, meanwhile, allowed the provision of basic services. And though their criminal-justice systems remained woefully underfunded, both were finally able to start providing citizens with basic protections. While the legal, social and political improvements in South Africa between 1994 and 2004 were impressive, in poorer Mozambique, the improvement was smaller but still marked.

*** No human dream is more universal than the longing for a paradise on earth, a place free of the ravages of time and disease, where the best in nature flourishes while the worst is forbidden to enter. By definition, such magical lands can’t be near at hand; they must be remote and inaccessible – destinations to be reached by pilgrimage or a heroic journey. Ancient Tibetan Buddhist texts spoke of just such a kingdom, where wise kings, blessed with long life spans, await the day when they will take power over the world, ushering in a golden age of peace and justice. This mythical kingdom was called “Shambala,” and its location was believed to be a valley in northern India. Said to be enclosed by a double ring of snow-capped mountains, this fabled valley of Shambala resembled a “mandala,” which is Buddhism’s circular symbol of the unity of all creation. Known only to a few European enthusiasts of Asia in the 19th century, the myth of Shambala was popularized in the 20th century by the famous Russian mystic, Madame Blavatsky, who claimed she received telepathic messages from the mystic valley.

*** The late Chinese Prime Minister, Chou-En Lai, upon being asked whether the French Revolution had been a good thing in world history, was reported to have said: “It is still too early to tell.” Watching the Western media analyze the recent emergence of China as a major investor in Africa and likely to become a new factor to rival the historical EuroAmerican ascendancy in African politics, one is even more justified in saying that it is “too soon to tell” what the outcome of the Chinese love affair in Africa will be. Besides China, India is also increasingly mentioned as a new source of large-scale investments in Africa. For the moment, the emergence of the Asian superpowers, themselves once in the sphere of British imperialism, as investors and trading partners in Africa, seems to offer an opportunity of shaking the “marginalisation” into which most African countries have fallen since the collapse of the Soviet bloc in 1989.

*** Editors have two primary functions which sometimes overlap: finding/selecting manuscripts, then polishing them for publication. Acquisitions editors perform the first chore. The approach they adopt depends on several factors. The idea for a college text, for example, usually originates inside the publishing house; the acquisition editor’s job is then to choose a suitable author to produce the manuscript. In a trade book division, on the other hand, the acquisitions editor may be more passive, carefully reading manuscripts and queries that are mailed in, then recommending the best of these for development as a book. In the former case, the acquisitions editor may be knowledgeable in a given area (economics, perhaps, or one of the sciences) while the second type might be more of a generalist. Copyeditors, who whip the manuscript into shape for the press, must possess a superb background in English and bring to their work high standards of accuracy and thoroughness along with a remarkable attention to detail.

KPDS 2007 SONBAHAR

*** Today, the Berlin Crisis of 1961, in which the Soviet Union demanded that Western powers cut their ties with Berlin, may be the mostforgotten crisis in the annals of the Cold War. Even most Berliners who lived through the event remember little about it. Yet this crisis over Berlin brought America and the Soviet Union, the two superpowers of the post-World War II period, close to war. In fact, since the very end of World War II, Berlin had been the centrepiece of a struggle between these two superpowers. It was here that World War II ended in 1945 when, following the occupation and defeat of Nazi Germany by the allied armies of the United States, Britain, and France from the west, and the Soviet Union from the east, the city had been captured and divided into the separate zones of East and West Berlin. Soon afterwards Germany had itself split into East and West, and the border between the two had become the dividing line (the so-called “iron curtain”) between Sovietcontrolled Eastern Europe and free, capitalist Western Europe. Thus, situated behind this iron curtain and stuck a hundred miles inside Eastern territory, West Berlin was claimed, protected and supplied by the Western powers. In 1948, Stalin imposed a blockade, cutting West Berlin off from its Western suppliers. The United States responded with an airlift, keeping the zone alive for more than 300 days before Allied access was restored.

*** Narva is a quiet northeastern Estonian town bathed in sea breezes. Though small, with a population of just over 72,000, it occupies a large place in Russian history. It was here in 1700 that, by attacking the Swedes, who were then in control of much of the Baltic coast, Russia launched its final campaign in a centuries-long quest to become a European power. The battle ended in defeat for the Russians, but the war did not; by 1721 Russia had conquered the Baltic territories as far southwest as Riga, the capital of present-day Latvia, and had built a new capital, Saint Petersburg, on the Gulf of Finland. Later in that century, Russia, through a partition agreement with Austria and Prussia, gained control of the rest of the Baltics, and would retain them until the fall of the Soviet Union in 1991.

*** Set in 1941 in Leningrad, Helen Dunmore’s novel The Siege opens with deceptively gentle scenes of Chekhovian melancholy. After the death of her mother, 23-year-old Anna Levin, the protagonist, gives up her artistic studies to look after her 5-yearold brother and her politically suspect father Abraham, who, as a writer, has fallen out of favour with Stalin’s cultural police. So she jumps at the chance to make a drawing of the retired actress Marina Petrovna, with whom Anna’s father might once have had a romantic relationship. But Anna’s worries about art and romance are soon swept away as the Germans besiege her native city. At this point, Dunmore’s novel transforms abruptly as well, shifting from a romantic narrative into a study of survival under most extreme hardships. Anna’s abundant artistic creativity is put touse providing food and fuel for her helpless family, and her drawing skills are called on to sketch a neighbour’s starved baby so that the grieving mother might remember her lost child. Indeed, the novel presents a striking contrast between the gentle display of human emotions and the rude dictates of survival under the most inhuman circumstances.

*** Following World War II, European countries largely gave up their colonial possessions and, by the 1950s and 1960s, had already begun to receive growing numbers of immigrants from their former colonies. In many instances, these included the descendants of the slaves in the colonies, who had been forced to work. In this respect, Britain is a case in point. Though in small numbers, Africans and Indians had come to Britain long before the tens of thousands who came as colonial immigrants in the 1960s and thereafter. The first Africans who came to Britain were probably soldiers during the Roman possession of that country in antiquity. In modern times, especially in the eighteenth and nineteenth centuries, African and Indian princes and scholars visited Britain. Others coming to Britain were in service positions; for instance, inthe eighteenth century, black African and Indian young men were fashionable as servants in the homes of the wealthy. Africans and Indians also came to Britain as sailors and traders, and port towns, such as London, Glasgow, Bristol, Cardiff and Liverpool, developed small black populations in the early nineteenth century, some of which persisted into the twentieth century. Relations between these populationsand the native white population were varied, historians citing instances both of hostility and solidarity.

*** A couple of months ago NASA asked the scientific community what kinds of research it should conduct when it returns humans to the moon. In doing so, NASA wanted prioritized research objectives for the robotic orbiters and landers that will be used primarily for reconnaissance purposes prior to later explorations by astronauts of the lunar surface. Recommendations made by scientists varied greatly, but they can be summarized. The top priority that scientists have recommended is the development of programmes for lunar data analysis. Next is the exploration of the moon’s south pole, which is called “the Aitken basin,” an impact scar mostly on the moon’s back side. Then comes an instrument network for probing the interior of the moon, and this is followed by rock samplereturns, scientifically selected landing sites, and analysis of any icy polar deposits.

KPDS 2008 İLKBAHAR

*** In many primitive communities there is a taboo on mentioning a man’s name except in certain special circumstances, because his name is believed to contain within it something of himself, which would be lost and wasted if his name were uttered without first taking special precautions. This belief about words is widespread. Among the more primitive and the uneducated, it is universal. A remarkably matter-offact practical application of it occurs even in the present day in the Tibetan prayer-wheel. If, thinks the Tibetan peasant, a prayer uttered once does some good, then the same prayer uttered many times will do more good. Therefore, since he assumes that the efficacy lies in the prayer as an entity in itself, he writes it round the rim of a wheel, and then frugally employs the water of a mountain stream to turn it all day long, instead of wastefully employing his own lungs and lips to say it again and again.

*** Thomas Edison began conducting experiments during his childhood. To start with, there were hundreds of unsuccessful experiments but Edison eventually invented and patented 2,500 items, including the electric lamp and phonograph. He was determined to “give laughter and light” to people, but, until he actually managed to do so, most people ridiculed him. Without losing hope, Edison attempted over 1,000 unsuccessful experiments in his efforts to make an electric lamp. When people told him he was wasting his time, energy, and money for nothing, Edison exclaimed, “For nothing! Every time I make an experiment, I get new results. Failures are stepping stones to success.” Determined to give people electric lamps, Edison said he’d meet his goal by early 1880. In October, 1879, he created his first electric lamp, and in so doing, received much praise. People realized that Edison’s invention was not affected by rain or wind, remaining constant through bad weather. Just as he had hoped, Edison provided people with light and laughter.

*** Questions of education are frequently discussed as if they bore no relation to the social system in which and for which the education is carried on. This is one of the most common reasons for the unsatisfactoriness of the answers. It is only within a particular social system that a system of education has any meaning. If education today seems to deteriorate, if it seems to become more and more chaotic and meaningless, it is primarily because we have no settled and satisfactory arrangement of society, and because we have both vague and diverse opinions about the kind of society we want. Education is a subject which cannot be discussed in a void: our questions raise other questions, social, economic, financial, and political. And the bearings are on more ultimate problems even than these: to know what we want in education, we must know what we want in general, we must derive our theory of education from our philosophy of life.

*** The distinction between “journalism” and “literature” is quite futile, unless we are drawing such a violent contrast as that between Gibbon’s Historyand today’s paper; and such a contrast itself is too violent to have meaning. You cannot, that is, draw any useful distinction between journalism and literature merely on a scale of literary values, as a difference between the well-written and the supremely wellwritten: a second-rate novel is not journalism, but it certainly is not literature.The term “journalism” has deteriorated, so let us try to recall it to its more permanent sense. To my thinking, the most accurate as well as most comprehensive definition of the term is to be obtained through considering the type of mind, concerned with writing what all would concede to be the best journalism. There’s a type of mind, and I have a very close sympathy with it, which can only turn to writing, or only produce its best writing, under the pressure of an immediate occasion; and it is this type of mind which I propose to treat as the journalist’s. The underlying causes may differ: the cause may be an ardent preoccupation with affairs of the day, or it may be (as with myself) laziness requiring an immediate stimulus, or a habit formed by early necessity of earning small sums quickly. It is not so much that the journalist works on different material from that of other writers,as that he works from a different, no less and often more honourable, motive.

*** Leonardo da Vinci is a member of a very small class of “transformative geniuses,” not ordinary or common geniuses, who have contributed abundantly to their fields, but rather the ones who have created or defined entire fields. In literature, no one asks, “Who was the greatest writer?” Honest debate can start at Number Two. Shakespeare, the consensus choice as greatest writer, is a member of this class of transformative geniuses. Similarly, Isaac Newton is recognized as the greatest among scientists and mathematicians; Ludwig van Beethoven, and possibly Bach and Mozart, are the transformative geniuses among composers. The most recent transformative genius the world has seen may have been Albert Einstein, a scientist like Newton – and Time Magazine’s “Man of the Century” for the 20 th century. In ranking artists,one can start the debate at Number Three – a rank for which Raphael and Rembrandt are candidates, or perhaps one of the great French Impressionists, or the 20 th century’s most famous artist, Picasso. The ranks of Number One and Number Two, however, are reserved for Leonardo and Michelangelo, taken in either order. These two are far above all other artists. Michelangelo lived a very long lifetime of eighty-nine years, and was productive to the end. Leonardo, on the other hand, lived sixty-seven years, and left behind just a dozen paintings. And only a half of these are incontrovertibly one hundred per cent by him. In contrast, Rembrandt painted hundreds of paintings, 57 of himself alone; van Gogh created nine hundred paintings in a period of nine years. So how can we put Leonardo at the very pinnacle? The answer is really quite simple: his dozen or so paintings include the Number One and the Number Two most famous paintings in the history of art – The Last Supperand Mona Lisa.

KPDS 2008 SONBAHAR

*** The major sciences of the Hellenistic Age were astronomy, mathematics, geography, medicine, and physics. The most renownedof the early Hellenistic astronomers was Aristarchus of Samos (310-230 B.C.), sometimes called the “Hellenistic Copernicus.” His primary accomplishment was his deduction that the Earth and the other planets revolve around the Sun. This view was not accepted by his successors because it conflicted with the teachings of Aristotle and also with the Greek conviction that humanity, and therefore the Earth, must be at the centre of the universe. Later, in the second century A.D., Aristarchus’s fame was overshadowed by that of Ptolemy of Alexandria. Although Ptolemy made few original discoveries, he systematized the works of others. His principal writing, The Almagest,based on the view that all heavenly bodies revolve around the Earth, was handed down to medieval Europe as the classic summary of ancient astronomy.

*** The most significant political change in eastern Europe during the late 1980s was the collapse of communism in East Germany and the unification of East and West Germany. Although long considered the most prosperous of the Soviet satellite countries, East Germany suffered from severe economic stagnation and environmentaldegradation. Waves of East Germans registered their discontent with worsening conditions by massive illegal emigration to the West. This exodus together with evidence of widespread official corruption led to the resignation of East Germany’s longtime, hard-line premier, Erich Honecker. His successor, Egon Krenz, promised reforms, but he was nevertheless faced with continuing protests and continuing mass emigration. In the end, on 4 November 1989, the government, in a move that acknowledged its powerlessness to hold its citizens captive, opened its border with Czechoslovakia. This move effectively freed East Germans to travel to the West. In a matter of days, the Berlin Wall, which had been the embodiment of the Cold War, the Iron Curtain, and the division of East from West, was demolished by groups of ordinary citizens. Jubilant crowds from both sides walked through the gaping holes that now permitted men, women, and children to take the few steps that symbolized the return to freedom and a chance for national unity.

*** The causes of World War II were rooted in the peace settlement at Versailles in 1919-1920. The peace had created as many problems as it had solved. The senior Allied heads of state yielded to demands that involved annexing German territory and creating new states out of the eastern European empires. In doing so, the peacemakers created fresh bitterness and conflict. The Versailles treaty and its champions, such as US President Woodrow Wilson, proclaimed the principle of self-determination for the peoples of eastern and southern Europe. Yet the new states created by the treaty crossed ethnic boundaries, involved political compromises, and frustrated many of the expectations they had raised. The unsteady new boundaries would be redrawn by force in the 1930s. The Allied powers also kept up the naval blockade against Germany after the end of World War I. This forced the new German government to accept harsh terms that deprived Germany of its political power in Europe. The blockade and its consequences created grievances that made the German people feel angry and completely humiliated.

*** The French Revolution transformed the political and diplomatic landscape of Europe suddenly and dramatically. The transformation of industry came more gradually. By the 1830s or 1840s, however, writers and social thinkers in Europe were increasingly aware of unexpected and extraordinary changes in their economic world. They began to speak of an “industrial revolution,” one that seemed to parallel the revolution in politics. The term has stayed with us. The Industrial Revolution spanned the hundred years after 1780. It represented the first breakthrough from an agricultural and overwhelmingly rural economy to one characterized by large-scale manufacturing, more capital-intensive enterprises, and urbanization. It involved new sources of energy and power, faster transportation, mechanization, higher productivity, and new ways of organizing human labour. It triggered social changes with revolutionary consequences for the West and its relationship with the world. Of all these changes, perhaps the most important one was to be seen in energy. Over the space of two or three generations, a society and an economy that had traditionally drawn on water, wind, and wood for most of its energy needs came to depend on steam engines and coal. In other words, the Industrial Revolution brought the beginnings of “the fossil fuel age.”

*** The Renaissance originated in Italy for several reasons. The most fundamental reason was that Italy in the later Middle Ages was the most advanced urban society in all of Europe. Unlike aristocrats north of the Alps, Italian aristocrats customarily lived in urban centres rather than in rural castles and consequently became fully involved in urban public affairs. Moreover, since the Italian aristocracy built its palaces in the cities, the aristocratic class was less sharply set off from the class of rich merchants than in other European countries. Hence, whereas in France or Germany most aristocrats lived on the income from their lands while rich town dwellers, called bourgeois in French, gained their living from trade, in Italy so many town-dwelling aristocrats engaged in banking or mercantile enterprises, and so many rich mercantile families imitated the manners of the aristocracy that, by the fourteenth and fifteenth centuries, the aristocracy and upper bourgeoisie were becoming virtually indistinguishable. For instance, the noted Florentine family of the Medici, which had emerged as a family of physicians (as the name suggests), made its fortune in banking and commerce, and rose into the aristocracy in the fifteenth century. Obviously, social mobility as such brought about a great demand for education in the skills of reading and counting necessary to become a successful merchant, but the richest and most prominent families sought above all to find teachers who would impart to their sons the knowledge and skills necessary in politics and public life.

KPDS 2009 İLKBAHAR

*** The 16 th century in England is generally known as the Tudor period, which historically lasted from 1485 to 1603. Among the famous Tudor sovereigns were Henry VII, Henry VIII, and Elizabeth I. In fact, the early years of the Tudor period were marked by significant changes in trade and in the arts of war. Henry VII made commercial treaties with European countries. Economically, England, which had always been a sheep-raising country, was by now manufacturing and exporting significant amounts of cloth. As lands were enclosed to permit grazing on a larger scale, people were driven off the land to the cities, and London grew into a metropolitan market with sophisticated commercial institutions. These changes had an impact on the traditional feudal social order, which also began to decline; also, due to the introduction of cannons and firearms, the feudal system of warfare became obsolete. Yet, it would be a mistake to imagine these changes as sudden and dramatic. In fact, it was a slow and long process whereby England was transformed into a modern sta

*** In the last third of the 19 th century, new technologies transformed the face of manufacturing in Europe, leading to new levels of economic growth and complex realignments among industry, labour and national governments. Like Europe’s first industrial revolution, which began in the late 18 th century and centred on coal, steam and iron, this “second” industrial revolution relied on innovation in three key areas: steel, electricity, and chemicals. For instance, steel, which was harder, stronger and more malleable than iron, had long been used as a construction material. But until the mid-nineteenth century, producing steel cheaply and in large quantities was impossible. That changed between the 1850s and 1870s, as new and different processes for refining and mass-producing alloy steel revolutionized the metallurgical industry. Although iron did not disappear overnight, it was soon eclipsed by soaring steel production. So, steel began to be used for various purposes. In Britain, for example, shipbuilders made a quick and profitable switch to steel construction, and thus kept their lead in the industry. Germany and the US, however, dominated the rest of the steel industry. By 1901, Germany was producing almost half as much steel as Britain and was able to build a massive national and industrial infrastructure.

*** In 1764, there was a serious quarrel over taxation between the British government and its colonies in America. The British government continued to think of the colonists as British subjects. In 1700, there had been only 200.000 colonists, but by 1770 this number rose to 2.5 million. Obviously, such large numbers needed to be dealt with carefully. Some American colonists decided that it was not lawful for the British government to tax them without their agreement. Political opinion in Britain was divided. Some felt that the tax was fair because the money would be used to pay for the defence of the American colonies against French attack. But several important politicians agreed with the colonists that there should be “no taxation without representation”. Hence, in 1773, at the port of Boston, a group of colonists threw a shipload of tea into the sea rather than pay tax on it. The event became known as “the Boston Tea Party”. The British government responded by closing the port. But the colonists then decided to prevent British goods from entering America until the port was opened again. The colonists’ action was regarded as a rebellion, and the British government decided to defeat it by force. Thus, the American War of Independence got underway. The war lasted from 1775 until 1783. The British government had no respect for the colonists’ fighting ability. The result was a disastrous defeat for the British forces. At the end of the war, Britain lostall the American colonies except Canada.

*** The Mississippi is one of the world’s great continental rivers, like the Amazon in South America, the Congo in Africa, or the Volga inEurope. Its waters are gathered from two-thirds of the US and, together with the Missouri, which is its chief western branch, the Mississippi flows some 6.400 kilometres from its northern sources in the Rocky Mountains to the Gulf of Mexico, which makes it one of the world’s longest waterways. The Mississippi has been called the “father of waters”. Through all its lower course, it wanders along, appearing lazy and harmless. But people who know the river are not deceived by its benign appearance, for they have had many bitter struggles with its floods. They have had to learn that nothing is to be gained by fighting against the rages of the mighty stream. To control it, Americans have had to accept some of the river’s own terms and to undertake the patient work of conserving and rebuilding soil, grasslands and forests, far back where the waters begin to gather.

*** The Copernican revolution began over 500 years ago with the realization that the Earth was not the centre of the universe, but we still await its grand finale: the anticipated discovery of life elsewhere. Where else might we find life? The vast scale of the universe makes it virtually certain that there are other Earthlike settings. In our own solar system, Mars’s distance from the Sun makes it sufficiently Earth-like; so, especially with increasing evidence for occasional liquid water, many are looking there for the first signs of extraterrestrial life. Recently, however, a new contender has emerged, and surprisingly it is from the cold outer solar system: it is Jupiter’s moon Europa. As one of the four satellites of Jupiter, discovered by Galileo in 1610, Europa is now believed to have water in a liquid state, even though it is so far from the Sun. Thus, the possibility of liquid water on Europa has opened the door to speculation about life on this satellite of Jupiter.

KPDS 2009 SONBAHAR

*** A key feature of globalization has been the transformation of the world economy, highlighted by the rapid integration of markets since 1970. In a series of historic changes, the international agreements that had regulated the movement of people, goods, and money since World War II were overturned. To begin with, the postwar economic arrangements sealed by various treaties steadily eroded in the late 1960s, as Western industrial nations faced a double burden of inflation and economic stagnation. A crucial shift in monetary policy occurred in 1971, when the United States abandoned the postwar gold standard and allowed the dollar to range freely. As a result, formal regulations on currencies, international banking, and lending among states faded away. They were replaced with an informal network of arrangements managed autonomously by large private lenders, their political friends in leading Western states, and independent financial agencies such as the International Monetary Fund (IMF) and the World Bank. The economists and administrators who dominated these new networks steered away from the interventionist policies that had shaped postwar planning and recovery. Instead, they relied on a broad range of market-driven models termed “neoliberalism.”

*** During the latter part of 1980, Iraq invaded Iran and hoped to seize its southern oil fields. Iran counterattacked. The result was a murderous eightyear conflict marked by the use of chemical weapons and human waves of young Iranian radicals fighting the Soviet-armed Iraqis. The war ended with Iran’s defeat, but not the collapse ofits theocratic regime. In the short term, their long defence of Iranian nationalism left the mullahs more entrenched at home, while abroad they used their oil revenues to back grassroots radicalism in Lebanon and militants elsewhere who engaged in anti-Western terrorism. Over the years, the strongest threats to the Iranian regime ultimately have come from within, from a new generation of young students and workers who have discovered that their prospects for prosperity and democratic rights have not changed much since the days of the shah.

*** Until the late thirteenth century, European maritime commerce had been divided between a Mediterranean and a North Atlantic world. Starting around 1270, however, Italian merchants began to sail through the Strait of Gibraltar and on to the woolproducing regions of England and the Netherlands. This was the essential first step in the extension of Mediterranean commerce and colonization into the Atlantic Ocean. The second step was the discovery by Genoese sailors, during the fourteenth century, of the Atlantic island chains known as the Canaries and the Azores. Efforts to colonize the Canary Islands and to convert and enslave their inhabitants began almost immediately. But an effective conquest of the Canary Islands did not begin until the fifteenth century, when it was undertaken by Portugal and completed by Spain. The Canaries, in turn, became the base from which further Portuguese voyages down the west coast of Africa proceeded. They were also the “jumping-off point” from which Christopher Columbus would sail westward across the Atlantic Ocean in hopes of reaching Asia.

*** Like nearly all the peoples of the ancient world, the Romans took slavery for granted. Nothing in Rome’s earlier experience had prepared it, however, for the huge increase in slave numbers that resulted from its western and eastern conquests. In 146 B.C., fiftyfive thousand Carthaginians were enslaved after the destruction of their city; not long before, one hundred and fifty thousand Greek prisoners of war had met the same fate. By the end of the second century B.C., there were a million slaves in Italy alone, making Roman Italy one of the most slave-based economies known to history. The majority of these slaves worked as agricultural labourers on the vast estates of the Roman aristocracy. Some of these estates were the result of earlier Roman conquests within Italy itself. But others were constructed by aristocrats buying up the land holdings of thousands of small farmers who found themselves unable to compete with the great estate-owners in producing grain for the market.

*** The finest example of Byzantine architecture is the church of Saint Sophia (Holy Wisdom) in İstanbul, constructed by the emperorJustinian in the sixth century. Evidently, its structural design was something altogether new in the history of architecture. The central feature of the design was the application of the dome principle to a building of square shape. The church was designed in the form of a cross, with a magnificent dome over its central square. The main problem for the architects was how to fit the circumference of the dome to the square area it was supposed to cover. The solution was to have four great arches spring from pillars at the four corners of the square. The rim of the dome was then made to rest on the keystones of the arches, with the curved triangular spaces between the arches filled with masonry. The result was an architectural framework of marvellous strength, which at the same time made possible a style of imposing grandeur and delicacy. The dome itself hasa diametre of 107 feet and rises to a height of nearly 180 feet from the floor. So many windows are placed around its rim that the dome appears to have no support at all but to be suspended in midair.

KPDS 2010 İLKBAHAR

*** The heroic myths and epics of a society teach its members the appropriate attitudes, behaviour, and values of that culture. These myths are of particular interest and value to us. Not only are they exciting adventure stories, but in these myths we see ourselves, drawn larger and grander than we are, yet with our human weaknesses as well as our strengths. As for heroes, they are the models of human behaviour for their society. They earn lasting fame by performing great deeds that help their community, and they inspire others to emulate them. Heroes are forced by circumstance to make critical choices where they must balance one set of values against competing values. They achieve heroic stature in part from their accomplishments and in part because they emerge from their trials as more sensitive and thoughtful human beings. Yet heroes are not the same throughout the world. They come from cultures where individuals may earn fame in a variety of ways. This permits them to express their individuality. However, in spite of their extraordinary abilities, no hero is perfect. Yet their human weaknesses are often as instructive as their heroic qualities. Their imperfections allow ordinary people to identify with them and to like them, since everyone has similar psychological needs and conflicts.

*** Our knowledge of the Mycenaean civilization in Greece is based primarily upon what archaeologists have been able to discover. Fortunately, they have located and studied the ruins of a number of important Mycenaean sites both in Greece and in Troy, the site of Homer’s “Ilium” in Turkey. The material available to archaeologists is very limited, due to the ravages of time, weather, fire, and theft. The materials that have survived include objects such as jewelry, pottery, metal utensils, and various kinds of weapons. In addition, archaeologists have found a large number of clay tablets, inscribed with a language called “Linear B,” which they can read. It now becomes clear that the Mycenaean civilization in full bloom far surpassed in complexity and wealth many of the Greek civilizations that followed it. The Mycenaeans were an aggressive people who loved fighting, hunting, and athletic contests. Their land was mountainous and their soil rocky and dry. Therefore, they took to the sea and became fearsome raiders of other communities. In this way they acquired extraordinary wealth.

*** The loss of global biodiversity is occurring at an alarming rate. Since the 1970s, the area of tropical forests destroyed worldwide exceeds the land mass of the European Union. Animal and plant species are disappearing. Overfishing has depleted stocks around the world. Poor farming practices have depleted soils while allowing the invasion of harmful species. Destruction of wetlands has left low-lying areas extremely vulnerable to storms and natural disasters. Especially in Europe, ecosystems have suffered more human-induced damage than those on any other continent. Only about 3 per cent of Europe’s forests can be classified as undisturbed by humans, and the continent has lost more than half of its wetlands. The spread of urbanization and the overexploitation of resources is having an enormous impact on biodiversity.

*** The father of modern socialism, Karl Marx (1818-1883) was barely known in the early nineteenth century. His reputation rose later, after 1848, when a wave of revolutions and violent confrontation seemed to confirm his distinctive theory of history and make earlier socialists’ emphasis on peaceful reorganization of industrial society seem naive. As a child, he grew up in Trier, in the western section of Germany, in a region and a family keenly interested in the political debatesand movements of the revolutionary era. His family was Jewish, but his father had converted to Protestantism in order to be able to work as a lawyer. Marx studied law briefly at the University of Berlin before turning instead to philosophy and particularly to the ideas of Hegel. With the so-called Young Hegelian, a group of rebellious students who hated the narrow thinking of a deeply conservative Prussian university system, Marx appropriated Hegel’s concepts for his radical politics. His radicalism made it impossible for him to get a post in the university. He became a journalist and, from 1842 to 1843, edited the Rheinische Zeitung (Rhineland Gazette). The paper’s criticism of legal privilege and political repression put it on a collision course with the Prussian government, which closed it down and sent Marx into exile – first in Paris, then Brussels, and eventually London.

*** Oil has provided humanity with many benefits, including affordable energy to reduce our workloads and improve our mobility. Because oil is such an important and visible part of our daily lives, and because it is exceptionally open to political manipulations, it often receives an enormous amount of attention. This is especially true whenever its price increases sharply, and experts immediately get to work to diagnose the cause and consequences of the price increase. In fact, the future of oil is not that much different from its past: undoubtedly, oil production and consumption will become cleaner and more efficient, but prices will continue to be volatile, and the oil industry will continue to be blamed for conflicts, corruption, and pollution. And for all the current talk about the end of the oil age, it will remain a vital source of energy as it is now, nearly a century after the first warnings about soaring consumption and limited resources.

KPDS 2010 SONBAHAR

*** In antiquity, prior to the third century B.C., physics had been a branch of philosophy. It was made a separate, experimental science by Archimedes of Syracuse, who lived between 287 and 212 B.C.. He not only discovered the law of floating objects, or specific gravity, but also formulated with scientific exactness the principles of the lever, the pulley, and the screw. Among his memorable inventions were th compound pulley and the screw propeller for ships. Although he has been considered the greatest technical genius of antiquity, in fact he preferred to devote himself to pure scientific research. Tradition relates that he discovered “Archimedes’ principle,” that is, specific gravity, while pondering possible theories in his bath; when he reached his stunning insight, he dashed out naked into the street crying “Eureka!” (“I have found it!”).

*** The Romans were descended from a cluster of peoples who had crossed the Alps into Italy during the second millenium B.C. and spoke a variety of Indo-European dialects. Recent archaeological research has pushed the origins of the city of Rome back to at least the tenth century B.C., several centuries earlier than the traditional date 753 B.C., which the Romans themselves considered their city’s foundation year. Rome’s strategic location along the Tiber River brought it many different advantages. Trading ships could navigate the Tiber as far as Rome, but no farther; the city could thus serve as a port without being threatened by attack from the sea. Rome’s famous hills increased the defensibility of the site. In other words, as a city, Rome was situated at a junction across the Tiber, making it a major land and river crossroads.

*** When air pollution, including acid rain, is combined with other environmental stresses, such as low winte temperatures, prolonged droughts, insects, and bacterial, fungal, and viral diseases, it can cause plants to decline and die. More than half of the red spruce trees in the mountains of the northern United States have died since the mid-1970s. Other tree species, such as sugar maples, for example, are also dying. Many still-living trees are exhibiting symptoms of forest decline, characterized by a gradual deterioration and often eventual death. The general symptoms of forest decline are reduced vigour and growth, but some plants exhibit specific symptoms, such as yellowing of needles in conifers. Air pollutants may or may not be the primary stress that results in forest decline, but the presence of air pollution lowers plant resistance to other stress factors. When one or more stresses weaken a tree, then an additional stress may be enough to cause death.

*** Government planners and social scientists from many countries are developing a number of strategies to help us adapt to global warming. For example, what should people living in coastal areas do? They can move inland away from the dangers of storm surges, although this solution has high economic costs. An alternative plan, which is also extremely expensive, is to build dikes to protect coastal land. The Dutch, who have been doing this sort of thing for several hundred years, have offered their technical expertise to several developing nations threatened by a rise in sea level. We also have to adapt to shifting agricultural zones. Many temperate countries are in the process of evaluating semitropical crops to determine the best ones to substitute for traditional crops if or when the climate warms. Drought-resistant species of trees are being developed by large lumber companies now, because the trees planted today will be harvested many decades later when global warming may already get much worse.

*** In England, transportation had improved a great deal during the years before 1830, but moving heavy materials, particularly coal, remained a problem. It is therefore significant that the first modern railway, built in 1825 for the transportation of coal, ran from the Durham coal field of Stockton to Darlington near the coast. Coal had traditionally been transported short distances via tramways, or tracks along which horses pulled coal carts. The Stockton-toDarlington railway was a logical extension of a tramway, designed to answer the transportation needs arising from constantly expanding industrialization. The man primarily responsible for the design of the first steam railway was George Stephenson, a selfeducated engineer who had not learned to read until he was seventeen. The locomotives on the Stockton Darlington line travelled at fifteen miles an hour, the fastest rate at which machines had yet moved goods overland. Soon they would move people as well, transforming transportation in the process.

KPDS 2011 İLKBAHAR *** An Australian historian proposed that the key to understanding Australia was “the tyranny of distance”. Australians were far removed from their British ancestors, far from the centres of power in Europe and North America and far from each other -with the major cities separated by distances of some 800 km. Time, however, has broken down that sense of distance. Australians today do not see London or New York as the centre of the world. The proximity to Asian economies like China is an economic strength. Transportation and communications links have taken away the sense of remoteness felt by past generations. However, the technology that truly promises to end the tyranny of distance is high-speed broadband, whose benefits we are still only beginning to understand though it has already been a decade since the frenzied dotcom era. That is why the Australian government is rolling out the world’s most ambitious broadband project - a national network that will bring fibre to homes in more than 1,000 cities and towns covering 93% of residences. Next generation wireless and satellite technologies will cover the other 7%. The network will operate at lightning speeds and involve an estimated investment of $40 billion through an independent state-owned enterprise in partnership with the private sector.

*** Not long afterthe Euro came into being in J anuary 1999, Germany was mocked as being the sick man of Europe, its economic fortunes in sharp contrast to the fast-growing countries at the geographical borders of the new currency zone. More than a decade on, however, the tables have turned. Even as the peripheral economies of Spain, Greece and Ireland continue to struggle, 2012 will be the year in which Germany puts a firm stamp on the Euro zone. This will be felt in three related spheres: in Germany’s new-found economic strength, in its preference for, and insistence on greater honesty in public finances and in its growing influence on the European Central Bank. Europe’s economy is set to slow in 2012 as governments address their increasing budget deficits. Germany will enjoy faster gross domestic product growth than the average in the richer parts of the currency zone (whose membership keeps on increasing). Germany is less burdened by household debt and has a smaller budget deficit than almost all its peers - and so has less need to raise taxes or curb public spending. The country is also better placed to benefit from the boom in emerging markets.

*** In 1993, Frances Rauscher and her team published a scientific paper that changed the world. She had taken a number of students and randomly divided them into three groups. One group listened to Mozart’s Sonata for Two Pianos in D Major, the second group heard a standard relaxation tape, and the third sat in silence. Everyone then completed a standard test of spatial intelligence. Those who had listened to Mozart scored far higher than those in the other two groups. Journalists reported the findings, with some exaggerating the results, declaring just a few minutes of Mozart led to a substantial, long-term increase in intelligence. The idea spread, some reporting that even babies became brighter after listening to Mozart. But when other scientists tried to replicate Rauscher’s results, they concluded that the effect, if it existed, was much smaller than was first thought. For instance, Glenn Schellenberg had children learn keyboard skills, have voice training, take drama classes or, as a control, do nothing. Clear IQ improvements were observed in children who were taught keyboard skills or given voice lessons, whereas those given drama lessons were no different from the control group. It seems that the focused attention and memorization required in certain tasks, not just listening to Mozart, helps children’s self discipline and thinking.

*** The idea that American Indians could have built something resembling a city was so foreign to European settlers that when they encountered the Cahokia Mounds in Illinois in Midwest America, they thought they must have been the work of a foreign civilization: either the Phoenicians or the Vikings. Even today the idea of an Indian city runs so contrary to American notions of Indian life that no Anglo Saxon American can absorb it. The first person to write an account of the Cahokia Mounds, the earliest and finest city built by Indians, was Henry Brackenbridge in 1811. When he reported his discovery, likening it to Egyptian pyramids, newspapers widely ignored it. He complained of this to his friend, former president Thomas Jefferson, and the word of “Cahokia” did eventually get around. Unfortunately, most Americans were not very interested. The United States was trying to get the Indians out of the way, not appreciate their history. The Indian Removal Act of 1830 which ordered the relocation of eastern Indians to lands west of the Mississippi was based on the assumption that Indians were nomadic savages with no ability to make good use of land. Evidence of an ancient city, close to the size of Washington, D.C. at that time, would have spoiled the story line.

*** Women seem to be particularly vulnerable to depression during their reproductive years: Rates of the disorder are highest in females between the ages of 25 and 45. New data indicate that the incidence of depression in females rises after giving birth. In 2007 Patricia Dietz reported that 10.4% of 4,398 mothers had been depressed in the nine months following childbirth compared with 8.7% in the nine months before pregnancy and 6.9% during pregnancy. More than half of the women with post natal depression had also been depressed during or before pregnancy suggesting that a previous occurrence of depression may be the biggest risk factor for acquiring the illness postpartum depression. But the hormonal changes that occur in a new mother’s body are also thought to contribute to postpartum depression. During pregnancy, a woman experiences a surge in blood levels of oestrogen and progesterone. Then, in the first 48 hours after childbirth, the amount of these two hormones falls suddenly, almost back to normal levels. This chemical instability could contribute to depression. Of course, hormonal flux does not fully explain postpartum depression. After all, this biochemical fluctuation occurs in all new mothers and yet only a relatively small proportion of them become depressed.

KPDS 2011 SONBAHAR

*** Americans tend to think that varieties of English are more determined by region than by any other factor, such as age, ethnicity, gender and social class. The linguist Henry Smith, for instance, maintained that each region of American English is highly distinctive. Scholars who have investigated the matter have been influenced by the theory of dialect geography formulated in the 19 th century by European dialectologists. As a result, investigations have presumed the idea of long-settled and stable regions – an idea appropriate for Europe but less suitable to the more recent and fluid settlement patterns of the US. Even so, American English dialects are conventionally treated under four headings: North, Coastal South, Midland, and West. The Northern dialect stretches from New England to New York and was shaped by migration from the 17 th century colonial settlements. The Coastal Southern dialect centres on the Atlantic port cities of the states of Virginia, the Carolinas and Georgia, formed in a time of plantation and ranch agriculture. The Midland dialect is spoken between North and South Midlands according to some dialectologists while others emphasize its affiliation with its neighbours and describe it as Lower North and Upper South. Finally, the Western dialect is used in the area that covers California and the Pacific Northwest.

*** To succeed in school, children must master three skills – reading, writing and arithmetic – but not all students readily grasp these basic skills. Among English-speaking children, an estimated 2 to 15% have trouble with reading or spelling, broadly classified as dyslexia. From 1 to 7% struggle to do math, a disability known as dyscalculia. Statistics vary but dyslexia appears to be more common among English speakers than among speakers of highly phonetic languages such as Turkish and Italian. It is believed that at least one child in most elementary school classes in the US suffers from dyslexia. Both dyslexia and dyscalculia defy easy explanation. Neither disorder is the result of faulty eyesight or hearing, both of which can also delay language acquisition but are easily corrected. Instead, children with dyslexia and dyscalculia have working sensory organs, apparently normal sensory and motor development and, sometimes, above-average intelligence. After more than 15 years of research, investigators now believe these conditions frequently involve so-called partial functional deficits of the senses: In affected children, the eyes and ears accurately register sights and sounds, letters, numbers and spoken syllables, but that information is misinterpreted as it is processed in the brain.

*** The First World War could be called the War of the Ottoman Succession. It was, in part, a struggle between Austria and Russia for domination in the areas in the Balkans once ruled by the Ottoman Empire. Its first shots were fired in the former Ottoman city of Sarajevo. Throughout the summer and autumn of 1914, as the European powers were locked in battle, the Ottoman government hesitated. Finally, at the end of October, against the wishes of his colleagues, Enver Pasha decided to attack Russian targets with the new warships in the Black Sea. His decision led to war across Europe, the collapse of the Ottoman Empire and the end of stability in the Middle East. Initially, the alliance between the Ottoman Empire and the Central Powers worked well. In the first half of the 20 th century, Germany was not the source of horror that it later became. Britain, France and Russia were the enemies to be feared and resented. By comparison, Germany appeared friendly. The Ottoman government calculated that its alliance with the Central Powers would restore the glory of the empire, help it recover some of the islands lost to Greece in 1913, and perhaps lead to an extension of territory in Turkish-speaking central Asia.

*** The Dead Sea is a place of mystery: the lowest surface on earth, the purported sites of Sodom and Gomorrah, a supposed font of curative waters and, despite its name, a treasure trove of unusual microbial life. Yet its future is anything but a mystery. After centuries of stability – owing to a delicate equilibrium between freshwater supply from the Jordan River and evaporation under the relentless Middle Eastern sun – the sea is now disappearing. Jordanians to the east, Israelis to the west and Syrians and Lebanese to the north are pumping so much freshwater from the river catchment that almost none reaches the sea. Israel and Jordan are also siphoning water from the Dead Sea to extract valuable minerals, hastening the decline. Thousands of sinkholes have formed in the receding sea’s wake, curtailing tourism and development along the border because no one can predict where the next gaping hole will suddenly open, potentially swallowing buildings, roads or people. Concerned over losing a valuable natural and cultural resource, officials from Israel, Jordan and the Palestinian Authorities have proposed an enormous conveyor system that would steadily refill the Dead Sea with water from the Red Sea to the south. Scientists are testing how the mixing of the waters might affect the lake’s chemistry and biology.

*** For some people, living in an affluent area can actually be a health hazard. This is the provocative conclusion of a study of the death records of more than 8,000 people living in four major US cities. The ill effects of being poor or living in economically disadvantaged areas have been demonstrated before, but it is unusual to consider that poor people living in richer areas may be no better off. Marilyn Winkleby, a researcher at Stanford University in California, decided to look into this and was surprised to find that the death rates in four Californian cities were actually highest for poor people living in the richest neighbourhoods. Her study offers two possible explanations: Poorer people living in rich areas may have to pay proportionally more for housing, intensifying the effect of poverty; alternatively, their health may suffer from the stress caused by continually being reminded that they are at the bottom of the pile. Another researcher, Richard Wilkinson, from the University of Nottingham in the UK, also suspects that stress is largely to blame. He reviewed more than 150 studies and concluded that health is generally poorer when differences in income are larger.

KPDS 2012 İLKBAHAR

*** Deception gains a slight edge over deception detection when the interactions are few in number and are among strangers. If you spend enough time with the people you interact with, they may leak their true intent through their behaviour. However, when interactions are anonymous or infrequent, behavioural cues cannot be read against a background of known behaviour, so more general attributes must be used. Because of the negative consequences of being detected, people are expected to be nervous when lying. In response to concern over appearing nervous, people may exert control, trying to suppress behaviour, with possible side effects detectable by the listener such as a planned, rehearsed impression. Lying is also cognitively demanding. You must suppress the truth and construct a falsehood that is plausible, then tell it in a convincing way and remember the story. Cognitive load appears to play the biggest role. When lies are not well-rehearsed, people have to think too hard, and this causes several effects, including overcontrol that leads to blinking and fidgeting less and using fewer hand gestures, longer pauses and higher-pitched voices. Of course, if self-deception is involved, you are less likely to give off the normal cues of lying that others might perceive.

*** A behavioural pattern is considered to be innate when it is essential for survival and already present at birth, as it is predetermined by the genetic make-up of the organism. A reflex is the simplest form of an innate behaviour. It is a programmed reaction to an outside stimulus that is carried out unconsciously. For example, the eyelids close automatically as soon as a draft of air stimulates the surface of the eye and the pupils of a cat will contract as soon as it looks into bright light. These are reflexes that an organism does not have to learn; they are referred to as unconditioned reflexes. An unconditioned reflex is always an unconscious response, and therefore it is impossible to suppress it at will. Such a reflex always requires a stimulus that triggers a certain behaviour. Many unconditioned reflexes exist in order to protect the organism, for example coughing, nausea, or the draw back reflex of the body part that touches a hot object. Anatomically, a reflex is based on a chain of stimulus and reaction, which is referred to as a reflex arc. A well-known example is the knee jerk or patellar reflex in humans, which is triggered by a light hit to the patellar tendon in the knee. The knee jerk reflex is often used in medicine to test the function of the spinal cord and associated nerves. The real purpose of this reflex is to protect humans from injury when tripping.

*** Relatively few people enjoy the opportunity to travel to other countries. By far the most common form of travel is that by residents of a country within that country. International travel, although given high priority by segments of the populations of industrialized nations, is still a minority activity. As a very rough guide, we estimate that expenditure worldwide on domestic tourism may be worth up to ten times that amount on international tourism. Ironically, there are relatively few countries that collect domestic travel and tourism statistics, while much more information is available on international tourism. Why is this? First of all, international travel involves, by definition, the crossing of a frontier. It is therefore easier to observe and monitor. Domestic tourism involves movement internally and is therefore more difficult to research. Countries that only make use of registration forms at hotels miss out on all aspects of domestic tourism that involve staying in other accommodation establishments or with friends or relatives. A number of countries do not even try to measure domestic tourism due to its very nature. For example, in many developing countries, very little domestic movement involves staying in paid accommodation, and so it does not compete with demand from international visitors.

*** During the 1990s, the country that was viewed by American leaders and many others in the West as the most important challenge for a transition to democracy was Russia. The Clinton administration emphasized that one of its high priorities in foreign policy was the success of the movement to democracy and a market economy in the states of the former Soviet Union, in particular Russia. A senior official asserted that “helping the Russian people to build a free society and market economy is the greatest strategic challenge of our time,” and that “Russia was the single most important foreign policy priority” of the Clinton administration. Russia was by far the largest of the former republics of the Soviet Union in both population and land area. In addition, its geographical location gave it influence on issues in several regions in which the US was interested, and it had greater strategic military capability than any other country except the US. On a deeper level, Russia represented what remained of the former geopolitical and ideological rival of the US. If the state that had been the core of the superpower which was considered to be the main adversary of the US and of democracy could, within a relatively short time, be changed into an ideological soul mate of the US, the symbolic implications would be profound.

*** Species diversity provides the foundation for individual ecosystems and thus is the prerequisite for the functioning of the biosphere. It is an immeasurable source of food and medicinal products, and an irreplaceable resource as a gene pool. Due to the increasing destruction and pollution of natural habitats, over-fishing and hunting, humans are destroying the biological multitude of life, and with that, the basis of their own well-being. The public is concerned when a well-known species like the tiger, whale, or mountain gorilla becomes endangered; however, the majority of other cases are hardly ever noticed by the public. Apart from the decline of species diversity within habitats, loss of genetic diversity within individual species has also been observed. The destruction of individual populations leads to a reduction in genetic regeneration capacity within a species. For instance, selective deforestation reduces the quality of the genetic material of affected tree species, since only strong, healthy trees are logged and the weaker ones are left behind. Humankind’s intentional or unintentional introduction of exotic animal and plant species to new habitats plays an important role as well. These so-called “neozic species” can become a threat to local species and lead to their complete extinction. For instance, the flightless Kiwi bird in New Zealand is threatened with extinction by introduced rats and feral cats.

KPDS 2012 SONBAHAR

*** For hundreds of thousands of years, human civilizations tended to barter for goods, trading shells and precious stones for food and other important commodities. For the first evidence of money as currency, we need to go back 5,000 years to where modern-day Iraq now sits, to find ‘the shekel’. Though this was the first form of currency, it was not money as we know and understand it today. It actually represented a certain weight of barley, a kind of plant, equivalent to gold or silver. Eventually, the shekel became a coin currency in its own right. In much the same way, Britain’s currency is called ‘the pound’, because it was originally equivalent to a pound of silver. The ancient Greeks and Romans used gold and silver coins as currency, with the Latin ‘denarius’ ultimately giving birth to ‘dinar’ in various countries including Jordan and Algeria, and providing the ‘d’ that served as an abbreviation for the British penny before decimalization in 1971. It also gives us the word for money in Spanish and Portuguese – ‘dinero’ and ‘dinhero’. The first ever banknotes were issued in 7 th -century China, though it took another 1,000 years before the idea of paper money was adopted in Europe, by Sweden’s Stockholms Banco in 1661.

*** British scientists have begun studying a rare meteorite to reveal more about the history of Mars. The rock, named ‘Tissint’ after the Moroccan area where it crashed in July 2011, was recovered from the ground just five months later – not enough time to be too contaminated. “The Tissint sample is probably the most important meteorite to have landed on the Earth in the last 100 years,” says Dr. Caroline Smith, curator of meteorites at the Natural History Museum in London. An analysis of the rock revealed its Martian origin. It would have been removed from Mars when an asteroid struck the planet, staying in space as debris before being attracted by the Earth’s gravity. Of the 41,000 officially recognized meteorites, 61 come from Mars and the Tissint rock is only the fifth that was witnessed falling. Dr. Tony Irving of Washington University, who performed some initial analysis on the sample, does not think there is much chance of finding fossilized life within it. But the British team could reveal whether minerals have been affected by water or contain elements such as carbon. Smith says “We’re not looking for microbes, but we’re looking for the chemical and environmental signatures to indicate whether Mars, at some point in its past, may have provided a suitable environment for life to exist.”

*** While playing computer games is sometimes seen as a solitary pursuit, a study at Brigham Young University shows that it actually enhances social connections. Studying the effect of multiplayer online games on marriages, researchers found that in the 76% of the cases where the couple played together, games actually aided the relationship. In other words, couples that gamed together stayed together. Games may have other effects on us too. The famous psychologist, Philip Zimbardo, recently spoke out on the subject. In his 1971 Stanford Prison Experiment, in which volunteers were randomly assigned the roles of prisoner or guard, he showed that human behaviour is heavily influenced by environmental and social pressures. More recently, Zimbardo even suggested that exposing children to morally ambiguous situations in games could be useful in helping them develop their own moral compass. One possibility is to explore virtual worlds through computer games that could enable people to experience and understand concepts that they would otherwise find difficult to imagine. Games about society, populated by real people and open to all, could help test how different cultural backgrounds could be brought together in peace.

*** Although many community newspapers are justifiably proud of their hard-hitting local editorials, perhaps half of all community papers carry no editorials at all. Publishers who refuse to editorialize often claim that editorial harassing is resented in small communities. Others are fearful of alienating readers and advertisers. Still others say they do not have enough time to develop polished, well-researched editorials on a regular basis. Many publishers are leaders in the commercial and political lives of their towns, and are so much a part of the local power structure that their editorials would not be persuasive anyway. Those who editorialize assert that editorials and opinion columns give identity to their newspapers and leadership to their communities. Indeed, some of the most inspired writing the US has produced – the ‘Crisis’ essays of Tom Paine, the Federalist Papers explaining and defending the Constitution, the stirring commentary of William Allen White of Kansas – first saw the light of day as editorial or column material in a community newspaper. Courageous hometown editors regularly win Pulitzer Prizes and other professional honours for crusading editorials on local issues.

*** Innovation is not a synonym for invention – an invention has to be taken to the market to be regarded as innovation. Innovation must change the way people do something. In an essay on creativity, Teresa Amabile and others describe innovation as ‘the successful implementation of creative ideas within an organization’. Creativity, which includes invention, is only the starting point for innovation, which is a necessary but not sufficient condition for it. As Amabile implies, the business of innovation needs to be managed all the way from the creative inspiration through to a launchable product or service. Innovation is not restricted to products and services. It might be internal to the business, in the form of new and more effective organizational structures or processes. It could be a new way of marketing or distribution, like online grocery deliveries. By today’s thinking, innovation can also be in the form of a significant improvement to an existing commodity. When you build a better product, not necessarily a revolutionary one, the whole world will want to buy it. A lot of small types of innovation like this are more akin to continuous improvement, which makes up 85-90% of the average corporate development portfolio.

YDS 2013 İLKBAHAR

*** History is one of the few school subjects commonly mandated in education systems throughout the world. Furthermore, the use of history textbooks to support student learning is an almost universally accepted practice. However, the widespread international presence of the humble history textbook should not disguise its ideological and cultural potency. Indeed, essential to understanding the power and importance of history textbooks is to appreciate that in any given culture they typically exist as the keepers of ideas, values and knowledge. No matter how neutral history textbooks may appear, they are ideologically important, because they often seek to inject the youth with a shared set of values, national ethos and an incontrovertible sense of political orthodoxy. Textbooks stand as cultural artefacts that embody a range of issues associated with ideology, politics and values which in themselves function at a variety of different levels of power, status and influence. Embedded in history textbooks are narratives and stories that nation states choose to tell about themselves and their relations with other nations. Typically, they represent a core of cultural knowledge which future generations are expected both to assimilate and support.

*** Farmers in many countries utilize antibiotics in two key ways: at full strength to treat animals that are sick and in low doses to fatten meat-producing livestock or to prevent veterinary illnesses. Although even the proper use of antibiotics can inadvertently lead to the spread of drug resistant bacteria, the habit of using a low dose is a formula for disaster: the treatment provides just enough antibiotic to kill some but not all bacteria. The germs that survive are typically those that happen to bear genetic mutations for resisting the antibiotic. They then reproduce and exchange genes with other microbial resisters. As bacteria are found literally everywhere, resistant strains produced in animals eventually find their way into people as well. You could not design a better system for guaranteeing the spread of antibiotic resistance. To cease the spread, Denmark enforced tighter rules on the use of antibiotics in the raising of poultry and other farm animals. The lesson is that improving animal husbandry – making sure that pens, stalls and cages are properly cleaned and giving animals more room or time to mature – offsets the initial negative impact of limiting antibiotic use.

*** “The Marshall Plan was not a simple program for transferring massive sums of money to struggling countries, but an explicit – and eventually successful – attempt to reindustrialize Europe.” say Erik Reinert and Ha-Joon Chang. It follows that if Africa really wants economic prosperity, it should study and draw valuable lessons from the Marshall Plan’s dark twin: the Morgenthau Plan implemented in Germany in 1945. Reinert tells the story best: When it was clear that the Allies would win the Second World War, the question of what to do with Germany, which in three decades had precipitated two World Wars, reared its head. Henry Morgenthau Jr, the US secretary of the treasury, formulated a plan to keep Germany from ever again threatening world peace. Germany, he argued, had to be entirely deindustrialized and turned into an agricultural nation. All industrial equipment was to be destroyed, and the mines were to be flooded. This program was approved by the Allies and was immediately implemented when Germany capitulated in 1945. However, it soon became clear that the Morgenthau Plan was causing serious economic problems in Germany: deindustrialization caused agricultural productivity to plummet. This was indeed an interesting experiment. The mechanisms of synergy between industry and agriculture worked in reverse: killing the industry reduced the productivity of the agricultural sector.

*** Imagine an industry that runs out of raw materials. Companies go bankrupt, workers are laid off, families suffer and associated organizations are thrown into turmoil. Eventually, governments are forced to take drastic action. Welcome to global banking, recently brought to its knees by the interruption of its lifeblood – the flow of cash. In this case, we seem to have been fortunate. In the nick of time, governments released reserves in order to start cash circulating again. But what if the reserves had not been there? What are we going to do when our supplies of vital materials such as fish, tropical hardwoods, metals like indium and fresh water dry up? We live on a planet with finite resources – that is no surprise to anyone – so why do we have an economic system in which all that matters is growth – more growth means using more resources. When the human population was counted in millions and resources were sparse, people could simply move to new pastures. However, with 9 billion people expected around 2050, moving on is not an option. As politicians reconstruct the global economy, they should take heed. If we are to leave any kind of planet to our children, we need an economic system that lets us live within our means.

*** Many athletes credit drugs with improving their performance, but some of them may want to thank their brain instead. Mounting evidence suggests that the boost from human growth hormone (HGH), an increasingly popular doping drug, might be caused by the placebo effect. In a new double-blind trial funded by the World Anti-Doping Agency, in which neither researchers nor participants knew who was receiving HGH and who was taking a placebo, the researchers asked participants to guess whether or not they were on the real drug. Then they examined the results of the group who guessed that they were getting HGH when, in fact, they had received a placebo. That group improved at four fitness tests measuring strength, endurance, power and sprint capacity. The study participants who guessed correctly that they were taking a placebo did not improve, according to preliminary results presented at the Society for Endocrinology meeting in June 2011. “The finding really shows the power of the mind” said Ken Ho,an endocrinologist at the Garvan Institute in Sydney, Australia, who led the study. She maintains that many athletes are reaping the benefits of the placebo effect, without knowing whether what they are taking is beneficial or not.

ÜDS FEN 2000 SONBAHAR

*** There are about forty distinct kinds of wild cats known to inhabit the earth today. They range in size from the mighty Siberian tiger to several little spotted species about the size of the average domestic cat. The cats are the most efficient land predators left on earth. They combine power, speed,patience, camouflage, and considerable individual skill. All swim well, most climb with great agility, and at least for short distances, most can move with amazing swiftness. The African lion can reach a speed of almost forty miles per hour when it charges.

*** We are warm-blooded animals. The temperature inside us is generally higher than the temperature outside us. It follows from this fact that, just as a kettle of hot water cools as it loses heat to the air around it, so the human body is continually losing heat. But, unlike the kettle, it does not cool down, for all the time fresh quantities of heat are being generated inside. The body is both making heat and losing some of it at the same time. The loss of heat is controlled by a very delicate mechanism. The body resembles a thermostat heater in that while it gives off heat it manages to remain at the same temperature.

*** Civil engineering offers a particular challenge because almost every structure or system that is designed and built by civil engineers is unique. One structure rarely duplicates another exactly. Even when structures seem to be identical, site requirements or other factors generally result in modifications. Large structures like dams, bridges, or tunnels may differ substantially from previous structures. The civil engineer must, therefore, always be ready and willing to meet new challenges.

***Genetics is the study of mechanisms of the hereditary process. Modern genetics began with the experiments of Gregor Mendel in 1865. He studied the inheritance of different factors in peas, and found that some traits were "dominant" and some "recessive", the "dominant" appearing in a ratio of very nearly three to one.Mendel's results were ignored for many years until their rediscovery at the beginning of the twentieth century.

*** All our sources of power are ‘natural’; we have found that matter can be turned into energy and energy into matter, but that nothing can be created. We can convert one into the other with relative ease, but all our power is based upon the control of natural sources, in the sense that the energy or fuel is never man-made. It already exists in the wind and in rivers; or it may be stored up as in oil or coal.

*** Geologists are especially interested in the mineral content of rocks. All rocks consist of one or more minerals, many of which are needed as raw materials for industry or have properties which make them valuable or useful. Gold, for example, is valuable. Diamonds are both valuable and useful. Coal is also found in rocks, usually underground and it is vitally important as fuel in modern life. Britain is rich in coal because it was covered in dense forest more than 300 million years ago. Coal is formed from the remains of trees and the other plants which have gradually been compressed and hardened in the rock structure of the earth.

*** To astronomers, the great accomplishment of the flights to the moon was the bringing back of rocks from the lunar surface. It was the first extraterrestrial material ever to reach Earth, with the exception of meteorites. The lunar rocks seemed to show that the moon was virtually free of water and of organic material and was, therefore, a world utterly without life. In fact, this had been suspected by astronomers, since the 1600s; but there had been some hope of traces of air and water that might have made possible very primitive life at the bacterial level, if nothing more.

*** Long after the discovery of electricity, man found that he could use the great power to produce it. At first, he used natural waterfalls.Later, man began to build dams to generate hydroelectric power. Dams are immense structures which hold back the water of a river and form a lake behind. The water is let through under control and allowed to fall through pipes to the turbines below. The rushing water drives the turbines, and as they revolve, they spin electromagnets; these magnets generate electricity.

ÜDS SAĞLIK 2000 SONBAHAR

*** Feeling the pulse is part of the doctors' routing in examining a patient in hospitals. The nurse counts the pulse night and morning. The pulse gives the doctor useful information about his patient's condition, so helping him to diagnose the trouble. The cause of the pulse is the beating of the heart. At each heart beat from four to six ounces of blood are pumped into the main artery of the body, and in consequence a wave of pressure goes along the arteries all over the body. It is this wave which is the pulse the doctor feels at the wrist. A beat of the heart is responsible for each wave.

*** A bruise is caused by damage to the tissues under the skin, which leads to bleeding. The common everyday bruise is due to the tearing of some small blood vessels and to the escape of blood beneath the surface: it usually causes no more than a slight bluish discoloration and clears up without treatment. A severe blow, however, may cause bruising in the muscles, usually accompanied by tears in the fibers of the muscle: the pool of blood that results may lead to a large and painful swelling.

*** Nobody should be ashamed of being depressed. It is either a natural aspect of grief, or an illness which is treatable. It is normal to be depressed after a divorce or the death of a loved one. But, in today's rushing society, people may feel they should recover from depression faster than is really natural. Recovery from mourning should be expected to be a matter of many months, not weeks, and nobody should be afraid to let this depression run its natural course. In fact, trying to rush the pace or brighten up with antidepressants is seldom the best route to a full recovery.

*** Tooth decay is dealt with by drilling out the decayed matter and filling up the resulting cavity. All decay and weakened areas must be removed, otherwise decay will continue beneath the filling. The cavity should be prepared so that the filling will stay in securely and withstand pressure from chewing. High speed electric drills are now usual and so is the use of an injection of a local anesthetic to make the procedure painless. A lining of chemical element is put into the prepared cavity to protect the pulp from heat and chemicals. The filling, placed on top of this, is usually an amalgam of silver, tin, copper, zinc alloy and mercury.

*** Studies show that more than two cups of coffee a day can cause unpleasant symptoms such as nervousness, irritability, stomach pain and insomnia. Thus, although many people build up a certain tolerance for the stimulant experiments indicate that caffeine users take longer to fall asleep than non-users and they also wake up more often. The effects ofcaffeine are similar in persons of all ages, but certain groups are particularly sensitive to the drug, including the elderly, children, pregnant women, and those suffering from heart disease, hypertension, and emotional illness.

*** Colour blindness is a genetically transmitted condition in which a person cannot detect all colours. The defect is more common in men than in women. Most colour blind people can see only two basic colours, and they tend to confuse other colours, especially red with green. This impairment can bring about problems because many colour blind people do not realize that their eyesight is defective. They have learned to use the colour terms that everyone else uses, and they are not aware that they do not see what others see. There is a risk that their condition might place them in danger.

*** The presence of fever in a patient does not necessarily prove that he is suffering from an infection. An accurate clinical diagnosis will depend on several important factors - the age and the sex of the patient, the clinical history and any physical signs which point to a focus of infection. Whatever may be the underlying cause of fever, the important thing is to keep the patient as comfortable as possible. This will include keeping the temperature down. Further, good nursing in clean surroundings with frequent bed-bathing will add greatly to the patient's wellbeing and may even prove lifesaving.

*** Excessive vitamin consumption is a potential problem. Some people take unnecessarily large quantities of 3vitamins and minerals for years, assuming that if a little is good for them, a lot must be better. There is no evidence to support their convictions. In fact, a study released last month in a public health journal says that people who take supplements are not healthier and do not live longer. According to researchers almost no one needs to take them. Vitamin deficiencies are almost nonexistent in the West except for among some elderly people whose diets do not usually include sufficient fresh fruit and vegetables.

ÜDS SOSYAL 2000 SONBAHAR

*** Any criminal justice system is an apparatus that society uses to enforce the standards of conduct necessary to protect individuals and the community. It operates by apprehending, prosecuting, convicting, sentencing these members of the community who violate the basic rules of group existence. The action taken against lawbreakers is designed to serve three purposes beyond the immediately punitive one. It removes dangerous people from the community; it deters others from criminal behavior and it gives society an opportunity to attempt to transform lawbreakers into lawabiding citizens.

*** The literature of any society reveals the values, the conflicts and the experiences, both past and present of its members. Through the literature of a people it Is possible to gain some insight into their attitudes, beliefs and problems. Much of human experience is similar because basically all men have similar needs, and the reader may find his own concern voiced by character in a novel written centuries before

*** Chronic psychological stress depends much more on the person than on what happens to him. Someone promoted above his abilities will probably be under constant stress but some personalities may not be aware of the pressure, or of not being up to the job. On the other hand, someone in a position well within his abilities may still find everything a strain. Perhaps it is because he does not realize he can do it easily or because he wants to win promotion; or simplybecause that is his way of going about things.

*** The emergence of hew processes for producing goods and services that provide better quality at lower cost them. Mass production has transformed the nature of work. This is a fundamental change in economic life. People perform a much more central role in the new production system. There is now more democracy in the workplace. A reduction of business hierarchies, the creation of production teams, more worker participation in decision making and employee ownership are some of the indicators of this economic democracy. The heart of this process is not technology; it's a new set of social relations in the workplace which means that most workers now enjoy more work satisfaction.

*** In a competitive economy, the consumer usually has the choice of several different brands of the same product. Yet underneath their labels, these products are often nearly identical. One manufacturer's toothpaste tends to differ very little from another manufacturer's. Two different brands of shampoo may vary only in scent and color. And the tobacco in two different brands of cigarettes frequently come from the same fields. This close similarity means that a shopper has little reason to choose one brand over another. Thus, manufacturers are confronted with a problem: how to keep sales high enough to stay in business. Manufacturers solve this problem by advertising.

*** The urge to dance is one of the most powerful of human İnstincts. Nearly all dances from antiquity to the 19th century had one important common characteristic. They alternated between collective movements and solo twins, in which every dancer or couple in succession would become the centre of attention and then merge back into the crowd. Thus dancing expressed both the communal and the individualistic impulse of humanity, holding the two in delicate balance tothe benefit of healthy society.

*** A great deal of archaeological evidence has revealed the importance of water supply systems in the ancient world. Probably the most impressive system were built by the Romans, whose aqueducts still stand in modern Italy, Spain, France and Turkey. Rome itself had a water supply estimated at 50million gallons a day or about 50 gallons a day for each resident of the city. The water has delivered to fountains where people collected it in pots and then carried it to their homes; only a few buildings and residences had connections to the main pipelines.

*** The term "Third World" is used to denote the poor nations of Africa, Asia and Latin America as opposed to the "Second World" of communist developed nations. The terminology is far from satisfactory, as there is a great social and political variation within the "Third World". Indeed, there are some countries where extreme poverty prevails, and these could be regarded as a "fourth" group.

ÜDS FEN 2001 İLKBAHAR

*** Weeds are plants out of place, either as the wrong plant in cultivated ground, or as any plant where none should be. They can cause considerable financial loss through the cost of their control and the damage they do to crops. Plants which become really troublesome as weeds are those which persist despite man's efforts to control them. Such persistency is due to several factors of which perhaps the most important are prolific seed production, coupled with die often remarkably long periods of dormancy of the seed, and the ability of vegetative parts of some plants to survive mechanical damage and adverse conditions and to set up new plants. Weeds may be controlled by hand, by cultivation and other mechanical means, by biological means and by chemical weedkillers. Chemical weedkillers are widely used, either to give a total kill and suppress all vegetation or to control weeds selectively in crops.

*** Strictly speaking the term "avalanche" should be restricted to falls of snow and ice in mountainous regions but popular usage has extended its meaning to cover rock fails and landslips in all environments. The period of greatest danger from avalanches proper is during a thaw, when melt-watermakes a good lubricant for the snow and ice banked steeply against rock faces. The rising cloud of white dust, the vertical grooves and patches of bare rock formed by the scouring action, and the dull roar of the avalanche are all common features of mountains above the permanent snow line. Rock fragments may also be carried down, for the recurrent freezing and thawing of water lodged in joints and crevices of the rock forms a powerful agent of disintegration. The action is the same as that which leads to burst pipes. Freezing causes expansion of the water in the spaces of a joint and produces a pressure sufficient to break the rock.

*** A typical explosives factory is divided into two parts: the "nondanger" and "danger" areas. The main business of the non-danger area lies in the manufacture of nitric and sulphuric acids for the nitration processes, including the recovery of these acids from the waste products of nitration. Other raw materials are also prepared in the non-danger area. The actual manufacture of explosives and their mixing and packing are carried out in the danger area, subject to rigorous safety measures. The main danger in manufacture is ignition by spark, friction or impact, the latter two being especially hazardous if the explosive is allowed to become contaminated with gritty material. Naked lights, steel tools or anything which might produce spark or flame are therefore excluded from the danger buildings. Each building has a "clean" floor which may be approached only in specially cleaned shoes, while the workers are provided with factory clothing to ensure that grit is not carried into the buildings.

*** Aircraft landing-wheel brakes are fitted to all sizes of aircraft for arresting motion after touch-down, for steering during taxiing by differential control of port and starboard brakes, and to hold the aircraft stationary while the engine is warmed-up or tested. Small aircraft have simple two-shoe internal expanding brakes manually operated and very similar to the standard road-vehicle brake, but the larger machines require power-operated brakes using compressed air or hydraulic pressure from compressors or pumps driven by the engine. Besides being as light and compact as possible, landing-wheel brakes must remain effective and balanced during very high rates of energy dissipation due to the great weight of the aircraft and the very high landing speeds.

*** The culmination of the classic age of the machine tool was the work of Joseph Whitworth. His pre-eminence lay not so much in any far-reaching innovations as in the quality and accuracy of the workmanship he was able to obtain. It was Whitworth who introduced the standard screw thread which was used in British engineering until 1948, and it was he who revolutionised standards of measurement. Indeed, the many measuring machines of the second half of the 19 th century, though increasing the facility, did not greatly increase the accuracy Whitworth had attained. At the Great Exhibition of1851 his planing, slotting, shaking, drilling, punching and shearing machines made him the outstanding machine-tool maker of the age.

*** The main advantages of electric traction on railways are that it is both pleasant and efficient. It brings the removal of a smoke nuisance from tunnels and from the vicinity of larger cities. Further, owing to high acceleration, it is possible to provide a more frequent and faster service on densely populated suburban lines. The track capacity is improved by electrification on mountainous lines because of increase of speed, both up and down the gradient, generally using electric forms of braking in the latter case. Some of the major electrification schemes of the world, for instance, those in Switzerland and Sweden, have been largely dictated by the desire to operate the railway system without dependence upon imported fuel.

*** Just as railway bridges were the great structural symbols of the 19 th century, highway bridges became the engineering emblems of the 20 century. The invention of the automobilecreated an irresistible demand for paved roads and vehicular bridges throughout the developed world. The type of bridge needed for cars and trucks, however, is fundamentally different from that needed for locomotives. Most highway bridges carry lighter loads than railway bridges do, and their roadways can be sharply curved or steeply sloping. To meet these needs, many turn-ofthe-century bridge designers began working with a new building material: reinforced concrete, which has steel bars embedded in it. And the master of this new material was Swiss structural engineer Robert Maillart, who designed some of the most original and influential bridges of the modern era.

*** Botanic gardens may be regarded as having a threefold function: to please and educate the public; to carry out investigations regarding the economic value of native and foreign plant products and acclimatisation of plants; and to act as centres of information and scientific investigation in various fields of botany, such as anatomy, morphology and physiology, for which museums, libraries and laboratories are also needed. The search for drugs and spices particularly has tempted men from early times to explore all parts of the world and this has promoted a close link between exploration and botanic gardens. One well-known botanic garden is the Royal Botanic Garden at Edinburgh which was founded in 1670 by Robert Sibbald for the cultivation of medical plants. Since that date it has been removed to several different sites. It is now one of the major botanic gardens in Britain with an area of over 60 acres.

ÜDS SAĞLIK 2001 İLKBAHAR

*** Aspirin used to be regarded as a safe cure for numerous ills, and was widely used both in its soluble and insoluble form. Now, however, it is not so well thought of. It is useful for the relief of headache or other pains and it will reduce a fever for 2 or 3 hours and so make a patient feel more comfortable, but except in very rare cases it will cure nothing. Moreover, there have been some very serious cases of poisoning as a result of taking aspirin. This is why children's aspirins are not recommended, for they are often nicely flavoured, so the children are tempted to eat them like sweets. For small children, suitably small quantities of ordinary adult soluble aspirin should be used after having checked the dose with the doctor.

*** Combined active and passive prophylactic immunisation is designed to obtain both the immediate but temporary benefit of protection: afforded by serum and the more remote but lasting benefit afforded by vaccine injection. It has been used chiefly in diphtheria. If, for example, this disease breaks out in a school, a small dose of serum, such as 500 antitoxic units, may be injected at once into all the children, and a first dose of diphtheria prophylactic vaccine given simultaneously. A second dose of vaccine is administered four weeks later. The serum confers passive protection during the time that active immunity is developing. Since the main effect of the serum passes off in 3-4 weeks and active immunity is not established for 5-6 weeks, there may be a short intermediate period of relative susceptibility, but it has been found in practice that, provided the children are protected torn infection by temporary segregation of carriers, there is very little risk of diphtheria breaking out again. An outbreak can thus be brought to an abrupt end.

*** Some underweight people enjoy an active, healthy life, but others are underweight because of smoking habits or poor health. An underweight person, especially an older adult, may be unable to preserve lean tissue during the fight against a wasting disease such as cancer or a digestive disorder, especially when accompanied by malnutrition. Without adequate nutrient and energy reserves, an underweight person will have a particularly tough battle against such medical stresses. In fact, many people with cancer die, not from the cancer itself, but from malnutrition. Underweight women may become infertile. Exactly how infertility develops is unclear, but contributing factors include not only body weight, but also restricted energy and fat intake and depleted body fat stores.

*** Worms are intestinal parasites, but the only common types found in Britain are . threadworms, the tiny thread-like worms which cause irritability and itching in the skin of children, less often in adults. Then there are round-worms, ' somewhat resembling the ordinary -garden earthworm, which seldom lead to symptoms. Finally the third group is the tapeworms which may reach a , length of 3 or even 6 m. Many parasitic worms lead a double life, they spend part of their life in the human intestine and the other part in the muscles of another animal. The tapeworm, for example, while in the human intestine, lays eggs which pass out of the body in the excreta, and are then swallowed by various animals, especially in those parts of the world where human excreta are used as manure in the fields.

*** For many years whooping cough has been regarded merely as a bother to the patient and a nuisance to others; as, in fact, a trivial disease. Unfortunately; this is not so: because statistics show that it has caused more deaths than polio, diphtheria, scarlet fever, and measles put together. Whooping cough begins in a child as an ordinary cold with cough and slight fever, and this stage lasts for a week or ten days. Then begins a series of coughs following in rapid succession, during which time, the patient is unable to breathe. The "whoop" is caused by the noisy in drawing of breath when the fit stops. The face may become blue and congested. Bronchitis is usually present, and bronchopneumonia may result as a complication, so inoculation of all children before the disease has a chance to strike them is most important.

*** Typhus used to be known as "jail fever" because it was frequent in prisons; but overcrowding, poverty, and bad hygienic surroundings anywhere are suitable conditions for epidemics of typhus. Improved conditions in industrialised countries have made it unusual, since typhus is carried from one person to another by infected body lice. Typhus comes on suddenly with a rise in temperature to about 39 °C, but within four days it may be as high as 42 °C. There may, or may not, be a rash at this time, and in the second week, when the temperature is at its highest, there is delirium, physical weakness, and a weak pulse.

*** Thalidomide was unique. In every animal test used in the late 1950s, it had a clean bill of health. It was chemically related to other drugs which had been in use for a long time. Over-dosage with thalidomide was unlikely to prove fatal. It was marketed in Europe and in Britain as a "safe sedative". The tragic results that followed its use by women in the early weeks of pregnancy are how well known. Babies were born with severe deformities of limbs, internal organs or both. That effect could not have been foretold from any animal tests in use at that time. Since that date new drugs have been subjected to strict testing in various animal species to check the effect on foetal development, along with the older tests for toxicity which had always been undertaken by well-known drug companies.

*** Basic sanitary facilities are absent in many parts of the tropics, particularly in rural areas, and this state of affairs is responsible for the prevalence of preventable diseases such as hookworm infection, dysentery and cholera. The solution lies in the provision of toilet facilities which are suitable for the local conditions, and the villagers must be educated to use them and to appreciate their value to the community. In view of the need to improve the fertility of the soil, local authorities are becoming increasingly interested in simple methods of composting village waste, in this way invigorating the soil with valuable humus without the risks of infection involved in the old practices of direct application.

ÜDS SOSYAL 2001 İLKBAHAR

*** If the term employment is used in its wider sense as meaning work which is of economic value not only to the family but to the community, women have at all times been thus employed. In primitive societies and in agricultural communities women shared in the productive work on the land, in the workshops and the home; they took part in trade and they cared for the old, the sick and the infirm at a time when there were no health and welfare services in the modern sense. Indeed, in any society in which the productivity of labour is low, women's active participation in the daily work is indispensable. To this day, women work in the fields in all agricultural communities; they spin and weave, do building work and various other kinds of hard physical labour in many parts of the world which have not yet reached the threshold of industrialisation.

*** Venice is famed for its architecture and for its paintings. Venetian architecture reflects the Byzantine influence, though Gothic became the main style in the 14th century; and the first Renaissance building dates from the second half of the 15th century. The Venetian school of painters did not come into prominence until the 15th century. The best known of this school are Gentile and Giovanni Bellini, Titian, Tintoretto and Veronese. The centre of Venetian life is St Mark's square which is paved with marble. Round this and the adjoining Piazzetta are grouped buildings of great beauty and interest. At the eastern end of the square is the basilica of St Mark, a fitting cathedral for the patriarch of Venice.

*** If you wish to glimpse the ancient trophies of Pedra Furada, in a difficult, mountainous area of northeastern Brazil, you must be prepared to face intense dry heat, treacherous rocky ground, sudden flooding, boulders falling from cliffs, aggressive snakes and swarms of mosquitoes. It's worth it, though, for the archeological treasures to be found there are exceptionally ancient. Rock paintings - vivid depictions of prehistoric ways of life, including dancing, hunting, war and sex rituals - have been dated at more than 12,000 years old, while curious pebble structures, primeval bonfires and early stone tools are up to 50,000 years old. The discovery of the remains had a profound effect on our knowledge of American prehistory, for they suggested that the first people in the New World arrived not via a bridge of ice from Siberia but by sea from Africa or possibly even Australia.

*** Radical changes in the life of western society were brought about by the new technical inventions of the 181 and 19th centuries which resulted in a gradual but complete reorganisation of the productive process. This is generally known as "the Industrial Revolution". The outstanding feature of this industrial revolution was the increasing specialisation of labour and, with it, the removal of more and more industries from the home to the factory. At the same time, a growing number of activities such as baking, soap making and dressmaking were taken over by industry. These changes profoundly affected the lives of women. Many of them worked in the new factories for very low wages and for excessively long hours; others worked at home for manufacturers.

*** New evidence suggests that Noah's flood really did happen. A recent expedition has confirmed that a huge flood occurred 7000 years ago in the Black Sea. The theory was first put forward last year by marine geologists William Ryan and Walter Pitman. In their book Noah's Flood, they argue that the great flood resulted from the last ice age, which peaked 12,000 years ago. When the poles froze, ocean levels dropped and cut off the Mediterranean from the Black Sea, which dropped 150 m to become a lake with fertile shores. When the ice thawed, the Mediterranean rose back up and broke through what is now known as the Bosphorus with a force equal to 200 Niagara Falls for a period of two years. The Black Sea rose, consuming a mile of shore a day. Those who had settled there fled, spreading their stories of the flood.

*** Winchester is a cathedral city in England. Once a royal city and residence of the kings of Wessex, Winchester competed with London to be capital of England and rivalled Salisbury as a great centre of learning. The progressively intensified agricultural use of the land and the increasing population of the nearby towns and villages gave prosperity to Winchester and turned it into a major economic centre. Wool was an important local product, and its collection and distribution formed part of the city's economy. Under King Alfred, whose statue stands in the city, the cultural and ecclesiastical life of Winchester became firmly established.

*** Mr Mellon senior was a Croesus whose golden touch gave him a grip on much of American industry, including power, mining, civil engineering and insurance. His son, Paul Mellon, was a very different sort of person but no less remarkable. His achievement was that he managed to dispose of more money and goods than any other American of his time, but did it in an unobtrusive and well-regarded way. The great capitalists of the 19th century, particularly Andrew Carnegie and John Rockefeller, pioneered American philanthropy. Carnegie said the same aggressive energy that had made a capitalist rich should be employed to return his profits to society. The man who died rich, died disgraced, Carnegie thundered. Without necessarily agreeing with such strictures, Paul Mellon set out to redistribute some of his wealth. Both men thought that what America needed was culture. Carnegie had favoured public libraries. Mr Mellon went for public art galleries.

*** George Vancouver was a British naval explorer who served as a seaman on Captain Cook's second voyage round the world (177275) and as a midshipman on his third voyage (1776-80). He then saw service in the West Indies. In 1791 he was placed in charge of an expedition to the northwest coast of North America to seek for a passage to the interior of the continent which was rumoured to exist in those parts. On the outward voyage by the Cape of Good Hope, a portion of the southwest coastline of Australia was examined, and Tasmania, New Zealand and Hawaii were visited. Vancouver spent three years in carefully surveying portions of the west coast of North America. He was the first to circumnavigate Vancouver Island, to which his name was given by the Spaniards tocommemorate his achievement. The standard of his survey was exceptionally high and worthy of his old captain, James Cook; and his voyage practically disproved the existence of a water-passage to the interior along these coasts.

ÜDS FEN 2001 SONBAHAR *** The Royal Society is the national academy of science for Great Britain and Northern Ireland but, unlike other national academies, is and always has been independent of state control; it is not maintained by grants from public funds and manages its own affairs. Since its foundation, however, kings, statesmen and government departments have regularly sought its advice on scientific matters; it has never hesitated to assist governments when convinced that the national interest called for scientific action. Within ten years of its foundation the society, at the invitation of Charles II and his ministers, grappled with problems of national food supply, arboriculture, naval architecture and navigation. Throughout the 18 th century it worked with the admiralty on what was then called “the problem of the longitude” in the solution of which are associated the names of the astronomers Edmond Halley and Nevil Maskelyne, the chronometer maker John Harrison and the navigator James Cook. It found a cure for jail-fever and advised on the protection of ships of war against lightning; it organized a geodetic survey of the British Isles and appointed scientific personnel to several Arctic and Antarctic expedition. *** The first flight by a power driven manned aeroplane took place in 1903 and its subsequent development as a military weapon was so rapid that all the belligerents entered World War I totally unprepared to defend themselves against it. The first bombing raids, however, compelled the consideration of anti-aircraft measures, and Britain, in particular, attacked by Zeppelin airships and Gotha aircraft was forced to develop a range of specialized anti-aircraft equipment which came to include guns, searchlights, sound-locators and predictors, giving it a qualitative ascendancy in this field retained until the end of World War II. Indeed the first night attack on London caused such public consternation that its gun defences had to be doubled within forty-eight hours and, though they hit few planes, their presence was of great psychological value.

*** The design of ships is governed by scientific principles and economic considerations but in practice it has many of the qualities of an art. The designer may be supplied with the precise and detailed requirements of an owner or he may receive only the barest outline of requirements such as the weight of cargo to be carried and the speed. The dimensions chosen and the main characteristics of the ship are governed by the trade in which the vessel is to compete. High-density cargoes such as iron ore require little cubic capacity; low-density cargoes such as bananas require vast cubic capacity. The ports which the vessel must enter may impose restrictions on length and draught. Passage through canals may restrict both draught and breadth. The nature of the cargo may determine the size of cargo holds and of the hatchways through which the cargo is loaded and unloaded. Available facilities at the ports to be entered affect the loading and unloading apparatus to be installed in the vessel.

*** Erosion is regarded not merely as the physical removal of soil by water and wind, but rather as the deterioration of all the component parts of the habitat in which man and his crops and livestock have to exist. Since there is no conclusive evidence for any major climatic change in historic times to explain this deterioration, we must conclude that the eroding ofthe total environment has been due primarily to thoughtless destruction of the vegetative cover. This has led to deterioration of the microclimate above and below the surface, generally in the direction of a general drying out of the soil which has exposed it to erosive action of wind and rainfall of high intensity or frequency, and to the loss of organic matter in the soil, thus reducing its capacity to resist erosion by conserving the water that falls on the surface. If everything possible is done within the total environment to conserve the naturally planted or cultivated vegetation, this will also ensure optimal conservation of soil and water.

*** Scientists can now speed up the process of genetic change through biotechnology. Farmers need no longer wait patiently for breeding to yield improved crops and animals, nor must they even respect natural lines of reproduction among species. Laboratory scientists can now select desirable traits from any of a number of species and insert those traits into the genetic material of crops and animals. Among the new products of biotechnology are tomatoes that stay fresh much longer than the usual ones and so promise less waste and higher profits. Normally, tomatoes produce a protein that softens them after they have been picked. Scientists introduce into a tomato plant a gene that is a mirror image of the one that codes for the "softening" enzyme. This gene fastens itself to the RNA of the native gene and blocks its action. A vine-ripe tomato with this special gene rots more slowly than a normal tomato, allowing growers to harvest at the most flavourful and nutritious red stage. The tomatoes will still last much longer during shipping and marketing than regular tomatoes harvested when green

*** The world's nuclear plants have accumulated vast stocks of highly radioactive waste. Worldwide, high-level waste is currently stored above ground, and no government has a clear policy on its eventual disposal. While most experts believe that burying the waste is the safest bet in the long term, the problem isfinding sites that everyone can agree are geologically stable. Decaying radioactive isotopes release heat. As a result, high-level waste must be constantly cooled; otherwise, it becomes dangerously hot. This is why many experts want to store waste above ground until it has decayed and is cool enough to be stored safely in sealed repositories several hundreds of metres below ground. According to one recent theory, however, waste should be lowered down boreholes drilled to 4 kilometres. The trick is to exploit heat generated by the waste to fuse the surrounding rock and contain any leaking radioactivity.

*** To obtain power from the sun's rays is to use nuclear power developed at no expense in a laboratory 93 million miles away, for the radiant energy of the sun is maintained by nuclear transformation of chemical elements occurring in the sun's interior at temperatures of many million degrees, and at pressures of many million atmospheres. The resources of solar power are enormous. If 100 per cent efficiency could be secured in the transformation of radiant solar energy into mechanical work, a horsepower per square yard of ground surface would be available under cloudless skies. The expense of collecting solar energy still prevents its competition with the usual power sources .Yet, unless the vague promise of safe thermonuclear power from oceans becomes realized, solar power must supply the enormous and growing requirements of posterity within two centuries. Because the ground sources (coal, oil and uranium) as they near exhaustion will become more costly than solar power.

*** Sounds produced by continuous vibration tones are spreads waves of compression through the air. Where there is a solid boundary such asthe walls of a room the sound waves are reflected so that the sounds within the room are prolonged beyond what they would be in the open. The sounds produced by the voice or by a musical instrument then reverberate through the room after the actual tone production has ceased. When the sound waves strike the walls some of the sound energy travels on and is either absorbed in the material or may penetrate to the other side; but with the usual hard, unyielding walls of which most buildings are made, more than 90% of the sound energy is reflected back into the room at each impact, so that some time must elapse before all is spent. It is this reverberation which, in its excess, is the prime cause of the faulty acoustics of many pre 20th century buildings.

ÜDS SAĞLIK 2001 SONBAHAR *** Europe faces "a serious risk of an uncontrollable resurgence of malaria", warns the WHO in a new report. Drainage, drugs and insecticides had eradicated malaria from the whole of Europe by the 1960s. Now civil disorder and irrigation threaten to bring it back unless controls are stepped up, the report says. It seems that more European travellers are bringing malaria back from countries where it is endemic, and the big fear is that local mosquitoes could acquire the parasite from such travellers and re-establish a local chain of transmission. Three recent cases in Luxembourg and two in New York have fuelled concern over air travel as a means of reintroduction. The cases in Luxembourg all occurred within a few kilometres of the country's international airport, and were probably caused by mosquitoes stowing away on aircraft coming in from the tropics.

*** The value of heat for the preservation of food has been known for thousands ofyears, but it was not realized until the nineteenth century that a very mild heat treatment far below boiling point, made liquid foods such as milk keep much longer. The discovery followed the work of the French scientist Louis Pasteur on wine and beer. The process, called after him "pasteurization", is a carefully controlled mild heat treatment. It was found that the process served two purposes; it delayed the souring of milk, and it destroyed the dangerous disease germs which sometimes occur in this product. These germs include the bacteria which cause tuberculosis, undulant fever, typhoid and paratyphoid fevers, dysentery, diphtheria, scarlet fever and septic sore throat.

*** Headache, like backache, is one of the commonest types of pain with which mankind is afflicted. It may arise under a diversity of circumstances. A blow to the head causes pain, and after a severe head injury with concussion, headaches may continue for weeks or months. The coverings of the brain, or meninges, are sensitive structures and, when inflamed, as in cases of meningitis, or irritated (as with meningeal haemorrhage), headache may be a prominent feature. The arteries of the brain are also sensitive, and many kinds of headache are referable to arterial disease, more especially to influences which distend the lumen of the arteries, or which distend and then contract the arterial walls. But the brain itself is insensitive and lacerations or gunshot wounds of the cerebral substance may produce headache only in so far as the bony skull and the meninges are at the same time damaged. Tumours of the brain produce headache, not because the brain tissue is involved, but because the raised intracranial tension alters the diameter of the intracranial arteries.

*** The brain, like all of the body's organs, responds to both inherited and environmental factors that can enhance or diminish its amazing capacities. One of the challenges researchers face when studying the human brain is to distinguish among normal age-related physiological changes, changes caused by diseases, and changes that result from cumulative, extrinsic factors such as diet. The brain normally changes in some characteristic ways as it ages. For one thing, its blood supply decreases. For another, the number of neurons, the brain cells that specialize in transmitting information, diminishes as people age. When the number of nerve cells in one part of the cerebral cortex diminishes, hearing and speech are affected. Losses of neurons in other parts of the cortex can impair memory and cognitive function. When the number of neurons in the hindbrain diminishes, balance and posture are affected. Losses of neurons in other parts of the brain affect still other functions.

*** People infected with tuberculosis are difficult to treat because the bacteria can lie dormant in the body. In this state, they are unaffected by antibiotics and do not spark an immune response. But they can reactivate and cause disease when the body's immune system is compromised. Some people believe that proteinsthat deliver a wake-up call to dormant bacteria could be used to fight tuberculosis. There are two ways wake-up proteins might be used therapeutically. One is to trick the bacteria out of dormancy so that they can be zapped with antibiotics. The other is to use the proteins as vaccines. An injection of wake-up proteins might prime the body to notice a subsequent infection earlier than would otherwise be the case and attack the bacteria as they activate.

*** Haemorrhage is an escape of blood from the vessels through which it normally circulates. The quantity lost may be microscopic, or may amount to quite a large quantity; large haemorrhages usually arise from a large artery or vein, while bleeding from a capillary may be shown only by a minute red spot in the skin. Many haemorrhages are trivial and require no specific treatment. Examples of these are such common domestic accidents as cut fingers and nose bleeds. Others form some of the major emergencies of medicine. The principles of treatment are to arrest haemorrhage, to combat shock by restoring normal blood volume, and to keep the patient quiet, comfortable and confident.

*** Physical activity deserves attention in any program to reduce coronary heart disease (CHD) risk. Some evidence suggests that weight training can raise high-density lipoprotein (HDL) if undertaken regularly, but frequent and sustained aerobic activity may be more effective in lowering low-density lipoproteins (LDL) and raising HDL. Furthermore, aerobic endurance-type activities, such as brisk walking, undertaken faithfully for 30 minutes or more as a daily or every-other-day routine can strengthen the heart and blood vessels; alter body composition in favor of lean over fat tissue; expand the volume of oxygen the heart can deliver to the tissues at each beat and so reduce the heart's workload; change the hormonal climate in which the body does its work in such a way as to lower blood pressure; and bring about a redistribution of body water that eases the transit of blood through the peripheral arteries. These changes are so beneficial that some experts believe that physical activity should be the primary focus of cardiovascular disease prevention efforts.

*** In America, Britain and several other countries, the years after World War II were notable for increased interest in, and research on, methods of teaching retarded children. Today no one educational procedure is generally accepted, but there is agreement that teaching should so far as possible be individually orientated. Obviously, special material must be used, suitable to the child's chronological age and general interests, and a variety of ways have been suggested to stimulate the apatheticand remedy loss of confidence. Since inability to analyze spontaneously and make deductions tends to characterize the intellectually dull, care must be taken at all stages of teaching to break down material and demonstrate each step clearly.

ÜDS SOSYAL 2001 SONBAHAR *** No eighteenth century king of England could have raised the vast sums needed to build a Versailles, and no English nobleman would have cared to compete with the German princelings in luxury and extravagance. Still, it is true that the building craze did reach England. The most striking example is Marlborough's Blenheim Palace, which is on a massive scale. But this is an exception. The ideal of the English eighteenth century was not the castle but the country house. The architects of these country houses usually rejected the extravagances of the Baroque style, it was their ambition not to break any rule of what they considered 'good taste", and so they were anxious to keep as closely as possible to the real or pretended laws of classical architecture. Architects of the Italian Renaissance who had studied and measured the ruins of classical buildings with scientific care had published their findings in textbooks to provide builders and craftsmen with patterns. The most famous of these books was written by Andrea Palladio. This book of Palladio's came to be considered as the ultimate athority on all rules of taste in achitecture in eighteenth-century England. To build one's villa in the Palladio manner was considered the last word in fashion. *** Throughoutthe history of music, composers have always sought to find new and original means of expression. Without this urge no progress could have been made. In our modem age, the desire to experiment is stronger than itever was before. This is partly because the instability of world affairs has given rise to a feeling of unrest and insecurity, and partly because more people are being better educated and concerned with culture. The arts are now, as a result, more sophisticated, even more artificial. In the past the ideal was the natural and spontaneous expression of beauty, but it no longer is. In order to avoid the obvious and the commonplace, some composers have, perhaps, moved too far from the main stream of music, and this has sometimes led to eccentricity. But there is a positive side to all this: there is a healthy lack of complacency. *** A state of war may be brought to an end in one of four ways. In the first place, one belligerent may completely overrun the territory of its enemy, whose armies and government are in consequence entirely disrupted. This state of affairs, known as debellatio, came to pass In the case of Germany in World War II. Secondly, states may drift from a state of war to one of peace without any formalities to mark the transition. Such was the case in the war between Poland and Sweden in 1716. Thirdly, formal declarations of peace may be made by one or both of the belligerents. An example of this is seen in the joint resolution of the United States Congress of 1920 by which the war with Germany was terminated. Lastly, there remains the most common method of ending a state of war, namely, by making a treaty of peace. By this means belligerents can define with precision the exact terms upon which they have ended the conflict *** By the 1950s, Africa was among the least developed of the continents and. according to United Nations estimates, as much as 70% of its resources of land and labour were devoted to subsistence production and only about 5% of its total population was engaged in wage-earning employment But even with agriculture there was increasing production for export: cocoa in Ghana, groundnuts and ail-palm products in Nigeria, cotton in Uganda, coffee in Kenya, and maize and sheep-rearing in southern Africa were outstanding examples. Minerals were particularly important in bringing about the opening up of the interior of Africa. For centuries some parts, like the Gold Coast (now Ghana) had produced gold, and during the 19 th century first diamonds and then gold were discovered in South Africa, and since then numerous other minerals have been exploited including copper and cobalt.

*** Most large firms have marketing or advertising departments that work in conjunction with their advertising agencies. Only one part of the company's sales budget will be spent through the advertising agency. Companies have their own sales force for calling on wholesale and retail organizations. In many cases, they also produce a proportion of their sales literature and shop display material. But with the larger companies sales promotion efforts are usually combined under a single marketing policy. which is often worked out by the company itself in conjunction with its advertising agency. The advertising agency may also be consulted over packaging systems and the distributing of new products that are being put on the market.

*** The term intelligence is often used by biologists as synonymous with the capacity to learn. An animal that learns to adapt itself to a wide range of situations is said to be more intelligent than one whose behaviour is largely governed by instinct. In psychology, on the other hand, so wide a definition is not satisfactory. The simpler forms of teaming, at least, can scarcely be said to manifest intelligence in any sense remotely consistent with the ordinary meaning of the term. It is customary, therefore, to distinguish between learning of a more or less repetitive and mechanical type and the solving of genuinely new problems by creative means. It is the latter rather than the former that we ordinarily ascribe to intelligence and most psychologists allow their definition of this term to be guided by popular usage.

*** Sir Walter Raleigh entered court under the protection of the Earl of Leicester, and was soon in high favour with Queen Elizabeth. Fuller's often quoted story of how Raleigh threw down his plush cloak on a muddy road for the queen to walk on, lacks authority, but it is in keeping with his faculty for quick decision and characteristic of the romantic sentimentality pervading the Elizabethan court. He owed his advancement to good looks, a plausible tongue, an ingenious wit and magnificent clothes. But he was proud, haughty and impatient, and everywhere except in his native Devon he made numerous bitter enemies. He was consulted confidentially on Irish affairs but at no time was he one of the Queen's official advisers, possibly because Elizabeth, despite her affection for him, saw through his ambitions and doubted his wisdom. But she was lavish in her favours, and in a few years raised him to affluence.

*** Sir Robert Peel will always be remembered as the one who created the Metropolitan police force but first he set about reducing the savagery of the criminal law and he also introduced various prison reforms. Such changes could not be risked without an efficient police force. So in 1829 he set about creating just that. The original characteristics of the new police were that they were to be an organized non-military force of paid constables (with officers) under two magistrates, as joint commissioners, responsible to the home secretary, an office like that of Minister of the Interior. To emphasize their civilian character, they wore as uniform a dark blue highcollared swallow-tail coat and a heavy chimney-pot hat (not to be superseded by the tunic and helmet until about 1865), and carried no arms but only a truncheon. Until 1885 they had no whistle, only a rattle for summoning assistance. Their earliest instructions were prefaced with the following words which still appear in the forefront of their standing orders: the primary object ofan efficient police force is the prevention of crime is committed. The protection of life and property, the preservation of public tranquility, and the absence of crime, will alone prove whether the objects for which the police we reappointed have been attained.

ÜDS FEN 2002 İLKBAHAR

*** The Rhine - Ruhr area became the greatest industrial region of Germany, because it had at its heart the great coal field of the Ruhr. Mining is now almost entirely northeast and westwards across the Rhine. The region contains the greater part of the German iron, steel and heavy engineering industries. The great integrated iron and steel plants mostly cluster on the Rhine waterway. Specialized steel plants and engineering works are more widespread. With a decline in coalmining and the dismantling after World War II of certain steel plants,some of the older Ruhr towns have diversified their industries considerably: vehicles, electrical goods and clothing are now being produced.

*** Post war radar has been developed for an enormous range of uses from police radar speed traps to the ballistic missile early warning systems. At sea it is used on ships of all sizes from the super tankers down to pleasure craft, and the air it guards military and civilian aircraft against collisions. It is even used to keep track of the orbital junkyard created by innumerable space launches. Radar found an unexpected use in astronomy and space navigation. Radar signals were bounced off the moon in 1946 and reflections were obtained from Venus and the sun in the late 1950s. Subsequently, radar maps were made of the moon and Venus - not that such long ranges are essential for radar maps to prove themselves useful. For example, satellite-borne radar aimed at the earth has actually led to the discovery of previously unknown remnants of a Mayan canal drainage system in Central America.

*** A contraption that automatically fits deer with a pesticide impregnated collar is helping to tackle the menace of Lyme disease, which is usually spread among people by ticks that live on the deer. This disease is now one of the fastest spreading infectious diseases in the US and can be fatal. Trapping and treating every deer in a forest with pesticides isn't easy, so a machine has been designed to do it. The animals are lured to a feeding tray where have to place their heads in a V-shaped through to get to the food. The machine keeps an open pesticide impregnated collar at the ready, drooping next to the trough where the deer will put its neck. As the animal takes the food, its neck presses down on a switch that triggers a spring- loaded arm. This propels one end of the open collar over the neck where it meets the other end. The two ends join using Velcro, so within seconds of the animal's arrival the collar is complete.

*** Transport represents 22 per cent of total energy consumption in industrialized countries, mainly in the form of automobiles. Although this is the fastest growth sector in such countries, the rate of increase in road transport energy demand has slowed in most developed countries since the late 1960s. This has reflected both improved vehicle efficiency and a slowing down in the level of acquisition of automobiles by households. These developments have encouraged hopes that saturation levels may operate at lower levels than sometimes projected. In developing countries, transport represents 14 per cent of total energy consumption but the number of automobiles is approximately 20/1000 people, compared to 600/1000 people industrialized countries. In attention to strictly technical improvements that can be made to automobiles and trucks, there is another important area of action which could help in the solution of the problems, namely, system operation. In this category, there is a variety of actions that could be performed more efficiently such as transporting passengers and freight by other means, such as bus and rail that would result in lower energy consumption and therefore, lower emissions.

*** Britain has a target to deliver 10 per cent of its electrical power from renewable resources by 2010. And despite what one might hear from some quarters, superb natural and technical resources already exist that could make this possible. All that is lacking is the political will; but at present, the government seems reluctant to take any positive action. At present "new" renewables, such as landfill gas, wind, solar, wave power and small-scale hydropower contribute around one per cent to the UK's electrical generating capacity. Generating power from landfill gas is already fully economic; but has limited scope for growth as the country moves away from land filling waste. Energy recovery from waste is highly controversial and also limited in capacity. So, if Britain is to meet her interim target of five per cent by 2003 and 10 per cent by 2010, she must look to other renewables for growth.

*** Glaciers originate in areas that lie above the limit of prominent snow. Thus in tropical climates glaciers are only to be found at very great heights, whereas in polar regions they flow into the sea. The largest glaciers are found in regions receiving the heaviest snowfall. The great glaciers of the Himalayas lie in the path of the monsoon, which deposits on them the full measure of its vast water vapour content. The largest glacierized areas after Antarctica are in Greenland, North America, and in central and south central Asia. It has been estimated that the volume of the world's glaciers and ice sheets exceeds 11,000,000 cubic miles which, if returned to the oceans, would raise the sea - level by some 200 ft, submerging all existing seaports and much land besides.

*** The report, Dams and Development, which has been recently published, provides stark evidence that the world's 45,000 large dams which block over half of the world's rivers, have been failed experiments. They have failed to produce as much electricity and water, or control as much flood damage, as their backers claim. They regularly suffer huge costoverruns and time delays. They have made up to 80 million people homeless, and their benefits have largely gone to the urban well-off not the rural poor they displace. Moreover, their effects on ecosystems have been disastrous.

*** Our understanding of submarine volcanic eruptions has improved substantially in the past decade owing to the recent ability to remotely detect such events and to respond rabidly with brief surveys and sampling at the eruption site. But these data are necessarily limited to observations after the event. In contrast, the 1998 eruption of the Axial volcano on the Juan de Fuce ridge was monitored by on site sea-floor instruments. One of these instruments, which measured bottom pressure, was overrun and entrapped by the 1998 lava flow. The data recorded by this discovered. The data recorded by this instrument reveal the duration, character and effusion rate of an eruption on a mid-ocean ridge.

ÜDS SAĞLIK 2002 İLKBAHAR

*** Ionizing radiation has proved to be most valuable, for example, in clinical diagnosis and radiotherapy. However, inadvertent exposure to relatively high doses of ionizing radiation is capable of injuring and killing cells, inducing mutations, producing developmental abnormalities in fetuses exposed in utero, or even producing latent cancers. On earth, it is impossible to escape exposure to radiation. Cosmic rays bathe the earth continuously, as do terrestrial concentrations of radionuclides, such as radon gas. The two constitute natural "background" radiation. Few humans in developed countries escape diagnostic X-rays, and many require radiotherapy as a potential cure for various types of neoplasia, The "early" injurious effects of radiation appear only when certain cumulative levels of exposure to radiation have been exceeded. However, the later appearing consequences may have no thresholds; hence, the public's concern about the possible carcinogen city of even low-level exposures.

*** For years, it has been assumed that obesity is the result of "too much food and too little exercise". While this maxim is largely correct, the etiology of obesity can be much more complex. There is a well-documented familial tendency, but whether this is of environmental or genetic origin is unclear. Studies of twins separated at birth and living apart provide strong evidence for a substantial genetic influence. Children of overweight parents, when adopted by "lean" families, have a greater tendency to become obese than do adoptees from non-obese natural parents. "Energy efficiency" may contribute to obesity; with reserves of fat deposits readily available to metabolize in the obese, a given amount of activity requires a smaller expenditure of energy. This theory has been invoked by those who complain that they "gain weight whether they eat or not", and indeed there is evidence of differences in energy efficiency among individuals. Similarly, obesity has been attributed to abnormally low basal metabolic rates (BMRs) since obese individuals do show lower BMRs. However, this fact is due to an artifact of BMR measurement; a larger proportion of the total fat mass of an obese person is inert, lowmetabolizing fat, a fact that makes BMR calculations lower.

*** The site of a hospital needs careful consideration. More and more people now agree that a hospital should, wherever possible, be part of the community it serves. This makes for the convenience of patients, particularly outpatients, and eases staff recruitment. Whilst it may be necessary to serve small communities by peripheral separate outpatient departments, in general, in-patient and outpatient buildings should be on the same site. It is not, however, always practicable to build extensively in a town or city, and it is difficult to make provision for expansion. The actual design of a hospital is also of great importance. Medicine is ever changing, and it is difficult to forecast changes that lie ahead. Doctors invariably call for flexibility in planning, which is really only practicable if hospital construction is on ground-floor level and if the wards and the special investigatory departments are so designed that they can be readily extended.

*** The government of Britain is again looking at fluoridation of the public water supply as a possible means of achieving nationwide improvements in dental hygiene. To the casual observer, given the highly erroneous impression that the presence of fluoride in toothpaste is beneficial in the prevention of dental caries, such a measure may understandably appear desirable and worthy of public support. However, many water authorities across the country have long understood the risks associated with fluoridation; but the government and the various official organizations that support such a measure continually deny these risks. It should be realized that there are two forms offluoride. One of these is calcium fluoride, which is a natural substance occurring in water at verylow levels of 0.01-1 parts per million and a substance which the various organizations involved in promoting fluoridation constantly draw attention to when attempting to justify their case. The other form of fluoride is sodium fluoride, which occurs alongside various related substances such as fluosilicic acid and is an extremely dangerous industrial byproduct produced by such industries as aluminum, ceramics, phosphate fertilizers and nuclear power. This form of fluoride is an accumulative poison even more toxic than lead and only slightly less toxic than arsenic.

*** The importance of early detection of deafness in childhood is generally recognized. Detection is normally a two-stage process of which the first is a screening test of hearing. Screening tests sift out children with impaired hearing from those with normal hearing and can be successfully administered at any time after the age of seven months. Children who fail a screening test are given a diagnostic test, as the second stage in the process, to determine the nature and extent of their hearing loss. Both screening and diagnostic tests must be appropriate to the developmental level of the children to whom they are administered. Special techniques for testing babies and young children have been evolved at Manchester University. When deafness has been diagnosed, parents are advised to seek guidance about the management and early training of their children at the audiology clinics established by an increasing number of local authorities and hospitals.

*** By far the most common sleep complaint is insomnia. About a third of Americans have trouble failing asleep or staying asleep, problems that result in listlessness and loss of alertness during the day. Most of the time the distress is temporary, brought on by anxiety about a problem at work or a sudden family crisis. But sometimes sleep difficulties can extend for months and years. Faced with a chronic situation, insomniacs frequently medicate themselves with alcohol or drugs. Doctors warn that in most cases sleeping pills should not be taken for longer than two or three weeks. Such drugs can lose their effectiveness with time, and it takes higher and higher dosages to work. People run the risk of becoming dependent on the pills.

*** Asthma may be defined as a malady characterized by attacks ofbreathlessness due to paroxysmal narrowing of the small bronchi and bronchioles. Asthma may start at any age, but most commonly in childhood. The typical attack starts suddenly with breathlessness and wheezing, the difficulty being mainly in expiration. A small amount of viscid mucus is usually expectorated towards the end of the attack. Attacks may occur at any time, but especially during the night or in the early morning. Their duration is variable, some ending in an hour or so, others, especially if complicated by bronchitis, continuing for days. The frequency of the attacks varies from one or two a year to several daily. During attacks the lungs become over-distended with air, since the obstruction to respiration is greater in expiration than in inspiration and the muscles of forced inspiration, which are brought into action, are stronger than the expiratory muscles.

ÜDS SOSYAL 2002 İLKBAHAR *** In surveys of British public opinion, journalists typically rank below politicians, lawyers and usedcar salesmen as trustworthy characters. And yet we depend upon journalists to guide us through today's rapidly evolving, information-rich "media age". The internet, digital television and technologies as yet unborn all promise to revolutionize how we learn about what's going on, in a world increasingly shaped by the forces of economic globalization. But there is surely no substitute for good-quality, probing journalism. After all, it is the great crusading craft. It is the great support of democracy. In every society, authority - whether government, corporate or pressure group - needs to be constantly and vigorously challenged by an independent press. In every society too, that challenge rarely comes from the right. Indeed, in theory at least, it should come from the campaigning, liberal media. *** As one steps out of the busy commercial streets in front of the station in a Japanese city and moves on into the side streets, one often encounters old shops and historic temples and shrines which hint at the former character of the city. It is in the side streets rather than the mainroads that the original face of a city is to be found. But even in antiquelooking houses in side streets, one can often see that their interiors may have been rebuilt and their fittings replaced in an attempt to keep abreast of the times. This applies in farming villages as well as cities; old-style houses and buildings decrease year by year, and in some cases the changes have been even more radical than those which have occurred in the cities. But although outward appearance and facilities may have been renovated, there has surely been no renovation in the sensibilities and attitudes of the people who live in these new environments.

*** In the past decade the term "internal marketing" has emerged in many companies to describe the application of marketing internally within the firm. This seems to be an area where practice appears ahead of theory. Despite the existence of many internal marketing programs no books, at least in English, have been published on internal marketing and only a handful of articles have addressed this important and emerging area. There are two key aspects to this. One involves the notion of the internal customer. That is, every person working within an organization is both a supplier and a customer. Here we are concerned with getting staff to recognize that both individuals and departments have customers and then determining what can be done to improve levels of customer service and quality levels within the organization. The second aspect is concerned with making certain that all staff work together in a manner that is attuned to the company's mission, strategy and goals. *** Hans C. Andersen, the Danish author, was born on 2 nd April at Odense in Funen. His father, a poor shoemaker, was devoted to reading and thinking, but died when Hans was a child. His mother was a simple, uneducated woman, who after her second marriage sank still deeper into poverty and took to drinking in her old age. Andersen, who loved her dearly, has told her story in "She was Worth Nothing". His grandmother did her best to spoil the boy, who was given to daydreaming. After a very meager education in a pauper-school it was intended to apprentice him to a tailor, but as a fortune-teller had foretold that Odense would one day be illuminated in his honor, his mother permitted him to go to Copenhagen, where he tried to become an actor or a singer, but cut a pitiable figure. Fortunately, kind people supported him. Thanks to the support and guardianship of Jonas Collin, an influential councilor of state, Andersen at the age of 17 was sent to school. In 1828 he matriculated and at once began to write, mostly plays and poems. In the 30s he traveled abroad twice. From 1835 his fairy tales began to appear in installments, and were soon translated into almost all the European languages, and gained for him a world reputation. The full acknowledgement of his own countrymen, for which he longed so much, came much later. But it came at last. He lived to see Odense, his native town, illuminated in his honor as prophesied.

*** The advertising techniques and methods used today were for the most part devised in Britain, Europe and America, and as a new technique evolved in one country it was quickly taken up elsewhere. The practice continues on a worldwide scale. The simplification of the words and illustrations in advertisements has come about as the result of the skill of copywriters, artists and photographers combined with the findings of advertising research workers. Advertisers have found it is often more effective to concentrate on putting across one aspect of their product than to go into lengthy descriptions. They have seized on the truth of the saying that a picture can be worth a thousand words. Some critics of advertising have argued that an advertiser should be content to furnish the public with information about his product and draw the line at persuasion, but in practice the dividing line between informing and persuading is impossible to draw. Persuasion starts at the point where information is first supplied, particularly when, as must happen for reasons of space and time, the information given is selective. *** The growth of the importance of libraries in both the social and the scientific spheres has led to a great development in library science and in educational training for the profession of librarianship. In Germany and France requirements for admission to the profession of librarian have been laid down by the state; in other countries where there are library schools and professional examinations, professional qualifications are usuallyexpected though they are not necessary for admission to the profession. The American Library Association, founded in 1876, was the first such association to be established in the world, and has been responsible in no small measure for the advances in library techniques in America and elsewhere. Library associations now exist in most countries; the English Library Association, founded in 1878, holds national and local conferences, maintains an excellent library and information bureau, promotes facilities for professional education, conducts examinations (elementary, intermediate and final) and maintains a register of qualified librarians. *** The art of Leonardo da Vinci, like his character, is full of conflicting tendencies and apparent contradictions. His tireless curiosity, combined with his deep feeling for all living things, led him as a scientist to explore the entire range of natural phenomena, while at the same time a fantastic creative imagination caused him as an artist to transform the results of his scientific researches in a thousand ways. These two elements of his nature alternated throughout his life and explain his restless changes of occupation and the fact that he never devoted himself exclusively to painting for very long at a time. Although in his versatility and scientific interests he is usually regarded as the quintessence of the man of the renaissance, yet some of the products of his tortured imagination are a direct reminder of the middle ages. Even in his artistic output conflicting tendencies are apparent. In the "Last Supper" he reaches the peak of his renaissance classicism, whereas in other works, such as the Anghiari cartoon, his sense of restless movement clearly foreshadows the baroque. Finally, in the "Deluge" drawings, he produces works, which are completely unrelated to European art and suggest that of the Far East. *** The extension of French as a second or acquired language is particularly striking. This was to some extent the result of military and political prestige, but the virtues of the language itself and French cultural achievements actually played a much greater role. The lead which France took in the 12 th century in literature and in other domains made its language a sort of lingua franca of the cultured classes of Europe; and in the 17 th and 18 th centuries French was cultivated assiduously by the royal courts and the upper classes of most European countries and was written by many nonFrench authors. But even before the end of the 18 th century French began to lose ground, and the movement was accelerated by the revival of national sentiment in Europe at the turn of the century and later by the rise of English under the influence of Britain and America. Many things, however, have found their supreme expression in French; and its precision, beauty and eloquence make it a classical language in the true sense of the word.

ÜDS FEN 2002 SONBAHAR

*** William Willcocks was born 150 years ago in a tent beside a canal in northern India, where his father worked for the colonial government. He learnt his engineering in India before heading for Egypt in 1883. There he rose to become director-general of reservoirs, and a legend on the banks of the Nile. He built the first Aswan dam, then the largest in the world, went on to revive the ancient irrigation systems of Mesopotamia, and watered deserts from south Africa to India. But he was deeply troubled by the discovery that much of what his fellow water engineers did in their colonial playgrounds was worse than useless.

*** The acronym ‘radar’, for radio detection and ranging, has been credited to the US Navy, which used it officially towards the end of 1940, but the concept of radar is somewhat older. Hertz showed that metals would reflect electromagnetic waves and Tesla is said to have suggested using this phenomenon in a radar-like manner in 1899. A few years later a German, Christian Hulsmeyer, received patents for a ship’s anticollision device. Also many radio engineers and experimenters observed that passing aircraft or ships interfered with their experiments. Although these features are all suggestive of radar, none was actually radar unless the term is very loosely defined. In the 1930s, however, several of the major powers became aware of the military possibilities of radar and work on it started immediately in the USA, Britain, France, Germany, Italy, Japan and the Soviet Union. By the end of World War II, military radar, and military radio navigation aids too, were well developed.

*** Aviation is about to go back to its roots. Nearly 100 years after the Wright brothers’ first heavier-than-air powered flight, the US Air Force is testing an experimental plane that uses “wing warping”, which is the steering and control technique that kept Orville Wright aloft in 1903. But this time round, it will be at supersonic speeds. Unlike conventional aircraft wings, which use movable surfaces like flaps on the wings and the tail, wing warping bends the entire wing. The USAF call it “active aeroelastic wing” technology, and is investing $41 million in the project in the hope that it will lead to lighter, more manoeuvrable supersonic planes.

*** Like so many American waterways, the Chesapeake Bay, an enormous, semi-saline body of water that is treasured for its aquatic life, became badly polluted during the 20 th century. But it has regained much of its biological vitality since the early 1970s, thanks to concerted ecological stewardship. In this effort, the Conowingo dam has provided valuable assistance. Environmentalists are not often fond of dams, which have a habit of trapping migratory fish and disrupting sensitive water ecosystems not to mention looking ugly. But the Conowingo dam on the Susquehanna river in eastern Maryland is an exception. Since its construction in 1928, the Conowingo dam has not only generated electricity but also trapped vast amounts of sediment behind its imposing walls. This was not a function the builders had planned for, but in recent years its value has become clear. The water flowing past the Conowingo dam is much cleaner than it would otherwise be. Even America’s onceendangered national symbol, the bald eagle, can be seen perching near the dam, waiting to swoop down and seize a meal of fish by its talons.

*** More than half of astronauts suffer from space sickness, also known as Space Adaptation Syndrome. Symptoms include headaches, nausea, vomiting and poor concentration. The main cause of space sickness is disorientation caused by exposure to zero gravity conditions. The human body is used to a much stronger gravitational field on Earth and organs in the inner ear, along with canals that sense motion, tell the brain about the location of the limbs relative to the ground. In other words, they’re responsible for balance. Unfortunately, the signals from these organs in the inner ear go wrong in zero gravity, leading to space sickness. Over time though, the brain learns to ignore them and relies instead on visual clues, such as the position of the feet, to determine balance. Astronauts quickly readapt to Earth’s gravity within a few days and there are probably no long-term effects from this strange affliction.

*** Why do transformers hum? This is one of those questions which seems easy but has surprising hidden depths. The simple explanation is that electric currents create magnetic fields, and the alternating current of mains electricity used by transformers creates a magnetic field that changes at 50 cycles a second. This in turn triggers a regular motion of the metal molecules inside the transformer, known as magnetostriction. It’s this motion that makes the surrounding air vibrate, creating the hum. But why does the metal respond in this way? The answer lies in a property of the electrons in the metal known as ‘spin’ - a property which can only be explained by reference to Einstein’s theory of relativity, which is beyond the understanding of most of us.

*** Time travel has been a favourite science fiction theme ever since it was first used in H.G. Wells’s trailblazing novel “The Time Machine”. But not everything it describes is science fiction: travelling forward in time, for example, is a proven fact. Einstein’s theory of relativity predicts that an observer moving relative to Earth can leap into Earth’s future, and the effect has been confirmed using atomic clocks. Dramatic time warps require speeds close to that of light, which is possible in principle but would take a major feat of engineering, not to mention a lot of money. Going back in time is far more problematic. Relativity does not rule out an observer being able to make a journey through space-time and return to the past. But all scenarios so far discussed require exotic circumstances.

*** In a biography of Bardeen, recently published, he does not fit the popular stereotype of scientific genius, for he is surprisingly sane and ordinary. As far as character goes, he had several assets. To start with he was a notable team builder. Tenacious when it came to attacking problems, he had the gift of breaking a large problem down into smaller, more soluble parts and then reassembling the whole. As a teacher, his habit of stopping to think allowed his students to do so too. Government and industry valued his advice - according to one commentator, he helped Xerox to build one of the finest industrial laboratories in the world in the fields of organic and disordered solids during the late 1970s. But, perhaps, the most telling aspect of Bardeen’s character was his willingness to share the credit with others. For example, he deliberately stayed away from the meeting of the American Physical Society in March 1957, at which his theory of superconductivity was first presented, so that the contribution of his young co-researchers would be recognised.

ÜDS SAĞLIK 2002 SONBAHAR

*** An antibiotic that removes metals from the brain is emerging as a prime candidate for treating Alzheimer's. This boosts a controversial theory that blames the accumulation of metals, rather than the formation of insoluble plaques, for the disease's characteristic mental deterioration. The antibiotic, clioquinol, binds to copper and zinc and is small enough to get into the brain. It is no longer manufactured but was last used in the 1970s to treat intestinal infections. Now, in various medical schools, efforts are being made to resurrect the drug to treat Alzheimer's.

*** Some 54 million Americans have disabilities and that number has been . increasing. It includes more than 10 million children with developmental disabilities - a number accentuated by the fact that 50 per cent of children with disabilities are not identified until school age. Early identification of developmental disabilities is crucial, as it has been shown to improve , outcomes. A good example of this is autism, where early identification and treatment can lead a child to impressive gains in communication, social skills and learning. It's also clear that having a developmental disability puts one at risk for secondary conditions. The lack of recreational opportunities, for example, can lead to the development of obesity as well as withdrawal, depression and isolation. Again, early identification can help prevent these problems.

*** Artificial sweeteners permit people to keep their sugar and energy intakes down, yet still enjoy the delicious sweet tastes oftheir favourite foods and beverages. The Food and Drug Administration (FDA) has approved the use of four artificial sweeteners - saccharin, aspartame, acesulfame potassium (acesulfame-K) and sucralose. Two others have petitioned the FDA and are awaiting approval -alitame and cyclamate. Saccharin, acesulfame-K and sucralose are not metabolised in the body; they pass through the kidneys unchanged. In contrast, the body digests aspartame as a protein. In fact, aspartame is technically classified as a nutritive sweetener because it yields energy, but for all practical purposes, that energy is negligible. Some consumers have challenged the safety of using artificial sweeteners. Considering that all compounds are toxic at some dose, it is hardly surprising that large doses of artificial sweeteners (or their components or metabolic byproducts) have toxic effects. The question to ask is whether their ingestion is safe for human beings in quantities people normally use (and potentially abuse). The answer is yes, except in the case of aspartame, which may present a problem for certain people and so carries a warning on its label.

*** An ulcer is an erosion of the top layer of cells from an area, such as the wall of the stomach or duodenum. This erosion leaves the underlying layers of cells unprotected and exposed to gastric juices. The erosion may proceed until the gastric juices reach the capillaries that feed the area, leading to bleeding, and reach the nerves, causing pain. If Gl bleeding is excessive, iron deficiency may develop. If the erosion penetrates all the way through the Gl lining, a life-threatening infection can develop. Many people naively believe that an ulcer is caused by stress or spicy foods, but this is not the case - at least not at first. The stomach lining in a healthy person is well protected by its mucous coat. What, then, causes ulcers to form? Three major causes of ulcers have been identified: bacterial infection with Helicobacter pylori, the use of certain antiinflammatory drugs such as ibuprofen and naproxen, and disorders that cause excessive gastric acid secretion. The cause of the ulcer dictates the type of drug treatment. For example, people with ulcers caused by infection receive antibiotics, whereas those with ulcers caused by drugs discontinue their use. In addition, all treatment plans aim to relieve pain, heal the ulcer, and prevent recurrence.

*** Cataracts are age-related thickenings in the lenses of the eyes that impair vision. If not surgically removed, they ultimately lead to blindness. Cataracts occur even in well-nourished individuals as a result of ultraviolet light exposure, oxidative stress, injury, viral infections, toxic substances and genetic disorders. Many cataracts, however, are vaguely called senile cataracts - meaning "caused by aging". In the United States, morethan half of all adults 65 and older have a cataract. Oxidative stress appears to play a significant rolein the development of cataracts, and the antioxidant nutrients may help minimize the damage. Studies have reported an inverse relationship between cataracts and dietary intakes of vitamin C, vitamin E and carotenoids. Taking supplements of vitamins C and E seems to reduce the likelihood of developing age-related cataracts. *** The treatment of hypochondriasis has traditionally been difficult. Where it is secondary to another condition it usually resolves with treatment of the primary disorder. For example, hypochondriasis, which is secondary to depression, will usually resolve when the depression is successfully treated. Recognition and early diagnosis are important. Reassurance, combined with a serious appraisal of symptoms and an explanation of psychological factors may then be effective. It is not enough simply to tell the patient that there is nothing wrong. It is important to acknowledge the patient's distress and provide an alternative model, introducing psychological factors. A dismissive attitude that "it is all in your head" is counterproductive. Management regimes based on regular brief appointments with one key doctor and avoiding admission to hospital are useful. They have been found greatly to improve physical functioning and cut costs on investigations and hospital admissions by one third. *** The birth of a younger infant whilst an older child is in the toddler or preschool period is such a normal event that the proper handling of the situation should be regarded as an integral part of childcare. Put in its simplest terms, the problem is one of the older child being displaced by the new baby as the centre of attention and focus of his mother's affection. Such a displacement is inevitable, but its effects on the older child can either be helpful to his development (as when he accepts the baby with pleasure into his family circle and so takes a step forward from the egotism of babyhood) or harmful when jealousy predominates over any pleasure in having a brother or sister and the child becomes hostile to his mother and more demanding of her attention, returning to many of the practices of babyhood which he had outgrown. Thus a child or three or four may demand to sleep in his old cradle, to take milk from the bottle or breast, to be constantly carried, or may restart wetting the bed. Any or all of these symptoms may appear transiently in a child who subsequently adapts well to the new situation, but when they persist they indicate that the child is receiving less attention and affection than he needs. *** Morphine, which is given as a painkiller to many people with cancer, might stimulate the growth of tumours, say researchers in the US. Their worrying findings have been questioned by others in the field, but all agree that further studies are urgently needed to settle the issue. In test-tube experiments and in mice, Kalpna Gupta and her colleagues found that morphine encourages the growth of blood vessels, known as angiogenesis. The increased blood supply accelerated the growth of breast tumours in mice. Although the researchers have not yet looked for this effect in people, Gupta warns that morphine could be harmful for patients with any form of solid tumour that depends on a healthy blood supply. She stresses that nobody should yet consider altering their use of morphine because of her findings. "But clinical studies must be done," she says.

ÜDS SOSYAL 2002 SONBAHAR

*** Cinema today is all too often just about plot or special effects. So really great films, successfully weaving together sound, image and time to tell a really cinematic story, are always special. On show in New York earlier this month was a remarkable example. Alexander Sokurov's "Russian Ark" explores the 300 -year- long history of the Hermitage museum in St. Petersburg, It takes you an uncut 90 - minute walk through 35 of its historic rooms and halls. This has never been done -before and the result demonstrates impressively how much film can achieve.

*** The global market for coffee has failed, and needs a complete overhaul so that farmers can get more for their beans. The plunging price of coffee over the past decade has certainly caused regrettable misery for many farmers. Big coffee companies now stand accused of placing profits before people. If efforts are made to cut down on the profits, the companies will almost certainly fight back to keep up their excessive profits. It is likely that they will win because the trade barriers set by rich countries means that it is hard for coffee farmers to change over and grow other crops.

*** For more than 40 years the radical thinker William Philips edited Partisan Review, a magazine of small circulation and little money but with a great deal of influence. Writers and commentators whose words later commanded audiences of millions first saw their names in print in a publication that might sell 15,000 copies if things were going well. Mary McCarthy, Bernard Malomud and Saul Bellow were apprentice contributors. Leading European writers such as Jean Paul Sartre and Albert Camus were introduced to American readers through the Review. The magazine was defending T. S. Eliot, Franz Kafka and James Joyce long before their acceptance as central to modern culture.

*** When economists try to explain why the internet is more popular in one country that another, they usually point to factors such as the number of PCs. telephone lines or average years of schooling. But something less quantifiable may be more important: trust. This, at least, is the result of a recent study, which compared 17 countries. The Internet's anonymity and vastness encourage misrepresentation and fraud. Thus, people who are normally suspicious (and there are a lot of them) lend to shun the medium, while more trusting ones embrace it

*** Southern Africa's food crisis looks like being the worst in a decade. Around 14.5 million people are dangerously hungry, and many have been reduced to eating wild leaves and herbs . One might then expect food aid to be welcomed. But Zambia is refusing to accept American donations because much of its corn and soya is genetically modified. Zambia's president, Levy Mwanawasa, calls he stuff "poison" and refuses to import, despite a warning from the UN World Food Programme, on September 16th that relief supplies in his country could run out in two weeks.

*** Which European country has the worst record for shoplifting? The answer is Britain and she holds the record now for the second year running, according to a survey released on September 19th. Britons not only steal more than their continental counterparts, they are also less competent employees on the shop floor, resulting in Britain having the worst overall rate of retail "shrinkage" –a measure of losses by retailers from theft, mispricing and other wastage. Continental Europeans are actually not much better. Shrinkage is increasing alarmingly in some countries and is generally on the rise. Denmark is a notable example. According to one survey, shrinkage there is 9 % up on last year. Shrinkage costs the European economy a surprisingly large amount, in fact, the total annual cost has been estimated at around 830 billion which is equivalent to a shocking 880 per person in the region. That is more than the costs of the much-higher—profile car crime or domestic burglary.

*** Afghanistan's terrain, climate and tradition of gardening make it a good place for growing fruit. In the 1970s, export of fruits and nuts provided about 40 % of the country's foreign exchange. About 60 % of the world's dried fruit came from Afghanistan. In the 1990s, the Food and Agriculture Organization (FAO) helped to create over 300 fruit - tree nurseries throughout the country to replace orchards destroyed during a decade of occupation by the Soviet Union. Further fighting ruined more of Afghan agriculture, including the vineyards of the Somali plain. But it was the lack of water and the lack of money that devastated the country's orchards, most of which are now in a sorry state. Such fruits and vegetables that are being produced now are difficult to distribute or export because many of the country's roads and bridges have been destroyed.

*** Sweden maintained a position of neutrality during both World Wars and this, in part at least, enabled her to build up an elaborate structure of welfare legislation that many larger nations were later to imitate. The first major step was the establishment in 1911, of old-age pensions. Economic prosperity based on its neutralist policy enabled Sweden, together with Norway, to pioneer in public health, housing, and job security programs. Forty-four years of Socialist government were ended in 1976 with the election of a conservative coalition. Presently, the Socialists were again returned to power, only to be ousted in September 1991. The new coalition of four conservative parties promised to reduce taxes and cut back on the welfare state but not alter Sweden's traditional neutrality. Under them, in a 1994 referendum, voters approved joining the European Union. Although supportive of a European monetary union, Sweden decided not to adopt the euro when it first appeared in 1999.

ÜDS FEN 2003 İLKBAHAR

*** It seems that a programme designed to destroy Columbia's huge illegal drugs business could be poisoning farmers and damaging the environment. Backed by 1.3 billion of US government funds, drug enforcers routinely identify fields of coca plants and opium poppies, and spray them from the air with herbicide. Around 120,000 hectares have been sprayed with the herbicide "glyphosphate". But although glyphosphate is considered to be relatively safe for humans and the environment, the Colombian government has received over a thousand complaints from people who claim to have suffered ill effects after coming into contact with the chemical. Reported symptoms range from skin and eye irritations to coughing and vomiting. Some critics suspect additives to the spray are responsible. Others on the ground complained that the spray had killed food crops when it drifted onto them from nearby fields.

*** Can coal ever become a friend of the environment? Coal-fired power stations supply half the electricity used in America, and a similar amount in many other industrial countries, but are responsible for 80% of the power industry's emissions of carbon dioxide the most worrisome of the so-called "greenhouse gases". Because of special exemptions, much of the country's coal-derived electricity comes from plants that are more than 30 years old. Many of these plants are approaching the end of their commercial lives, and the thought of having to replace a lot of dirty old power stations, with new ones that will have to comply with the Clean Air Act, is causing a nightmare in the power industry. Suddenly, energy engineers are talking about "clean coal" technology. That message has been heard before. There was similar excitement over clean coal in the mid-1980s and early 1990s. Large sums of taxpayers' money were handed out to firms developing clean coal. The difference this time, say energy engineers, is that a number of electricity suppliers have actually started building facilities that use clean coal.

*** To understand topics such as the origin of the universe, the ultimate fateof black holes and the possibility of time travel, we need to understand how the universe works. We now have a good idea about what the basic building blocks of matter might be. Physics in the 20 th century was built on the twin revolutions of quantum mechanics (a theory of matter) and Einstein's theory of space, time and gravitation known as relativity. But it's extremely unsatisfying to find two ultimate descriptions of reality when you're looking for just one. Trying to unify the two theories presents formidable technical and conceptual obstacles that have challenged some of the finest theoretical physicists for decades.

*** When it is completed late next year, a 39-storey apartment building under construction in San Francisco will be the tallest precast concrete structure ever built in an area of high seismic activity. Its builders are using an innovative new structural connection thatcould revolutionize the way buildings are built in seismic zones. Conventional cast-in-place and precast systems survive earthquakes by dissipating the energy through the structure, often doing irreparable damage to themselves in the process. The new connection, developed with help from the University of Washington, consistsof high-strength steel reinforcing cables and "mild" steel bars that stretch slightly during an earthquake, then pull the building back into place. The steel components also considerably reduce seismic energy before it can attack the structure. This means less damage to beams, walls and ceilings.

*** In terms of pure science, the discovery that the universe is in the grip of a strange "anti-gravity" force that is making it expand ever faster, is the most significant of the last decade. The possibility that such a force might exist has been known for years, with theorists finding that it kept reappearing out of Einstein's theory of gravity. For years they tried to avoid it coming up with all sorts of arguments for why the force couldn't really exist. Now they're being forced to face it, and to face the embarrassing fact that they can't explain the single most important force in the universe.

*** Air-starved soil could have been a key player in the largest extinction ever tostrike Earth. The claim follows the discovery of a rare mineral in ancient soil collected from Antarctica. The extinction, at the end of the Permian period 250 million years ago, wiped out virtually all marine life and some 70 per cent of land animals. But the reason for the extinction, which preceded the rise of the dinosaurs, has been a longstanding puzzle. Now a team of geologists think they have found what could be a major factor in the extinction. They collected fossilized soil samples that formed in Antarctica just after the Permian period ended. The soil contained noduleshaped minerals that have been identified as berthierine. This iron-rich mineral forms only in environments where oxygen is scarce. So, if the oxygen levels in the soil were low enough to allow berthierine to form, then it follows that the soil would not have been able to support plant life. Such intolerably low levels of oxygen would be enough to kill the plants off completely.

ÜDS SAĞLIK 2003 İLKBAHAR

*** Transplants of foetal eye tissue from aborted foetuses seem to have improved the vision of two out of four people with a degenerative eye disease. It is too early to be sure the improvements are real and lasting, but on the strength of the results, the team pioneering the surgery has asked regulators for permission to carry out further operations. Before the experimental surgery on her left eye a year ago, Elisabeth Bryant, who is 63, could barely see anything with it. "Now I can see people's eyes, noses and mouths when they're sitting across the room from me. Like the other patients in the trial, she has advanced retinitis pigmentosa, a hereditary disease that causes degeneration of the retina. It affects around 1 in 3500 people in Western countries. Those involved in the transplants admit that there is a danger of creating false hope, but point out that the potential benefits of the procedure are so great that work on it must continue. They believe it could lead to a treatment for common diseases, such as agerelated macular degeneration, which is responsible for half the blindness in Britain. This is a condition which seems to be on the increase and occurring at younger age levels.

*** Today when you are ill, you may try three different medicines before finding one that works - and sometimes none work at all. But soon a simple test could determine which medicine would be most effective before you begin treatment, saving you time, money and possibly your life. Experts estimate that as many as 40% of people taking medication respond less than perfectly to it. The result is that 2 million Americans are hospitalized for adverse drug reactions each year; 100,000 die. With 20% complete and a rough draft of the other 80%, the Human Genome Project will help eliminate such adverse reactions. One of the first genetic tests to predict a patient's drug response is being developed in Sweden. Only about 30% of Swedes with high blood pressure respond to ACE inhibitors - a class of approximately 20 drugs that lower blood pressure. That means the other 70% continue to suffer from high blood pressure and are also exposed to the drug's side effects, which include difficulty in breathing, kidney dysfunction and dizziness.

*** Fungi that infect the skin live only in the dead, topmost layer and don't penetrate deeper. Some fungal infections cause no symptoms or produce only a small amount of irritation, scaling and redness. Other fungal infections cause itching, swelling, blisters and severe scaling. Fungi usually make their homes in moist areas of the body where skin surfaces meet: between the toes, in the groin and under the breasts. Obese people are more likely to get these infections because they have excessive skin folds. Strangely, fungal infections on one part of the body can cause rashes on other parts ofthe body that aren't infected. For example, a fungal infection on the foot may cause an itchy, bumpy rash on the fingers. These eruptions represent allergic reactions to the fungus.

*** What would conversation be like without hand gestures? Difficult, and in countries like Italy, perhaps unimaginable. It was her travels to Italy, in fact, that inspired Jana Iverson, a psychologist at the University of Chicago, to see whether we learn gesturing from others or if it is an innate part of speaking. She asked 24 children, 12 of whom had been blind from birth, to compare the amounts of water in two identical glasses, then compare them again after the water in one glass was poured into a dish. (The blind children explored the water and receptacles with their hands.) Asked how they arrived at an answer, both blind and sighted children used the same gestures as they spoke, including cupping one hand into a C shape and imitating the act of pouring. Blind children gestured even when talking to an experimenter they knew was blind. The fact that someone who had never seen gestures before would gesture", says Iverson, "even to a partner who they know can't see, suggests that gesturing and speaking are tightly connected in some very fundamental way in our brains".

*** What differences are there in intellectual function between men and women? Major sex differences in function seem to lie inpatterns of ability rather than in overall level of intelligence (measured as IQ), although some researchers have argued that there exists a small IQ difference favouring human males. Differences in intellectual pattern refer to the fact that people have different intellectual strengths. For example, some people are especially good at using words, whereas others are better at dealing with external stimuli, such as identifying an object in a different orientation. Two individuals may have differing cognitive abilities within the same level of general intelligence. Sex differences in problem solving have been systematically studied in adults in laboratory situations. On average, men perform better than women at certain spatial tasks. In particular, men seem to have an advantage in tests that require the subject to imagine rotating an object or manipulating it in some other way. They also outperform women in mathematical reasoning tests and in navigating their way through a route. Women, on average, excel on tests that measure recall of words and on tests that challenge the person to find words that begin with a specific letter or fulfill certain specific conditions. They also tend to be better than men at rapidly identifying matching items and performing certain precision manual tasks.

*** Recent research in Canada suggests that our arteries may narrow slightly when we breathe in the sort of traffic pollutants found in urban areas during rush hour. The small restriction in blood flow may not be a problem for healthy people, but it could be fatal for those with cardiovascular disease. Researchers at the University of Toronto asked healthy volunteers to inhale a mixture of ozone and particles less than 2.5 micrometers across for two hours. The width of their brachial artery, a large vessel in the arm, reduced by between 2 and 4%. Breathing ozone or particulates by themselves, or breathing filtered air, did not cause constriction. It is unclear how this effect is mediated, but there can be no doubt that heart disease and airborne pollution are deadly allies. In the US, for example, the Environmental Protection Agency has estimated that air pollution contributed to 60,000 heartrelated deaths in 1996. With the health stakes so high, environmental scientists have been given the task of measuring levels of different particulates in the atmosphere, and tracking them back to their source. A particle's behaviour is partly determined by its density. But its size is the most important factor, and the smaller particulates are, the more potentially dangerous they are to health.

ÜDS SOSYAL 2003 İLKBAHAR

*** Leonardo's importance as a painter may be summarized by saying that he was the first master of the high Renaissance style, and of all his scientific researches that which he most fully devoted to this end was his study of light and shade. Many changes came about in Italian painting around the turn of the 15th-16th century, for which he more than anyone else was responsible. As regards subject matter and composition, painting became less realistic and more classical but as regards treatment it became more naturalistic and closer to life, which was partly the result of increased knowledge of anatomy and an interest in light and shade. Inhis rare excursions into classical subjects, such as the "Leda", Leonardo's approach is intensely personal. Indeed, drawings by Leonardo which seem to derive from antique art are excessively rare and there is no evidence of his interest in the celebrated collections of Lorenzo the Magnificent or Pope Leo X, even though he was living in close contact with both of these. This trend is not belied by the "Last Supper" even though it epitomizes the spirit of classical art.

*** At first sight, doing away with paper and letting computers register votes seems an attractive proposition, at least in theory. Electronic votes can be easily counted and recounted. At a mock election held in Palm Beach, USA, for instance, it took no more than an hour to count all the votes. Ironically it is computer scientists, not officials, who are counseling caution. There is no way to verify that ballots are recorded, transmitted and tabulated properly, argues one computer-science professor. For one thing, it is theoretically impossible to determine whether computer systems are free from programming bugs. Many people feel that elections should not be paperless. They would like to see touch-screen systems connected direct to a printer to produce physical evidence of a voter's choice. Such a printout could be inspected by the voter and kept as evidence in case there were problems with the electronic voting system.

*** Chief imperial architect Sinan was in his eighties when he built the Selimiye, which marks the zenith of his lifelong endeavour to perfect the domed building. The Suleymaniye Mosque had taken him nearly to the zenith of his creative power, but Selimiye was the crown of his genius. The mosque was built to commemorate the conquest of Cyprus, perhaps Selim ll's sole military achievement. Either because no hilltop remained in Istanbul worthy of such a monument or perhaps as a gesture of loyalty to a city where Selim had spent nine years of his life, it was decided that the mosque should be built in Edirne. The chosen site was the hill of Sanbayir overlooking the city, and the acquisition of land commenced. In those years horticulture was one of the main sources of income in Edirne, and Sanbayir was covered with flower nurseries growing principally tulips. The story goes that a woman who owned a small tulip garden here objected to selling her land, and resisted official pressure for a considerable time. Finally, they took her to see Sinan, hoping the architect could persuade her. She agreed, but on one condition, that in some part of the mosque there should be a sign that there had once been a tulip garden. Sinan kept his word and had an inverted tulip motif carved on one of the marble pillars at the eastern corner of the muezzin's gallery in the centre of the mosque.

*** Norway was transformed by the discovery of oil and gas in the North Sea in the 1960s and its previously agricultural-based economy saw an explosion of wealth. But since then a fierce but seemingly misplaced sense of national pride appears to be preventing Norwegian companies from making sensible business decisions to capitalize on the natural advantages the country has been handed. State intervention has supported entrepreneurial business ever since, but now times are tougher. Companies in other parts of Scandinavia have realized that in order to keep ahead in a global economy, and win foreign market share, they not only need greater financial power but also partners with local knowledge. However, rather than taking a similar proactive approach, Norwegian companies are adopting a defensive stance and seem perplexed by a sudden wave of hostile takeover bids.

*** It is thought that sails were invented by the Egyptians around 3400 B.C. Harnessing wind power for sea transport was a crucial factor in the development of civilisation, enabling Egyptian sailors to carry the timber they needed from the thickly forested coasts of Lebanon, and later taking them to Cyprus where there were plentiful deposits of copper and to the coasts of Anatolia. Trade began to flourish in the eastern Mediterranean, but ofthe thousands of ships which came and went from Anatolian harbours, some were inevitably wrecked in storms. The ships and cargoes which sank to the seabed over the centuries are today providing illuminating evidence about maritime and commercial history. Since the first underwater excavations were carried out in Turkish coastal waters in 1960, much has been discovered about shipbuilding technology in antiquity and the commodities the ships were carrying. Undoubtedly, the most important wreck excavated so far is the Uluburun Wreck, which is the earliest of all, dating from the late Bronze Age. This ship sank 3300 years ago and is accounted one of the most important ten archaeological discoveries of the 20th century. The gold seal of Queen Nefertiti and a two-leaf wooden book, the oldest book in the world, are among the hundreds of remarkable finds.

*** The serious writer is an interpreter, not an inventor. Like a good actor, he is an intermediary between a segment of experience and an audience. The actor must pay some consideration to his audience: he must be careful, for instance,to face toward it, not away from it. But the great actor is the one who is wrapped up in the thoughts and feelings of the role he is playing, not the one who is continually stealing glances at the audience to determine the effect of his last gesture or bit of business. The actor who begins taking his clues from the audience rather than from the script soon becomes a "ham": he exaggerates and falsifies for the sake of effects. The writer, too, though he must pay some consideration to his reader, must focus his attention primarily on his subject. If he begins to think primarily of the effect of his tale on his reader, he begins to manipulate his material, to heighten reality, to contrive and falsify for the sake of effects. The serious writer selects and arranges his material in order to convey most effectively the feeling or truth of a human situation. The less serious writer selects and arranges his material so as to stimulate a response in the reader.

ÜDS FEN 2003 SONBAHAR

*** The Michigan factory of a leading car manufacturing company already has its place in industrial history. It is famous primarily on account of record-breaking models produced there between 1927 and 1931. Now the refitted factory is back in the limelight as one of the world's most architecturally advanced buildings. The height of new technology will be its 42,000 square meter roof. Dubbed a "living roof by the company, it consists of a ground-hugging plant called sedum which is growing in a 7.6 cm-thick matlike material. The sedum insulates the roof, reducing heating bills by as much as five per cent and saving on replacement costs. It lasts twice as long as a standard roof because it doesn't shrink or expand when the temperature varies. It is also expected to become a habitat for butterflies and birds. Embedded in the roof are 36 skylights that let in natural light. On sunny days the factory will operate with up to half of its lights switched off. The company also intends to clear the soil around the factory to remove the chemical by-products of years of steel manufacturing. The car company will plant native bushes, flowers and trees that will break down polyaromatic hydrocarbons.

*** The amount of engineering and exertion required to do work in space came as a surprise in the early days of the manned space program. For instance, when the astronauts Eugene Ceman and Thomas Stafford launched into space aboard Gemini 9 on June 3rd, 1966, they had no way of knowing that a nightmare would begin as soon as Ceman began a space walk. From the moment he emerged from the capsule, everything Ceman did was much harder than he had expected. Every weightless movement triggered an equal opposite reaction, and he found himself repeatedly flying out to the end of the umbilical cord connecting him to the Gemini capsule and then rebounding in an unexpected direction. Stafford finally ordered Ceman to forget about the 10million-dollar backpack and return to the capsule. Doing so turned out to be the most alarming part of the space walk, as Ceman discovered, that his pressurized suit wouldn't flex enough to allow him back inside so that operation alone took him thirty laborious minutes. Then the struggle to close the hatch was so prolonged and difficult that Stafford decided he needed to lie, so the ground crew would not panic. "Coming in, no problem" he fibbed as he and Ceman improvised a lever to force the latch into position. It finally closed.

*** Ancient Babylonian manuscripts, Dark Age chronicles and New Age prophecies, all speak of catastrophes striking the Earth with devastating consequences; and all regard these disasters as instances of divine punishment. Many describe global inundations floods that engulf continents, wiping out entire civilizations. Others describe fires that rain down from the skies. Until recently few scientists took these tales of catastrophe seriously, regarding them as hangovers from a superstitious age. But today these ancient accounts are regarded as potentially valuable evidence for events that have played a key role in the history of our planet, from its very formation 4.5 million years ago. These events are every bit as catastrophic as the legends claimed: cosmic impacts that have dealt severe blows to life many times in the past - and will do so again. The dramatic view of Earth history now emerging could hardly be more different from that held by many scientists even as recently as the 1980s.

*** From its birth during World War II, there is one task to which cutting-edge computer technology has always been applied: codebreaking. In 1943, mathematicians and engineers at Bletchley Park, Buckinghamshire, built Colossus, the forerunner of the modern electronic computer. Its task was to break the Lorentz ciphers, used by Hitler and his generals for their most secret communications. Consisting of thousands of valves, optical devices and pulleys, Colossus looked ungainly but worked brilliantly. One of its major triumphs was to reveal that Hitler had been taken in by Allied deception tactics leading up to D-Day, thus clearing the way for the real invasion in 1944. Ever since, government codebreaking centers have bought the fastest supercomputers the moment they emerge, During the Cold War, the key challenge was to find and exploit the slightest weakness in the cipher systems used by the Soviet Union, acknowledged to be the most secure in the world. Their efforts ultimately led to the downfall of all of the Soviet's "Magnificent Five" British spies.

*** Central heating, available in the US since the early 19th century, became popular only after the Civil War. Typically, coal-burning furnaces fuelled the early systems. The furnaces warped and cracked, causing gases to escape, and had to be stoked frequently. It took years and countless small improvements, but by the mid1920s the systems had become reliable and, with the emergence of oilfired furnaces, more convenient. Natural gas, which became widely available with the building of a pipeline infrastructure after World War II, had developed into the leading fuel by 1960. Its acceptance resulted in part from its versatility - unlike oil, it can power appliances such as clothes washers and dryers, ovens, ranges and outdoor grills. Because it comes primarily from US and Canadian fields, natural gas is also less vulnerable than oil is, to war and embargo. Oil remains the predominant fuel in a few areas, such as New England, where natural gas pipelines have not yet thoroughly penetrated. Oil users in many regions have the advantage of being able to buy a season's supply in advance at favorable rates and, in contrast to most gas users, can easily change their supplier.

*** The US company, "Scientific Applications and Research Associates" (SARA) Inc, has developed crowd-dispersal weapons for the US military that generate high-intensity sound waves. Sound levels of 120 to 130 decibels cause pain and blurred vision; above 140 decibels, sound produces profound discomfort throughout the body. Ear plugs are no protection. These acoustic systems have a directed beam, projecting the sound in a narrow fan. They include high-intensity sound generators with power levels of up to 60 kilowatts and are combustion driven, using a series of small explosions to create a noise or drive a siren or whistle.

ÜDS SAĞLIK 2003 SONBAHAR

*** Dark-skinned people require longer sunlight exposure than lightskinned people: heavily pigmented skin arrives at the same plateau of vitamin D synthesis in three hours as fair skin in 30 minutes. The ultraviolet (UV) rays of the sun that promote vitamin D synthesis are blocked by heavy clouds, smoke or smog. Differences in skin pigmentation and smog may account for the finding that darkskinned people in northern, smoggy cities are more prone to rickets. For these people, and for those who are unable to go outdoors frequently, dietary vitamin D is most important. Deficiency is especially likely in older adults because they typically drink little or no milk, their exposure to sunlight is limited, and the skin, liver and kidneys lose their ability to make and activate vitamin D with advancing age. Depending on the UV radiation used, the UV rays from tanning lamps and tanning booths may also stimulate vitamin D synthesis but the hazards outweigh any possible benefits. If the lamps are not properly filtered, people using tanning booths risk burns, damage to the eyes and blood vessels, and skin cancer.

*** In the search to heal wounds without leaving a scar, researchers have looked at some 3,000 treatments. Many of these treatments have not lived up to expectations, and none can induce repair that leaves the skin in pristine condition. Now US and British scientists have come up with three different recipes for advanced bandages that give the repair of injured skin a quick start, but then break down, leaving behind only healed tissue. This type of bandage eliminates the need to change dressings, cuts the risk of infection and sometimes makes scariess healing possible. When skin is injured, the weave-like structure of collagen fibres is destroyed. To minimize blood loss and infection, the body produces a quick fix by using cells, called fibroblasts, which lay down thin linear strips of replacement collagen. When skin cells grow on the replacement collagen, they produce pale, less flexible material. Avoiding this scar tissue means getting the body to rebuild the complex fibrous structure of the original.

*** Despite millennia of preoccupation with every facet of human emotion, we are still far from explaining in a rigorous physiological sense this part of our mental experience. Neuroscientists have, in modern times, been especially concerned with the neural basis of such cognitive processes as perception and memory. They have for the most part ignored the brain's role in emotion. Yet in recent years, interest in this mysterious mental terrain has surged. Catalyzed by breakthroughs in understanding the neural basis of cognition and by an increasingly sophisticated knowledge of the anatomical organization and physiology of the brain, investigators have begun to tackle the problem of emotion. One quite rewarding area of research has been the inquiry into the relation between memory and emotion. Much of this examination has involved studies of one particular emotion - fear -and the manner inwhich specific events or stimuli come, through individual learning experiences, to evoke this state. Scientists have been able to determine the way in which the brain shapes how we form memories about this basic, but significant, emotional event. We call this process "emotional memory".

*** General levels of health in developed societies have steadily improved over the last hundred years or so, largely because of the introduction of public health services, and, indeed, the welfare state. This progress, however, seems to bear little relationship to the delivery of medicine. While scientific medicine has generated some solutions, it has also produced deadly problems. Crucially, it has led to a virtual takeover of healthcare by pharmaceutical corporations and an acceptance that pharmaceutical medicine is primary and best. Yet the extent of the damage done by drugs is enormous. In the US in 2000 it was estimated that there were over two million severe adverse drug reactions, of which 106,000 were fatal. Similar calculations suggest that in England, adverse drug reactions are the third most common cause of death. Drug companies have always supported clinical medicine. Medicine's high-ranking professional bodies were themselves set up with help from pharmaceutical companies, and still receive grants for running costs, hospitality, building and printing. Given this funding, is it surprising that reports sponsored by such bodies so often criticize the use of vitamins, food supplements, nutritional and alternative medicine? Today, however, the corporatization of medicine is gaining speed and reaches far beyond the professional institutions of medicine.

*** "Placebo" is a Latin word which means "I will please", in medicine it has come to mean a treatment - a sugar pill or saline injection - that, in itself, has no physical effect. In spite of this, placebos have often been found to relieve physical symptoms such as pain, swelling, inflammation, or even depression. Apparently, in such cases, it is the act of treatment rather than the treatment itself that is effective. Why there should be such a thing as the placebo effect remains problematic. Many doctors have suggested that it was selected by evolution as a means of managing the resources of the immune system. Given that placebos seem to work, in part, by suppressing the immune response, they may be a way of conserving the body's energy at critical moments. Patients may even get depressed to stop them from being too active and so slowing recovery. It is a high price to pay, but then evolution is not a merciful master.

*** According to the World Health Organization (WHO), around one in four people suffers from mental health problems at some stage in their lives, with depression being the single most common form. In Britain, suicide now accounts for around 20 per cent of all deaths among young people, with attempted suicide having increased by 50 per cent since 1990. Yet despite such stark statistics, the issue of mental fitness remains way off the agenda. Instead, people are relying on medication more than ever before which is like taking painkillers hoping they will turn you into a long-distance runner. How much better it would be if there were some way of training ourselves to become mentally as well as physically fit. Now, it seems, there is. It is a technique that has been around for a long time, but only very recently has hard scientific evidence emerged to back claims for its effectiveness. It does not involve any special equipment or special diets, and best of all it is free. It is called meditation.

ÜDS SOSYAL 2003 SONBAHAR

*** The ability to recognize people automatically by analyzing bodily characteristics such as fingerprints, faces and eyeballs, collectively known as biometrics, has long been a goal of technologists and governments alike. Plans for large-scale projects to incorporate biometric scans into passports, identity cards and visas are now under way in several countries. Since January 5 th the US has been scanning foreigners from particular countries as they arrive at its airports. Both the US and Europe plan to start issuing biometric passports as soon as next year. Biometric identity cards are being adopted in Hong Kong and Oman, and Britain plans to follow suit. Biometric technology has been around for quite a while, but has not been widely adopted, for good reason: while it can improve security in some situations, its costs more frequently far outweigh its benefits. Even the most advanced systems falsely reject a small proportion of legitimate users, and falsely accept illegitimate ones. At best, the introduction of biometric identity documents will produce a marginal increase in security, at enormous cost. But at worst, biometrics could, in fact, reduce security in several ways.

*** People are fascinated by juries. They are the focus of a disproportionate number of novels and movies, and the 1995 murder trial of O.J. Simpson virtually brought the United States to a standstill because people could not miss the exciting televised court proceedings. Juries represent one of the most significant decision-making groups, not only because they are presented as a symbol of all that is democratic, fair and just in a society but also because of the consequences of their decisions for defendants, victims and the community. A case in point is the 1992 Los Angeles riots, which were sparked by an unexpected 'not guilty' verdict delivered by an all-white jury in the case of the police beating of a black suspect. Juries are groups and thus are potentially prey to the deficiencies of group decision-making. In addition to these problems, there are a number of issues to do specifically with the task confronted by juries. One issue is the influence of laws and penalties on the jury. Harsh laws with stiff penalties tend to discourage juries from convicting, which is quite the reverse of the intention of legislators who introduce such laws. Juries have to deal with enormous amounts of information presented in court. It has been suggested that information delivered later in the trial is more heavily weighted in decision-making. Further, inadmissible evidence,that is the evidence given by witnesses or interjected by counsel but is subsequently ruled to be inadmissible for procedural reasons by the judge, still has an effect on jury deliberation.

*** A growing percentage ofthe American economy and of other advanced industrial economies in Europe and Asia depends on imports and exports. Foreign trade, both exports and imports, accounts for a little over 25 per cent of the goods and services produced in the United States, and even more in countries such as Japan and Germany. This percentage will grow in the future. The success of firms today and in the future depends on their ability to operate globally. Globalization of the world's industrial economies greatly enhances the value of information to the firm and offers new opportunities to businesses. Today, information systems provide the communication and analytic power that firms need for conducting trade and managing businesses on a global scale. Controlling the far-flung global corporation, which includes communicating with distributors and suppliers, operating 24 hours a day in different national environments and servicing local as well as international reporting needs, is a major business challenge that requires powerful information system responses. Globalization and information technology also bring new threats to domestic business firms: because of global communication and management systems, customers now can shop in a worldwide marketplace, obtaining price and quality information reliably, 24 hours a day. This phenomenon heightens competition and forces firms to play in open, unprotected worldwide markets. To become effective and profitable participants in international markets, firms need powerful information and communication systems.

*** Smoking has become unfashionable in most Western countries over the past decade or more, yet its incidence remains high. Even legislation against smoking in a shared work space and banning it on public transport has had limited success, when measured by a decline in the percentage of people still addicted to smoking. In these countries, the highest rates of smoking tend to be found among people in the 20-29 age group, teenage women and working-class groups. Smokers tend to be well-informed about illnesses related to their habit, such as lung cancer and heart disease. Despite this knowledge, current smokers tend to underestimate the risk of dying from smoking when compared with former smokers and those who have never smoked. Antismoking campaigns have used a wide variety of media and techniques to discourage smoking. For example, one campaign adopted a television commercial and poster, while another used a direct-mail approach, along with radio advertisements. Various celebrities have helped by performing at places of work and by recording verbal messages. There have been different target groups. One campaign aimed to reach women, who outnumber men in the under-18 smokers group, stressing the benefits of not smoking with respect to health, beauty and fitness. Another used baby stickers. Another campaign highlighted the benefits of a smoke-free workplace and was conducted in major clothing chain stores, supplemented by radio and television advertisements. There is a socially supportive context nowadays to quit, and the recognition that passive smoking is dangerous may help some in the future to quit permanently.

*** Gauguin began his career as a painter late in life. Born in Paris in 1848 he was raised by his widowed mother in Peru, where his paternal half Peruvian grandmother lived. "You know. I have Indian blood, Inca blood, in me" he later wrote, "and it's reflected in everything I do". At 17, Gauguin joined the French Navy and sailed around the world, encountering many native cultures during the ship's ports of call. In 1872, he married a Danish woman named Mette Gad and they settled in Paris, where he worked as a stockbroker for a decade. All the while, Gauguin collected contemporary art by the impressionists -in particular Renoir, Monet, Pissarro, who later became a friend, and Cezanne, whose primitive style and sweeping colors affected him deeply. In 1883, at 35 Gauguin abruptly left his wife, his five children and his bourgeois lifestyle to devote himself to art. He traveled to Brittany, where he painted and sculpted, and to Martinique, where he discovered the vibrant beauty of the tropics. Then he moved to Aries, in Provence, where he and his friend Vincent van Gogh painted colorful landscapes and planned their escape to Tahiti, which they imagined as exotic, bountiful and free of stifling European mores.

*** The typical African-American seems uninterested in saving for retirement. Statistics show that in recent years Americans in general have become increasingly less disciplined about saving. However, this trend is disturbingly prevalent among black Americans. Only 59% of AfricanAmericans say they or their spouses have saved for retirement, compared with 77% of white workers, according to a recent survey. What are the reasons for this disparity? Historically, there's been a cultural disconnection between the retirement portrayed in the glossy brochures - lavish vacations in high-priced villas - and the African-American reality. Most blacks expect to go on working even after they have officially retired, rather than lying around on a beach. Their experience in retirement has never been like mainstream white America. "Typically", they say, 'we work until we die". Researchers have often attributed the savings gap for African-Americans to lower wages or to a lower comfort level with the stock market. Those factors do play a part, but even educated professional blacks earning sixfigure incomes tend to ignore their retirement and follow a wasteful way of life. Economists say the behavior mirrors that of first-generation immigrants who engage in conspicuous consumption. "Black professionals today are often first-generation college graduates who, like other groups, want to symbolize that they have succeeded."

ÜDS FEN 2004 İLKBAHAR

*** A curious form of renewable-energy generation may be on the horizon - with underwater turbines. These units harness the power of tidally driven currents, which flow back and forth like clockwork, making it possible to generate electricity on a predictable schedule. In this respect, underwater turbines are more attractive than their wind-driven counterparts, which are now employed widely to help power electric grids. Underwater structures are also less likely to be damaged by violent storms which have little effect on submerged objects. And using underwater turbines sidesteps the common objections to conventional hydropower - that damming a river stops migrating fish and inundates land upstream. Underwater turbines have long been used on a small scale. But soon now they will be used on a large scale to produce megawatts of electric power.

*** Biologists value marine organisms because their primitive systems are good models for more complex organisms, such as humans- Despite being genetically further from us than more common research mammals, we share a surprising number of evolutionary links with these basic animals. By understanding their life processes and the way their systems react when things go wrong, scientists hope to continue making important discoveries that could help increase the chances of finding cures for serious diseases such as cancer and AIDS. As research continues, understanding increases about how our bodies and minds work, but there is still a lot to learn. Obviously it takes a long time to work out all the mechanisms, and there are still many new bioactive compounds to be found in marine organisms. But by using sea creatures to better understand human characteristics and disorders, progress is being made.

*** Architects and engineers engage in design quite explicitly, and they typically do so with distinct objectives. Architects tend to focus on form over function, whereas engineers tend to do the opposite. For most architects, the design of a building has firstly to do with how it looks, both inside and out, and how it fits in with nearby buildings. Architects are also expected to give considerable thought to how the building will be used, how people will move through it, how it will feel, although such considerations do not always seem to be foremost in their minds judging by results, indeed, if architectural criticism is taken at face value, architects do seem to be principally concerned with the texture of a building's facade, the appearance of its public spaces, the furniture with which it is filled. Architects seem to pay close attention to details, even down to the nature of the lighting fixtures and the hardware on doors and windows, but not always to how they will be operated or how they will fulfill their purpose. Nevertheless, such considerations collectively constitute architectural and interior design.

*** Many people who are perfectly relaxed driving along the highways become nervous when they get on an airliner, although most knowfull well that flying is safer than driving. The statistics are indeed clear on this point. For example, a paper published in 1991 documented the substantially lower risk of flying compared with driving in the United States. Some of the many millions of Americans who flew over the next few years probably derived comfort from such hard facts. But now, a decade later, things have changed. The hijacking of four large jets on September 11, 2001, and the disastrous events that ensued led many to avoid flying in the United States during the following months. For example, in the fourth quarter of 2001, there was a drop of 18 per cent in the number of passengers compared with the same time period in 2000. Many still avoid air-travel. Accordingly, it would be appropriate to again calculate the risks involved in flying and driving, taking into account the latest statistics, including the tragic deaths of the passengers on those four hijacked planes.

*** Electric heating, which appeared in less than 1 per cent of homes in the United States in 1950, now dominates most areas with mild winters and cheap electricity, including the South and the Northwest. Its popularity, at least in the South, was spurred by the low cost of adding electric heating to new houses built with airconditioning. In the Northeast and Midwest, electricity has not been a popular fuel because of its high cost for cold-weather heating and because it delivers heat at 90 to 95°F, compared with 120 to 140°F for gas and oil, which many in cold climates find preferable. In some areas, such as California, electric heating has not progressed because of building code restrictions. Bottled gas, which is somewhat more expensive than utility gas, is the fuel ofchoice in rural areas not served by utility pipelines. Wood, the dominant fuel throughout the US economy until the 1880s, is the leading heating fuel in just a few rural counties. Home heating, which accounts for less than 7 per cent of all energy consumed in the US, has had a commendable efficiency record: from 1978 to 1997, the amount of fuel consumed for this purpose declined 44 per cent despite a 33 per cent increase in the number of housing units and an increase in house size. This improvement came about thanks to better insulation and more efficient equipment following the energy crisis of the 1970s.

*** The American chestnut was once the most common canopy tree in the deciduous (non-evergreen) forests of the eastern United States. It shaded areas from New England to Georgia until the fungus Cryphonectria parasiticawiped out the species in the terrible forest disaster of the early 20th century. The fungus continues to kill chestnuts before they can mature. The vacuum left by the chestnut's destruction is now filled with other species. Considerable research has gone into understanding what happened to the American chestnut. But some scientists remain interested in getting to the root not of its destruction, but rather of the centuries of dominance enjoyed by the massive, fragrant and economically important tree.A team of scientists recently put forward the idea that the American chestnut may have been engaged in the chemically charged competition known as allelopathy. An allelopathic plant releases potentially toxic substances into the environment through its roots, its leaves or processes such as evaporation. Black walnut, sycamore and sassafras trees are just a few known allelopaths that limit the germination of competitor. It's likely that the list will soon branch out to include chestnut. ---

ÜDS SAĞLIK 2004 İLKBAHAR

*** For a drug to work, it has to get to the place in the body where the problem lies, and that's why the science of pharmacokinetics is important. Enough of the drug has to stay at the site of action until the drug does its job, butnot so much that it produces severe side effects or toxic reactions. Every doctor knows that selecting the right dose is a tricky balancing act. Many drugs get to their site of action through the bloodstream. How much time these drugs need to work and how long their effects last, often depend onhow fast they get into the bloodstream, how much of them gets into the bloodstream, how fast they leave the bloodstream, how efficiently they're broken down by the liver, and how quickly they're eliminated by the kidneys and intestines.

*** In this experiment, patients were placed into a refrigerated chamber for several minutes, and measurements were taken of both their lung capacity and the temperature within their trachea. On one of the experimental study days, the subjects were asked to breathe entirely through their nose; on the other, their nose was clipped tightly and they breathed through their mouth. The patients' lung capacity dropped significantly on the mouth-breathing day ofthe experiment; this correlated highly with a significant cooling of the trachea. On the day they breathed only through their nose, however, their lower airway function changed only minimally and their intra-tracheal temperature remained close to normal. These studies demonstrate that nasal breathing has an important protective effect on the lower airways, most likely because the nose is more effective at warming and moisturizing inspired air before it passes into the lungs. In addition, a nose that is clear and free of inflammation is better able to filter and remove airborne allergens and pollutants than is the mouth.

*** The Norwegian-born psychologist Lovaas devised a highly structured form of behaviour modification called "Applied Behavioural Analysis" (ABA) as an aid for autistic children. The theory behind ABA rests on the assumption that autistic children have inefficient neurocircuits - specifically, connections that are not as efficient as those found in normal children. In normal children, a connection may go from A to B, but in the autistic brain the connection may go from A to C to D to E before finally reaching B. Inefficient connections result in more "noise", which is believed to hinder social and cognitive development. Lovaas's therapy takes advantage of the brain's ability to adapt and be retrained. When people learn a particular skill, such as throwing a baseball, they are reinforcing neurocircuits that are specific to that skill. When autistic children are taught a specific behaviour through constant repetition,the therapy is training the neurocircuits to respond in a certain way and somehow teaching the brain to receive these signals, which would otherwise be drowned out in a sea of noise. In order to be effective, Lovaas has stressed that ABA therapy must be started early, must be sufficiently intensive, and must be carried out in part by the parents.

*** A British businessman, David Green, was determined to provide India with good-quality, lowprice lenses for use in cataract surgery. He got a group of expert engineers to design lenses that could be produced cheaply, and got support to start a lens factory, as part of Aurolab, in India. Now, the typical eye surgeon in India does 2,500 cataract operations a year as opposed to 125 or so in the US. Thanks in part to such a steady demand and to India's cheap labour, Aurolab today can sell roughly 700,000 lenses a year for as little as $4 each and still make a profit. Aurolab, which operates as an independent nonprofit organization, made money from the beginning, it reinvests all proceeds into expanding its operations and designing new products, such as sutures and eyeglasses. But even at $4 a lens, how can the very poor afford eye surgery? At this point the Aravind hospital provided the help that was needed. It set up a scheme in which patients pay whatever they can. Those who are relatively well-off pay more for their surgery, subsidizing the very poorest, who pay nothing. Even with these subsidies, the Aravind hospital makes $2 for every dollar it spends on cataract surgery. This has enabled it to open up five more hospitals, catering to some 100 million Indians.

*** The federal Elderly Nutrition Programme is intended to improve older people's nutrition status and enable them to avoid medical problems, live at home, and stay out of institutions. Its specific goals are to provide low-cost, nutritious meals, opportunities for social interaction and shopping assistance. The Elderly Nutrition Programme provides for communal meal programmes. Administrators try to select sites for communal meals so as to feed as many eligible people as possible. Volunteers may also deliver meals to those who are homebound either permanently or temporarily; these efforts are known as Meals on Wheels. The home-delivery programme ensures nutrition, but its recipients miss out on the social benefit of the communal mealsites; every effort is made to persuade older people to come to the shared meals, if they can. All persons aged 60 years and older and their spouses are eligible to receive meals from these programmes, regardless of their income. However, should demand exceed supply, priority is given to those who are economically and socially needy. An estimated 25% of the nation's elderly poor benefit from these meals every day of the week.

*** A chemical naturally produced in the body that works in a similar way tocannabis is thought to hold the key to treating obesity. A recent study found that oleylethanolamide (OEA) significantly decreased the appetites of lab rats, with a resulting loss of weight. Scientists are just beginning to understand OEA, which they believe is manufactured in the small intestine. It is a naturally occurring molecule in the same class of compound as cannabis' active ingredient and acts on the same receptors. However, rather than increasing the appetite,OEA has the opposite effect. The chemical did not affect various functions of the rats', nervous systems as other weight loss drugs do. "OEA is like a cousin to cannabis - it has similar properties but a different personality. In this case, it creates the opposite reaction", says the head researcher. OEA could solve the problem of obesity, which greatly increases the risk of diseases such as diabetes, heart disease, stroke and some cancers.

ÜDS FEN 2004 SONBAHAR

*** Florida's more than 1,000 miles of coastline contain about 150 drawbridges, more than are found in any other state. As these bridges age and Florida grows and changes, many of them now have to be replaced. But replaced with what? Recently there have been very many controversies over drawbridges. One involved the Belleair Beach Causeway, which was built in 1950. When it approached the end of its 50-year lifespan, the choice for a replacement came down to three options: a drawbridge the same height as the existing span (21 feet above high tide) but wider, to accommodate modern traffic levels; a drawbridge with a height of 45 feet; and a fixed span 65 feet tall. Each choice had its advantages and disadvantages. The two drawbridges would have created the least disruption in nearby communities. Opening a drawbridge inconveniences motorists, however, and can cause a dangerous delay for ambulances or in other emergencies. A drawbridge also costs more to build and maintain. A high fixed span would be cheaper and eliminate the delays, but it would require extensive property acquisition to accommodate its approaches. It would also tower over the area and block the views of many residents, and some tall boats would be unable to fit underneath. However after much debate and many hearings, the country board settled on a high fixed span.

*** At one point, during what was still a time of hopeful experimentation with nuclear technology, the US considered using atomic bombs to blow a trench through the isthmus of Panama. The idea was to replace the 50-year-old Panama Canal, whose locks were too narrow for the world's growing fleet of super-tankers and aircraft carriers, with one that would be more suited to modern conditions. The problem was not just the size of the locks but their very existence. Ships don't simply sail through the Panama Canal; they have to be lifted 85 feet to the elevation of the highest point along the way and then brought down to sea level again. Each ship's trip through requires 52 million gallons of fresh water, more than most American cities use in a day, all of it flowing through huge gravity-fed tubes. The source of this water supply is a vast artificial lake whose dam also provides the electrical power for the whole operation. To fill up and empty a single lock takes 40 minutes on average, and when any one of the 12 locks shuts down for maintenance, ship traffic can back up for days. Finally, with all that complicated machinery, if the canal were to be sabotaged (as had happened to the lockless Suez Canal in 1956), correcting the damage might take years.

*** A new and quicker method to check or prove the existence of weapons of mass destruction (WMD) is being developed. First, one needs to know where they may be hidden and then a high-velocity projectile can be fired at the target. High-tech sensors packed into the projectile will then instantly beam back confirmation that the weapons are there if indeed they are. It's a high-risk concept that raises many questions, not least its technological feasibility and the political protests that would follow if such a device were ever built or used. But the US military is taking the idea seriously. Last year, in a two-page research paper commissioned by the army, experts from the Institute for Advanced Technology detailed real test results of a prototype projectile designed to verify the existence of WMDs. They say such a device offers a way to inspect for weapons without permission or cooperation. To inspect reinforced concrete bunkers or factory buildings suspected of housing WMDs, the researchers designed a projectile that can penetrate several meters of hardened concrete, without damaging its load of sensors. Its casing is built from AerMet 100, a nickelcobalt steel with traces of molybdenum and chromium. Heat-treating the casing after it is made gives it an extremely hard surface. The tapering projectile is 230 millimeters long, with a maximum calibre of 45 millimeters.

*** If the solar company's claims are borne out, its high- efficiency cells will bring solar power closer to becoming a practical option for home owners. The average power demand of a household is 2,000 to 3,000 watts. With solar cells around 20 per cent efficient, this demand can be met with about 15 square meters of solar panels, which is compact enough to fit on a rooftop. It has been estimated that ready-to-install arrays will sell for around $10 per watt. That cost may eventually fall further. For, as one spokesman for the company said: “The trend is towards higher efficiency". Others agree, but say silicon may not be the material that ultimately delivers it. Thin films of cadmium telluride, for instance, are showing promise in the lab. One London-based solar energy systems supplier is very impressed with some new solar cells that have just come on the market, both on account of their efficiency and also because they are practical. They are very easy to connect together," he points out, you can just put tabs on the sides of the back contacts and connect one cell to another without wires."

*** To engineers, design typically has less to do with aesthetics and appearance and more to do with fabrication and performance. Engineers tend to focus on the structure behind the façade. They worry about how the building will be built, how it will stand, whether it will sway too much in the wind, whether it will survive an earthquake, whether it will crack or leak. Engineers designing the structural frame of hotel buildings take into account the strength and stiffness of ballroom floors, where large crowds will gather and rhythmic dancing will occur. Engineers are expected to think about how a building will be heated and cooled, how air will circulate among its spaces, how energy efficient it will be. In the ideal world, the design efforts of architects and engineers complement each other, resulting in a building that is both a joy to look at and a pleasure to use. But all too often in practice, things do not work out like that, and the users of the building pay the price. In most buildings, the work of the architect masks, cloaks and hides the work of the engineer. Engineering criticism is almost unheard of in public discussions of building design, although it does sometimes come to the fore when buildings fall down, as in the case of the collapse of the World Trade Center towers.

*** Smoke is clouding our view of global warming, protecting the planet from perhaps three-quarters of the greenhouse effect. That might sound like good news, but experts say that, as the cover diminishes in coming decades, we're in for a dramatic escalation of warming that could be two or even three times as great as official best guesses. This was the dramatic conclusion reached last week at a workshop in Berlin. Scientists have suspected for a decade that aerosols of smoke and other particles from burning rainforest, crop waste and fossil fuels are blocking sunlight and counteracting the warming effect of carbon dioxide emissions. Until now, they reckoned that aerosols reduced greenhouse warming by perhaps a quarter, cutting increases by 0.2°C. So the 0.6°C of warming over the past century would have been 0.8°C without aerosols. But the Berlin workshop concluded that the real figure is even higher -aerosols may have reduced global warming by as much as threequarters cutting increases by 1.8°C. If so, the good news is that aerosols have prevented the world getting almost two degrees warmer than it is now. But the bad news is that the climate system is much more sensitive to greenhouse gases than previously guessed. In fact, warming could go up by 7 to10°C.

ÜDS SOSYAL 2004 SONBAHAR

*** The German economy isn't getting any healthier. Growth remains stagnant and businesses remain pessimistic about the future. Real gross domestic product in the third quarter grew by 0.3%, after a 0.2% rise in the previous quarter. Household spending was the main engine of growth in the period. Exports also helped, rising 2.7%. The weakness came largely in business investment, and construction spending also fell again. Government loans slowed. Companies' views of current conditions have stabilized. However, future expectations worsened, with the index dropping to 95.8. That doesn't promise well for a rebound in business spending. And recent rises in taxation and spending cuts by the government could weigh on consumers.

*** Modern psychology considers childhood an extremely important period of human development. Western culture views children as vulnerable and requiring a great deal of attention, care and shelter from harm. Many laws are designed to protect children from dangerous toys, dangerous substances and even dangerous parents. Our belief that all children ought to have a free public education and that they should remain in school until adolescence similarly reflects the view that childhood is a special and important time. But these attitudes toward children reflect a relatively recent conception of early development. The Greek and Roman civilizations, for instance, which extended from about 600 B.C. to about 400 A.D., are usually regarded as periods of great enlightenment. Yet the status of children during those times was hardly enviable. Although such great Greek thinkers as Plato and Aristotle wrote of the importance of education, they also defended practices that today would seem unthinkable. Infanticide, the killing of newborns, was routine and viewed as an appropriate way to deal with babies who were illegitimate, unhealthy or simply unwanted.

*** Non-lethal weapons are typically given names that make them sound ac ceptable; for example, the term "rubber bullet" was adopted to suggest something soft and harmless. Jelly batons, sponge grenades and bean bags sound like something from a children's party, and you need to be aware that these "bean bags" are canvas bags of lead shot. Similarly, the very term "stink bombs" makes them sound like a practical joke instead of a chemical warfare agent. Sometimes language is deliberately used to mislead, as in the case of "rubber-coated bullets". These are steel balls the size of marbles with a thin rubber coat, able to penetrate the skull. There are also so-called "plastic bullets", which are high velocity bul lets made of PVC.

*** For more than ten years, Russia's relations with the advanced countries of the western world had been a torrid and unsatisfying mixture of unrequited love, misunderstanding and dashed hopes. Actually, recently there have been big shifts on both sides. The West is no longer trying to recast Russia in its own image. The days are gone when politicians in the US used the International Monetary Fund (IMF) as a foreign policy fund to promote particular policies and politicians in Russia. These days, Russia runs itself more or less as it likes. Its economic growth may be slow but it is encouraging: Russia registered its third consecutive year of real growth in 2001, with the average income up by 5% and at a time when the world's big economies were sluggish. Although good official connections in business still matter hugely, the days of central planning are gone for good, as is the wild era of looting and barter that followed the collapse of communism. Russia delivers its oil and gas on ti me and is steadily paying off its foreign debts. Compared with what might have happened, the outside world finds that cheering.

*** The Postal Service has the longest history of monopoly power in the country and has the distinction of being mentioned in the US Constitution. In the past, the post office was impressive in using its monopoly to maintain high wages despite a widespread reputation for poor service. But these past accomplishments have been severely diminished by its inability to control the entry of competitors. First came FedEx Corp. and its associates, then the fax machine, and then, most destructively, the Internet and e-mail. The failure to get Congress to classify all these innovations as first class mail and therefore the exclusive domain of the Postal Service demonstrates a tremendous loss of muscle. Therefore, this contestant fails to measure up and should probably not even be nominated in the future.

*** Behavioral scientists have identified many powerful factors that drive us to war - factors so numerous and so compelling that it is hard to imagi ne how we will ever overcome them. Evolution seems to have equipped us with strong tendencies to organize and kill. As General John J. Pershing stated, "Men go to war because they enjoy it." Like many mammals, we also possess the natural tendency to protect our territory. Society is capable of suppressing genetically-based tendencies, but when it comes to war, most cultures actually fuel the flames. We deliberately instill nationalistic pride in our children, and we teach them to assume roles and follow orders which are all characteristics of the good soldier. In addition we reduce the individuality of people by giving them uniforms; we diffuse responsibility by having them use weapons in teams; we dehumanize enemies by labeling them heathens, animals and so on. Throw in financial incentives, some propaganda and a charismatic leader or two, and we become more prone to war than ever.

ÜDS FEN 2005 İLKBAHAR

*** On 31 October 1994, a turboprop airliner heading for Chicago, Illinois, crashed into a soybean field at Roselawn in Indiana. All 68 people aboard died. Although the weather was cold and damp that day, no one could believe it when investigators revealed that the crash was caused by a build-up of ice on the wings. Not only did this modern plane have a fully functional de-icing system, but according to US Federal Aviation Administration (FAA) standards, the French-built ATR-72 should have had no problems flying in the cold, damp conditions. The pilots even knew their craft was icing up and attempted to clear it, following de-icing procedures exactly.

*** After a heavy rain one day 200,000 years ago, someone small walked across some sand. Sand covered the tracks and eventually they hardened into sandstone. More centuries passed, and the sandstone eroded. Some construction workers on a break in Nahoon, South Africa, discovered the tracks, which were thought to be 30,000 years old. Now geologist Dave Roberts, at the Council for Geoscience in Cape Town, has redated the impressions and says they are 200,000 years old, the oldest human footprints on Earth. "We have far more powerful dating techniques now", says Roberts. He used thermoluminescence, a dating method that measures when sand grains were last exposed to light. Only about seven inches long, the prints clearly show five toes and a welldeveloped arch.

*** It cannot be denied that buying locally grown food really does offer big advantages, not just in freshness, but also in environmental savings. Imported foods, especially those flown in from the opposite hemisphere, use up huge amounts of jet fuel – 127 calories of fuelper calorie of Californian lettuce flown to Britain, and 66 calories of fuel per calorie of South African carrot; that at least is what a green research group based in London has estimated. Much of that cost is hidden from consumers, because air fuel attracts no tax – an advantage guaranteed by international treaty. And under the Kyoto Protocol, carbon emissions from international transports aren't added to national carbon-emission tallies, because nobody can agree whose account to charge them to. But the fuel used to import food and drink to Britain continues to account for four million tonnes of CO2 emissions annually, which is about 2.5 per cent of the national total.

*** By 2020, total meat consumption in developing countries is predicted to have more than doubled. Yet livestock, especially in wealthy countries, already munches its way through 36 per cent of the world's grain harvest. All that grain exacts a heavy environmental cost, because the increased demand pulls farmers towards growing maize and soybean and away from pasture and fodder crops such as alfalfa that do so much for soil health. And grain-fed animals tend to be housed in huge feedlots, where crowding causes disease and veterinary surgeons use more antibiotics, and where vast mountains of manure pollute the surroundings. Consumers could reverse many of these problems by demanding meat from grass-fed livestock, or simply by eating less meat. And as an added bonus for overfed Westerners, the meat from grass-fed cattle is leaner than that from grain-fed animals.

*** In the first half of the 20th century, acid rain - whether natural or unnatural - was a hidden phenomenon. By the 1950s, however, its effects were becoming apparent to scientists in Scandinavia with the loss of fish from mountain lakes. A network of rainwater monitoring stations was set up across Europe, which very soon proved the existence of acid rain as a result of human activities. Acid rain was also charged with killing vast tracts of forest in Germany. The effects of long-range sulphur pollution were looking so worrying that the issue was placed before the world at the UN Conference on the Human Environment in Stockholm in 1972. The outcome was that states are now responsible for ensuring that their activities do not damage the environment of other states. This paved the way for the development of programmes for reducing long-range air pollution. The most important of these was the Convention on Long-Range Transboundary Air Pollution, attended in 1979 by the world's leading industrial nations.

*** When it came to replacing the bridge at John's Pass, there were certain special problems to be taken into consideration. The previous bridge had had a life span of only 30 years, but in that time the currents had caused extensive damage. But a more serious problem connected with the currents is that the bridge has to open on demand, rather than on a fixed schedule. Currents at the inlet are particularly fierce, and making boats wait would be too dangerous. This unpredictability makes life even tougher for motorists. Yet a fixed bridge would make a stretch of the Intracoastal Waterway impassable to boats taller than 65 feet, so that option was ruled out almost immediately. When an examination of the bridge's records showed that increasing the height would reduce the frequency of closings only slightly, the state decided to build a new drawbridge with the same height as the old one.

ÜDS SAĞLIK 2005 İLKBAHAR

Despite all the attention we give to our hair, and putting aside the fact that the first synthetic hair dyes were created in 1907, it has really only been in the last 50 years or so that hair has been scientifically studied. Before that, it was deemed too trivial to be worthy ofthe attention, but the amount that scientists can now tell about a person from the study of their hair,often simply by looking at it under a microscope, is remarkable. Humans have around five million body hairs which is as many as a chimpanzee has, although ours are smaller and finer. They come in three types. An unborn baby has a kind of fine down all over its body that begins to grow about 12 weeks after conception. Normally, these "lanugo" hairs are shed a few weeks before birth, although some premature babies are born with them. After birth and throughout our lives, humans are covered in short "vellus" hairs just a centimetre or two long and with little or no pigment. Finally, the pigmented, thicker hairs that grow in varying quantities on our heads, groin, armpits, forearms and legs, and (on men) chests, stomachs and faces, are "terminal" hairs.

*** Evidence from epidemiological studies carried out in the US shows that a sedentary life-style increases the incidence of more than 20 chronic diseases. One such study, a Nurses' Health Study, concerned 70,000 female nurses aged 40 to 65 years, it was found that, when nurses walked the equivalent of three or more hours per week at a brisk pace, they had 30 per cent less coronary artery disease, ischemic stroke and type-2 diabetes, as compared to sedentary nurses. In another study, some site-specific cancers were found to be also more prevalent when moderately active females become inactive. Breast cancer rates rose 25 per cent among the sedentary in one study, and each one-hour decrease of recreational physical activity per week during adolescence was associated with a 3 per cent increased risk of breast cancer. Indeed, physical inactivity is now the third leading cause of death in the US and contributes to the second leading cause, which is obesity. Physical inactivity is one of the causes of at least 1 in 8 deaths.

*** Health care professionals may quickly dismiss alternative therapies as ineffective and perhaps even dangerous, but their clients think otherwise. In a survey of over 1,500 people, 1 out of every 3 had used at least one alternative therapy in the past year for a variety of medical complaints from anxiety and headaches to cancer and tumours. Visits to alternative therapies outnumbered visits to primary care physicians. Interestingly, those who seek alternative therapies seem to do so not so much because they are dissatisfied with conventional medicine, but because they find these alternatives more in line with their beliefs about health and life. Most often, people use alternative therapies in addition to, rather than in place of, conventional therapies. Only a few of the people surveyed saw an alternative therapist without also seeing a physician; all of those with life-threatening conditions such as cancer, diabetes or lung problems who used alternative therapies saw a medical doctor as well. In fact, it seems that most people seekalternative therapies for nonserious medical conditions or health promotion. They simply want to feel better, and access is easy.

*** A century ago, most people who suffered traumatic injuries or contracted serious infections died soon afterward. Also those who developed heart disease or cancer had little expectation of a long life after the disease was diagnosed. Death was a familiar experience, and most people expected little more than comfort care from doctors. Today, death is often seen as an event that can be deferred indefinitely rather than as an intrinsic part of life. The leading causes of death for people over age 65 years are heart disease, cancer, stroke, chronic obstructive pulmonary disease, pneumonia and dementia. Medical procedures commonly extend the lives of people who have these diseases, often giving people many years in which quality of life and function are quite good. Sometimes, however, procedures extend life, but the quality of life and function decline.

*** The veterinarians who oversee laboratory animals often find themselves in the position of having to perform procedures that may not be in the best interest of their nonhuman patients. The vet's goal then is to produce the best possible outcome for the animal under the circumstances. When Larry Carbone started out in that profession, he hoped to improve the condition of laboratory animals while at the same time accommodating the needs of researchers. Unfortunately, he found that, in practice, the ability of veterinarians to advocate effectively for animals is limited by a variety of factors. His new book is the fruit of the extensive research he conducted to discover what determines how we view laboratory animals and why policies concerning their care have developed as they have. He examines the objectivity of those who presume to know what these animals want and to speak for them, showing that personal beliefs, theories and ideologies colour even the most scientific reports.

*** Discussion about the use of antidepressants by children, which is always a hot topic, boiled over in September when hearings revealed that both the drug industry and the Food and Drug Administration (FDA) had concealed strong evidence about the dangers associated with the most widely prescribed drugs, a class known as selective serotonin reuptake inhibitors (SSRIs). The analysis of the test results, which pharmaceutical companies had failed to release to the public and which the FDA had sat on for a year, indicated that these SSRIs double the suicide risk in depressed juveniles yet help no more children in trials than placebos do. The highprofile congressional and FDA hearings were made all the more dramatic as parents recounted how their children had moved from moderate depression to suicidal depression within days of starting SSRIs. Some of these children had died during the year of delayed FDA action.

ÜDS SOSYAL 2005 İLKBAHAR

*** From birth to age 10, our developmental focus is on learning how to be human beings - learning to move, to communicate, tomaster basic skills. These often require the mastery of learned social and cultural conventions, traditions, and rituals, such as movements associated with various games, differences in spoken or written languages, and our culture's definition of good manners. The initial development is slow and awkward, but children generally function at a rapid automatic level by age 10. Adults usually allow young children to make mistakes. We smile indulgently and offer support rather than criticism as toddlers trip and as 2-year-olds make language errors. We are there principally to protect their safety and to applaud their successes because we realize that toddling leads to walking and running, and babbling leads to speaking, reading and writing.

*** In his three-and-a-half years in the job, the President of Harvard University seems to have upset a large number of people. First, he said students were getting too many "A" grades because of grade inflation, {which was correct). Then he attacked a professor on account of his extra-curricular activities. Now he has suggested that one of the reasons women achieve less in science and maths is that they have less innate ability. The President's -comments were off the record; but he has since confirmed that he did draw attention to the possibility that innate differences, rather than social factors, such as education and treatment in the workplace, might have a rote to play. This has resulted in a great deal of violent criticism from many quarters. But, scientifically speaking is he correct?

*** The reasons for language death range from natural disasters to different forms of cultural assimilation. Small communities in isolated areas can easily be wiped out by earthquakes, hurricanes, tsunamis and other natural disasters. On 17th July 1998, a 7.1 magnitude earthquake off the coast of Papua New Guinea killed more than 2.2Q0 people and displaced a further 10,000 people. Many villages were destroyed; and some 30% of the villagers were killed. The people in these villages had been identified as being sufficiently different from each other in their speech to justify the recognition of four separate languages, but the numbers were very small. Moreover, as the survivors moved away to care centers and other locations, these communities and thus their languages could not survive the trauma of displacement.

*** Immigrants have long complained about US employers who cheat or abuse them and threaten to have them deported if they protest. Generally, the problem has been confined to the lowest rungs of the workforce, such as Mexican farm hands who enter the country illegally. But nowadays, the weak US economy has sparked an outbreak of abusive treatment among the legions of professional immigrant employees who flocked to the US on perfectly valid visas during the late-1990s boom. Usually, theirs are the cases of employers who do not pay full salary or benefits.

*** In London's theatres, tastes seem to be changing. Though audiences are not falling, that's mostly thanks to the allure of musicals, not plays. The commercial London Theatres ran at 65% capacity in 2003, the most recent year for which figures are available. But this disguises a big difference between musicals and plays. For the musicals, attendance averages 68% of capacity; for plays, attendance is somewhat lower, at 56%. So if a show doesn't contain some singing and plenty of dancing, half the chairs are likely to remain empty. And In a business in which the costs are all fixed, a few more tickets sold can make all the difference. However, London's subsidized theatres are doing unusually well. For example, at the National Theatre, which receives around £14 million in public money every year, attendance has been running at over 90% of capacity for the past 20 months. That's partly thanks to sponsorship and partly to aggressive programming.

*** Spain's third-largest bank, Banco Popular, raised some eyebrows recently when it agreed to buy Portugal's Banco Nacional for $638 million. Shareholders voted overwhelmingly in approval of the acquisition. Normally, however, Popular's approach is far more conservative: a singleminded devotion to organic retail growth in its home market as opposed to takeovers. Over the past decade, the bank has worked hard to build its base, cultivating the market for mortgages, consumer loans, and, recently, loans to small and midsize businesses. As rivals looked to the New World and bought up smaller banks at home. Popular expanded its retail network to 2,200 branches throughout Spain, it also added Internet and telephone banking operations that are based in the branches. The effort has paid off. In each of the past three years, the bank has gained 300,000 customers.

ÜDS FEN 2005 SONBAHAR

*** There have been stories in the press about mobile phones sparking explosions at petrol stations. But according to the GSM Association, a worldwide body for mobile phone makers, none of these reports has ever been traced back to a real event. But there is a real safety concern, and it's not about radio emissions from mobiles as you might have thought. Instead, the GSM Association says there is a theoretical risk that if a hand-held phone is dropped and the battery separates from the phone, it could cause a spark across the contacts. This is equally true of other battery-powered devices such as torches, Walkmans and CD players. But it's far more likely that mobile phones cause a hazard at petrol stations by distracting their users while they're operating a petrol pump.

*** Formerly, potatoes were grown on unirrigated land, which often meant they were small and probably misshapen. Now, however, farmers routinely irrigate their lands to produce products acceptable to the fast-food industry for its French fries. But in Minnesota the groundwater that farmers pump for potatoes has turned out to be the same water that helps to sustain the Straight River, a major trout fishery. Even modest pumping for potatoes, a federal study eventually concluded, had the potential to reduce the river's flow by one third during the irrigation season, with adverse impact on the brown trout. For now, the trout are not in danger, but that could change if Minnesota were to approve applications from farmers still eager to see potato planting and irrigation widen.

*** All multi-engined aircraft are designed to keep flying in the event of engine failure. Losing thrust from one "side of an aircraft unbalances it and causes the nose of the aircraft to turn in the direction of the failed engine. Aircraft have a vertical stabilizer (the upright at the back) to keep the aircraft's nose into the wind, with a rudder attached to it for fine tuning. Moving the rudder into the airflow creates a sideways force, which turns the aircraft around its vertical axis. When this is applied in the direction of the failed engine, the force created by the rudder will counteract the turn induced by the uneven engine thrust. The vertical stabilizer and rudder are sized to control the uneven thrust caused by an engine failure at the most demanding limits or the flight envelope, as they must be capable of generating powerful turning forces.

*** If engineers waited for the development of scientific knowledge to use and organize into technological achievements, ours would be a very different world from what we know. In engineering, it is not so much science as it is ingenuity that is applied to solve problems and satisfy needs and wants. If this were not so, the steam engine would never have been invented in the absence of thermodynamics. The Wright Brothers would not have flown since they had no aerodynamics textbooks. The astronauts would never have landed on the Moon nor the rovers on Mars without firm geological knowledge of their surfaces. Rather than following scientific theories and discoveries, engineering leads them. Operating steam engines prompted the development of thermodynamics, actual powered flight drove aerodynamics, and Moon and Mars missions brought back samples and sent back data that led to increased scientific knowledge about those extraterrestrial bodies.

*** Gamma-ray bursts (GRBs) are among the most intense areas of research in high-energy astrophysics, and they represent the largest known explosions in the universe. Last year, NASA launched the Swift satellite to rapidly locate and observe GRBs and their afterglows at x-ray, ultraviolet and optical wavelengths. These afterglow observations are particularly important for the understanding or these enormous explosions that occur at cosmological distances near the edge of the observable universe. Amateur astronomers, due to their large numbers, their ability to respond quickly to the randomly located GRBs and the availability of highly sensitive CCD cameras, have been able to provide important, early data on the optical afterglows of GRBs in the past few years. Indeed, at least one optical afterglow from a GRB was discovered by an amateur astronomer. Observations can be done with even a modest-size telescope, provided the observations are made quickly enough.

*** The people of Blaenavon in South Wales were understandably worried. The opencast mine only 1 kilometer north of their town had served the great Blaenavon ironworks when they opened in the Iate18"1 century. But it had been lying derelict for decades and now British Coal Opencast wanted to mine the remaining 320,000 tonnes of coal. At opencast sites, for every tonne of coal recovered, up to 40 tonnes of rock has tobe extracted. So, in the case of Blaenavon, unless precautionary measures were taken, the air in the town was going to be thick with dust. Precautionary measures were taken, based on well-tried techniques. For instance, tall spray masts were erected around dumps to provide a curtain of rain to entrap and wash out dust from the air, and the wheels of every vehicle leaving the site were washed. Further, reclaimed areas were planted with trees and grass from day one. With these and other measures this turned out to be a success story for all concerned. In fact, during the entire mining and reclamation project not a single complaint about nuisance dust was made to the local authority.

ÜDS SOSYAL 2005 SONBAHAR

*** To some people, branding is about the art of lying successfully and creating value out of nothing. A good brand inspires trust, but the relationship between the brand and the reality of the product offered is frequently elastic. Branding is therefore an art and the people who do it best are in great demand. In many business segments entry is easy as barriers are low and there is scarcely any means of differentiating yourproduct from that of the competitor. Branding is all there is left to make a difference. Mobile phone companies are a classic lease, in which one company can copy the competitor's contracts.Each company offers virtually the same handsets. Apart from network coverage, brand is everything. Another classic example is airlines. Anyone can charter the same Boeing 737and run an airline as long as it can secure slots. The difference is finally down to service and marketing.

*** Almost 50 years ago, the scientist and novelist Charles Percy Snow delivered a lecture at the University of Cambridge in which he described a problematic situation that he termed "the two cultures". According to C. P. Snow, as he came to be most commonly known, it was the circumstances of his involvement in both the physics and the writing communities, mostly in Britain, that gave him an unusually diverse perspective on intellectual life at mid-century. Although he noted thatmembers of the two groups that he moved among had similar social origins, possessed comparable intelligence and earned about the same amount of money, they barely communicated with each other. Snow observed that their intellectual, moral and psychological climates had so little in common that they may as well have come from different parts of the world. He feared that the intellectual life of the whole of western society was increasingly being split into two polar groups characterized by physical scientists and "literary intellectuals".

*** In an agreement made outside the courts in Basle, Switzerland, Nigerian state representatives, Swiss federal officers and executives from a dozen European banks have agreed on the return of around 1.1 billion euros of the late dictator Sani Abacha's investments, to Nigeria. The conclusion of what has been one of the world's largest investigations into money laundering comes as a huge relief to regulators and bankers in Switzerland, the UK, Liechtenstein, Luxembourg and Jersey, the destinations that provided safe havens for Abacha's stolen millions. In exposing just how easily Abacha was able to make deposits of tens or hundreds of millions of dollars in European banks that pride themselves on watchfulness and integrity, the Nigerian investigators highlighted major failings in both the legislative and executive competencies of the European banking community. Reform has already taken place. Undoubtedly more will follow, but the sorry tale of how Europe's banks aided the dictator's outrageous years of theft cannot be forgotten.

*** As for the lifestyle in the Netherlands, the Dutch have a deserved reputation for being easygoing. But as with any country, there are some surprises for newcomers to get used to. For instance, the amount of bureaucracy strikes them as remarkable when they first move to the Netherlands. Want to light a small fire in your garden? You'll need permission from the local government. Fancy painting your house? Better check with the government first. A major problem is the tightening up of immigration laws on a large scale. The problem is that the Netherlands is a small country that is facing a lot of immigration, so they are making it tougher and tougher. But, actually, racism is foreign to the Dutch culture. They are very accepting; they take you as you are.

*** With their magnificent architecture and sophisticated knowledge of astronomy, and mathematics, the Maya boasted one of the great cultures of the ancient world. Although they had not discovered the wheel and were without metal tools, the Maya constructed massive pyramids, temples and monuments of stone both in large cities and in smaller ceremonial centers throughout the lowlands of the Yucatan Peninsula, which covers parts of what are now southern Mexico and Guatemala and essentially all of Belize. From celestial observatories, they tracked the progress, for example, of Venus and developed, a calendar based on a solar year of 365 days. They created their own system of mathematics, using a base number of 20 with a concept of zero. And they developed a hieroglyphic scheme for writing, one that used hundreds of elaborate signs.

*** Patent laws do little or nothing to help poor countries dig their way out of poverty, and could even make matters worse. That's the controversial conclusion of an independent commission on intellectual property rights appointed by the British government. Its report, launched recently in Geneva, contains 50 radical recommendations to help make these rights work to the advantage of poor countries. Britain's Department for International Development has already promised to look hard at the suggestions, but it's unclear whether the rest of the international community will listen. The World Trade Organization has persuaded most countries to sign an intellectual property rights (IPR) agreement that obliges them to impose Western style laws on everything from patents to copyrights by 2006. But the new report argues that these laws only benefit rich countries with strong traditions of invention, and do little to aid the transfer of technology to poor countries. The report argues that poor countries should be given a lot more flexibility So as to customize those laws, and up to a decade longer to do so. At present, many poor countries don't have intellectual property laws at all. That means local inventors can't get protection for their ideas, but it also means people can buy cheap versions of medicine or software that have been patented elsewhere. Since poor countries often have little to patent in the first place, the benefits of having no laws can outweigh the disadvantages.

ÜDS FEN 2006 İLKBAHAR

*** How have terrestrial organisms met the environmental challenges of living on land? Life began in the oceans, but many life forms have since adapted to terrestrial life in a sea of air. Every single organism living on land has to meet the same environmental challenges: obtaining enough water; preventing excessive water loss; getting enough energy; and in polar regions, tolerating widely varying temperature extremes. How those challenges are met varies from one organism to another, and in large part explains the diversity of life encountered on land today. Some animals avoid colder temperatures by migrating to warmer climates for the winter, whereas others avoid the cold by passing the winter in a dormant state called hibernation. Many plants also spend winter in a dormant state. The aerial parts of some plants die during the winter, but the underground parts remain alive; the following spring they resume metabolic activity and develop new aerial shoots. Many trees are deciduous; that is, they shed their leaves for the duration of their dormancy. Shedding leaves is actually an adaptation to the “dryness” of winter. Roots cannot absorb water from ground that is cold or frozen; by shedding its leaves the plant reduces water loss during the cold winter months when obtaining water from the soil is impossible.

*** Except perhaps for some remote island dwellers, most people have a natural tendency to view continents as fundamental, permanent and even characteristic features of Earth. One easily forgets that the world’s continental platforms amount only to scattered and isolated masses on a planet that is largely covered by water. But when viewed from space, the correct picture of Earth becomes immediately clear. It is a blue planet. From this perspective it seems quite extraordinary that over its long history, Earth could manage to hold a small fraction of its surface always above the sea — enabling, among other things, human evolution to proceed on dry land. Is the persistence of highstanding continents just an accident? How did Earth’s complicated crust come intoexistence? Has it been there all the time, like some primeval icing on a planetary cake, or has it evolved through the ages? Such questions engendered debates that divided scientists for many decades, but the fascinating story of how the terrestrial surface came to take its present form is now partly resolved. That understanding shows, remarkably enough, that the conditions required to form the continents of Earth may be unmatched in the rest of the solar system.

*** Henrik-Jan van Veen has carried out a great deal of research into spinning. This is especially true for “graveyard spins”, the term for what happens when fighter pilots get so disoriented they miscalculate how to get their plane back on course. They can end up in a dangerous and often fatal spin. Van Veen works at a research lab run by the Netherlands Organization for Applied Scientific Research, the TNO. The range of research covered by the TNO is vast, and it sees itself as a practical problemsolver. And for the Dutch air force, the graveyard spin is certainly a problem that needs solving. Van Veen’s specialty is “vibrotactile devices”, which use vibrations to convey information. His latest project is a vest studded all over with small discsthat can each vibrate independently. In a test room, a pilot is strapped into a seat in a “cockpit”. At the push of a button, the lights go out and the chair starts spinning. After a while the chair is stopped. “He’ll think he’s spinning the other way now”, says van Veen. The pilot is told to correct the spin, but instead, he overcorrects massively, and the chair begins spinning again. In the next test, the pilot dons van Veen’s vest and is told that the patch of the vest that is vibrating will indicate the direction he should force the joystick to correct a spin. This time, when the chair stops spinning the pilot manages to keep the seat still. Van Veen thinks the vibrotactile vest could do more than save the lives of fighter pilots. He’s now working on linking the system to a GPS receiver so that tourists in a foreign city or blind people in an unfamiliar environment can use the vest to find their way around.

*** In his preface to Spaceflight Revolution, David Ashford recalls how he started his research into rocket motors. As he later explains, these were motors that would power aspace plane — one that would launch space travellers and satellites cheaply and reliably into orbit. That was 1961. Ashford admits he would probably have taken another job if he’d known that, 42 years later, satellites would still be launched by rockets descended from ballistic missiles. The technology is there, but political and budgetary decisions have so far stopped space planes getting off the ground. But Ashford presents a compelling argument that a small orbital space plane would cost relatively little to design and develop — the equivalent of just two shuttle flights.

*** For almost 200 years, the idea of cosmic events affecting life on Earth was viewed as heretical by the church, which regarded catastrophe as proof of divine intervention, and as nonsense by the scientific establishment, which dismissed it as superstition. Yet in the end, the sheer weight of evidence has swept away all doubt about the reality of global catastrophes. Attempts to make scientific sense of the many legends of global catastrophes date back to the dawn of modern science itself, in the 17 th century. Following the publication of Newton’s laws of motion and universal gravitation in 1687, Edmond Halley decided to apply them to the mystery of comets. By studying records of their appearance, Halley argued that the bright comets of 1456, 1531, 1607 and 1682 were in fact one comet, laterknown as the “Halley” comet, that followed a vast elliptical orbit around the Sun in agreement with Newton’s laws. But Halley noted something else as well: a comet crossing the orbit of the Earth might one day collide with us with devastating consequences.

*** The Wireless Museum has several of the earliest crystal wireless sets from the 1920s which ran on electromagnetic waves with no external power source, and were easily made at home. Valve radios, which came along in the 1930s, needed electricity to heat up the valves and the museum has both mains and batterypowered valve radios on display. The collection also has some rare wartime civilian receivers — the only type of valve radio manufactured during the Second World War. This was by order of the government, because at this time most manufacturing was focused on the war effort. There are also plenty of modern day transistor radios including a collection of novelty radios dating from the sixties and seventies.--

ÜDS SAĞLIK 2006 İLKBAHAR

*** Aging involves multiple harmful biological events that accumulate in different tissues overtime and gradually reduce an organism’s state of maintenance and function. Calendar time, however, serves as an imperfect measurement of the physiological processes involved in aging.We all know individuals who are the same chronological age but appear to be very different when it comes to physiological age. Rather than counting years — or gray hairs, for that matter — modern gerontologists turn to biological markers, or biomarkers, of aging. These physiological parameters indicate an individual’s functional level and some biomarkers, such as insulin levels, correlate with mortality. The presence of such biomarkers depends indirectly on patterns of gene expression, which are induced by a variety of internal or external stimuli.

*** Cardiac transplantation, once considered an experimental procedure, has emerged as the therapy of choice for many appropriately selected patients with life-threatening irremediable heart disease. Congestive heart failure (CHF), the primary indication for cardiac transplantation, is the most commonly reported reason for hospital admission. A knowledge of cardiac transplantation medicine is therefore important for all physicians, as transplantation should be considered a therapeuticoption for many of these patients. In the past, post-cardiac transplant care was largely performed by specialized transplant physicians, primarily cardiologists and cardiovascular surgeons. As survival after cardiac transplantation has improved markedly over the last decade, the population of patients who are long-term survivors after heart transplantation has grown. Primary care physicians, as well as cardiologists not based at cardiac transplant centres, often assist inthe care of these patients, most often inconsultation with cardiac transplant physicians. In addition, a physician may be called on to assist in the management and evaluation of a potential cardiac donor.

*** When a patient sees a doctor, the patient is seeking help –- to regain or retain health. The physician’s task is to work for the patient’s health. The doctor does so by treating disease, by relieving discomfort, by assisting the patient with any disability, by preventing premature death, and by maximizing contentment. (Some have summarized these activities as tackling “the fiveD’s” of health – disease, discomfort, disability, death, and dissatisfaction). Often there is success in all these areas. In the best of circumstances, the doctor is able to prevent disease and help the patient remain healthy. In other cases, disease and death defeat us. In some cases none of the goals are achieved, but even that outcome must not stop us from trying. By focusing on the health of the patient, the doctor tests the myriad activities of clinical medicine against the health outcome of the patient.

*** Cancer describes a class of diseases characterized by the uncontrolled growth of aberrant cells. Cancers kill by the destructive invasion of normal organs through direct extension and spreadto distant sites via the blood, lymph, or serosal surfaces. The abnormal clinical behaviour of cancer cells is often mirrored by biologic aberrations such as genetic mutations, chromosomal translocations, expression of fetal or other discordant ontologic characteristics, and the inappropriate secretion of hormones or enzymes. All cancers invade or metastasize but each specific type has unique biologic and clinical features that must be appreciated for proper diagnosis, treatment and study. About 1.2 million new cases of invasive cancer are diagnosed each year in the United States, and about 500,000 people die annuallyof the disease. Cancer is the second most deadly disease and is expected to surpass heart disease early in the twenty-first century to top that vicious list. Over the past half century, the frequency of most cancers has been stable, but some dramatic changes have taken place. Steady declines in stomach and uterine cancerhave occurred, the latter undoubtedly due to routine cytologic screening for cervical cancer. The cause of the decline in stomach cancer is unknown. The most striking change has been the increases in lung cancer in both men and women, undoubtedly related to smoking.

*** Medicine is not a science, but a profession that encompasses medical science learning as well as personal, humanistic, and professional attributes. Nonetheless, the delivery of Western medicine depends totally on science and the scientific method. Since Flexner issued his famous report on the subject in 1910, American medical education has striven to develop a strong scientific base as an integral part of medical education at every level: premedical, medical, residency and continuing medical education. Biomedical science is fundamental to understanding disease, making diagnoses, applying new therapies and appreciating the complexities and opportunities of new technologies. The process of becoming a physician and being committed to lifelong learning requires that one possess the scientific base not only to acquire and appreciate new knowledge but to see new ways for applying it to patient care as well. The physician must be able to understand reports of current research in the medical literature in order to grasp and evaluate the newest and latest approaches, no matter how complicated the field may become.

*** In the past, lead poisoning was thought to arise from pica(abnormal ingestion) among children living in old, broken-down houses with peeling layers of leadbased paints. In the past two decades, lead intoxication has occurred with decreasing frequency. This may in part be relatedto less use of lead in paint and leaded gasoline; several studies relate environmental lead contamination to traffic density patterns. In the United States, hundreds of occupations involve potentially significant exposure. It is estimated that more than 800,000 American workers have potentially significant lead exposure. Lead and other metal workers or miners, storage battery workers, and pottery makers are particularly heavily exposed. Workers in auto manufacturing, ship building, paint manufacture and printing industries are also at substantial risk, as are house painters and those who repair oldhouses. In past centuries lead was addedto wine to sweeten it, a deception that was eventually made punishable by death. Recently, adding leadto various herbal and folk medicines has resulted in poisoning. Bullets left in the body can result in lead poisoning, especially if a joint is involved, because synovial fluid appears to be a good solvent for lead. The interval between the bullet getting embedded in the body and clinical evidence of lead poisoning has ranged from 2 days to 40 years.

ÜDS SOSYAL 2006 İLKBAHAR

*** The US Supreme Court is not aradical institution, nor is it likely to become one asa result of any particular presidential election. The risks for the judiciary in presidential elections are a lot lower than many people imagine. This is not because there are no significant ideological or methodological differences among judges. Differences do exist, and they display party affiliation to some extent. And they matter – not just on public issues such as abortion rights and racial discrimination but also in those procedures that actually guide the way lower courts handle a large variety of legal cases. That said, the courts have pretty strong institutional defences against radicalism of any kind. For one thing, the judiciary’s power is spread among more than 800 federal judges, no one of whose views matter all that much in the broad scheme of things. Even onthe Supreme Court the idiosyncrasies or ideological extremism of any one judge can have only a limited effect. Without four likeminded judges, his or her views are just noise.

*** Governments have learned tovalue innovation these days for good reason. Far from being simply some missing factor in the growth equation, innovation is now recognized as the single most important ingredient in any modern economy. It actually accounts for more than half of economic growth in America and Britain. In short, it is innovation, more than the application of capital or labour, that keeps the world economy going. As a result, economists have decided that the innovators of the world are due some special recognition. It is not possible to recognize all the countless innovations that have helped to spread wealth, health and human happiness around the world. But a handful of people who have made the biggest contribution to the wealth-creation process in their own fields over the past few years, have been nominated for awards.

*** American schools need moretime if they are to teach efficiently. The school year is fixed at or below 180 days in all but a handful of states – down from more than 190 in the late nineteenth century, when Saturday-morning sessions were common. The instructional day is only about six hours, of which much is taken up with nonacademic matters. In 1994, a national commission calculated that in four years of high school a typical American student puts in less than half as much time on academic subjects as do students in Japan, France and Germany. Extending the school day orthe school year can get expensive and complicated, and reducing nonacademic electives and physical education brings complaints from parents and students alike. But there is one quite cheap and uncomplicated way to increase study time: add more homework. You may not be surprised to learn that homework raises student achievement, at least in the higher grades. For young children homework appears not to be particularly helpful. Even among older students it is hard to be sure of the extent to whichmore homework may lead to higher achievement.

*** A nonprofessional-class working mother, who has been forced unwillingly into the labour market, is oppressed by various unique forces. She is oppressed by the fact that her work is oftentimes physically exhausting, ill-paid, and devoid of benefits such as health insurance and paid sick leave. She is oppressed by the fact that it is impossible to put a small child in reliable day-care if you make only a minimum wage, and she is oppressed by the terrible child-care options that are available at an inexpensive rate. She is oppressed by the fact that she has nothing to fall back on. If she is out of work, and her child needs a visit to the doctor and antibiotics, she may not be able to afford those things and will have to treat her sick child with unprescribed medications, which themselves are far from cheap.

*** During the Renaissance, especially in the sixteenth century, it was customary to debate the preeminence of the arts, particularly as between painting and sculpture. The more commonly accepted opinion is represented by Benvenuto Cellini, who thought that sculpture is eight times as great as any other art based on drawing, because a statue has eight views and they must all be equally good. A painting, he said, is nothing better thanthe image of a tree, man, or other object. In fact, the difference between painting and sculpture isas great as between a shadow and the object casting it. Leonardo, on the other hand, thought that painting is superior to sculpture because it is more intellectual. By this he meant that as a technique it is infinitely more subtle in the effects that it can produce, and infinitely wider in the scope it offers to invention or imagination. Michelangelo, when the question was referred to him, in his wise and direct way said that things which have the same end are themselves the same, and that therefore there could be no difference between painting and sculpture except differences due to better judgment and harder work.

*** Because a play presents its action through actors, its impact is direct, immediate, and heightened by the actor’s skills. Instead of responding to words on a printed page, the spectator sees what is done and hears what is said. The experience of the play is registered directly upon his senses. It may therefore be fuller and more compact. Where the work of prose fiction may tell us what a character looks like in one paragraph, how he moves or speaks in a second, what he says in a third, and how his auditors respond in a fourth, the acted play presents this material all at once. Simultaneous impressions are not separated. Moreover, this experience is interpreted by actors who may be highly skilled in rendering nuances of meaning and strong emotion. Through facial expression, gesture, speech rhythm, and intonation, they may be able to make a speaker’s words more expressive than can the reader’s unaided imagination. Thus, the performance of a play by skilled actors, expertly directed, gives the playwright a tremendous source of power.

ÜDS FEN 2006 SONBAHAR

*** Over billions of years, life has evolved into a spectacular diversity of forms – more than a million species presently exist. For each, the source of its uniqueness is the particular combination of proteins found within its cells. Yet in the midst of this diversity, the similarities between living things are profound. For example, although the fruit fly genome encodes about 14,000 different proteins, and humans have two to three times that number, many proteins are still recognizably similar in sequence and task, reflecting their common ancestry. In fact, when scientists have put human disease genes into flies, they often cause the same symptoms in the insects as they do in people. Furthermore, addition of a normal human gene can sometimes compensate for the deletion of the same gene from the fly.

*** The most common view among scientists is that mathematics and physics are quite different. Physics describes the universe and depends on experiment and observation. The particular laws that govern our universe, such as Newton’s laws of motion, must be determined empirically and then asserted like axioms that cannot be logically proved, merely verified. Mathematics, on the other hand, is somehow independent of the universe. Results and theorems, such as the properties of the integers and real numbers, do not depend in any way on the particular nature of reality in which we find ourselves. Mathematical truths would be true in any universe.

*** Carbon dioxide (CO2), like water and most other pure substances, exists in solid,liquid, and gaseous states and can undergo changes from one state to another. Solid CO2, however, has an interesting property: at normal pressures, it passes directly to the gaseous state without first melting to the liquid state. This property, together with the fact that this change occurs at -78°C, makes solid CO2useful for keeping materials very cold. Because solid CO2cools other objects and does not leave a liquid residue, it is called “dry ice”. As for liquid CO2, it is obtained by putting carbon dioxide gas under pressure. When liquid CO2evaporates, it absorbs large quantities of heat, cooling as low as -57°C. Because of this property, it is often used as a refrigerant. If the compressed gas from the evaporating CO2liquid is allowed to expand through a valve, the rapidly cooled vapour forms solid carbon dioxide “snow”. This CO2 snow is compacted into blocks and is the source of dry ice.

*** The entire future of human space exploration rests on a patch of lunar ice. For the past two years NASA has focused on designing a new crew vehicle and launch system that could return astronauts to the moon by 2018. The agency’s ultimate goal is to establish a permanent lunar base and use it for a human mission to Mars. But the grand plan depends on a risky prediction that NASA will find water ice in a permanently shadowed crater basin at one of the moon’s poles. Plentiful ice deposits would be an asset for lunar colonists, who could use the water for life support or convert it to hydrogen and oxygen rocket fuel. And two orbiters sent to the moon in the 1990s, Clementine and Lunar Prospector, found evidence of ice in perpetually shadowed polar areas where consistently frigid temperatures would preserve the water carried to the moon by comet and meteorite impacts. But somescientists have disputed Clementine’s radar data, and the anomalous neutron emissions observed by Lunar Prospector could have been caused by atomic hydrogen in the lunar soil instead of ice.

*** The primary means of reproduction and dispersal for Earth’s most successful plants is seeds, which develop from the female gametophyte and its associated tissues. Seed plants show the greatest evolutionary complexity in the plant kingdom and are the dominant plants in most terrestrial environments. Seeds are reproductively superior to spores for three main reasons. First, a seed contains a multicellular, well-developed young plant with embryonic root, stem, and leaves already formed, whereas a spore is a single cell. Second, a seed contains a food supply. After germination, the plant embryo is nourished by food stored in the seed until it becomes selfsufficient. Because a spore is a single cell, few food reserves exist for the plant that develops from a spore. Third, a seed is protected by a resistant seed coat. Like spores, seeds can live for extended periods of time at reduced rates of metabolism, germinating when conditions become favourable.

*** Despite bacteria’s presence in all parts of the planet, their diversity in the world’s soils is poorly understood. To better understand what makes the organisms thrive, Duke University researchers trekked far and wide to collect a few centimetres of dirt as samples from 98 locations across North and South America, then analyzed each sample for genetic variation. To their surprise, the strongest predictor of high diversity was neutral pH. The acidic soil of the Peruvian Amazon,for example, harboured far fewer bacterial species than did the neutral dirt of the arid American Southwest. “There are a lot of variables that didn’t turn out to be very important,” says the researcher Robert Jackson, who adds that a more complete search for different habitats might turn up other stimulators of diversity, such as carbon abundance.

ÜDS SAĞLIK 2006 SONBAHAR

*** Few recent papers in biology have received as much praise as Woo Suk Hwang’s 2004 and 2005 announcements of cloning human embryonic stem cells – or fallen as fast into disrepute with the discovery that they were rank fakes. Embryonic stem cell (ESC) research is no less promising today than it was before Hwang’s deceit was revealed; most investigators continue to believe that it will eventually yield revolutionary medical treatments. The fact that no one has yet derived ESCs from cloned human embryos simply means that the science is less advanced than has been supposed over the past two years. Still, Hwang has badly undermined the reputation of a field that already has more than its share of political and public relations problems. Some longtime opponents of ESC research will undoubtedly argue that Hwang’s lies only prove that the investigators cannot be trusted to conduct their work ethically, and the public may believe them.

*** Bats are creatures of the night that are commonly held in fear. At first glance, those fears might seem to have some medical justification. Long known as vectors for rabies, it is now thought that bats may be the origin of some of the most deadly emerging viruses, including SARS. From research with other viruses, virologist Linfa Wang of the Australian Animal Health Laboratory knew bats could get chronic infections from viruses while not getting sick, making them ideal carriers for disease. Bats, civets and a menagerie of other animals were often found caged near one another in live-animal markets in Asia. So Wang hypothesized that bats might harbour SARS as well. Wang and his colleagues analyzed blood, throat and faecal swabs from 408 wild bats from China. Genetic analysis revealed five bats, which represented three of nine species of horseshoe bats tested, possessed viruses closely related to SARS. They reported last September that the genetic variation within those coronaviruses was far greater than that seen in human or civet SARS. Therefore, bats, probably having lived longer with the diseases, may be the origin of the coronaviruses seen in other species.

*** The increasing visibility of homeless mentally ill individuals, particularly in large cities, has aroused public concern and prompted a move toward reinstitutionalization. However, an important ethical issue is involved. If such people are not readjusting to society, should they beinvoluntarily committed to a mental hospital? One of the most cherished civil rights in a democratic society is the right to liberty. It is essential that any action toward commitment safeguard this right. Some experts believe that legal action is warranted only if a person is potentially dangerous to others. The rare, but highly publicized, occasions when a mentally ill person experiencing a psychotic episode attacks an innocent bystander have generated fears for public safety. But dangerousness is difficult to predict. Studies have shown that mental health professionals are poor at predicting whether a person will commit a dangerous act. Moreover, the legal system is designed to protect people from preventive detention.

*** Much work on aging brains has focused on their failings, but a new study looks at how they succeed. In a University of Michigan at Ann Arbor report on which brain regions respond to challenging tasks, researchers found that aging brains function differently than young brains. Cindy Lustig of Ann Arbor used functional magnetic resonance imaging to observe the brains of young adults (aged 18 to 30) and seniors (65 to 92) as they tackled simple and difficult mental exercises. For the easy tasks, brain activity was very similar, but tougher challenges prompted differences. The seniors activated several frontal brain regions that the young adults did not. In addition, the younger people “turned off” parts of the brain not used during the tasks, but the elders kept those regions active. Lustig concludes that “older adults’ brains can indeed rise to the challenge, at least in some situations, but they may do so differently”.

*** Researchers are a step closer to understanding how Alzheimer’s disease takes shape – literally. A sign of Alzheimer’s is the presence of protein aggregates in the brain known as plaques. They are made up of various lengths and conformations of the beta amyloid protein. The proteins link end to end, forming long, threadlike structures called fibrils. Now biologist Roland Riek and his colleagues have constructed a three-dimensional model of the fibrils based on their own experiments and earlier data published by others. Riek says the model will help investigators to understand protein structure, which could lead to better targeted drugs. For example, molecules could be engineered to act as protein binding partners, thus interfering with fibril formation. Such a sticky molecule could also be used to diagnose the disease early. The model work might lend insight to other neurological disorders that involve fibril formation, such as Parkinson’s disease. Riek says his group will extend the three-dimensional work to other variations of the amyloid protein, because it undergoes many conformational changes on its way to forming a fibril. “We need to try to trap them in these intermediate states”, he explains.

*** Treating depression could change significantly following the results of a small Canadian clinical trial that ended in 2005. The procedure used in the study freed several patients from heavy depression that had resisted medication, talk therapy and even electroconvulsive (shock) treatment. Study co-author Helen S. Mayberg cautions that any trial so small – just six patients – must be considered provisional. Yet four of the six subjectsfelt dramatic and lasting effects. University of Toronto neurosurgeon Andres Lozano implanted battery-powered, pacemakerlike devices underneath a patient’s clavicle, then ran flexible, hair-thin electrodes to the subgenual cingulate, a well-buried cortical area that Mayberg had previously found active in depressive or sad states. The electrodes delivered pulses of four volts, 130 times a second. Mayberg hypothesized that in badly depressed patients the subgenual cingulate acts like a switch left open, allowing depressive circuits to fire more than is necessary. Her results suggest that the regular stimulation might moderate that activity. In 2005, after a year of living with the continuous impulses, the four patients had lowered their scores on the Hamilton Depression Rating Scale from the soul-deadening high 20s to between one and eight – quite healthy.

ÜDS SOSYAL 2006 SONBAHAR

*** The young child’s reliance on visual impressions is made clear by an experiment on the conservation of number. If two rows of checkers are matched one for one against each other, young children will say, correctly, that the rows have the same number of checkers. If the checkers in one row are brought closer together to form a cluster, 5-year-olds say there are now more checkers in the straight row – even though no checkers have been removed. The visual impression of a long row of checkers overrides the numerical equality that was obvious when the checkers appeared in matching rows. In contrast, 7-year-olds assume that if the number of objects was equal before, it must remain equal. At this age, numerical equality has become more significant than visual impression.

*** Not long ago, football was not a good example of globalization. The labour market in international club football was highly protected. National leagues like Italy’s Serie Aand Spain’s La Ligaimposed quotas on their teams, allowing them to import only a limited number of players. Some teams could have only two foreign players on the field. This arrangement, however, began to crumble in 1995, when the European court ruled that the difference of treatment of nationals from other EU countries was anticonstitutional. This permitted players to move freely within the EU, and made the club teams much more multi-national. Now it is not unusual for a majority of the players on a successful league team to be foreign nationals.

*** In 1964, Kitty Genovese was murdered outside her home in New York City late at night. She fought back, and the murder took over half an hour. At least 38 neighbours heard her screams for help, but nobody came to her aid. No one even called the police. The American public was horrified by this incident, and social psychologists began to investigate the causes of what at first was termed “bystander apathy”. Their work showed that “apathy” was not a very accurate term, however. It is not simple indifference that prevents bystanders from intervening in emergencies. First, there are realistic deterrents such as physical danger. Second, getting involved may mean lengthy court appearances or other entanglements. Third, emergencies are unpredictable and require quick, unplanned action; few of us are prepared for such situations. Finally, one risks making a fool of oneself by misinterpreting a situation as an emergency when it is not. Researchers concluded that “the bystander to an emergency situation is in an unenviable position. It is perhaps surprising that anyone should intervene at all”

*** Despite various scientific advances, in the early 1900s the public still did not understand mental illness and viewed mental hospitals and their inmates with fear and horror. Clifford Beers undertook the task of educating the public about mental health. As a young man, Beers developed a bipolar disorder and was confined for 3 years in several private and state hospitals. Although chains and other methods of torture had been abandoned long before, the straitjacket was still widely used to restrain excited patients. Lack of funds made the average state mental hospital –with its overcrowded wards, poor food, and unsympathetic attendants –a far from pleasant place to live. After his recovery, Beers wrote about his experiences in the now-famous book A Mind That Found Itself(1908), which aroused considerable public interest. Beers worked ceaselessly to educate the public about mental illness and helped to organize the National Committee for Mental Hygiene. In 1950, this organization joined with two related groups to form the National Association for Mental Health. The mental hygiene movement played an invaluable role in stimulating the organization of child-guidance clinics and community mental health centres to aid in the prevention and treatmentof mental disorders.

*** During the “hunger winter” of 1944 in Amsterdam, over 20,000 people died of starvation. Many of the city’s trees were cut down, and the interiors of abandoned buildings broken up for fuel. When peace came this once most beautiful and urbane of cities was in urgent need of large-scale reconstruction. In the years following the end of World War II in Europe, modern architecture had an unprecedented opportunity to demonstrate a socially minded, urban style. The consensus today is that in most places it failed. The young Dutch architect Aldo van Eyck was one of the earliest critics of the mechanistic approach taken by his modernist colleagues to urban reconstruction. The failure of architecture and planning to recreate forms of urban community and solidarity has become a problem in post-war Europe, as so many acclaimed housing estates, new towns, or newly designed urban quarters, around Europe, have been troubled by vandalism, disrepair and abandonment. Van Eyck saw this coming. In 1947 at the age of 28, he went to work for the Office for Public Works in Amsterdam and, as his first project, built a small playground. This was in line with his belief that by promoting and shaping the daily “encounter” or “inbetween-ness” of social space, architecture could humanize cities and create public trust.

*** Each year in the touristic town of Agrigento, Sicily, hundreds of illegally-built houses are bulldozed by the local government. New construction in Agrigento, home to many ancient temples which tourists come to see, has been banned since 1968. In spite of this, hundreds of new and half-built houses can be seen in the hills surrounding the archaeological park. Not only do these buildings spoil the landscape, but many are also unsafe and unsanitary. Some of the people living in these buildings pour sewage into the sea and pile garbage on roadsides since their houses are illegal and they aren’t allowed to use the city sewage system and garbage service. Several of these houses are also built on dangerous cliffs, sites that would never be allowed by Italy’s strict building codes.

ÜDS FEN 2007 İLKBAHAR

*** During our visit in the summer of 1994 to the Chernobyl Exclusion Zone, a region within a 30 km radius of the Chernobyl Nuclear Power Plant, we were amazed by the diversity of mammals living in the shadow of the ruined reactor only eight years after the meltdown. During our excursion through the woods, we trapped some of the local mice for examination in a makeshift laboratory. We were surprised to find that, although each mouse registered unprecedented levels of radiation in its bones and muscles, all the animals seemed physically normal, and many of the females were carrying normal-looking embryos. We found that the mice did not have any obvious chromosomal damage. We wondered whether the absence of injury could be explained by some sort of adaptive change, perhaps a more efficient DNA-repair mechanism, after many prior generations had been exposed to radiation. But when we transplanted wild mice from uncontaminated regions into cages in the Exclusion Zone and then examined their chromosomes, they were likewise unaffected by the radiation. In at least this respect, the mice seemed to have a natural “immunity” to harm from radiation.

*** The concentrations of methane (CH4) and carbon dioxide (CO2) gases in the atmosphere have both risen dramatically since the start of the Industrial Revolution. However, unlike its more familiar greenhouse-gas cousin, atmospheric methane has recently stopped increasing in abundance. This development wasn’t entirely unanticipated, given that the rate of increase has been slowing for at least a quarter-century. The recent stabilization of methane levels is something that some scientists are trying very hard to explain. Methane has many sources. Some are natural, such as wetlands and plants, and some are the consequences of modern society, such as landfills and wastewater treatment. Methane is destroyed principally by its reaction with the hydroxyl radical (OH) in the lower atmosphere. One theory about the stabilization of methane levels is that deforestation has reduced the number of plants contributing to atmospheric methane. Another idea is that an increase in the prevalence of tropical thunderstorms may have raised the amounts of the various nitrogen oxides high in the atmosphere. There, these gases have the side effect of boosting the production of OH, which in turn acts to destroy methane.

*** In an attempt to settle the question of whether ice exists on the moon, NASA plans to launch the Lunar Reconnaissance Orbiter (LRO) in 2008. Travelling in a polar orbit only 50 kilometres above the moon’s surface, the probe will focus a high-resolution neutron sensor on the suspected ice deposits to determine their precise locations. But because the ice is probably buried and mixed with lunar dirt, NASA will also need to land a probe to dig up and analyze soil samples. This mission, scheduled for 2011, is a challenging one because instruments operating in shadowed areas cannot use solar power. The craft could land at a sunlit site and send a battery-powered vehicle into a dark crater, but the batteries would quickly die. A radioisotope thermal generator could provide electricity using heat from plutonium decay, but NASA is leaning against this option because it is expensive and controversial. Another idea under consideration is sending a probe that could hop from place to place on the lunar surface by restarting its landing rockets, lifting the craft to 100 metres above its original landing site and moving it to another spot in the crater basin to hunt for ice. Investigating more than one site is crucial because the ice may be unevenly distributed. Yet another alternative would be to fire groundpenetrating instruments at several places in the shadowed basin, either from a lander at the crater’s rim or from an orbiting craft.

*** In 1980, the physicist Luís Alvarez and his son Walter advanced a startling theory about the demise of the dinosaurs: that it was caused by forces that came from beyond this world. They hypothesized that perhaps a meteor impact had ended the age of the dinosaurs. The primary evidence was that in soil core samples taken in locations around the globe, iridium, a substance very rare on Earth but prevalent on asteroids, had been found in a thin layer of clay separating the fossil-rich rock of the late Cretaceous period (the end of the dinosaur age) and the sparsely fossiled rock of the Tertiaryperiod that followed. The Alvarezes hypothesized that a very large extraterrestrial object had slammed into the planet, sending an enormous fireball into the stratosphere, along with vast amounts of debris. A great cloud of dust enshrouded Earth, blocking sunlight for months, even years, and plants and animals perished in the ensuing cold and dark. When the dust finally settled back to Earth, it formed the telltale worldwide layer of iridium in the clay. The scientific world was not impressed by the theory. Indeed, some scientists scoffed at the Alvarezes’ hypothesis, but in 1990 scientists realized that a crater of 112 miles in diametre in Mexico and dated at 65 million years old might be evidence that the dinosaurs had indeed died out due to the effects of a giant meteor.

*** Stem cells, unlike all other cells in the body, can copy themselves indefinitely. So-called adult stem cells are found in many parts of the body, constantly rejuvenating the brain, remodelling arteries so blood can bypass clogs, and growing new skin to heal wounds. However, adult stem cells have more limited power than embryonic stem cells, which can turn into any type of cell in the body. Indeed, scientists are hoping that embryonic stem cells could be turned into neurons to fix damaged brains, cardiac cells to repair damaged hearts, or pancreatic cells to create insulin for people with diabetes. Maybe they could even be used to regenerate whole organs. To date, scientists worldwide have made more than 100 different human embryonic cell lines. Still, the existing lines have serious limitations. Most have been grown on a lattice of mouse embryonic skin cells for support. Consequently, the human embryonic cells are contaminated by mouse cells, and though they’re still useful for research, they cannot at present be used to develop therapies for humans.

*** Mount Vesuvius in southern Italy is actually a volcano inside the exploded skeleton of an older volcano. Looked at from above, the remaining ridge of a much larger volcano can be seen on the north side. This older volcano had probably erupted violently long before human settlement. Southern Italy is unstable ground. The African continental plate, on which most of the Mediterranean Sea rests, is actually diving beneath the European plate. That kind of underground collision produces molten rock, or magma, rich in volatile gases such as sulfur dioxide. Under pressure underground, these gases stay dissolved. But when the magma rises to the surface, the gases are released. Accordingly, when volcanoes like Vesuvius erupt, they tend to erupt explosively. To this day, in fact, Vesuvius remains one of the world’s most dangerous volcanoes; some 3.5 million Italians live in its shadow. Although monitoring devices are in place to warn of the volcano’s activity, if there were a major eruption with little warning, there could be a tremendous loss of life.

ÜDS SAĞLIK 2007 İLKBAHAR

*** The most common cause of cyanidepoisoning is smoke inhalation. A source to the public is acetonitrilein the form of artificial fingernail remover. Cyanide poisoning produces cellular hypoxia by binding with the ferric iron of mitochondrial cytochrome oxidase, disrupting the electron transport chain and the ability of cells to use oxygen. Patients who inhale cyanide may rapidly develop coma, shock, seizures, lactic acidosis, and respiratory and cardiac arrest. Mild exposures following smoke inhalation are now being described. Diagnosis may be difficult in these patients, and emergency administration of an antidote may be lifesaving. Patients with smoke inhalation who show evidence of lactic acidosis should be suspected of cyanide poisoning. The body has a natural enzyme, called “rhodanese”, which can complex cyanide and sulphur to form mildly toxic thiocyanate. Intravenous administration of sodium thiosulphate provides the sulphur necessary to produce thiocyanate and is relatively safe. Sodiumnitrite may also be administered, but its use is reserved for the most critical cases only, because it causes hypertension and methemoglobinemia.

*** Stimulation of several areasof the hypothalamus in the brain causes an animal to experience extreme hunger, a voracious appetite, and an intense desire to search for food. The area most associated with hunger is the lateral hypothalamic area. Damage to this area sometimes causes the animal to lose desire for food, sometimes causing lethal starvation. On the other hand, a centre inthe hypothalamus that opposes the desire for food, called the satiety centre, is located in the ventromedial nucleus. When this centre is stimulated electrically, an animal that is eating food suddenly stops eating and shows complete indifference to food. However, if this area is destroyed bilaterally, the animal cannot be satiated; instead, its hypothalamichunger centres become overactive, so that it has a voracious appetite, resulting in tremendous obesity.

*** When doctors suspect meningitis, they must quickly determine whether it is from a bacterial, viral, fungal, or other type of infection or from irritation caused by something other than an infection (for example, a chemical). The possible causes are many, and the treatment differs for each. The test usually used to diagnose meningitis and determine its cause is the spinal tap, or lumbar puncture. A thin needle is inserted between two bones in the lower spinal column to withdraw a sample of cerebrospinal fluid from an area just below the spinal cord. The doctor then examines the fluid for bacteria under a microscope and sends a sample of it to the laboratory to be cultured and identified. The bacteria can be tested for susceptibility to treatment with different antibiotics. The sugar level, an increase in protein, and the number and type of white blood cells in the fluid also help determine the type of infection.

*** The ultimate importance of the pulmonary ventilatory system is to continually renew the air in the gas exchange areas of the lungs, where the air is in proximity to pulmonary blood. These areas include the alveoli, alveolar sacs, alveolar ducts and respiratory bronchioles. However, during normal quiet respiration, the volumeof air is only enough to fill the respiratory passageways down as far as the terminal bronchioles, with only a small portion of the inspired air actually flowing all the way to the alveoli. Some of the molecules in the inspired air travel the final short distance to the alveoli through diffusion, but most never reach the gas exchange areas of the lungs, instead going to fill respiratory passages where gas exchange does not occur, such as the nose, pharynx and trachea. This air is called dead space airbecause it is not useful for the gas exchange process; the respiratory passages where no gas exchange takes place are called dead space. On expiration, the air in the dead space is expired first, before any of the air from the alveoli reaches the atmosphere. Therefore, the dead space does not aid in removal of the expiratory gases from the lungs.

*** In dialysis with an artificial kidney, the rate of movement of solute across the dialyzing membrane depends on four variables: the concentration gradient of the solute between the blood and the dialyzing fluid, the permeability of the membrane to the solute, the surface area of the membrane, and the length of time that the blood and fluid remain in contact with the membrane. Thus, the maximum rate of solute transfer occurs initially when the concentration gradient is greatest (when dialysis is begun) and slows down as the concentration gradient is dissipated. In a flowing system such as haemodialysis, in which blood and dialysate fluid flow through an artificial kidney, the dissipation of the concentration gradient can be reduced, and diffusion of solute across the membrane can be optimized by increasing the flow rate of either or both the blood and dialyzing fluid.

*** Virologist Robert Webster thinks that the H5N1 strain of the avian influenza virus poses the most serious public health threat since the Spanish flu pandemic of 1918, which killed an estimated 40 million to 100 million people worldwide. Although the H5N1 strain has so far shown no signs that it has acquired the ability to transmit easily from person to person, Webster says that it is only a matter of time before it does. For that to happen, Webster and others believe that a version of the human flu virus, which is easily transmittable between people, and the H5N1 avian virus would have to infect the same mammalian cell at the same time and recombine their DNA. If H5N1 picks up those genes from the human flu virus that enable it to spread from person to person, Webster says that virtually nobody will have immunity to it, and many deaths may ensue.

ÜDS SOSYAL 2007 İLKBAHAR

*** The Roman city of Pompeii in A.D. 79 was a thriving provincial centre, a few miles from the Bay of Naples, with a population of between 10,000 and 20,000 people. Its narrow streets, made narrower by street vendors and shops with cloth awnings for shade, were full of shoppers, tavern-goers, slaves, and vacationers from the North. A huge new aqueduct supplied running water from the Lower Apennine mountains, which flowed from fountains throughout the city, even in private homes. But the key to Pompeii’s prosperity, and that of smaller settlements nearby like Oplontis and Terzigna, was the region’s rich black earth provided by Mount Vesuvius’ volcanic eruptions. “One of the ironies of volcanoes is that they tend to produce very fertile soils, and that tends to tempt people to live around them”, says geologist Philip Janey. Had Roman knowledge in the summer of A.D. 79 been less mythological and more geological, the Pompeiians might have recognized the danger signs from Mount Vesuvius and escaped the volcanic eruption that was to follow.

*** Artist Paul Cézanne wanted to make paint “bleed”. The old masters, he said, painted warmblooded flesh and made the trees look warm and alive, and he would too. He wanted to capture “the green odour” of his Provence fields and “the perfume of marble from Saint-Victoire”, the mountain that was the subject of so many of his paintings. He was bold, spreading and slapping paint onto his still-lifes with a palette knife. “I will astonish Paris with an apple”, he boasted. In the years when his friends Manet, Monet, Pissarro and Renoir were finally gaining acceptance, Cézanne worked furiously and mostly in isolation, ridiculed by critics and mocked by the public, sometimes tearing up his own canvases. He wanted more than the quick impressions of the Impressionists, and devoted himself to studying the natural world. He called himself a “slave to nature”, but he knew that he could never completely capture the natural landscape on canvas.

*** Angola’s emergence as a serious player in the global oil sector has been underlined by the publication of its latest production figures. After several years of slow output growth, the fruits of the multi-billion dollar deepwater investment programme are finally feeding through. Thus, production reached 1.3 million barrels a day during the final quarter of 2005. With the new deepwater fields now coming into production, the government’s target of producing 2 million barrels a day by 2008 now seems eminently within reach. In the meantime, official Angolan government figures indicate that oil production averaged 1.25 million barrels a day during the course of 2005, a steep jump on the year before. In addition, the government’s estimate of proven oil reserves has finally been increased from 5.4 billion barrels to 12.4 billion barrels. This shows that, as sub-Saharan Africa’s second biggest oil producer after Nigeria, Angola is in an excellent position to overtake countries such as Libya and Algeria in the table of oil powers on the continent as a whole.

*** Supporters of today’s international criminal tribunals say that their work builds on the post-World War II tribunals in Nuremberg and, to a lesser degree, Tokyo. As a matter of legal doctrine, that is true. The category of “crimes against humanity”, for example, was developed at Nuremberg and is now a central element in many prosecutions. But there is a critical difference between now and then. The courts in Nuremberg and Tokyo were part of a broader political project that aimed to rehabilitate Germany and Japan, respectively, both socially and economically, not simply to try guilt or innocence or hand out harsh punishments. These were military courts that operated with military efficiency, and the Allies could then focus fully on the reconstruction of these countries. Yet, the international courts for the former Yugoslavia, Rwanda, and the new International Criminal Court (ICC) in the Hague, on the other hand, operate under civilian law and provide generous protection to defendants. The result is a ballooning of the court timelines and costs. For instance, it tookthe International Criminal Tribunal for Rwanda (ICTR) 10 years to complete the same number of trials that Nuremberg conducted in less than a year. Indeed, it is clear that, because of their protracted proceedings and excessive costs, today’s war crimes trials cannot serve the decisive political and social function that Nuremberg did.

*** The ability to deal with numbers and mathematical concepts reveals an interesting pattern of differences between girls and boys. Girls usually begin counting before boys. Throughout the primary-school years and middle school, girls are better at computational problems, whereas boys do better with mathreasoning problems. During this period, girls also tend to get higher grades. By high school, however, boys begin to perform better, especially at the higher levels of ability. Some psychologists believe this advantage is genetic, but others think that it may derive, in part, from males’ use of more effective strategies and their lower level of anxiety when approaching mathematics problems. It has also been suggested that the sex differences come about to some degree because girls view math as a male activity (and, therefore, have less interest in it) and because some parents and teachers offer greater encouragement to males in this area. Some studies support this analysis, but others do not. In this area, too, biological and socialization factors probably combine to produce the observed differences.

*** The printing press was first introduced into England by William Caxton in the last quarter of the fifteenth century. During his earlier travels in Europe, he had seen the newly invented system of printing from moveable type in Germany. He set up his own press in London in 1476. This initiated a major change in English literature. Now books did not have to be laboriously copied by hand. Soon, they would become relatively cheap. With books easily obtainable, more people could learn to read, and more books would be produced. The experience of literature would soon shift from the breathless group of listeners gathered in a hall or around a fire, hearing an old tale told once more, to the solitary individual, alone with the thoughts and feelings of another person speaking from the printed page.

ÜDS FEN 2007 SONBAHAR

*** Robots make unlikely green warriors, but they could soon be doing their bit for the environment. Trials of a Danish robot that maps the position of weeds growing among crops suggest that herbicide use could be reduced by 70 per cent if farmers used it to adopt more selective spraying techniques. Actually, the robot drives across fields scanning the ground for any weeds and noting their positions. A later version will be able to kill the weeds too by applying a few drops of herbicide. But the longer-term goal is to avoid herbicides altogether by having the robot pluck the weeds out of the ground rather than poisoning them. Although weedkilling robots have already been put to work in the United States, they cannot be used for agricultural purposes because they do not distinguish between plant species and tend to treat anything green as a weed. Instead, they are used to clear unwanted plants from railways and airport runways.

*** The world now recycles just over 50 per cent of the paper it uses. Reprocessing plants are being established in most countries. However, trees will never be fully spared because of the use of wood fibres themselves. Pure pulp is rich in water, which provides for ample hydrogen bonding that holds fibres together when made into paper. But each time a fibre is cleaned, de-inked and dried in a reprocessing plant, only 80 per cent of the bonds are recovered. After four or five recyclings, a fibre can no longer make strong enough bonds. Engineers can do little that is economically viable to overcome this physical limitation, so they focus on reducing the cost of reprocessing fresher fibres. One main challenge is finding a better way to neutralize “stickies”, which is the mess of adhesives from stamps, labels, seals, tape, magazine spines and various other sources, that jam the machinery. The industry has been working for a decade to find a chemical process that will break down stickies, but no full solution has been found yet.

*** The first documented scheme for in-flight refuelling came from a young Russian aviator named Alexander de Seversky. His father owned a plane and taught him to fly when he was in his early teens. In 1917, when he was 23, Seversky proposed a method for extending flight: One plane could carry extra fuel and deliver it to another through a hose. After the Russian Revolution, Russia’s new Bolshevik government sent him to the United States to study aircraft design, and he stayed there when political developments made his return to Russia dangerous. He got a job as an aeronautical engineer for the US War Department and was awarded the world’s first patent for air-to-air refuelling, in which large fuel tankers would supply fuel to fighter aircraft while in flight. Seversky went on to a distinguished career in airplane design and achieved perhaps his greatest fame as the author of the influential 1942 book Victory through Air Power. He never put his refuelling plan into action, though, and other aviators later came up with ideas of their own.

*** Mount Everest is the highest mountain on Earth above sea level, but it is not the world’s tallest. That honour goes to the Hawaiian volcano Mauna Kea. When measured from its base on the Pacific Ocean floor, it is about 1,000 metres taller than Mount Everest. Mauna Kea is part of a 5,600-kilometre-long chain of volcanoes stretching westward from the main Hawaiian island. This volcanic chain is formed by small convection streams called “hot spots”, just below the Earth’s crust, where magma rises from the hotter parts of the mantle, the region between the crust and the core of the earth. These hot spots melt sections of the tectonic plates moving above them, causing magma and bits of the molten plate to erupt onto the sea floor. Over time, the lava accumulates, forming a mountain that rises above sea level. The moving tectonic plates carry the newly-formed mountain away from its original location, as newer volcanoes continue to form in the same spot.

*** One of the most pressing international priorities is to control the dissemination ofnuclear materials that could be used in attacks by terrorists or rogue states. Nuclear materials contain unstable isotopes, which emit x-rays and gamma rays. The characteristic energies of these photons provide a fingerprint revealing which radioactive isotopes are present. Unfortunately, some isotopes that occur in benign applications emit gamma rays with energies that are very similar to those emitted by materials used in weapons, which leads to ambiguous identifications and false alarms. This problem has been worrying the United States, which is installing thousands of radiation portal monitors to detect the gamma rays emitted by nuclear materials carried by vehicles crossing the Canadian and Mexican borders. One of the worst fears of the authorities is that terrorists might smuggle highly-enriched uranium into the country to build a crude Hiroshima-style atomic bomb.

*** Meteorites offer glimpses of the earliest stages of planetary formation. Stony-iron meterorites come in two main classes: pallasites and mesosiderites, and it was previously thought they may have had similar origins. A new study, however, has revealed that their oxygen isotope properties differ and that they come from distinct places. Accordingly, the characteristics of mesosiderites suggest they came from the third largest asteroid, Vesta, which is the target of the NASA Dawn Mission. On the other hand, pallasites are made of mixed core-mantle material from a disrupted asteroid, indicating that extensive asteroid deformation was an integral part of planetary enlargement in the early solar system.

ÜDS SAĞLIK 2007 SONBAHAR

*** Pain is a signal that protects us from harmful stimuli. An excess of any type of stimulus such as pressure, heat, cold, excessive mechanical stretch, and specific chemical compounds stimulates pain receptors. In the human body, pain receptors are the tips of certain sensory neurons found in almost every tissue. However, most internal organs are poorly supplied with pain receptors. For this reason, pain from internal structures is often difficult to locate. In fact, pain is often not projected back to the organ that is stimulated. Instead, it is referred to an area just under the skin that may be some distance from the organ involved. The area to which the pain is referred is connected to nerve fibres from the same level of the spinal cord as the organ involved.

*** Protected inside the bony vertebrae of the spine is an inch-thick gelatinous bundle of nervous tissue called “the spinal cord”, which acts as the central communication conduit between the brain and the rest of the body. Millions of nerve fibres carry motor information from the brain to the muscles, while other fibres bring sensory information from the body to the brain. In its structure and functions, the spinal cord may be compared to a transcontinental telephone cable jam-packed with wires, each of which carries messages both ways. But what happens if that cable is cut? Signals cannot get through, communication is lost, and the cable must be repaired or replaced. In humans, though, this is not a simple process due to the sensitive nature of the spinal cord. The spinal cord is rarely severed because the vertebrae provide rigid protection. However, a traumatic blow to the spinal column and subsequent bleeding, swelling and scarring can crush the delicate nerve bundles and prevent signals from passing. The result may be a debilitating injury.

*** Our kidneys excrete metabolic wastes and help regulate the volume and composition of body fluids. Their vital function is compromised in more than 13 million people in the United States who suffer from kidney disease. In fact, kidney disease ranks fourth in prevalence among major human diseases in the United States. Kidney function can be impaired by infections, poisoning by substances such as mercury, lesions, tumours, kidney stones, shock or circulatory disease. For instance, one of the most common kidney diseases both in the United States and in the world is glomerulonephritis,which is related to the damage of the kidney’s filtering units. The damage is thought to result from an autoimmune response.

*** Immune deficiency, which in fact means an inadequate immune response, may occur for several reasons. For example, it is a side effect of most chemotherapy drugs used to treat cancer. Immune deficiency also occurs naturally. Although the immune system is not functional at birth, infants are protected by antibodies they have received from their mother through the placenta, and infants who breastfeed also receive antibodies from their mother’s milk. These antibodies offer protection until the infant’s own immune system develops during the first months of life. In rarecases, the immune system fails to develop, leaving the child without immune protection. Exposure to any virus or bacterium can be fatal to these children. Therefore, they are sealed into sterile quarters to isolate them from the microorganisms that are part of the normal world.

*** The causes of schizophrenia are unknown, although the disease has a strong genetic component. Studies of identical twins show that if one twin has schizophrenia, there is a 50% chance that the other twin will have it, too. Since identical twins share identical genes, this indicates that schizophrenia has an equally strong environmental component, the nature of which has not been identified. Current treatments for schizophrenia focus on brain pathways that use dopamine as a neurotransmitter. Despite their ability to alleviate symptoms, many of the drugs used to treat schizophrenia have such negative side effects thatpatients frequently stop taking them. Now that the human genome has been sequenced, there is a vigorous effort under way to find the mutant genes that predispose a person to the disease. This effort includes sequencing DNA from families with a high incidence of schizophrenia.

*** People who believe they may be infected with HIV, as well as those who know they are, can benefit from various psychological interventions. People with high-risk behaviours may have difficulty deciding whether to be tested for HIV, and psychologists can provide both information and support for these people. A significant minority of homosexual and bisexual men, intravenous drug users, and a larger proportion of heterosexual men and women with multiple partners and inconsistent users of condoms have never been tested for HIV. Indeed, an estimated 70% of people who are HIV-positive have not been tested and thus do not know their HIV status. Because HIV infection has a long incubation period, at-risk heterosexual men and women may contaminate others for years before they learn they have HIV. However, people learning of an HIVpositive test result typically react with increased anxiety, depression, anger and distress. Therefore, trained psychotherapists are needed to help such people cope with their diagnosis.

ÜDS SOSYAL 2007 SONBAHAR

*** We commonly speak of both lawand laws,and these terms, though not used with precision, point to two different aspects under which legal science may be approached. The laws of a country are thought of as separate, distinct, individual rules; the law of a country, however much we may analyse it into separate rules, is something more than the mere sum of such rules. It is, rather, a whole, a system which orders our conduct and in which the separate rules have their place and their relation to each other and to the whole. Moreover, it is never completely exhausted by any analysis, however far the analysis may be pushed, and however much the analysis may be necessary to our understanding of the whole. Thus, each rule which we call a law is part of the whole we call the law. Lawyers generally speak of law; laymen more often of laws.

*** We have all heard people from other countries described in very general terms. For instance, it has been said that “Germans work hard” and “Americans are friendly”. Such generalizations or stereotypes are very crude, and common sense tells us that not all Germans work hard and not all Americans are friendly. At the same time, there appears to be some truth in these generalizations since people from different countries share different characteristics. What these crude statements acknowledge, however, is that people from differentcountries have distinctive cultures and social customs. A society’s culture includes its customs, values, beliefs, ideas and the artifacts it produces. Attitudes towards such things as work, leisure, wealth, the role of women, and the value of education in one society’s culture might be significantly different from the attitudes and values found in another society’s culture. This is also the case regarding attitudes found in different countries towards politics and the political system.

*** Four years ago in 2003, when Paul Barrett first began planning an ambitious book on Muslims in America, who would have thought thatthe topic would still be of such urgent interest by the time it was published early this year? But, if anything, intervening events have made Barrett’s exploration of American Muslims more timely and important than ever. With the United States even more deeply embroiled in warfare in Iraq and Afghanistan, with sectarian conflict exploding within the Muslim world, with tensions high among Muslim populations in Europe, and with relations strained between some leaders of the Christian and Islamic religions, readers need as much information as they can get about Islam and its adherents. In his book, Barrett notes that, despite being targets of suspicion in the wake of the attacks of September 11, Muslims in America, as a group, offer a perfect illustration of old-fashioned American assimilation. Overall, they are prosperous, well-educated, politically active, and successful in business and the professions.

*** The way in which British people view Britain’s role in the world is still influenced by its past. Today Britain is an important regional power, but in the recent past it was a world power. Until World War II, Britain ruled the largest empire that the world has ever known. Incredible as it may seem today, during the 1920s, almost one-fifth of the world’s population lived under British rule. But the empire disappeared rapidly during the 1940s, 1950s and 1960s as the colonies that had made up the empire gained their independence. In the aftermath of the empire, British leaders were not very successful in adapting to play a much smaller part in world affairs. However, Britain’s allies made it clear that they no longer saw Britain as a major force in world politics. The “special relationship” which was said to exist between Britain and the United States weakened as other European countries, particularly Germany, recovered after World War II. So, whereas London had been recognized by American leaders as the “capital” of Europe during the 1940s and the 1950s, Bonn was seen as the new capital of Europe during the 1960s.

*** Kierkegaard was one of the most original thinkers of the nineteenth century in Europe. He wrote widely on religious, philosophical, and literary themes. However, his peculiar manner of presenting some of his leading ideas initially obscured their fundamental significance. He developed his views in strong opposition to prevailing opinions, such as certain metaphysical claims about the relation of thought to existence. He reacted against the ethical and religious theories of Kant and Hegel. Moreover, he opposed the doctrines and ideas which were being advanced by some of his contemporaries like Feuerbach and Marx. His discussion of the human condition, which emphasizes the significance of individual choice, has arguably been his most striking philosophical legacy, particularly for the growth of existentialism.

*** Developing markets, historically the domain of hyperinflation and political manipulation, now enjoy high surpluses, thanks to record commodity prices and severe fiscal discipline. Since 2001 these economies have achieved three times the average annual per-capita economic growth of their developed counterparts and now represent a quarter of global output. Stocks in emerging markets are causing much excitement among investors. However, too much excitement invites peril. Emerging markets have undeniably changed in the past decade, but lately they are looking overgrown, and even a minor crisis could send them tumbling. And while the potential triggers for a fall have changed, they are still there. As economies in the developing world get stronger, governments are getting more assertive and meddling with both companies and neighbouring countries, increasing political risk.

ÜDS FEN 2008 İLKBAHAR

*** There were many heated debates in the nineteenth century about the relationship between chemical reactions and living organisms. Some scientists felt that fermentation was an activity of living things and, therefore, could not take place outside of living cells. This was proved by the work Louis Pasteur undertook for the French wine industry. Indeed, in the 1850s, the French wine industry was having serious trouble with wine that had spoiled. The French emperor, Napoleon III, called in Pasteur to help. Pasteur knew that the fermentation which produced wine was caused by living yeast cells. But now he found that certain bacteria could also carry out fermentation. He discovered that fermentation by bacteria spoils wine because it produces vinegar (acetic acid) instead of the alcohol produced by yeast. Pasteur suggested that the winemakers heat the wine for a short time to destroy the bacteria. They were horrified, but it worked. The process, pasteurization, is still usedtoday, especially for milk.

*** Pluto, which was until recently regarded as the outermost and smallest planet in the solar system, has never been visited by an exploring spacecraft. So little is known about it that it is difficult to classify. Its distance from Earth is so great that the Hubble Space Telescope cannot reveal its surface features. Appropriately named for the Roman god of the underworld, it must be frozen, dark, and dead. Its mean distance from the Sun is 5,900 million kilometres. In fact, it has the most eccentric orbit in the solar system, bringing it at times closer to the Sun than Neptune. Furthermore, there is evidence that Pluto has an atmosphere, containing methane, and a polar ice cap that increases and decreases in size with Pluto’s seasons. It is not known to have water. The Hubble Space Telescope’s faint-object camera revealed light and dark regions on Pluto, indicating an ice cap at the north pole. It is not known if there is an ice cap at Pluto’s south pole.

*** Fossils are the remains of organisms which have endured for fantastic periods of time. Fossils can be bones or teeth or even plant or animal imprints preserved in rock since prehistoric times. The appearance of fossils in rock has been a source of wonder and fascination to man for centuries. The fossil of an ancient sea animal was even found among the possessions of a prehistoric man. Many people have tried to explain fossils. Aristotle believed they were the remains of living creatures, but thought the creatures grew in the rocks. Some people believed that fossils were placed in rocks by evil spirits. Other explanationswere remarkably modern. For example, Herodotus, an ancient Greek historian, observed fossil seashells in the Libyan desert in 450 B.C. and guessed that the Mediterranean Sea had once reached much farther south than it does today.

*** Today the world faces a growing crisis over the management of its great rivers. In recent years, most of the great rivers in the world, such as the Yellow River in China, the Indus, the Colorado, and the Nile, have all periodically run empty because mankind has used their every last drop.Indeed, there is a huge unmet demand in the world for water. More than a billion people have no access to clean drinking water, and while it is hoped that this figure will be halved by 2015, nobody is sure where the water will come from. With today’s trends, one-third of the world population will be seriously short of water by 2025. Politicians in China, India, Pakistan, Egypt and other waterstressed countries want their water engineers to find solutions – and fast.

*** A population is a group of individual organisms of the same kind that are limited to some particular space. The most familiar example is the human population, but there are also populations of animals and plants everywhere on Earth. In fact, scientists regard a population as a biological unit that has both structure and function. The parts of a population are its individual members. The functions of a population are similar to those of other biological units: growth, development, and self-maintenance in a changing environment. Individuals enter a population by birth and by moving in, that is, by immigration. Individuals leave a population by death and by moving out, that is, by emigration. If the environment of a population remains the same, loss and replacement of members are in balance. The population will be able to survive in that particular environment. If the environment changes, however, loss or addition of members increases or decreases the size of the population.

*** The huge ice sheet covering Greenland, which is the world’s largest island, provides a habitat for many arctic species and holds nearly 8 per cent of the world’s freshwater. It is, on average, 5,000 feet thick and is constantly being replaced as snow falls each winter. Over the course of centuries, the snow compacts into ice, which slides towards the ocean. In recent years, higher atmospheric concentrations of heat-trapping gases have accelerated that process. As temperatures rise, the top layers melt, giving way to darker, heat-absorbing ice and liquid water. The meltwater seeps down to the rock below, lubricating the ice mass and speeding its slide into the sea.

ÜDS SAĞLIK 2008 İLKBAHAR

*** Autism, from the Greek word for “self,” was first identified as a disorder in 1943. Initially, it was thought to be a psychological disorder brought on by cold or unemotional mothers, and curable by intensive sessions of psychotherapy. During the 1960s, specialists realized that autistics frequently had epilepsy and abnormal brain scans, which led to the condition being recognized as a brain disorder by the 1970s. Autism is now known to be a hereditary neurological condition, about three times more common in boys than girls. Usually, autistics lack the ability to relate normally to other people and have an anxious desire to maintain a routine, which evolves with age into intense interests or obsessions. Many autistic people deliver monologues on topics while unaware of other people’s comments or possible discomfort. There are several related, but different, forms of autism. Depending on the severity, symptoms can sometimes be alleviated with carefully controlled antidepressants, although sufferers typically find it difficult to function normally in society.

*** People who fail to eat enough food to meet energy needs risk nutrient deficiencies, including thiamin deficiency. Inadequate thiamin intakes have been reported among malnourished and homeless people. Similarly, people who derive most of their energy from empty-kcalorie items, like alcohol, risk thiamin deficiency. Alcohol contributes energy, but provides few, if any, nutrients and often displaces food. In addition, alcohol enhances thiamin excretion in the urine, doubling the risk of deficiency. Prolonged thiamin deficiency can result in the disease “beriberi,” which was first observed in East Asia when the custom of polishing rice became widespread. Rice provided 80 per cent of the energy intake of the people of that area, and ricehulls (the outer skin of rice) were their principal source of thiamin. When the hulls were removed, beriberi spread like wildfire. Because thiamin participates in nerve processes, paralysis sets in when it islacking. The symptoms of beriberi include damage to the nervous system as well as to the heart and other muscles.

*** Because oxygen is one of the major substances required for chemical reactions in the cells, it is fortunate that the body has a special control mechanism to maintain an almost exact and constant oxygen concentration in the extracellular fluid. This mechanism depends principally on the chemical characteristic of haemoglobin, which is present in all red blood cells. Haemoglobin combines with oxygen as blood passes through the lungs. Then, as the blood passes through the tissue capillaries, haemoglobin, because of its own strong chemical affinity for oxygen, does not release oxygen into the tissue fluid if too much oxygen is already there. If the oxygen concentration is too low, however, sufficient amounts are released to re-establish adequate tissue oxygen concentration. Thus, the regulation of oxygen concentration in the tissues depends principally on the chemical characteristics of haemoglobin itself.

*** The single most effective step people can take against hypertension is to find out whether they have it. At check-up time, a health-care professional can provide an accurate resting blood pressure reading. Under normal conditions, blood pressure fluctuates continously in response to a variety of factors including such things as talking or shifting position. Some people react emotionally to the procedure, which raises the blood pressure reading. For these reasons, if the resting blood pressure is above normal, the reading should be repeated before confirming the diagnosis ofhypertension. Thereafter, the blood pressure should be checked regularly. In general, efforts to reduce high blood pressure focus on weight control, because excess body fat, especially abdominal fat, can precipitate hypertension. Indeed, weightloss alone is one of the most effective nondrug treatments for hypertension. Those who are using drugs to control their blood pressure can often reduce or discontinue the drugs if they lose weight. Even a modest loss of 5 kilograms may significantly lower blood pressure.

*** The evidence linking dietary fat with cancer is less conclusive than for heart disease, but it does suggest an association between total fat intake and some types of cancer. Dietary fat seems not to initiate cancer development but to promote cancer once it has arisen. Some studies report a relationship between specific cancers and saturated fat or dietary fat from animal sources, which is mostly saturated. Thus, health advice to reduce the risk of cancer parallels that given toreduce the risk of heart disease: reduce total fat intake, especially saturated fat. The relationship between dietary fat and the risk of cancer differs for various types of cancers. In the case of breast cancer, some studies indicate little or no association between dietary fat and cancer. Others find that total energy intake is a better predictor than the percentage of kcalories from fat. In the case of prostate cancer, there does appear to be a strong association with fat. This association appears to be due primarily to saturated fat from meats; fat from milk or fish has not been implicated in cancer risk.

*** Oral cancers develop in 30,000 Americans and cause 8,000 deaths each year, mostly in people over age 40. This represents about 2.5 per cent of cancer cases and 1.5 per cent of all cancerrelated deaths. Clearly this is a high rate considering the small size of the mouth in relation to the rest of the body. Along with cancers of the lung and skin, cancers of the mouth are more preventable than most other cancers. Non-cancerous and cancerous growths can originate in any type of tissue in and around the mouth, including bone, muscle, and nerve. Rarely, cancers found in the mouth region have spread there from other parts of the body – most commonly the lung, breast and prostate. Screening for oral cancer should be an integral part of medical and dental examinations because early detection is critical. Cancers less than a half inch across can usually be cured easily. Unfortunately, most oral cancers aren’t diagnosed until they’ve spread to the lymph nodes of the jaw and neck. Because of delayed detection, 25 per cent of oral cancers are fatal.

ÜDS SOSYAL 2008 İLKBAHAR

*** The most important idea of the nineteenth century in Britain was that everyone had the right to personal freedom, and this became the basis of capitalism. This idea, which had originated with Adam Smith in the eighteenth century, spread widely due to the popularity of his book The Wealth of Nations. After Adam Smith, several capitalist economists argued that the government should not interfere in trade and industry at all. Fewer laws, they claimed, meant more freedom, and freedom for individuals would lead to happiness for the greatest number of people. These ideas were eagerly accepted by the growing middle class. However, it soon became very clear that the freedom of factory owners to do as they pleased had led to slavery and misery for the poor, not to happiness or freedom. By 1820, more and more people had begun to accept the idea that the government must interfere toprotect the poor and the weak. The result was a number of laws to improve working conditions. For instance, one of the laws, which went into effect in 1833, limited the number of hours that women and children were allowed to work.

*** During the economic depression that affected the whole Western world in the 1930s, with its mass unemployment, poverty and other social ills, governments, for the mostpart, did nothing. The accepted wisdom was that, given time, the free market would solve its own problems and that government interference would only make things worse. John Maynard Keynes, the British economist who challenged this belief, argued that it was the proper responsibility of governments to prevent both booms and recessions in order to maintain gradual economic growth and permanent full employment. He maintained that this could be done by manipulating taxation, credit and public expenditure. If the economy was growing too fast, then money and, therefore, demand could be taken out of the economy by higher taxes, lower government spending and by making it harder to borrow money. If there was recession and growing unemployment, then the government could put money into the economy through lower taxes, higher public expenditure and easier credit. Thus, demand could be encouraged. If, as a result, there was money in people’s pockets, then more would be spent on goods and more people would be needed to make the goods to fulfil the extra demand, and this would reduce unemployment.

*** The seventeenth century is probably the first in English history in which more people emigrated than immigrated. In the course of the century, something over one-third of a million people, mainly young adult males, emigrated across the Atlantic. The largest single group made for the West Indies; a second substantial group made for America, in particular Virginia and Catholic Maryland, and even Puritan New England. The pattern of emigration was a fluctuating one, but it probably reached its peak in the 1650s and 1660s. For most of those who emigrated, the search for employment and a better life was almost certainly the principal cause of their departure. For a clear minority, however, freedom from religious persecution took precedence. Moreover, an increasing number were forcibly transported as a punishment for criminal acts. In addition to these transatlantic emigrants, an unknown number emigrated to Europe and settled there. The largest group were probably the sons of Catholic families making for religious houses in France and elsewhere. There were also some adventurers who were willing to fight in any cause if the pay were good.

*** Since the dawn of civilization, the Middle East, a region at the crossroads of Africa, Asia and Europe, has been important to large and small powers alike, from the empires of the Eastto the imperial powers of the West. The opening of the Suez Canal in 1869, which transformed maritime travel between Europe and Asia, added to European interest. The region’s other riches also encouraged European intervention and rivalries. This resulted in a series of confrontations between the Ottoman Empire and its European adversaries, and finally in the collapse of the former and the direct or indirect European colonization of large parts of the region in the course of the nineteenth and early twentieth centuries. But what added to the Middle East’s importance in the twentieth century was oil, which was found in abundance in the Persian Gulf and in parts of North Africa. Moreover, in the strategic context of the Cold War, the region’s geopolitical importance provided an additional reason for the superpowers to increase their role and presence.

*** Ancient Greece consisted of a number of city-states, of which Athens was one of the greatest. In the fifth century B.C., all citizens native to Athens could both vote and speak in a government assembly; but this, of course did not apply to women and slaves. This system of “direct democracy” was feasible because Athens was a small community. Each individual could be involved, gathering collectively in the public square where decisions on government matters, such as laws and foreign affairs, were made. City administrators were expected to account for their decisions. What counted in ancient Athens was the authority of the community as a whole. This took precedent over the liberty of the individual. The freedom of the individual to make private decisions, such as choosing a religion, was restricted on the grounds that the interests of society were paramount. However, this simple form of democracy had its drawbacks. While subsequent political thinkers praised the concept of directpolitical involvement, it was recognized that this would be impractical in larger communities. Indeed, societies with populations of thousands or millions would never be able to manage the logistical problem of direct participation. It was, therefore, natural that in modern times there emerged the idea of representative democracy.

*** Until the early 1960s, the picturesque ruins of Aphrodisias were scattered in and around the very pretty village of Geyre, where the houses had been built largely from remnants of the ancient city. But the present excavations, which began in 1961, have now reached such a scale that the village and its inhabitants have been moved to another site nearby. Some of the superb sculptures unearthed are now exhibited in a new museum, which is located in what was once Geyre’s village square, while others can be seen around the archaeological zone, one of the most interesting and beautiful sites in all of Turkey. Surprisingly, the excavations at Aphrodisias have unearthed remains of a settlement dating back to about 5,800 B.C. The site seems to have been a very ancient shrine of Ishtar, the fertility goddess of Nineveh and Babylon, who was one of the predecessors of Aphrodite, the Greek goddess of love. In fact, the earliest Greek sanctuary of Aphrodite on this site dates from the sixth century B.C., and it was from this sanctuary during the next four centuries that the cult of Aphrodite spread throughout the Graeco-Roman world.

ÜDS FEN 2008 SONBAHAR

*** Dark matter is the invisible and mysterious material that makes up 22 per cent of the stuff in the universe. It is one of the greatest scientific unknowns. It does not emit light; nor does it reflect light or absorb it. While we are unable to see dark matter itself, we are able to create maps of it. We can clearly pinpoint its location by observing the effects of its mass on light from distant galaxies. This can be explained with reference to Einstein, who points out that a massive object will curve the fabric ofspace and that light will follow this deformed path. So we can look at how light from galaxies has been bent and, consequently, infer the quantity and location of the matter that did the bending. In fact, by using this method, a team of astronomers have recently managed to create the first threedimensional map of the immense structure of dark matter.

*** Hurricanes, which are circular storms spinning around a region of low atmospheric pressure, are powered by energy released by spiralling surface winds that draw heat from the ocean. Warmer seas provide more energy and make hurricanes stronger. This is what happened during Hurricane Katrina in August 2005, which submerged New Orleans and the vicinity. In fact, according to climate scientists, both the intensity and destructiveness of hurricanes have increased markedly since the 1970s. In other words, the energy released by an average hurricane appears to have increased by about 70 per cent within the past 30 years. This increase correlates very closely with rises in sea surface temperatures. Furthermore, tropical oceans have warmed about one degree Fahrenheit in the past 50 years, a rise that is believed to be chiefly the result of global warming.

*** Using coal to make electricity accounts for about a third of America’s carbon emissions. As a result, tackling emissions from coal-fired power plants represents our best opportunity to make sharp reductions in greenhouse gases. Fortunately, the United States already has the technology to do that. Unfortunately, right now the country is addicted to coal, a cheap, abundant power source. Burning coal produces more than half the country’s electricity, despite its immense human and environmental costs. Air pollutants from coal-fired power plants cause somewhere between 20,000 and 30,000 premature deaths in the United States each year. Besides, fifty tons of mercury are pumped into the atmosphere annually from coal plants. In addition, the extraction of coal, from West Virginia to Wyoming, devastates the physical environment, and its processing and burning produce gigantic volumes of waste.

*** Rivers and streams generally support communities of organisms quite different from those of lakes and ponds. A river or stream changes greatly between its source and the point at which it empties into a lake or the sea. Near the source, a stream’s water is usually cold, low in nutrients, and clear. The channel is often narrow, with a swift current that does not allow much silt to accumulate on the bottom. Most of the organisms found here are supported by the photosynthesis of algae attached to rocks or by organic material, such as leaves, carried into the stream from the surrounding land. Downstream, a river or stream generally widens and slows. The water is usually warmer and may be cloudier because of sediments and other particles suspended in it. Worms and insects that burrow into the mud are abundant, as are waterfowl, frogs, fish, and other water animals.

*** According to the most accurate scientific theory ever created and generally known as the standard model, all of space is filled with a mysterious stuff called “the Higgs field”. Unlike magnetic or gravitational fields, which vary from place to place (as, for instance, the fact that things weigh more on Earth than on the surface of the Moon), the Higgs field is exactly the same everywhere. What varies is how the different fundamental particles interact with it. That interaction, the theory goes, is what gives particles mass. In other words, the Higgs field is what makes some particles, such as protons and neutrons, relatively heavy, others (like electrons) subatomic lightweights, and still others (like photons) utterly massless. If photons weren’t so light, a person would be shredded by a photon hailstorm every time he or she was exposed to a sunbeam. Then again, if protons and neutrons weren’t so heavy, one wouldn’t dare to go outside to sunbathe anyway. So without mass and its affinity for gravity, there would be no galaxies, no stars, and no us.

*** People have been pushing into forestlands for thousands of years, but during the last century, scientists say, the rate of global forest reduction has reached alarming levels. About 50 million acres of forest are cleared every year. Much of Europe’s original forests are gone. The forests of North America, which once dominated the landscape, have shrunk by almost 40% in the last two centuries to make room for people and meet the demand for lumber and paper. Not only have many of the animals that depend on these ecosystems disappeared, but various species of trees have also been depleted. Timber farms on land that once sustained natural forests have little of the biodiversity of the original forests, with pesticides and other chemicals allowing the land to support only a few kinds of life.

ÜDS SAĞLIK 2008 SONBAHAR

*** Viral infections of the respiratory tract are certainly the most common cause of infectious illness in most countries. People in all age groups are susceptible, but for those at the extremes of life these illnesses are particularly hazardous. Viral respiratory infections are more common in children under the age of five years, but in the first year of life, they are more severe. In the elderly, degenerative processes of the heart or lungs make pneumonia a more frequent and serious complication, and the same is true for patients of all ages suffering from chronic cardiac disease or chronic bronchitis. On the whole, the vast majority of viral respiratory infections are mild though often uncomfortable conditions, but sometimes the illness is severe and constitutes a threat to life. Unfortunately, there isno method whereby the spread of infection can be prevented. Adequate ventilation and avoidance of crowds are clearly advisable.

*** Gingivitis is the inflammation of the gums. Under this condition, the gums become red and swollen and bleed easily. An extremely common condition, gingivitis can develop any time after a person’s teeth come in, and it is almost always the result of inadequate brushing and flossing, which allows plaque to remain along the gumline of the teeth. Plaque, which is a soft, sticky film made up primarily of bacteria, accumulates especially in faulty fillings and around the teeth next to poorly cleaned partial dentures, bridges, and orthodontic appliances. When plaque stays on the teeth for more than 72 hours, it hardens into tartar, which can’t be completely removed by brushing and flossing. Although plaque is the main cause of gingivitis, other factors can make the inflammation worse, especially pregnancy, puberty, and birth control drugs.

*** As adolescents begin to assert their individuality, family tensions increase, and battles are fought over clothes and hair styles, late nights and so on. The doctor may find himself consulted; some parents feel that, if their children get into trouble or disagree with them, they must be ill. Others seek a referee or an accomplice in the battle ofthe generations. Often the younger doctor in a partnership has an advantage in dealing with such problems, being able to bridge the generation gap and communicate well with both parents and teenagers. Both generations need education about the other and particularly about current norms of behaviour. Children may have to be reminded that their parents also have rights, and parents, especially those with unrealistic ambitions for their offspring, or those determined to live their lives again through their children, must be taught to give their children more independence.

*** Narrowly defined, fitness refers to the characteristics that enable the body to perform physical activity. These characteristics include flexibility of the joints, strength and endurance of the muscles, including the heart muscle, and a healthy body composition. A broader definition of fitness is the ability to meet routine physical demands with enough energy reserve to rise to a sudden challenge. This definition shows how fitness relates toeveryday life. Ordinary tasks such as carrying heavy suitcases, opening a stuck window, or climbing four flights of stairs, which might strain an unfit person,are easy for a fit person. Still another definition is the body’s ability to withstand stress, meaning both physical and psychological stress. These definitions do not contradict each other; all three describe the same wonderful condition of the body.

*** Most of the functions of the brain are still unknown, and the ones we know about are very poorly understood. The brain is assumed to be the organ of higher mentalfunction, of the mind and intellect, but there is surprisingly little evidence for this, and no one has any idea what physical structures or mechanisms perform these functions. The brain is known to control all bodily functions by means of motor and other nerves which carry impulses from the brain outwards to all parts of the body. Sometimes these are under our voluntary control; mostly, they are involuntary, reflexive or automatic. Reflexive actions are the result of impulses passed inwards from the body towards the brain by means of sensory nerves. Information arriving in the brain about various sensations like heat, pain,touch, position, the need for saliva or gastric juice or even the thought or smell of food are acted on in the various “centres” in the brain.

*** The immediate cause of obesity is the prolonged consumption of a diet containing more calories than are needed to provide for the body’s tissue repair, vital functions and physical activities. In modern society, food has become very plentiful and attractive, and the physical effort demanded by many occupations has diminished. Most people in civilized communities eat more than they require, and it is surprising that obesity is not more common than it is. It is difficult to escape the conclusion that there exists some unknown mechanism by which the body is enabled to get rid of the surplus calories which would otherwise be stored as fat. If there were not such a mechanism, obesity would be much more common.

ÜDS SOSYAL 2008 SONBAHAR

*** In 1786 Sir William Jones, a British judge serving in India, made a discovery that transformed knowledge about prehistory and began the formal study of historical linguistics. Turning his spare time towards the study of Sanskrit, the ancient language from which the predominant languages of the South Asian subcontinent derive, Jones discovered that Sanskrit shares features of grammar and vocabulary with Latin and ancient Greek to an extent inexplicable by sheer coincidence. His interest further aroused, he then examined the early Germanic language called “Gothic”, the ancient Celticlanguages of Europe, and Old Persian, and found thatthey, too, exhibited marked similarities to Sanskrit. He concluded that all these languages must have evolved from a common but now-extinct linguistic source. In the early nineteenth century, both this ancient language and the later languages that derived from it, were labelled “Indo-European,” reflecting their wide distribution from India and Ireland.

*** In 1914, Europe had built a seemingly stable peace. Through the complex negotiations of great power geopolitics, Europe had settled into two systems of alliance: the Allied Powers which consisted of Britain, France and Russia, and the Central Powers that included Germany, Austria,and Italy. Within this balance of power, the nations of Europe challenged one another for economic, military, and imperial advantage. The rivalry for colonies abroad accompanied a fierce arms race at home, where military leaders assumed that superior technology and larger armies would result in a quick victory in a European war. Indeed, inthe prevailing atmosphere of international suspicion, such a war seemed likely to many of Europe’s political and military leaders. Yet none of them predicted thatthe war would break out so soon. Nor did many expect that the assassination in June 1914 of the Austrian archduke and his wife would spark off that war, which engulfed all of Europe in just over a month’s time.

*** The Stone Age is, in fact, divided into various stages. Dominating the period is the Palaeolithic Age, which most anthropologists would extend down to roughly 11,000 B.C. Within the Stone Age in general, however, scholars also speak of an Upper Palaeolithic Era, beginning around 40,000 B.C. They draw attention to some significant changes in human behaviour around this date, including the appearance of sophisticated cave paintings, and evidence of religious ideas. Humans also began producing the most effective, finely crafted tools such as fishhooks, arrowheads, and sewing needles made from organic materials, such as wood or animal bone. Yet, despite these important developments, the basic patterns of human life changed little during this era. Virtually all human societies before 11,000 B.C. consisted of small bands of hunter-gatherers that moved incessantly in search of food. Because they could not stay in any one location for long, these groups left no continuous archaeological record whereby we might trace the development of their culture. Our knowledge of them is, therefore, very limited.

*** Since 1993, China has invested in more than fifty oil and gas projects in some thirty nations. In particular, China has focused on acquisitions and partnerships in Sudan and Iran. In Sudan alone, China has reportedly spent $15 billion developing oil fields. In the meantime, China has also begun to use its military to protect its oil investments abroad. Reportedly, troops disguised as oil workers patrol Chinese oil infrastructure in Sudan. Moreover, in recent years, China has strengthened its military presence in the oil-and gas-rich parts of the South China Sea, over which sovereignty is still disputed. Perhaps most significant in the short term is China’s relationship with Iran. With Saudi Arabia and Iraq clearly within the American sphere of influence, China has been steadily courting Tehran and aims to become the biggest buyer of Iranian oil. In return for oil, China has supplied Iran not only with conventional weapons but also with technology and materials that can be used for the manufacturing of nuclear weapons.

*** By the end of the twentieth century, East Asia had become a centre of industrial and manufacturing production. Especially China began to establish commercial ties with the West in the 1970s and became the world’s leading heavy industrial producer by the year 2000. Its state-owned companies acquired contracts from Western firms to produce products cheaply and in bulk, for sale back to home markets in the United States and Europe. Moreover, the Chinese government established semicapitalist commercial zones around major port cities like Shanghai. These commercial zones were intended to encourage massive foreign investment on terms that left China a favourable balance of trade for its huge volume of cheap exports. Yet, in practice, they enjoyed only mixed success. Problems in farming and a looming energy crisis hampered prosperity and economic growth, but Hong Kong only managed to maintain its traditional economic and cultural ties with the rest of the world. However, in recent years, China has overcome most of these problems and radically upgraded its economic performance.

*** The term “imperialism” means the process of extending one nation’s control over another; it is a process that takes many forms. Historians distinguish between “formal imperialism” and “informal imperialism.” Formal imperialism is colonialism, and it was exercised by the Europeans in the past mainly by direct rule: the colonizing nations annexed territories outright and established their own governments to subjugate and administer the peoples of these territories. Sometimes formal imperialism was exercised through indirect rule: the conquering nations reached agreements with native leaders and governed them. There was no single practice of colonial management, and resistance from the natives forced colonial powers to shift strategies frequently. As for “informal imperialism,” it refers to a more subtle and less visible exercise of power, in which the stronger nation allows the weaker one to maintain its independence while reducing its sovereignty. For the Europeans in the past, informal imperialism took the form of carving out zones of European sovereignty and privilege, such as treaty ports, within other countries. Essentially it meant using European economic, political, and cultural power to get advantageous treaties or terms of trade. Informal imperialism was not only common, it played an even more fundamental role in shaping global power relations in the 18th and 19th centuries.

ÜDS FEN 2009 İLKBAHAR

*** Artificial Intelligence (AI) is the ability of a digital computer or computer-controlled robot to perform tasks commonly associated with intelligent beings. The term is frequently applied to the project of developing systems endowed with the intellectual processes characteristic of humans, such as the ability to reason, discover meaning, generalize, or learn from past experience. Since the development of the digital computer in the 1940s, it has been demonstrated that computers can be programmed to perform very complex tasks, such as discovering proofs for mathematical theorems or playing chess, with great proficiency. Still, although there are continuing advances in computer-processing speed and memory capacity, there are as yet no programs that can match human flexibility over wider domains or in tasks requiring much everyday knowledge. On the other hand, some extraordinary programs have attained performance levels beyond those of human experts and professionals engaged in certain specific tasks. AI, in this limited sense, is used efficiently and found in applications as diverse as medical diagnosis, computer search engines, and voice or handwriting recognition.

*** The magnets that are used most commonly, such as the ones on compasses, those used for fridge decorations, and in many other everyday tools, are called permanent magnets.This type of magnet produces an external magnetic field that attracts or repels iron, and it may lose its strength when mistreated. Inside a magnetare groups of atoms called domains. The magnetizing process, which exposes a material to increasingly strong magnetic fields, aligns these domains in a single direction, where they become locked in a crystalline structure. High heat, radiation, strong electrical currents, or other nearby magnets, though, can damage that structure, nudging the domains out of alignment and diminishing the attractive force. Electromagnets, or non-permanent magnets, a less familiar type, have magnetic fields that rely on an electric current. They, thus, do not lose their strength; instead, the strength of the field can be varied as needed. This makes them appropriate for various applications, such as telephone receivers.

*** According to a recent study by the University of Alberta, parasitic sea lice are killing a population of young wild pink salmon along Canada’s west coast in alarming numbers. The authors of the study say that the entire wild population may be gone within eight years. With their protective scales, adult salmon can safely harbour the lice, but young salmon do not yet have the protective scales,leaving them prone to deep, infection-prone wounds left by the lice. Juveniles live in coastal waters, which are normally far from parasite-carrying adults living farther out to sea. Now, however, aqua farms are often located in these same waters, destroying the young salmon’s safe haven. Scientists argue that fish farms must be relocated or reduced, but so far, no government regulations have been launched to this end.

*** Glucose, nature’s most abundant sugar, may soon be petroleum’s fiercest rival. Chemists have long searched for cheap, renewable, and non-polluting alternatives to the 245 million tonnes of petroleumbased plastics produced annually. For years, they have been able to convert sugars into the chemical hydroxymethylfurfural (HMF), which can be used to make plastic. But the process, which used acid catalysts to break the sugars down, was costly and complicated by impurities and low yields. Researchers at the Pacific Northwest National Laboratory (PNLL) in Washington replaced the acid catalyst with a metal catalyst, chromium chloride, and used it to break down glucose, a sugar found in plant starches and cellulose. The result: HMF yields increased 10 to 70 percent over the old processes and impurities were eliminated. The next step to replacing petroleum is to find a low-impact renewable source for the glucose. Scientists hope to soon obtain glucose from cellulose rather than from plant starches. Cellulose is found in straw and sawdust, two waste products from the agricultural and wood industries that do not require precious farmland to be taken away from food crops.

*** Much has been said and written about the declining numbers of and disappointing lack of diversity among college students majoring in engineering. Among the factors cited to explain this paucity are the lack of exposure of high school students to the very idea of engineering and the fact that many have insufficient mathematics and science background to gain entrance to engineering school, even if they identify the profession as a possible career. This is unfortunate, for the ideas of engineering should be integrated into the curricula not only of high schools but also of middle and primary schools. By not being exposed properly throughout their education to engineering activities, children are being done a disservice. After all, even preschool children have the necessary conditions in their play for appreciating exactly what engineering is: design. Indeed, design is practised throughout their school day, even in their before- and after-school activities. It only should be pointed out to them that they are designing something, and, therefore, are future engineers in the making.

*** Contrary to popular belief, underground fires are a surprisingly frequent phenomenon, the fuel being coal and the fire travelling along the seams, or the thin layers of rock or mineral. Such fires travel slowly due to the limited supply of oxygen, but can burn for a very long time: the underground fire at Burning Mountain Nature Reserve in Australia is thought to have been continuing for the past 5,500 years. The number of such subterranean fires worldwide is countless. According to one study, subterranean fires in China alone are consuming some 200 million tonnes of coal a year and pumping into the air as many pollutants as all the cars in the United States. Along with numerous human-related factors, such fires are also contributing substantially to global warming.

ÜDS SAĞLIK 2009 İLKBAHAR

*** What controls our breathing? We obviously have some conscious control over it because we can voluntarily hold our breath for a short while or breathe faster and deeper. Most of the time, however, automatic control centres in our brain regulate our breathing movements. Automatic control is essential, for it ensures coordination between the respiratory and circulatory systems and the body’s metabolic needs for gas exchange. Anatomically, our breathing control centres are located inparts of the brain called “the pons” and “the medulla.” Nerves from the medulla’s control centre signal the diaphragm and rib muscles to contract, making us inhale. These nerves send out signals that result in about 10-14 inhalations per minute when we are at rest. Between inhalations, the muscles relax, and we exhale. The control centre in the pons smooths out the basic rhythm of breathing set by the medulla.

*** Josh Villa was 26 and driving home with a friend when his car mounted the kerb and flipped over. Villa was thrown through the windscreen, suffered massive head injuries and fell into a coma. Almost a year later, there was still little sign of improvement. He would open his eyes, but he was not responsive to any external stimuli inhis environment. He was then enrolled in a six-week study in which an electromagnetic coil was held over the front of his head to stimulate the underlying brain tissue. Such transcranial magnetic stimulation (TMS) has been investigated as a way of treating migraine, stroke, Parkinson’s disease and depression, with some promising results, but this is the first time it has been used as a potential therapy for someone in a comalike state. Certain improvements were observed; for instance, he began to say single words. The case has been described as “intriguing,” but it has also given rise to a lot of cautionary warning.

*** The vertebrae of the spinal column are separated by disks made of cartilage. Each disk has a strong outer layer and a softer inner part that acts as a shock absorber to cushion the vertebrae during movement. If the disk degenerates, for example following an injury or with aging, the inner part of the disk can bulge or rupture through the outer layer. The ruptured inner part of the disk can compress or irritate a nerve root and may even injure it. Most ruptured disks are in the lower back and usually affect only one leg. Such a rupture can cause pain not only in the lower back but also down the sciatic nerve, which runs from the spinal column to the buttocks, leg, and heel. Ruptured disks in the lower back can also cause leg weakness, and a person may especially have difficulty lifting the front part of the foot. A ruptured disk that is very large and centrally located in the spinal column can affect nerves that regulate bowel and bladder function, impairing the ability to defecate or urinate and making urgent medical attention necessary.

*** Muscles can obtain the carbohydrate they need, not only from glycogen stores but also from sugar taken during activity, which elevates blood glucose and enhances endurance. Normally, insulin stimulates all the tissues of the body todrain glucose from the blood and store it; however, this is exactly the opposite of what is needed for performance. During physical activity, the body’s release of the hormone epinephrinekeeps insulin from rising in response to glucose entering the blood. Physical activity also enhances muscle sensitivity to insulin so that the muscles become the primary recipient of blood glucose. Consuming sugar is especially useful during exhausting endurance activities lasting more than an hour. Endurance athletes often run short of glucose by the end of competitive events, and they are wise to take light carbohydrate snacks or drinks periodically during activity. During the last stages of an endurance competition, when glycogen is running low, glucose consumed during the event can make its way slowly from the digestive tract to the muscles and increase the body’s supply of glucose enough to prevent exhaustion.

*** A low calcium intake during the growing years limits the bones’ ability to achieve an optimal mass and density. Most people achieve a peak bone mass by about age 30, and dense bones protect against agerelated bone loss and fracture. Starting before the age of 40, all adults lose bone as they grow older. When bone loss reaches the point at which bones fracture under common, everyday stresses, the condition is known as osteoporosis. Today, worldwide, this is one of the most prevalent diseases of aging. For instance, in the US, it afflicts more than 25 million people, mostly older women. Unlike many diseases that make themselves known through symptoms such as pain, shortness of breath, skin lesions, tiredness, and the like, osteoporosis is silent. The body sends no signals saying bone loss is occurring. Blood samples offer no clues because blood calcium remains normal regardless of bone content, and measures of bone density are rarely taken. However, there are various strategies to protect against bone loss, and eating calciumrich foods is only one of them.

*** Fitness is determined more by the intensity of exercise than the duration. Workouts should be energetic enough that the muscles are somewhat sore the next day but fully recovered the day after that. To strengthen the heart, exercise must be performed at an intensity that increases heart rate at least 20 beats above the resting heart rate. The harder a person exercises, the faster the heart beats and the stronger the heart muscle becomes. Heart rate is determined by how hard the skeletal muscles contract. When a person starts to exercise, the skeletal muscles contract and squeeze the veins near them, forcing blood towards the heart. When the skeletal muscles relax, these veins fill with blood. The alternating contraction and relaxation of the skeletal muscles serve as a second heart, pumping extra blood to the heart. The increased blood flow causes the heart to beat faster and more forcefully. So the harder the skeletal muscles contract, the faster the heart beats.

ÜDS SOSYAL 2009 İLKBAHAR

*** Of the wealth of geographical and cartographical data remaining from the Eastern world and the Greeks, the Romans were interested only in those elements that best suited their essentially practical needs. They virtually abandoned the studies of cosmography and mathematical geography, preferring to devote maximum effort to land surveys carried out by specialized military corps of land surveyors. From their topographical surveys and onthe-spot reconnaissance of places and cities, they formulated their itineraria scripta, publications similar in many respects to our tourist guides, and their itineraria picta, a kind of road map on which they stressed only those topographical features of use to travellers.

*** Charlie Chaplin, who was born in Britain but spent most of his life in the United States, is one of the pivotal figures in film history. He is especially remembered for his work in the silent movies. Chaplin knew that a successful scene was not simply about the starring actor, butabout everything else. The only way to achieve that unity was to get personally involved in every stage of the film; from starring in his films to producing, directing, editing them, and even to composing the music for them. It was not uncommon for him to decide half-way through a film that an actor wasn’t suitable for a certain role, and start over with someone new. This constant attention to detail ran many features overtime and over-budget, but the public reaction assured him and the studios that what he was doing worked. Chaplin typically improvised his story in front of the camera with only a basic framework of a script. But on consideration, his art turned out to be firmly rooted, and could be seen, for example, to draw much of its strength from his successful fusion of English and American cultures and traditions.

*** The greatest feat of civil engineering since Roman times was the French achievement between 1666 and 1681 of the Languedoc canal to link the Mediterranean Sea with the Atlantic Ocean. It was conceived of by King Francis I and discussed by him with Leonardo da Vinci. It was first surveyed in 1539. As a result of the civil and religious disorders in France, however, nothing happened until 1661 when Pierre Riquet worked out a scheme for supplying enough water to the summit of the canal. Jean Colbert secured the interest of the young King Louis XIV, and in 1666 the work started. The whole canal was 150 miles long with 102 locks, a tunnel, and 3 aqueducts. It evoked world admiration, and was the prototype of all future European grand canal designs even though its usefulness to France was quickly nullified by the rapidly increasing size of oceangoing ships.

*** The most important influence on the style of English furniture was that of the Chippendale family. Chippendale furniture is the creation of Thomas Chippendale, a London-based cabinet-maker. His Gentleman and Cabinet-Maker’s Director, a folio of furniture designs, was published in 1754 and had a wide circulation. Chippendale himself never marked his creations, making it harder today for collectors to locate original pieces. One of the best ways to find original Chippendale furniture is to look for uneven joints and tool marks, as the furniture was made entirely by hand and does not have the perfect angles of a mass-produced product. Though an article of furniture made in Chippendale’s manner may bear his name, it is by no means an indication that it was actually made in his workshop. The 18th century was a time when artisans were beginning to exploit various styles, leading to widespread adoption of his name in revivals of his style, so much so that dealers spoke of “Chinese Chippendale”, “Gothic Chippendale”, and even “Irish Chippendale”. Many of these later designs that attach his name bear little relationship to his original concepts.

*** Pottery was one of man’s first artefacts. It is the presence of pottery, rather than of the polished stone, that marks the passage fromthe Mesolithic to the Neolithic Ages, when agricultural peoples settled both in the Mediterranean area and in the Middle East. It is commonly believed that the earliest pottery receptacles copied those of other materials, such as gourds or baskets. From the fingerprints on them, it is possible to deduce that they were made principally by women. Originally, any decoration was indented; that is, patterns were pressed into the soft clay, and it remained so for a long period until new situations, at different times in different parts of the world, produced painted decoration.As a widespread form of culture, permanently bearing in its shapes and decoration the character of individual periods and peoples, the finding of pottery has been of supreme importance to the archaeologist.

*** Although most cities seem to form by accident, for thousands of years some of them have been designed. Whether for defence, beauty, or practicality, urban designers have imposed their ideas of what a city should be about. But ideas are subject to changing needs and fashions. Centuries ago, a moat or a castellated wall would have been essential. Now, greenery is in vogue. While existing cities look for ways of becoming more environmentally friendly, a number of new ones are planned that intend to be totally green. One of these is Masdar. Masdar’s advertising states that “one day, all cities will be built like this.” This is not the case. For one thing, Masdar is experimental and a work in progress. What emerges will not necessarily translate well elsewhere. Each green city is unique, and getting it to work depends on its location and economy.

ÜDS FEN 2009 SONBAHAR

*** Palaeontology was once limited to digging up fossils and trying to deduce their age with inaccurate methods. However, fossil analysis improved dramatically in the 1960s, with the advent and refinement of two techniques: radiometric dating and stratigraphy. The first radiometric method was also known as carbon-14 dating, and it was usable for specimens younger than 50,000 years. Later, potassium-argon dating revolutionized the field by enabling scientists to detect the radioactive decay of elements found naturally in rocks and soil surrounding much older fossils. Stratigraphy, which is the study of rock layering, actually was developed well before the 1960s, but that was the decade scientists began to better understand how geological conditions, earthen layers, and fossil records all relate. The resulting refinement of biostratigraphy, i.e., the study of the complete life of a stratum of earth, allowed scientists to determine the environment and lifestyle of human ancestors based on fossilized flora and fauna found within the same layer as the hominine fossils. Since the 1960s, DNA testing has come to be used widely. As all living organisms have the same genetic code, scientists can use DNA variations as a molecular clock. After splitting with a common ancestor, each generation develops a constant rate of genetic mutations. The molecular clock allows scientists to calculate how long ago the split occurred based on the number of differences between species. The method is now helping scientists map the routes that humans took out of Africa.

*** Scientists are exploring waysof producing spider silk artificially, a process difficult to repeat effectively. A spider’s silk gland is a very efficient chemical factory. Inside its gland, the spider stores a mixture of liquid proteins, which it is able to transform into light, strong fibres. Artificial spider silk could have many applications, from lightweight and durable packing materials to parachutes, surgical sutures, and even bullet-proof vests. Producing it synthetically is a twopart process: scientists must first manufacture the proteins and then find a way to form them into superfine threads. They have had success with the first part, by producing proteins through genetic modification. Binding proteins into fibres as thin and strong as spider silk, however, has proved to be a challenge. Recently, however, a group of German scientists have attempted to solve that problem by using a device modelled on a spider’s glands. Like the arachnid method, the proteins are mixed with potassium phosphate, and then the pH is lowered before pressure is applied as the mixture flows through tiny channels, hardening and binding the proteins. So far, researchers have been able to make fibres of only a fraction of an inch long, but they hope to be able to produce longer, stronger fibres in the future.

*** Baleen whales and toothed whales each have a unique way of vocalizing. Only baleen whales produce long sequences of deep sounds known as whale songs. They have a larynx, an organ at the top of the trachea, which may be involved in sound production. Researchers are unclear about the organ’s role in the songs as whale larynxes are unlike those of humans, which have vocal chords. Toothed whales, on the other hand, rely on sequences of high-pitched clicks and whistles for both echo location and communication with their mates. Their phonic lips, a structure analogous to human nasal passages, press together when air is forced through them, vibrating the surrounding tissue. The sound waves then penetrate an oily organ in the whale’s head, called the “melon”, where they are focused into a beam of sound. When this beam strikes a fish, the seabed, or another object, the sound is reflected back to the whale as an echo. Toothed whales can thus locate prey and navigate in total darkness. However, during their long, deep dives, toothed whales cannot inhale air every time they want to produce a sound. So they collect it in a sac at the back of their head and reuse it.

*** An organism discovered deep in the ground has taken astrobiologists by surprise. The organism’s unique ability to live in complete isolation from other species, or even light or oxygen, suggests it could be the key to life on other planets. It was discovered in fluid-filled cracks in a South African gold mine, nearly three kilometres beneath the Earth’s surface. When US scientists analyzed the fluid, they expected to find genes from a mix of species.Instead, they found that 99.9 per cent of the DNA belonged to just one bacterium, a previously unknown species. Such a selfsufficient organism is virtually unheard of. It means that this organism extracts everything it needs from an otherwise dead environment. Almost all other known organisms on the Earth that do not use sunlight directly do use some product of photosynthesis. However, this newly-found organism gets its energy from the radioactive decay of uranium in the surrounding rocks. It also has genes to extract carbon and nitrogen from its environment, both of which are essential for making proteins. Scientists believe that this organism is just the type that could survive on a planet other than the Earth.

*** The human population continues to grow by more than 75 million people annually. Since the first Earth Day in 1970, emission rates have remained steady at about 1.2 metric tons of carbon per person per year. Unfortunately, the 1997 Kyoto Protocol has had little measurable effect on these per-capita emissions, even in the countries that have agreed to national targets. More than any other factor, population growth drives rising carbon emissions, and the US Census Bureau and United Nations both project that the global population, currently 6.6 billion, will surpass 9 billion before 2050. The implication is that one of the best strategies for reducing future greenhouse gas emissions ispopulation stabilization, as quickly as can be achieved by noncoercive means.

*** A scientific view of something is always an intimate mixture of theories and observed facts. The theories are broad, general ideas together with arguments based on them. The arguments are designed to show that, if the general ideas are accepted, then this or the other thing ought to be observed. If this, that, or the other actually is observed, then the theory is a good one; if not, then the theoreticians have to think again. Thus, theoretical ideas and arguments are continually subjected to the severe test of comparison with the facts,and scientists are proud of the strictness with which this is done. On the other hand, theories often suggest new things to look for; in other words, they lead to predictions. These predictions are frequently successful, and scientists are entitled to be proud of that, too. But it follows that no theory is immutable; any scientific view of any subject may, in principle, be invalidated at any time by the discovery of new facts.

ÜDS SAĞLIK 2009 SONBAHAR

*** The brain’s capacity for finding new informationprocessing pathways is thought to explain the success of artificial cochleas, which have been implanted in the ears ofapproximately 100,000 hearing-impaired people around the world. They typically have an array of electrodes, each of which channels electrical signals toward the auditory nerve. The electrodes can stimulate not just a single neuron in the brain but many simultaneously. When cochlear implants first appeared in the 1980s, many neuroscientists expected them to work poorly, given their primitive design. But the devices work well enough for some deaf people to converse over the telephone, particularly after an adjustment period during which channel settings are fine-tuned to provide the best reception. Patients’ brains somehow figure out how to make the most out of the strange signals. The surprising effectiveness of artificial cochleas – together with other evidence of the brain’s adaptability – has fuelled optimism about the prospects for brain/machine substitution. A case in point is an ongoing project at the University of Southern California that seeks to create implantable brain chips that can restore or enhance memory.

*** By six months of age, the infant’s capacity to digest and absorb a variety of dietary components as well as to metabolize and excrete the resulting products is near the capacity of the adult. Consideration of the long-term effects of inadequate or excessive intakes during infancy now assumes greater importance. These considerations about delivery of adequate amounts of nutrients are the basis for many of the feeding practices advocated during the second six months of life. Although it is clear that all nutrient needs during this period can be met with reasonable amounts of currently available infant formulas, addition of other foods after four to six months of age is recommended. In contrast, the volume of milk produced by many women may not be adequate to meet all nutrient needs ofthe breast-fed infant beyond about six months ofage, especially iron. Thus, for breast-fed infants, complementary foods are an important source of nutrients. Complementary foods (i.e., the additional foods, including formulas, given to the breast-fed infant) or replacement foods (i.e., food other than formula given to formula-fed infants) should be introduced step by step to both breast-fed and formula-fed infants, beginning between four and six months of age.

*** Following the growth of biological knowledge in the past few decades, a few researchers now believe extension of the human lifespan might be within reach. Why do organisms – people included – age in the first place? Like machines, people wear out. However, a machine can usually be repaired. A good mechanic with a stock of spare parts can keep it going indefinitely, to the point where no part of the original remains. The question arises, of course, of whether the machine is worth repairing. It is here that people and nature disagree. From the individual’s point of view, survival is a must. You cannot reproduce unless you are alive. Since ageing is a sure way of dying, it is no surprise that people want to stop it from advancing. Moreover, even the appearance of ageing can be harmful. It reduces the range of potential partners who find you attractive – since it is a sign that you are not going to be around for a very long time to help bring up the baby – and this, in turn, restricts your ability to reproduce. There is a paradox, however: the individual’s evolved desire not to age is opposed by another evolutionary force, the disposable soma. The soma is all of a body’s cells apart from the sex cells. Its role is to get the sex cells, and thus the organism’s genes, into the next generation. But evolutionary logic seems to require the soma to age and die in order for a species to continue. There is thus a premium on reproducing early rather than conserving resources for a future that may never come.

*** Individual “banks” of immune cells taken from pigs might one day be used to boost our own immune systems or to fight HIV and cancer. Our immune system’s T-cells, which play a key role in fighting off diseases, are sharpened during childhood to attack particular pathogens after encountering them. This flexibility diminishes after a child reaches young adulthood, but researchers at a US university have come up with a way to reviveit. According to them, if a human’s immune cells are transferred into a young pig, they could be brought up to maximum effectiveness (as in a child’s body), then implanted back into the person they came from. The research team has already had success with experiments where human stem cells were injected into developing pig foetuses; whenthe piglets were born, the injected cells had multiplied and matured into a diverse range of human T-cells, alongside the pig’s own immune cells, that were shown to be fully functional. The chief researcher envisions this approach eventually being used to make human cells that fight specific diseases. The necessary technology is available now to introduce the technique widely, provided that regulatory authorities can be convinced that it can be safely tested in humans. However, the fearis that dormant pig viruses buried in their DNA could be spread to humans. Another potential danger is that humanderived cells might pick up surface molecules from the pig. This could make the transferred cells themselves targets for immune destruction. The pigs might also produce too few human cells to fight disease.

*** Asthma is a life-threatening, allergy-driven lung disease common in wealthy countries. But exactly what causes it is unknown. Researchers at Washington University believe the direct cause of asthma is a chemical distress signal produced in skin that is damaged by another hazard of modern life: eczema. Unlike asthma, it is not dangerous, so people rarely worry about it. Nevertheless, 17% of children in America have it, and similarly high figures are found in Australia, Britain, and New Zealand. What is particularly intriguing is that many people with eczema go on to develop asthma (in America, the figure is 70%). That compares with an asthma prevalence of 4-8% in the general population. The Washington University group theorizes that the link between the two conditions is formed by thymic stromal lympho-poietin (TSLP), a signalling molecule secreted by damaged skin cells that elicits a strong immune response from the body to fight off invaders. Thus, eczema-induced TSLP enters the bloodstream and, when it arrives at the lungs, sensitizes them so that they react to allergens that would not previously have bothered them. In other words, they become asthmatic. Several experiments carried out by the researchers, only on mice, have confirmed that skin damage creates susceptibility to asthma by releasing TSLP.

*** Patients are admitted to critical care units from a variety of settings, including the emergency department, medical or surgical service, or operating room. Most critical carepatients are acutely and severely ill, commonly with dysfunction or failure of more than one organ system. The initial assessment must be rapid and focus on real or potentially lifethreatening processes that require immediate intervention. An example is the resuscitation of a patient with cardiopulmonary arrest. The pace of resuscitation is necessarily quick; physical examination may be restrictedinitially to the central nervous, cardiovascular, and respiratory systems, and interventions may be limited to the essential ABCs of airway, breathing, and circulation. Later, continuous electrocardiographic monitoring, measurement of blood pressure, and other standard procedures should start. In general, management of the critically ill patientshould be based on an understanding of physiology and pathophysiology. Indeed, although the contributions of cell and molecular biology to critical care medicine are substantial, the critical care unit more resembles a physiology laboratory, since the effects of its interventions can be directly observed.

ÜDS SOSYAL 2009 SONBAHAR

*** Scottish philosopher and historian David Hume emerged as an economist also with the publication of his Political Discourses. The famous Adam Smith was a friend of his and may have been influenced by Hume: they had similar principles, and both were very good at illustrating and supporting these from history. Although Hume did not formulate a complete system of economic theory, as did Smith in his Wealth of Nations, he introduced several of the new ideas around which the “classical economics” of the 18th century was built. His economic philosophy can be understood from his main arguments: that wealth consists not of money but of commodities; that the amount of money in circulation should be kept related to the amount of goods in the market; and that poor nations impoverish the rest because they do not produce enough to be able to take much part in trade. Beyond this, he urged society to welcome the shift from an agricultural to an industrial economy, without which civilization could not be achieved.

*** The people of Hong Kong have been experiencing an identity crisis ever since the British returned the colony to China in 1997 and it became a Special Administrative Region with special privileges (for 50 years). Although they are proud of their Chinese ethnicity, culturally they have always felt overwhelmingly Western and therefore much different from their cousins on the mainland. Now they have a new worry: a growing threat to Hong Kong’s economic success. The Chinese government recently announced its plan to turn the city of Shanghai into a global financial and shipping centre by 2020, a move that is seen as weakening Hong Kong’s traditional and profitable position as international gateway to mainland China. Even worse, China’s friendlier relationship with former enemy Taiwan is already reducing transit commerce through Hong Kong. After an economic contraction of almost 8% earlier this year, Hong Kong is feeling real pain, and the jobless rate could approach record levels. So the normally hands-off local government has sprung into action: it has announced two rounds of tax cuts and various handouts to the poor and to businesses. In addition, the city’s long-term planners have recommended that Hong Kong’s government focus on developing six fields – including education, environmental-related industries, and medical technology – in which Hong Kong already has an edge.

*** On July 1, 2009, the US state of California began enforcing a new menu-labelling law that requires chain restaurants to post on their menus the calories contained in their food items. Three other states – Oregon, Maine, and Massachusetts – have already passed similar regulations, as have 11 city and county governments. The trend has gathered strength quickly, mostly because of concern about the nation’s expanding waistlines. The next step is to deploy the practice nationally, and the Congress is about to debate such a law. Pressure for this type of move is coming from the obese, who represent more than a third of American adults, and their defenders. Overweight people often struggle to estimate the number of calories they consume when eating out and make mistakes when calculating how much food they should order. Proponents of menu labelling hope that knowing what is in their food may direct people to healthier items. In Los Angeles, for example, officials optimistically predict that menu labelling could prevent nearly 40% of the annual weight gain there. However, the effect of menu labelling on dietary choices remains unclear, and the regulations are too new to produce much evidence. Furthermore, some critics of the trend believe the public-health benefits of the new legislation are irrelevant. For them, the new regulations are welcome as part of a consumer’s wide-ranging right to know.

*** Although an extension of the worldwide ban on ivory exports to discourage the illegal killing of African elephants has been greeted enthusiastically in many places, the rhinoceroses (rhinos) of southern and eastern Africa are still paying with their lives for their horns, which remain prized by the Chinese for their medicinal and aphrodisiac qualities, and by the Yemenis for making dagger handles. According to a group, called Traffic, thatmonitors the wildlife trade throughout the world, this illegal business is on the rise. Last month, the group called for stronger international cooperation along smuggling routes and for more secure management of legal horn stocks. For its part, Zimbabwe, where there are a lot of illegal killings, has taken a very radical decision: it says it will start dehorning its rhinos. Today only five species of rhino survive in Africa and Asia. In the past, especially in the 19th and 20th centuries, they were slaughtered on a large scale by white hunters. By the 1960s, fewer than 70,000 black rhinos were left in Africa, and, over the next two decades, illegal hunters wiped out 96% of them. But since 1995, thanks to vigorous conservation efforts, the number of black rhinos has gone up again, to around 3,700. The number of white rhinos has nearly doubled over the same period, to over 14,500.

*** Many governments these days feel that the path to happiness for society as a whole lies through spending on the welfare of its youngest members: their health, education, and general well-being. A recent report from a leading international organization, the OECD, examined these efforts among its 30 member countries in order to learn if the aim was being achieved. Specifically, the researchers investigated 21 variables that were then grouped into six main categories. The results surprisingly showed that while some kinds of spending on children do work, many should be improved or scrapped. Also, total government spending per child was seen to vary considerably, as did outcomes, but the correlation between these was not strong. Moreover, the differences in spending levels among countries were not directly linked to their relative levels of prosperity. For example, rich Sweden is, as expected, kind to its children, but poorish Hungary turns out to be generous, too. Up-and-coming South Korea might be expected to be a bit reluctant to part with so much money, but the stinginess of Switzerland is totally unexpected. Children’s lobbies always want more funds, but the OECD report suggests that more money does not reliably yield better results. America has one of the highest levels of spending per child, and among the worst outcomes. In contrast, Australia spends less, with better outcomes.

*** He is young, dark, and handsome, with a beautiful light tenor voice, and he swept the Latin Grammy Awards in 2008. But should Juan Esteban Aristizabal, better known as Juanes, perform his songs at an upcoming “peace concert” in the Cuban capital, Havana? The debate over this Colombian rock star, who is based in the US city of Miami, has been raging on that city’s Spanishlanguage radio and television shows. Older Cuban-Americans, who left Cuba in the immediate aftermath of the 1959 revolution there, vehemently oppose the concert. They argue that it is just the latest attempt by the Cuban regime to manipulate public opinion. Traditionally, these older exiles have held all the political power in Miami. But younger ones are pushing back, especially when it comes to the arts. Beyond the overall Cuban-American community, the Juanes concert is seen as a potential great turning point in USCuban relations. A successful outcome could smooth the way for a further softening in American policy towards Cuba. Artists other than Juanes have attempted this kind of bridge-building with Cuba before; for example, left-leaning musicians like Bonnie Raitt and The Police appeared there in 1999, but they had an overtly political agenda. However, Juanes himself, who is widely admired for his humanitarian work in his native Colombia, denies having any thought of politics. “My only message is one of peace, of humanitarianism, and of tolerance,” he said recently.

ÜDS FEN 2010 İLKBAHAR

*** Solar panels turn the sunlight into energy when the sun shines directly on them, but as soon as the sunlight decreases, sodoes efficiency. A new antireflective film coating could help panels collect sunshine at 96 per cent efficiency from nearly any angle. The newly-developed film consists of seven layers of nanoscopic silicon and titanium-oxide rods arranged in increasing densities, with the topmost nearly as porous as air. This funnel-like structure captures light from almost every direction and focuses it onto the photovoltaic panel while also inhibiting reflection. The film, which is about one hundredth as thick as a human hair, could easily be applied to any solar panel and would help collect 20 per cent more light while eliminating the need for the expensive hardware usually used to rotate solar panels as the sun moves. Before the new film can be marketed, the nanoscientists who developed the film must find a way to protect the outermost layers from wind and heat, a process that might take another year.

*** It is to a plant’s advantage to be visually attractive to a specific pollinator so that those animals will seek out and concentrate on that particular plant during their search for nectar. This keeps pollen from being spread to other plant species, where pollination won’t take place. Flowers appeal to their pollinators’ sensory systems by using signals such as alluring odours or colours. Flowers pollinated by nocturnal animals like bats and moths, which rely more on hearing than smell and sight, usually have dull colours but powerful scents. Flowers pollinated by daytime animals like birds and bees, however, rely on a range of colours. Birds see a spectrum somewhat similar to ours but are especially receptive to red, so blossoms pollinated by them tend to be red or orange. Bees, on the other hand, see a different spectrum composed of yellow, blue, green, and ultraviolet. Flowers pollinated by bees, hence, tend to be in those colours and usually have special markings that are visible only in ultraviolet. Like runway lights, these markings guide insects to the right place to land and find nectar, and in the process, pollinate the plant.

*** The hope with biofuels is that they can offer a carbon-neutral energy source, because the crops that are grown for fuel will remove as much carbon from the atmosphere as will be released when they are eventually burned. This is basically the same thing that happens when we burn coal or oil. The difference is that in the caseof the latter, the carbon was absorbed hundreds of millions of years ago, and this is part of the problem. Humans will take just a few hundred years to burn through tens of millions of years of oil deposits. To grow enough crops to keep up with our current demand will require lots of additional lands to be cleared for agriculture. If rainforests are cleared to plant sugar cane, all the carbon that is currently locked in the trees will be released. The other problem is that adding nitrogen fertilizer to these crops releases nitrogen oxide, which is another greenhouse gas. A recent study showed that burning maize biofuels actually increases greenhouse gas emissions for this reason. However, this is more the fault of the choice of biofuel crop and the production method rather than a flaw in the biofuel concept as a whole. The shrub “jatropha”, for instance, can be grown on land too poor for trees or other crops to grow on, and is already used for biodiesel in India, Cambodia, and some African countries.

*** A 37,000-year-old baby mammoth could help to explain why the ancient species became extinct as well as giving an insight into climate change. Researchers at a Japanese medical school carried out a computed tomography (CT) scan of the mammoth, which was found frozen in Northern Siberia. They produced some highresolution 3D pictures which are being analyzed to find out about the animal’s internal organs and diet, and to work out how she died. The mammoth’s tissues and skeleton have been studied at a zoological museum in Russia. Air samples from her lungs will also be analyzed for clues to the Earth’s atmosphere at the time of her death. The mammoth, named Lyuba, was found by a reindeer herder buried in permafrost. She is unusual because of the proportionof her body that is preserved. According to one expert working on Lyuba, with fossils, scientists generally get only bones and teeth, but this specimen is special in that there are also the organs and muscles. The same expert notes that it will be interesting to see how this animal managed to adapt to life high in the Arctic and was able to survive, especially in the Ice Age.

*** Cats are famous for their aversion to water. However, the fishing cat, a wild Asiatic species, has no such tendency. In fact, these felines, about twice the size of typical house cats, prefer to be in proximity to water, making their homes in the near rivers and marshes. As their name indicates, the cats fish for their meals, sitting by the water and tapping their paws to create ripples on the surface that resemble insect movements to lure their prey. Their webbed front paws help the fishing cats to remain dry while scooping fish, frogs, and snails out of shallow water. However, they also dive right in to grab large fish and birds in their jaws. Once in the water, the cats can swim on the surface or even glide underwater. Their flat tails, significantly shorter than those of house cats, serve as rudders, helping them to adjust direction below the water surface. Unfortunately, habitat loss and overfishing have decreased the number of these cats by about 50 per cent over the past three generations. Recently, the International Union for the Conservation of Nature (IUCN) changed the status of these cats to “endangered” from the less-severe “vulnerable”.

*** Several years ago, biochemists studying marine ecosystems noticed something unusual: a sponge thriving in the middle of a coral reef that was dying from bacterial infection. The researchers identified a substance made by the sponge in order to defend itself from harmful microbes. They realized that it was a natural antibacterial molecule called “ageliferin”. This molecule can break down the formation of a protective biofilm coating that bacteria use to shield themselves from threats,including antibiotic drugs. Now the same researchers are using this natural compound to create innovative ways to fight drugresistant bacteria. They have recently modified the structure of ageliferin to make it more potent and formulated to help conventional medications combat otherwise drug-resistant bacteria, such as staph and cholera. The newlydeveloped chemical does not stop bacteria from proliferating, but it allows the antibiotic to work again. The researchers hope eventually to incorporate the altered ageliferin as a helper drug within commercial antibiotic products, allowing them to fight off formerly drug-resistant strains of diseases.

ÜDS SAĞLIK 2010 İLKBAHAR

*** A new vaccine that may offer lifetime protection against the flu has shown promise in human trials. It works on the deadly type A strain, responsible for pandemics. Current flu vaccines work by giving immunity against two proteins, called haemagglutinin and neurominidase, found on the surface of flu viruses. However, as these proteins continually mutate, vaccines have to be reformulated every year to keep on working. The new vaccine, known as ACAMFLU-A, gets around this problem by homing in on a protein called M2, found on all type A strains, that does not mutate so readily. The vaccine could be quickly produced in response to a flu outbreak. The doses can be mass produced and used at any time, because there is no need to identify the most prevalent strains. In theory, a single injection could offer lifetime protection. Whether this will work in practice is a matter for future trials.

*** A team of scientists have recently identified one of the molecules responsible for the tubular shape of blood vessels. Originally known for regulating blood vessel development for life,the vascular endothelial growth factor (VEGF) proteins analyzed by the research team have proven to be more interesting than initially thought. The team found that a certain variation of VEGF attracts an “instructor” protein. When this variant, with the instructor protein attached, docks with a receptor on the surface of a cell, a signal telling the cell to form a tube-like shape with its neighbours is sent to it. When the instructor cell is absent, the cells line up next to one another to form a sheet. Scientists believe that not only could this contribution allow blood vessels to be created from stem cells, but the discovery might be employed in other tube-like structuresin the body, such as the lungs and the intestines. Far from being confined to blood vessels, the discovery thus opens the door to resolving the problem of three-dimensional reconstruction of organs from stem cells.

*** The loss of one sense encourages the development of the four others. This has now been demonstrated convincingly in a study by a team of doctors. During five days, volunteers takingpart in the study were blindfolded and asked to carry out exercises designed to stimulate the sense of touch. When their brains were then tested using an MRI, an area of the visual cortex was shown to have been activated as if, being underused, this area was brought in to help the subjects when they had to rely on touch. About 24 hours later, after the blindfolds came off, the participants lost this aptitude. Previously, scientists had always believed that the brain was organized into distinct and highly-specialized systems. This new study, however, shows that the human brain has the ability to reorganize itself. In addition, the rapid reversibility of the process suggests that it is not based on the creation of new nervous connections but on the activation of previously inhibited zones.

*** A study by a group of scientists has resulted in a major step forward in overcoming drug addiction. As addiction was known to cause molecular changes in the brains of addicts, causing their neurons to transmit much stronger signals of dopamine, a messenger molecule involved in reward-seeking behaviours, the scientists were hoping to prove experimentally that certain key proteins in the dopamine-producing neurons influence drug addiction. It was an inspired guess, as the study found that mice in which these key proteins had been selectively switched off displayed clear addictive behaviour. For instance, mice in which the CluR1 protein was switched off showed a much longer period of dependence. Conversely, re-administration of cocaine after a long break immediately re-ignited the addiction, but mice whose NR1 protein had been deactivated resisted relapsing into addictive behaviours. The ability of these proteins to determine addictive behavioural patterns makes them fascinating.

*** Post-traumatic stress disorder (PTSD) is an anxiety disorder that can develop after exposure to a terrifying event or ordeal. While many scientific studies focus on the molecular mechanisms for learning and memorization, scientists need to address the “unlearning” process to tackle PTSD. Researchers have discovered that a receptor for glutamate, the most prominent neurotransmitter in the central nervous system, plays a key role in the unlearning process. They made this discovery in experiments where they trained mice to fear a sound by coupling it with an electric shock to the foot. They found that if, following this fear conditioning, the mice are repeatedly exposed to the sound without the electric shock, their fear eventually subsides. However, mutant mice lacking the gene-coding for metabotropic glutamate receptor 5 (mGluR5) are unable to shake off their fear of the now harmless sound. The researchers believe that a similar mechanism might be perturbed in PTSD sufferers and that metabotropic glutamate receptor 5 (mGluR5) may provide a potential target for new therapeutic treatments.

*** A deficiency in Vitamin D is known to cause various diseases due to insufficient calcium or phosphate in the bones. Vitamin D is actually an umbrella term that covers a group of steroid molecules. Of these, only Vitamin D3 requires sunlight to synthesize. It is formed in the skin of all mammals when light energy is absorbed by a precursor molecule called 7-dehydrocholesterol. A recentstudy found that at the height of summer, two minutes’ exposure of the face and arms to the sun, three to four times a week, could supply enough Vitamin D3. This rose to 15 minutes in the winter. Ironically, Vitamin D deficiency, which may also lead to skin cancer, is actually very common in some of the sunniest but most underdeveloped countries in the world. This is not because of malnutrition or a lack of dietary supplements, as most people mistakenly think. In fact, it is because very dark skin colour slows the rate of Vitamin D3 production by a factor of six, and people in these countries are usually heavily veiled when outside, as well. In addition, wearing sunscreen with a sun protection factor greater than eight will also block Vitamin D3 production.

ÜDS SOSYAL 2010 İLKBAHAR

*** Several EU member states enjoy long-standing political and economic links with Latin America. In fact, it was in the 1960s and 1970s that the EU first began expanding its ties to the region through a series of diplomatic initiatives and agreements aimed at promoting democracy, addressing development issues, and boosting trade and investment. Moreover, the EU actively worked for peace in the troubled Central American region in the 1980s. In the meantime, the accession in 1986 of Spain and Portugal to the EU furtherstrengthened region-toregion ties. EU engagement in Latin America increased during the 1990s as a formal political dialogue was put in place to advance issues of common interest, including how the EU and Latin America together can act in concert with other nations and international organizations to address global issues and challenges.

*** The Agta Negritos of the Philippines, a present-day tribal people, are an example of a culture whose women and men share all subsistence activities. Most interestingly, the Agta Negritos women hunt large game with bows, arrows, and hunting dogs. The women are prevented from hunting only during late pregnancy and the first few months after giving birth. Teenagers and women with older children are the most frequent hunters. The women space their children to allow for maximum mobility. They keep their birth rate down through the use of herbal contraceptives. By studying these ethnographic examples and by questioning the assumptions that have been made about female and male roles in prehistory, anthropologists have concluded that Western society’s traditionally low view of women’s status is by no means universal.

*** For the past 300 years, musicians and scientists have puzzled over the unparalleled quality of classical Cremonese violins made by the Italian master Antonio Stradivari. These classical violins have become the benchmark against which the sound of all other violins is compared. There are many theories as to the “secret” of Stradivarius violins. What was obviously first explored was the exact size of the violins and ratio of the parts to each other. Although instrument makers have disassembled their violins, calibrated every dimension of the pieces to within the hundredth of an inch, and replicated the measurements perfectly in new instruments, they have failed to duplicate the Stradivarius magic. It is also well-known that the density of the material through which a sound propagates influences significantly the vibration efficiency of the material, therefore the tonal qualities of the instrument. It is also a widely held belief that Stradivari’s well-guarded varnish formula was not just a protective coating of the instrument, but actually the most important secret to his violins.

*** Satellite images of the upper Amazon Basin in Brazil taken since 1999 have revealed hundreds of circles, squares, and other geometric shapes once hidden by the Amazon rain forests. They hint at a previously unknown ancient society that flourished in the Amazon. Now researchers estimate that nearly ten times as many such structures, of unknown purpose, may exist undetected under the Amazon forest cover. The discovery adds to evidence that the hinterlands of the Amazon once teemed with complex societies, which were largely wiped out by diseases brought to South America by European colonists in the 15th and 16th centuries. Since these vanished societies had gone unrecorded, earlier research had suggested that soils in the upper Amazon were too poor to support the extensive agriculture needed for such large, permanent settlements. The researchers say “We found that this view is wrong, and there is a lot more to discover in these places”.

*** A company’s public relations officer is responsible for creating and maintaining relationships between clients and customers. Through areas such as brand management, advertising, media relations and crisis management, public relations officers seek to foster interest, trust and belief in the company and its products. They are aware of how best to carry this out when dealing within their own nations and cultures. However, when dealing with a foreign audience, it is critical thatcross-cultural differences are recognized. By way of illustrating the impact cross-cultural awareness can have on the success or failure of a public relations campaign, a brief example can be cited: an American company tried to sell its toothpaste in Southeast Asiaby emphasizing that it “whitens your teeth”. Theyfound out that the local natives chew betel nuts to blacken their teeth because they found it attractive.

*** Following the terrorist attacks of September 11, 2001, Americans understandably rallied around the flag. Having just suffered the deadliest attack ever on the US soil, a great majority of the people believed another attack was imminent. But Americans also had enormous faith the “Global War on Terror” would help keep them safe. Just one month after 9/11, for instance, 94 per cent of Americans fully approved of how the fight against terrorism was being handled. The United States then quickly went to war in Afghanistan, closing down a terrorist camp and capturing or killing a number of high-level al Qaeda operatives in the process. However, since 2001, terrorists have found their targets on almost every continent, with bombings in Bali, London, Madrid, Istanbul, and elsewhere. Now Americans appear less convinced that their country is winning the war on terror. In the face of persisting threats, including a growing number of terroristattacks around the world, numerous reports show that Americans are losing faith in their government’s ability to wage the war successfully and to protect them from the terrorists’ next major attack.

ÜDS FEN 2010 SONBAHAR

*** Wildfires are an important environmental hazard in many geographical areas. Those areas most prone to wildfires have wet seasons followed by dry seasons. Vegetation that grows and accumulates during the wet season dries out enough during the dry season to burn easily. When lightning hits the ground, it ignites the dry organic material, and a fire spreads through the area. Actually fires have several effects on the environment. First, burning frees the minerals that are locked in organic matter. The ashes remaining after a fire are rich in potassium, phosphorus, calcium, and other minerals essential for plant growth. Thus, vegetation flourishes following a fire. Second, fire removes plant cover and exposes the soil, which stimulates the germination of seeds requiring bare soil, and encourages the growth of shade-intolerant plants. Third, fire can cause increased soil erosion because it removes plant cover, leaving the soil more vulnerable to wind and water.

*** Commercial logging, mostly for export abroad, accounts for 21% of tropical deforestation. Most tropical countries allow commercial logging to proceed at a much faster rate than is sustainable. For example, in parts of Malaysia, current logging practices remove the forest almost twice as fast as the sustainable rate. If thiscontinues, Malaysia will soon experience shortages of timber and will have to start importing logs. When that happens, Malaysia will have lost future revenues, both from logging and from harvesting other forest products, from its newly vanished forests. Moreover, in addition to commercial logging, cattle ranching also causes deforestation. In fact, approximately 12% of tropical rainforest destruction is done to provide open rangeland for cattle. After the forests are cleared, cattle can be raised on the land for six to ten years, after which time shrubby plants take over the range. Much of the beef raised on these ranches, which are often owned by foreign companies, is exported to fast-food restaurants.

*** Moisture is removed from humid air by mountains, which force the air to rise. As it gains altitude, the air cools, clouds form, and precipitation occurs, primarily on the windward slopes of the mountains. As the air mass moves down on the other side of the mountain, it is warmed, thereby lessening the chance of precipitation of any remaining moisture. This situation exists on the west coast of North America, where precipitation falls on the western slopes of mountains that are close to the coast. The dry lands on the sides of the mountains away fromthe prevailing wind are called “rain shadows.” Generally, differences in elevation, in the steepness and direction of slopes, and in exposure to sunlight and prevailing winds may produce local variations in climate known as “microclimates,” which can be quite different from their overall surroundings. For an organism, the microclimate of its habitat isof primary importance, because that is the climate an organism actually experiences and knows how to cope with.

*** In the early part of the twentieth century, the experiments carried out by Ernest Rutherford and his colleagues led to the idea that at the centre of an atom there is a tiny but massive nucleus. At the same time that the quantum theory was being developed and that scientists were attempting to understand the structure of the atom and its electrons, investigations into the nucleus itself had also begun. An important question to physicists was whether the nucleus had a structure, and what that structure might be. In fact, it has so far turned out that the nucleus is a complicated entity, and even today, it is not fully understood. However, by the early 1930s, a model of the nucleus had been developed that is still useful. According to this model, a nucleus is considered as an aggregate of two types of particles: protons and neutrons. A proton is the nucleus of the simplest atom which is hydrogen. The neutron, whose existence was ascertained only in 1932 by the English physicist James Chadwick, is electrically neutral as its name implies. These two constituents of a nucleus, neutrons and protons, are referred to collectively as “nucleons.”

*** Today scientists draw attention to some of the potential consequences of global warming on wildlife. They point out that each species reacts to changes in temperature differently. Some species will undoubtedly become extinct, particularly those with narrow temperature requirements, those confined to small reserves or parks, and those living in fragile ecosystems, whereas other species may survive in greatly reduced numbers and ranges. Ecosystems considered most vulnerable to species loss in the short term are polar seas, coral reefs, mountains, coastal wetlands, tundra, taiga, and temperate forests. On the other hand, some species may be able to migrate to new environments or adapt themselves to the changing conditions in their present habitats. Also, some species may be unaffected by global warming, whereas others may emerge from it as winners, with greatly expanded numbers and ranges. Those considered most likely to prosper include weeds, pests, and disease-carrying organisms that are already common in many different environments.

*** Earth and Venus, being roughly the same size and distance from the sun, are often regarded as twin planets. So it is natural to wonder how the crust of Venus compares with that of our own world. Although centuries of telescopic observations from Earth could give no insight, beginning in 1990 the Magellan space probe’s orbiting radar penetrated the thick clouds that enshroud Venus and revealed its surface with stunning clarity. From the detailed images of landforms, planetary scientists can surmise the type of rock that covers Venus. It seems that our sister planet is completely covered by rocks of basaltic composition, which are very much like the dark, finegrained rocks that line the ocean basins of Earth. Magellan’s mapping, however, failed to find extensive areas comparable to Earth’s continental crust.

ÜDS SAĞLIK 2010 SONBAHAR

*** Parkinson’s and Alzheimer’s diseases are the most common diseases which affect many people in the world. Approximately, 1 million people in the world suffer from Parkinson’s disease, a motor disorder characterized by difficulty in initiating movements and slowness of movement. Patients often have a masked facial expression, poor balance, and a flexed posture. Like Alzheimer’s disease, which is characterized by confusion, memory loss, and a variety of other symptoms, Parkinson’s disease is progressive, and the risk increases with age. The incidence of the Parkinson’s disease is about 1% at the age of 65, and about 5% at the age 85. Parkinson’s disease appears to result from a combination of environmental and genetic factors. Evidence for a genetic role includes the fact that some families with an increased incidence of Parkinson’s disease carry a mutated form of the gene for a protein which is important in normal brain function. The symptoms of Parkinson’s disease result from the death of neurons in the midbrain. As a result, at present, there is no cure for Parkinson’s disease, although various treatments can help control the symptoms.

*** Gene replacement therapy is being developed for several genetic diseases. Because many difficulties are inherent in treating most serious genetic diseases, scientists have dreamed of developing actual cures. Today, genetic engineering is bringing these dreams closer to reality. Such therapy could take two main forms. One approach would be to introduce copies of a normal gene into a fertilized egg, using modifications of the technology already used to produce transgenic animals. In some transgenic animals the introduced gene can remain stable from generation to generation, constituting a true “genetic cure.” However, this approach raises such complex ethical problems that it is not being actively pursued at this time. A second strategy – to introduce the normal gene into only some body cells (somatic cell gene therapy) – is receiving increased attention today. The rationale is that, although a particular gene may be present in all cells, it is expressed only in some. Expression of the normal allele in only the cells that require it may be sufficient to give a normal phenotype. Although this approach presents a number of technical obstacles, which must be overcome, gene therapies for a number of genetic diseases are undergoing development or are being tested on patients in clinical trials.

*** The pirate look is a time-honoured way to fix children’s “lazy eye”. The patch over the good eye forces the weak one to work, thereby preventing its deterioration. Playing video games helps, too. The neural cells corresponding to both eyes then learn to fire in synchrony so that the brain wires itself for the stereo vision required for depth perception. Left untreated past a critical age,lazy eye, or amblyopia, can result in permanently impaired vision. New studies are now showing that this condition, which affects up to 5 per cent ofthe population, could be repaired even past the critical age. What is more, amblyopia may provide insights into brain plasticity that could help treat a variety of other disorders related to faulty wiring, including schizophrenia, epilepsy, autism, anxiety, and addiction. These ailments are not neurodegenerative diseases that destroy part of the neural circuitry. So, if the defective circuits could be stimulated in the right way, the brain could develop normally.

*** Many drugs, whether prescribed or abused, affect the nervous system. While about 25% of all prescribed drugs are taken to alter psychological conditions, almost all the commonly abused drugs affect mood. In particular, levels of serotonin and dopamine are thought to influence mood. For example, when excessive amounts of norepinephrine are released, people feel energetic and stimulated, whereas low concentrations of this neurotransmitter reduce anxiety. Habitual use of almost any mood-altering drug can result in psychological dependence, in which the user becomes emotionally dependent on the drug. When deprived of it, the user craves the feeling of euphoria (well-being) that the drug induces. Some drugs induce tolerance after several weeks. This means that response to the drug decreases, and greater amounts are required to obtain the desired effect. Tolerance often occurs because the liver cells are stimulated to produce more of the enzymes that metabolize and inactivate the drug. Use of some of the drugs, such as heroin, tobacco, and alcohol, may also result in addiction (physical dependence), in which physiological changes occur that make the user dependent on the drug. Addiction can also occur because certain drugs, such as morphine, have components similar to substances that body cells normally manufacture on their own. The continued use of such a drug causes potentially dangerous physiological effects.

*** Anyone who has spent even a little time with an autistic boy or girl soon becomes familiar with the behaviours that set these children apart. But how do parents and doctors know if a baby has autism? Early diagnosis has proved difficult, but if it were possible, it would lead to much moreeffective treatment for the younger a child is the more malleable is the brain. So, the inability to detect autism until a child is two or three years old is a terrific disadvantage, as it eliminates a valuable window of treatment opportunity, when the brain is undergoing tremendous development. Researchers, however, are closing in on techniques that could detect autism in babies as young as six months and perhaps even at birth. The results of these new tests are expanding the understanding of autism and raising hopes for much earlier, specialized care that could improve a toddler’s chances for a more normal life as a child, teenager and adult.

*** For humans to be able to hear a sound, it must be both loud enough and within the right frequency range – as measured by the number of vibrations per second, or hertz (Hz). The average person is most sensitive to sounds in the 1,000-5,000 Hz range, and most lose the ability to hear very high frequencies (above around 20,000 Hz) with age. Even so a sizeable proportion of the population do seem to remain sensitive to the very low frequency “infrasound”. High-frequency sounds have more than just audible effects as teenagers in Swindon discovered in 2006. Tired of having crowds of youngsters collecting around the town theatre, the owners installed the Mosquito, a device that emits sonic energy at very high frequencies. Only the teenagers could hear it and it forced them to meet elsewhere.

ÜDS SOSYAL 2010 SONBAHAR

*** When Timemagazine declared its 2006 person of the year to be “You”, the magazine was pointing to an undeniable reality: anyone with an Internet connection can be a reporter, political commentator, cultural critic, or media producer. Around the same time, the media scholar H. Jenkins and his colleagues published a paper appreciating the “participatory cultures” of creation and sharing, mentorship, and civic engagement that were emerging online, especially among young people. AlthoughTimedid not explicitly frame participation in the new media as a youth phenomenon, most of the fifteen “citizens of digital democracy” who were featured in its December 13 article were under the age of thirty-five. Jenkins and his colleagues strongly suggest that young people are especially well-poised to take full advantage of Web 2.0. On the other hand, ever since digital technologies were made available, scholars, educators, policymakers, and parents have been debating their implications for young people’s literacy, attention spans,social tolerance, and tendency for aggression. Considerable strides are now being made in scholarship in many of these areas.

*** Although the United Statesand the Soviet Union became allies during World War II, there seemed to be little doubt that their opposing ideologies would ultimately produce a cold war. The cold war was a global phenomenon and was clearly conditioned by the political, economic, and social aspirations of the two superpowers. Between 1945 and 1989 Europe became a testing ground for the cold war itself. The Soviets were convinced that Eastern European buffer states had to be created to protect the Soviet Union from future invasions. As for the United States, the Truman Doctrine, Marshall Plan, and NATO were all intended to keep Russian ideology within its own borders. When the cold war came to an end following the collapse of the Berlin Wall in November 1989, the Soviet ideology was considered a spent force in Eastern Europe.

*** The last decade has seen notable changes in disability policy in Europe. Changed assumptions about the concept of disability have been reflected in the adoption of new national and pan-European legislation. As a consequence, the policy which has sought to separate and segregate people with disabilities in “special schools”, labour markets, residential accommodation and transport has, to some degree, and in some countries, been reconsidered. Attempts have been made to develop an integrated approach, opening up jobs, services and housing to all people irrespective of their ability or disability. A key element of this new approach has been the recognition that segregation and exclusion is not a necessary consequence of a physical or intellectual impairment, but the result of conscious policy choices based on false assumptions about the abilities of the people with disabilities. The new approach recognizes the role which discrimination plays in disadvantaging people with disabilities and, conversely, how legislation seeks to combat elements of disability discrimination and creates equality of opportunity for people with disabilities.

*** For the present, NASA appears to be committed to maintaining its human spaceflight program, whatever the cost. However, in the next decade, it may discover that it does not need human characters to tell compelling stories. Insteadof gazing at posters of astronauts, children are now playing with toy models of Mars rovers. The next generation of space adventurers is growing up with the knowledge that one can visit another planet without boarding a spacecraft. Decades from now, when those children are grown-ups, some of them will lead the next great explorations of the solar system. Sitting in quiet control rooms, they will send instructions to far-away probes already launched and make the final adjustment that points us towards the stars.

*** When prehistoric man returned home from a hunt, he was almost certainly asked the question we would like to ask today: “What happened?” Quite possibly, he replied in a factual manner, providing a short report of the land covered, the number of animals spotted, and the results. His face-to-face communication was limited only to those within the sight and sound of the speaker. Either because of this or because he thought his communication should be recorded in more permanent form, the caveman eventually began to draw his message, the report of his latest adventurous hunt, on the wall of the cave. This opened up a whole range of possibilities: The wall was there twenty-four hours a day, seven days a week. The caveman could go about his other business, whatever that may have been, and still know that his message was being communicated, for the audience was communicating not with the caveman himself, but with the wall. This was the beginning of mass communication – impersonal communication with a diverse audience that has a limited opportunity to respond – and much was gained from it.

*** From the mid-fifteenth century on, most of Europe had enjoyed steady economic growth, and the discovery of the New World seemed the basis of greater prosperity to come. By the middle of the sixteenth century, however,the situation changed. Nothing like the upward price trend that affected Western Europe in the second half of the sixteenth century had ever happened before. Since Europe’s population began to grow vastly and the food supply remained constant, food prices were driven sharply higher by the increased demand. At the same time, wages stagnated or even declined. On the other hand, the enormous influx of silver from Spanish America into Europe, where much of it was minted into coins, caused a dramatic increase in the volume of money in circulation. This, of course, fuelled the spiral of rising prices.

ÜDS FEN 2011 İLKBAHAR

*** The book by physicist Lee Smolin, The Trouble with Physics,is an all-out attack on string theory in theoretical physics. String theory aims to unify the laws governing all physical forces by combining quantum mechanics with general relativity. It is not very intuitive as it posits the existence of 10 space-time. Smolin points out that, not once in its 30 years of existence, has string theory been validated by a test result. Although they acknowledge this weakness, the theory’s advocates claim that it helps to clarify a number of concepts and, most important of all, it holds the promise of a grand unification. Smolin’s historical account is both brilliant and lively. The most interesting feature of the book is his sociological analysis of the way in which string theory has taken root in academic circles and the mechanisms that allowed it to gain its present almost total dominance. How can a community of likeminded scientists have secured such a powerful position that it is now able to determine the course of research, to monopolize public funding and to decide careers, to the point of abolishing all alternative approaches? Indeed, his analysis is applicable to many other fields and disciplines.

*** Stars are believed to begin life as collapsing masses of hydrogen gas. which are called “protostars”. As collapsing masses start contracting, they heat up. When the temperature in them reaches 10 million degrees, nuclear fusion begins and forms heavier elements, mainly helium at first. The energy released during these reactions balances the gravitational force, and the young star stabilizes as a main- sequence star. The tremendous brightness of stars comes from the energy released during these thermonuclear reactions. After billions of years, as helium is collected in the core and hydrogen is used up, the core contracts and heats further. The envelope expands and cools, and the star becomes a red giant. The next stage of stellar evolution depends on the mass of the star. Stars of residual mass less than 1.4 solar masses cool further and became white dwarfs, eventually fading and going out altogether. if the star’s residual mass is greater than two or three solar masses, it may contract even further and form a black hole, which is so dense that no matter or light can escape from it.

*** The sheer scale of European information storage systems and the interoperability of numerous existing databases inevitably raise a string of privacy questions. This explains the public mistrust of these technologies. In France, the Edwige police database for retrieving information on all kinds of activists from age 13 has been met with public outrage. In Germany, the creation of a vast “antiterrorist” database has provoked a wave of protest. The UK is also regularly attacked by ethical questions concerning its database of DNA fingerprints from 4.5 million individuals involved in a major or minor crime. No wonder people have grave concerns. How much trust can be placed in digital data storage tools when, in 2007, a British company simply mislaid the personal data of 25 million individuals or when, in 2008 confidential information on 30 million clients of a German bank appeared on the internet for 48 hours? The biggest danger is not that these tools could be used by officials who are dishonest about civil liberties, but that they are not secure from external intrusion or negligence. The architecture of these security systems ought to be foolproof.

*** Ozone molecules in the stratosphere absorb incoming solar ultraviolet radiation. With depletion of the ozone layer, more ultraviolet radiation reaches the Earth's surface. Excessive exposure to ultraviolet radiation is linked to a number of human health problems. These include cataracts, skin cancer, and a weakened immune system. However, this is not the end of the list. Much scientific evidence also documents crop damage from exposure to high levels of ultraviolet radiation. Moreover, biologists are seriously concerned that the ozone hole over Antarctica could damage plankton that forms the base of the food web for the surrounding ocean. A 1992 study confirmed that increased ultraviolet radiation is penetrating surface waters around Antarctica. This extra amount of ultraviolet radiation is negatively affecting Antarctic phytoplankton. The productivity of Antarctic phytoplankton has declined by at least 6% to 12% as a result. If the productivity of phytoplankton continues to decline, the complex food web of Antarctica, which includes fishes, seals, penguins, whales, and vast populations of birds, will beat risk.

*** Are we born with a limited number of heart cells or is the heart able to generate new ones? To this long- debated question, researchers at the Karolinska Institute have come up with an answer: Heart cells are continually replaced. The renewal rate is 1 % per year up to age 20 years and decreases over the years, reaching 0.5% in the ’70s. Thus, over a lifetime, less than half of the myocardial cells are renewed. The method used by Jonas Frisenna and his team to uncover the heart’s ability to produce new cells is totally Innovative. They determine the age of heart cells using the carbon-14 dating method. Following the aerial nuclear explosions conducted during the Cold War in the 1950s, large quantities of this radioactive isotope were released into the atmosphere and absorbed by plant, animal and human cells and DNA. But since the Nuclear Test Ban, these quantities of carbon-14 have decreased quite rapidly. Scientists have analyzed the carbon14 content of the DNA of heart cells of people born before and after the nuclear tests to determine when these cells were generated. The results of this study open new perspectives in the search for therapies to alleviate cell death in myocardial infarction.

*** Proteins are of central importance in the chemistry of life. These macromolecules serve as structural components of cells and tissues; growth and repair, as well as maintenance of the organism depend on an adequate supply of these compounds. Many proteins serve as enzymes, molecules that speed up the thousands of different chemical reactions that take place in an organism. The protein constituents of a cell are the clues to its lifestyle. Each cell type has characteristic types, distributions, and amounts of protein that determine what the cell looks like and how it functions. A muscle cell differs from other cell types by virtue of its large content of the proteins myosin and actin, which are largely responsible for its appearance as well as for its ability to contract. The protein haemoglobin, found in red blood cells, is responsible for the specialized function of oxygen transport. Although carbohydrates and lipids tend to have the same structures, among different species, most proteins are species-specific; that is, their structures vary from species to species. The specific proteins present are largely responsible for differences among species.

ÜDS SAĞLIK 2011 İLKBAHAR

*** The common cold is a misery, a nuisance and a financial burden which costs the United States economy alone an estimated $5 billion every year. The main problem is that it is not caused by one virus, but by a family of viruses of over 200, making it impossible to vaccinate against. Once infected, all one can really do is to ease the symptoms. Drugs relieve sinus pain and headache, while decongestants reduce blood flow to mucus membranes, unblocking the stuffy nose. More powerful symptomatic treatments are unlikely. Medication that carries even the slightest risk of side effects would never be approved to treat a condition which, for most people, is non-lethal, but simply a nuisance. Yet, for the sufferers of chronic lung diseases, such as asthma and bronchitis, colds can be deadly. For these people, the new drugs that are being developed could offer a lifeline. Antiviral drugs, which target the virus directly, are currently in development. These could kill up to half of the coldproducing viruses, greatly lowering the chance of infection.

*** A person’s emotional state is not just signalled to others by facial expression, but by body posture as well. This is the finding of researchers at the Harvard Medical School. They showed seven people some images of body posture - happy, fearful, and emotionally neutral like opening a door or pouring a glass of water. The subjects' emotional response to these images was evaluated by studying their brain activity with functional magnetic resonance imaging (fMRI) scans. It was found that viewing fearful whole- body expressions produced higher activity in areas known to process emotional information than viewing images of meaningful but emotionally neutral body actions. In contrast, viewing happy postures produced higher activity in areas of the brain that process visual information. The research showed that when it comes to conveying emotion, the body could be just as important as the face. However, almost all the studies so far into the perception of emotion have focused mainly on the brain activity generated by images of facial expressions. Further developments on these evaluations will follow after the invention of more advanced devices.

*** It is generally believed that DNA was discovered by the Cambridge scientists Francis Crick and James Watson, who won a Nobel Prize in 1962 for revealing its purpose. In fact, everyone is wrong: DNA was actually discovered in 1869, and its purpose revealed years before Crick and Watson. In fact, the real credit should have gone to Oswald Avery and his team at Rockefeller University, New York, who, in 1944, used bacteria to show that DNA passed genetic information from one organism to another. The trouble was that all the experts, including even those who advised the Nobel committee, then claimed that DNA was too simple to do this and could not possibly carry all the information needed to build a living organism. This was almost impossible. Thus, Avery was repeatedly denied the prize. However, by the early 1960s the Nobel committee agreed, and accepted Avery and his team had been right all along and deserved the Nobel Prize. But, by then it was too late as Avery had died in 1955. This is one of the sad stories in science.

*** Scientists have thought about the brain’s intricate form for centuries. In the early 1800s, German physician Franz Joseph Gall proposed that the shape of a person’s brain and skull spoke volumes about that individual’s intelligence and personality which is a theory known as “phrenology”. This influential, even though scientifically unsupported, idea led to the collection of “criminal” , “degenerate" and “genius” brains. Then, in the latter part of the 19th century, Swiss anatomist Wilhelm His claimed that the brain develops as a sequence of events guided by physical forces. British polymath D’Arcy Thompson built on that foundation, showing that the shapes of many structures, biological and Inanimate, result from physical self-organization. Provocative though they were, these early suppositions eventually faded from view. Phrenology became known as a pseudoscience, and modern genetic theories replaced the biomechanical approach of the 19th century and furthered our understanding of the structure of the human brain. Thus, no matter how exciting the theories may be more reliable information is a prerequisite for their acceptance.

*** The ear feels blocked when the air pressure in the space behind the eardrum, that is to say, the middle ear, is different from external air pressure. Normally this is not a problem because air is supplied to the middle ear from the outside through a narrow tube at the back of the nose, which is called “the Eustachian tube". This is usually closed, but yawning or swallowing opens it up sufficiently to permit the Internal and external pressures to equalize. Tilting the head back also helps because of the way in which the Eustachian tube is positioned in the head. If the tube is blocked, as when we have a cold, a difference in pressure gradually builds up. This stretches the eardrum and makes sounds seem muffled. Problems can also arise when there is a rapid change in external pressure during an aircraft descent or an underwater dive. This is one of the reasons why infants and children start crying while the aircraft is descending. The secret in all cases is to use your cheek and throat muscles to create a pressure that opens up the Eustachian tube.

*** The revelation that H2S is produced in the cardio-vascular system and helps to control blood pressure caught the attention of many researchers who had been looking for novel ways to protect the heart against damage from oxygen deprivation, as occurs when a clot prevents blood from bringing oxygen to the heart, leading to the death of cardiac tissue. In 2006, Gary Baxter reported that in isolated rat hearts, which were first provided with saline solution to mimic blood supply and then deprived of the saline to mimic a heart attack, administering H2S to these isolated hearts before halting the saline supply reduced the extent of cardiac muscle damage. Later, David Lefer showed that mice engineered to produce more H2S in the heart were better able to tolerate oxygen deprivation caused by a clot and more resistant to the damage that often ensues when blood flow is restored to tissues after a period of deprivation. Findings such as these suggest that H2S could be used to prevent or treat hypertension, heart attacks and strokes in humans. But the gas’s ability to relax blood vessels means that its potential applications could extend to other blood vessel problems too - including erectile dysfunction.

ÜDS SOSYAL 2011 İLKBAHAR

*** Nowadays, we all like to think we have got past the racist nonsense of previous centuries, when even the most eminent scientists, white ones, of course, declared white people to be the pinnacle of human progress and other races to be inferior. We now accept that no race is superior to another. There is no question that most societies have made enormous progress in eliminating such overt racial prejudice. But an unsettling study published by researchers in the United States suggests that there is still a long way to go. Even today, the study finds, Americans of various races still unconsciously dehumanize their black fellow citizens by subtly associating them with apes. In an experiment in which students were subliminally flashed a photo of either an African- American or a European-American face, and then shown a blurry picture of an ape, those shown the black face were quicker to recognize the ape. More troubling still, this association is not just confined to psychologists’ tests: It also appears to bias people’s judgements about whether specific instances of police violence are justified.

*** For centuries, when one country disapproved of another country’s behaviour, political leaders sought ways to communicate their displeasure short of going to war. That is the idea behind economic sanctions. Sanctions allow countries to punish another government without having to resort to violence. At least, that was the idea. But a recent study reveals that sanctions actually make it far more likely that two states will eventually meet on the battlefield. The researchers examined more than two hundred cases of sanctions and found that, when sanctions are added to the mix, military conflict is extremely likely to occur between two countries than if sanctions had not been imposed at all. Because countries generally prefer to enact sanctions that are not especially costly to themselves, target countries often interpret the action as a lack of resolve. This interpretation may lead the country being sanctioned to become provocative in its actions, which may in turn pave the way for a military confrontation.

*** Best known for his novels 20,000 Leagues Under the Seaand Around the World in 80 Days,Jules Verne, the French science fiction pioneer, has always been a major cultural figure in his native land, France. His hometown of Amiens continues to remember his contributions with parades, exhibitions and literary conferences. Yet, in the English-speaking world, Jules Verne has been pigeonholed as merely a young boy’s adventure writer, even though he was successfully able to foresee heavier-than-air flying machines and moon voyages. In addition, twentieth century pioneers such as the polar explorer Richard Byrd, the rocket scientist Wernher von Braun and the astronaut Neil Armstrong have all said that Jules Verne’s writings inspired them. But now, Jules Verne enthusiasts are pushing for a reconsideration of the writer as an influential literary figure, whose 64 novels and stories - of admittedly varying literary quality - offer not only startling prophecies but also offer provide a sharp commentary on the Europe and America of his day.

*** The most striking achievement of the successful Danish economy probably relates to labour market dynamism. Such an achievement Is hard to find throughout the rest of the world. The unemployment rate has been brought down from a previously high level and is now amongst the lowest across Europe. The Danish labour market model has been called “flexlcurity" which Is a combination of flexibility and security. How is it possible to blend labour market flexibility, as in the British model, with a generous unemployment benefit system, as in the continental European model, while still maintaining high job security and satisfaction levels? The Danish labour market model is based on three pillars. First, moderate employment protection makes it easy for firms to adjust their workforce to new conditions - easy to fire, easy to hire. Second, a generous unemployment benefit system protects job losers against significant drops in income, especially for those at the lower end of the income scale, where job insecurity is also highest. Third, active labour market policies are used extensively to bring relevant qualifications to the unemployed and to test availability for work.

*** A battle between traditional and progressive educators has raged since the 1930s, when modern approaches to musical teaching were first proposed. Stephanie Pitts, author and editor of the British Journal o f Music Education,charts the course of this battle in her book A Century of Change in Music Education.Both camps argued for music’s inclusion in the school curriculum, but their justifications and proposed teaching methods conflicted. On the one hand, declared traditionalists, music was a body of knowledge made up of the great symphonies and folk songs of Britain's repertoire. Students could be taught to appreciate and sing this “national songbook” as part of a cultural education. This was the method that dominated the classroom for decades, and is still part of the primary school approach. On the other hand, responded the progressives, if music was an expression of our inner psyche, then children should be encouraged to play and experiment in order to learn more about themselves. “Bring something to make noise with", they shouted.

*** Few operas are as rooted in one place as Benjamin Britten’s Peter Grimes.The title character is a dark- souled fisherman who goes mad after the death of his apprentice. Grimes was the invention of the poet George Crabbe, who grew up in Aldeburgh, on the eastern coast of England, in the later part of the eighteenth century. Crabbe apparently based Grimes on a detested local character. Montaga Slater, the opera’s librettist, wove his elaboration of the tale into various Aldeburgh settings. Britten, who was a resident of the same town for most of his adult life, brilliantly evoked its sights and sounds in his music - the crying of gulls, the creaking of buoys, the endless booming of the waves. The obvious way to stage Grimesis to re-create the original setting of Aldeburgh and let Britten’s flawless score do the rest. This was the approach taken by Tyrone Guthrie, who first directed the opera at Covent Garden's famous opera house, in 1947.

ÜDS FEN 2011 SONBAHAR

*** Early in the 20th century, volcanologist Giuseppe Mercalli created a scale to categorize earthquakes based on the level of damage incurred. This Mercalli intensity scale ranges from Level 1, which is registered only by seismographs, to Level 12, which results in severe changes on the Earth’s surface and the destruction of almost all buildings. The more popular and more scientific Richter scale, developed by Charles Francis Richter, calculates the intensity of an earthquake as “magnitude” (M) on a logarithmic scale. The M-value is determined from the distance between the hypocenter of the earthquake and the seismological recording station, as well as the amplitudes recorded on seismographs. Earthquakes with magnitudes less than 2.0 are not perceivable by people. Each number on the Richter scale represents an earthquake ten times more powerful than the number below it. Today, scientists use the more precise moment-magnitude scale. To calculate the M-value, this scale multiplies the area of the fault’s rupture by the distance moved along the fault. The study and measuring of earthquakes is crucial in aiding scientists and engineers with planning for future occurrences, especially because there could be deadly consequences.

*** Saving the planet might be the selfless motive for driving electric cars, but most owners derive the greatest pleasure from the silence of their vehicles. However, it’s a pleasure soon to disappear as lawmakers in Europe and the US prepare rules to make them noisier. The argument is that pedestrians would be safer, especially those with impaired vision and hearing. Although some figures from the US do suggest that proportionately there are higher collision rates with electric vehicles, conclusive data is nevertheless scarce. Do people really navigate the urban jungle using only their ears? Or indeed the latest diesel-engined vehicles, which, at low speeds, are very quiet? In fact, electric cars offer a rare opportunity to cut both chemical and noise pollution and we should embrace them. The sound made by internal-combustion engines is just an unwelcome waste product. Some people may find it useful, but then so did those who gathered up the dung from all our horse-driven carriages to use as fertiliser. Insisting that electric cars make a noise would be like passing a law in the early 20th century, obliging drivers of the new-fangled “motor vehicle” to deposit a load of well-rotted compost on the side of the road every 50 miles.

*** Calculus was independently developed by Gottfried Wilhelm Leibniz and Isaac Newton in the late 17th century. Leibniz based his theory on the use of geometric processes to solve mathematical problems. He viewed a curve as being made up of infinitely small segments, whereby the slope of the tangent could be calculated for each segment. He recognized the relationship between differential and integral calculus. Newton, on the other hand, was more interested in solving a physics problem: how to determine the instantaneous speed of an accelerating object. He viewed a curve as a reflection of constant acceleration and imagined a point as an infinitely small segment of a line. The time interval between observations of an object’s motion could be reduced to the point that the change in speed disappears. Thus, acceleration or deceleration can be calculated as the sum of the instantaneous speeds of the observed object. Leibniz was later accused of stealing Newton’s ideas from the correspondence exchanged by the two, and the Royal Society of London, influenced by Newton, erroneously pronounced him guilty. However, Leibniz’s system eventually became the dominant form of calculus, thanks to its elegant notation and simplicity.

*** For decades, Germany has had some of the most enlightened energy policies in Europe. It has long been admired for setting world-leading growth in wind and solar. But, its decision to end nuclear power by 2022 will set back efforts to decarbonize the electricity supply by 10 crucial years, and could prove expensive for every household in Europe. Germany’s sudden about-turn, like all decisions on nuclear energy, was highly political. Last year, the government, headed by Angela Merkel, made the sensible but unpopular decision to extend the life of Germany’s nuclear plants to 2036 as a “bridge technology” towards “the age of renewable energy”. But, after the disaster at the Fukushima Daiichi nuclear plant in Japan, public hostility intensified and Merkel retreated. The U-turn may help her in the 2013 federal elections, but it is a major step back for the climate. About 23% of Germany’s electricity comes from nuclear and 17% from renewable energy sources. That’s a 40% share for zero-carbon in total. The government has admirable plans to raise renewable electricity to 35% of consumption by 2020. However, even this planned increase falls 5% short of filling the hole in zero-carbon electricity left by abandoning nuclear power.

*** The fate of the dinosaurs may have been sealed half a billion years before life even appeared, by two geological time bombs that still exist near our planet’s core. A controversial new hypothesis links massive eruptions of lava that coincided with many of the Earth’s largest extinctions to two unusually hot sections of mantle 2,800 kilometres beneath the Earth’s crust. These sections formed just after the Earth itself, 4.5 billion years ago. If the hypothesis is correct, they have periodically burst through the planet’s crust, creating enormous oceans of lava which poisoned the atmosphere and wiped out entire branches of the tree of life. Debates still rage over what caused different mass extinctions, including the one that wiped out the dinosaurs. An asteroid that smashed into the Earth 65 million years ago is no doubt partially to blame for the demise of the dinosaurs. But, a less-known school of thought has it that this and other extinctions occurred when cracks in the crust let huge amounts of lava pour out from the centre of the Earth. Each event flooded at least 100,000 square kilometres, leaving behind distinct geological regions known as large igneous provinces (LIPs), such as India’s Deccan traps, which were formed during the time when the dinosaurs became extinct.

***There have been very few studies comparing the microbiological safety of organic and conventional food production. In theory, organic food could be more prone to microbial contamination due to the lack of preservatives and the use of animal waste or manure as fertilisers are more commonly referred to. However, the results of the present studies have not been conclusive due to a number of factors, including a small sample size and a failure to take into account seasonal and regional variations. Clearly, organic and conventional foods are susceptible to contamination by pathogenic microorganisms at every point in the food chain. It can occur during production from manure and water, during processing from environmental sources and during the final handling and packing, possibly as a result of poor human sanitation. One area where organic production systems might pose a higher risk is through the use of newer untreated manure as fertiliser. Studies carried out on organic and conventional produce found that E. coli contamination was 19 times greater on organic farms that used manure or compost less than 12 months old than on farms that used older materials. Though the risks are reduced as manure matures, researchers have found that many pathogenic organisms such as E. coli and salmonella can still survive up to 60 days or more in compost and in the soil, depending on temperature and the condition of the soil.

ÜDS SAĞLIK 2011 SONBAHAR

*** Conservation biologists are applying their understanding of population, community, ecosystem, and landscape dynamics in establishing parks, wilderness areas, and other legally protected nature reserves. Choosing locations for protection often focuses on biodiversity hot spots. These relatively small areas have a large number of endangered and threatened species and an exceptional concentration of endemic species, those that are found nowhere else. Together, the “hottest” of Earth’s biodiversity hot spots total less than 1.5% of Earth’s land but are home to a third of all species of plants and vertebrates. There are also hot spots in aquatic ecosystems, such as certain river systems and coral reefs. Because endemic species are limited to specific areas, they are highly sensitive to deprivation of their natural environment. At the current rate of human development, some biologists estimate that loss of habitat will cause the extinction of about half of the species in terrestrial biodiversity hot spots in the next 10 to 15 years. Thus, biodiversity hot spots can also be hot spots of extinction. They rank high on the list of areas demanding strong global conservation efforts. Concentrations of species provide an opportunity to protect many species in very limited areas. However, species endangerment is truly a global problem, and focusing on hot spots should not detract from efforts to conserve habitats and species diversity in other areas. *** A contagious tumour threatens to wipe out the famous Tasmanian devil, a ferocious marsupial animal. Could contagious cancers arise in humans, too? Given that humans have great genetic diversity and can avoid behaving in ways that would foster tumour transmission, it might seem safe to assume that our species can readily avoid the fate of the Tasmanian devil. Indeed, if a person were bitten by an infected Tasmanian devil or by a dog with the canine transmissible tumour, the person’s genetic makeup, being so different from that of the animals, would probably ensure a strong immune response able to detect and kill the invading cells. Thus, the bitten individual would not get sick or start spreading the disease to others. There are grounds for concern, though. Contagious cancers could, in theory, arise in a group of great apes with low genetic diversity because of population declines. If they were hunted by human populations with many members having impaired immunity, the close contact might enable tumour cells to transfer to humans and then spread. Such conditions exist where humans with a high HIV prevalence hunt endangered apes. Although this scenario is possible, we suspect that cross-species transmission is not the most likely way that a contagious cancer would arise in humans. We hold this view in part because no known cases of cross-species transmission of the dog cancer have occurred in nature, although the disease has been experimentally transferred to related canines in the laboratory.

*** Proper diet is an important contributor to preventing tooth cavities. Although all carbohydrates can cause tooth decay to some degree, the biggest culprits are sugars. All simple sugars have the same effect on the teeth, including table sugar and the sugars in honey, fruits, and milk. Whenever sugar comes in contact with plaque, Streptococcus mutans bacteria in the plaque produce acid for about twenty minutes. The amount of sugar eaten is irrelevant; the amount of time the sugar stays in contact with the teeth is the important issue. Thus, sipping a sugary soft drink over an hour is more damaging than eating a candy bar in five minutes. A person who tends to develop cavities should eat sweet snacks less often. Rinsing the mouth after eating a snack removes some of the sugar; brushing the teeth is more effective. In fact, brushing prevents cavities from forming on the sides of the teeth, and flossing gets between the teeth where a brush can’t reach. Drinking artificially sweetened soft drinks also helps, though diet colas contain acid that can promote tooth decay. Drinking tea or coffee without sugar can also help people avoid cavities, particularly on exposed root surfaces.

*** Patients with hearing loss not correctible by medical therapy may benefit from hearing amplification. Contemporary hearing aids are comparatively free of distortion and have been miniaturized to the point where they often may be contained entirely within the ear canal. To optimize the benefit, a hearing aid must be carefully selected to conform to the nature of the hearing loss. Digitally programmable hearing aids are now widely available and allow optimization of speech intelligibility and improved performance in difficult listening circumstances. Aside from hearing aids, many assistive devices are available to improve comprehension in individual and group settings, to help with hearing television and radio programs, and for telephone communication. For patients with severe to profound sensory hearing loss, the cochlear implant – an electronic device that is surgically implanted into the cochlea to stimulate the auditory nerve – offers socially beneficial auditory rehabilitation to most adults with acquired deafness. New trends in cochlear implantation include its use for patients with only partial deafness, preserving residual hearing and allowing both acoustic and electrical hearing in the same ear, as well as bilateral cochlear implantation.

*** In 1883, Sigmund Freud was a young neurologist living in Vienna and struggling to make ends meet. Like many doctors, he became interested in a South American drug that was all the rage, cocaine. On April 30, 1883, he took a dose of pure cocaine for the first time. He evaluated its effects on mood, strength and reaction times and wrote up his findings in a pamphlet called Über Coca, or “On Cocaine”. His comments read: “Long, intensive physical work is performed without any fatigue… This result is enjoyed without any of the unpleasant after-effects that follow exhilaration brought about by alcohol… Absolutely no craving for the further use of cocaine appears after the first, or even after repeated taking of the drug.” Like other doctors of his time, Freud failed to recognize that cocaine is highly addictive – he actually recommended it as a treatment for morphine addiction. But, he did make one fascinating observation. Applied to the tongue or nose, cocaine produces a profound local numbing. Freud mentioned this to his colleague Karl Koller, an eye specialist who immediately saw its potential and later used it to transform eye surgery. In non-addictive forms, cocaine’s offspring are now widely used as local anaesthetics in medicine. If Freud had followed through with his original insight, that cocaine is an anaesthetic, he would probably have stayed a neurologist and never found the time to invent psychoanalysis.

*** Most people are surprised to learn that fat has some virtues. Only when people eat either too much or too little fat does ill health follow. It is true, though, that in our society of abundance, people are likely to encounter too much fat. Fat is actually a subset of the class of nutrients known as lipids, but the term “fat” is often used to refer to all the lipids. The lipid family includes triglycerides (fats and oils), phospholipids, and sterols, all important to nutrition. The triglycerides provide the body with a continuous fuel supply, keep it warm, and protect it from mechanical shock; their component fatty acids serve as starting materials for important hormonal regulators. The phospholipids and sterols contribute to the cells’ structures, and the sterol cholesterol serves as the raw material for some hormones, vitamin D, and bile. In foods, triglycerides are the solid fats and liquid oils. The triglycerides carry with them the four fatsoluble vitamins – A, D, E, and K – together with many of the compounds that give foods their flavour, texture, and palatability. Fat is responsible for the delicious aromas associated with sizzling meat and hamburgers on the grill, onions being sautéed, or vegetables in a stir-fry. Of course, these wonderful characteristics lure people into eating too much from time to time.

ÜDS SOSYAL 2011 SONBAHAR *** Since its economic reform began in 1978, China has gone from being a poor developing country to the second-largest economy in the world. It has also emerged from isolation to become a political superpower. Its meteoric rise has been one of the most important global changes of recent years. However, when it comes to science and technology, most people think of China as being stuck in the past and only visualize a country with massive steelworks and vast smoking factories. That may have been true a few years ago, but it is no longer the case. Very quietly, China has become the world’s second largest producer of scientific knowledge, surpassed only by the US, a status it has achieved at an awe-inspiring rate. If it continues on its current trajectory, China will overtake the US before 2020 and the world will look very different as a result. The historical scientific dominance of North America and Europe will have to adjust to a new world. In the West, people are largely familiar with research systems in which money, people, and output stay roughly the same from year to year. Research spending in Europe and North America has outpaced economic growth since 1945, but not by a dramatic amount. Not so with China.

*** To fully understand unemployment, one must consider the causes of recorded long-term unemployment, that is, the government assistance programs. For one thing, government assistance increases the measure of unemployment by prompting people who are not working to claim that they are looking for work even when they are not. The work registration requirement for welfare recipients, for example, compels people who otherwise would not be considered part of the labour force to register as if they were a part of it. This requirement effectively increases the measure of unemployment in the labour force even though these people are better described “noemployed” – that is, not actively looking for work. Similarly, unemployment insurance induces people to say they are job hunting in order to collect benefit. These programs also contribute to long-term unemployment by providing an incentive, and the means, not to work. Each unemployed person has a “reservation wage” – the minimum wage he or she insists on getting before accepting a job. Unemployment insurance and other social assistance programs increase the wage, causing an unemployed person to remain unemployed longer.

*** In 1895, Alfred Nobel drafted a holograph will, replacing one that left his vast fortune essentially to relatives, servants, and friends. The new will, for which Nobel will be forever remembered, substantially reduced his personal bequests. It directed that his estate be invested conservatively and that the income from these investments be used to establish annual prizes to be awarded with no reservations regarding nationality to those people whose activities are deemed to be of the greatest benefit to humankind in the fields of physics, chemistry, physiology or medicine, literature, and peace. Nobel’s will was disputed legally for more than three years. Eventually, a system was established for the distribution of the income in the form of Nobel Prizes, the first set of which were awarded in 1901. As the income from the Nobel trust has increased, the size of each award has grown to the point that in 2005, the typical prize was worth over $1.3 million, more than thirty times what the same award had been worth fifty years earlier. The list of Nobel laureates, which has now been expanded to include a sixth field, economics, contains the names of international giants in their fields. The Nobel legacy is great because of the endowment he established to recognize those who contribute most to the benefit of humankind.

*** The recent dramatic unrest in the UK is described as the world’s first decentralized riots, facilitated by social media such as Twitter and the BlackBerry Messenger network. But deep psychological forces are also at play. In times of social unrest, violent acts that would usually seem extreme can quickly become the norm among groups of people identifying with each other in terms of age, social status or other attributes. When people see looting and rioting happening elsewhere, it shows them what can be realized. Feeling empowered, they think they can do pretty much what they like, and they have a good time doing it. They simply don’t think of it as unacceptable as they would at other times. Such collective action is driven by social identity – by people sharing a common social place. It is too early to know what the rioters might have in common, but it is suspected that it boils down to social class and is defined by economic deprivation. They seem to be targeting the middle class. It’s like a kind of class warfare on the streets of Britain. Alienation from their families, the local community and mainstream society stops agitators from caring about the harm they cause others. It is necessary to challenge the rioters’ current perception that there are no opportunities for a better future.

*** Since early 2010, global food and oil prices have been on a sustained and synchronized upward trend. According to a recent survey by the United Nations, it is estimated that oil price increases will reduce growth in some developing Asia-Pacific economies, as well as putting pressure on inflation and adversely affecting current accounts. High oil prices will increase costs for domestic industry and push up the price of imports and reduce demand for exports. Food prices have increased by up to 35%. While adverse climatic conditions have affected supply in many countries, increasing conversion of food crops into biofuels, export bans, and heightened speculative activity in food commodities have exaggerated the price surge. Rising food prices are having dire effects on the poor, and reserving hard won development gains. Due to the higher food and energy prices, up to 42 million additional people across Asia and the Pacific may remain in poverty in 2011 in addition to the 19 million already affected in 2010. In the worst-case scenario, in which food price inflation doubles in 2011 and the average oil prices rises up to 130$ per barrel, achieving the Millennium Development Goal for many least developed countries would be postponed for at least a half decade.

*** Lower birth-rates and longer lives lead to population aging, which matters for many reasons, but first and foremost because of the costs of retirement. These costs are borne principally by the government and funded through taxes on the working-age population. The old-age-dependency ratio – that is, the population aged 65 and over divided by the population aged 15 to 64 – is a key indicator of population aging. Other things being equal, the tax rate for pensions will be proportional to this ratio. In the developed world, this ratio rose from .12 in 1950 to .21 today, and is estimated to increase to .44 in 2050. If, in the developed countries, the elderly in 2050 are to receive the level of benefits given to the current elderly, then the level of payroll taxes needed to fund government pensions will more than double by 2050. Due to higher fertility and immigration, the US population is projected to remain younger than those of other OECD countries, and the pension problem will be less severe. Health costs, however, pose an even more difficult problem due to the socialized health-care system for the elderly in the US. As the population ages and spending per elderly person rises, government spending on health care will likely soar.

ÜDS FEN 2012 İLKBAHAR *** Scientists were initially unconvinced that the decrease in the number of amphibians – animals such as frogs that live on land and in water – was real, because amphibian populations are notorious for fluctuating widely. However, after statistical evidence showed that the declines were far more widespread than would reasonably be expected by chance, most researchers agreed that something was seriously wrong. Reports of declines and extinctions accelerated during the 1990s, and the observations indicated that something specific and troubling was happening to amphibians. At one locale in Costa Rica, 40 per cent of the local amphibian species disappeared over a short period. The loss of amphibian species not only contributes to the world’s biodiversity crisis but also has remarkable implications for the ecosystems where the declines occur. Without amphibians, links to food webs are broken, and other organisms suffer in often unpredictable ways. Although some of the earliest amphibian declines were recorded in the US, much of the scientific literature focuses on decreases in tropical countries, where losses have often been more dramatic and have involved a larger number of species. This has led to the curious problem of declines in temperate amphibian species receiving insufficient attention.

*** Humans have evolved to work best in the atmosphere and gravity that exist on the Earth’s surface. To survive in space, astronauts have to take an Earth-like environment with them such as fresh oxygen, which is circulated around the craft for them to breathe. The main difference in space is the weightlessness causing astronauts to float around. As soon as astronauts go into space, their bodies start adapting to this weightlessness. Muscles, bones, heart and blood all undergo changes. At least half of all astronauts suffer an unpleasant reaction to weightlessness. On the Earth, gravity exerts a force on our bodies, which gives us weight and keeps us rooted to the ground. It also pulls body fluid downwards. In space, astronauts lose their sense of balance. They can feel sick, and go off their food. It can take two weeks for the digestive system to fully adjust. Therefore, NASA plans no spacewalks during the first three days of a mission, because an astronaut who vomits inside a spacesuit risks suffocation.

*** As well as being the largest mountain range on the planet, the Himalayas is also one of the youngest. Just seventy million years ago, a very short time in geological terms, the Himalayas did not exist. As the Indo-Australian tectonic plate collided with the Eurasian plate at the rate of about 15 centimetres a year, the ocean floor in between began to rise up to form the mountain range. This means that much of the rock out of which these towering peaks are made was formed at the bottom of an ocean, only to be lifted up thousands of metres into the air. The evidence for this extraordinary journey is not difficult to find. If you look closely at any piece of Himalayan limestone, you will see it has a chalky, granular structure. What you are looking at are the remains of sea creatures. Given a relatively short timescale and a bit of pressure, these biological remains are quickly converted into solid rock. Limestone can also be formed by the direct precipitation of calcium carbonate from water, although the biological sedimentary form is more abundant. We know that the Himalayan limestone is predominantly biological because we have found fossils at the top of Mount Everest. There is perhaps no better example of the endless recycling of Earth’s resources that has been going on since its formation almost five billion years ago.

*** It is safe to bet that a flying motorcycle will never be a practical transportation option. Yet, this process has not stopped an engineering firm in California from playing the long odds. The company is building a prototype called the Switchblade, and it hopes to sell a do-it-yourself kit as early as 2015. Attractive design and the promise of having air and ground transport in one package have kept alive dreams of a flying vehicle. A three-wheel design was chosen because it meets the definition of a motorcycle, which is not as highly regulated as cars are. For example, motorcycles do not need bumpers, which would make a flying vehicle heavier and more expensive. As the company envisions it, occupants would sit in the aerodynamic Switchblade, in climate-controlled luxury with an instrument display that switches from air to ground readings on landing. The Switchblade will succeed, the company believes, because it will transform easily between transportation in the air and on the ground. If pilots encountered bad weather while flying, they could put down at an airstrip, fold in the wings and finish the trip by travelling on the ground with no manual disassembly. The reality, however, is more complicated, given that aircraft are prohibited from operating on roads and tightly regulated as to how close they can fly to homes, military installations and environmentally sensitive areas.

*** Noise drives many species of marine animals to change their behaviour markedly – their calling, feeding and migration patterns – sometimes onto a beach and ending their lives. Fish like cod and haddock in the Barents Sea have been found to escape from the area when oil-prospecting air guns start firing, drastically reducing fish catches for days. Large baleen whales communicate over vast distances in the same frequencies that ship propellers and engines generate. On most days, the area over which whales in coastal waters can hear one another shrinks to only 10 to 20 per cent of its natural extent. Christopher W. Clark studies endangered northern right whales, whose habitat includes busy shipping lanes for the port of Boston. “Shipping noise is always there,” Clark says. “It doesn’t have to be fatal to be problematic over time. The whales’ social network is constantly being ripped and reformed.” Unable to communicate, individual whales have trouble finding each other and spend more time on their own. The problem is getting steadily worse for another reason. As we are making more noise, we are also making the ocean better at transmitting it. Seawater is absorbing less sound as carbon dioxide from fossil-fuel burning seeps into the ocean and acidifies it.

*** NASA’s Deep Space Network is responsible for tracking, commanding and receiving data from space probes throughout the solar system. Some of the spacecraft are so far away and they use such small transmitters that radio signals received from them are about 20 million times weaker than a watch battery. To receive such weak signals, the Deep Space Network uses huge dish-shaped antennae up to 70 meters across. To dispatch commands to the most remote spacecraft, the same giant dishes are used to send radio signals from immensely powerful, 400-kilowatt transmitters. The Deep Space Network’s antennae are located at three sites – Madrid (Spain), Canberra (Australia), and California (the US). The sites were chosen because they are roughly 120 degrees and a third of the way round the world from each other. As the Earth turns, at least one of the stations is always in contact with a space probe. This ensures a 24-hour observation, with overlapping time to transfer the radio link to the next station. The sites are also surrounded by mountains, which protect them from radio interference.

ÜDS SAĞLIK 2012 İLKBAHAR *** The Marylebone Health Centre in London, which opened in 1987, was the first National Health Service practice to employ complementary therapists, and it is the subject of a long-term research study into the effectiveness of integrated medicine. The team comprises three full-time and two part-time family doctors, an osteopath, homeopath, naturopath, acupuncturist, massage therapist and a counsellor. Conventional medicine is still the foundation of the practice, but doctors have the option of suggesting a therapy if they consider it appropriate for the patient’s condition. Dr. Sue Morrison, who is in charge of the practice, says that she could not now imagine working without complementary therapies. “We have found that we can contain problems – emotional as well as physical – that are usually difficult to look after in normal practice.” Integrated medicine may also be a part of the reason why the Marylebone Health Centre has a low referral rate to specialists and a drug-prescribing rate that is half of the national average. At the Marylebone Health Centre, patient care is “relationship-centred”, meaning that the alliance of patient and practitioner is central to all treatment and healing, which is a key element in integrated medicine.

***The hygiene hypothesis was first described in 1989 by David P. Strachan, a British epidemiologist, who noticed that the more children in a family, the lower the rates of allergies and eczema. Children in large families tend to exchange colds and other infections more often than children with fewer siblings, and this increased exposure to pathogens perhaps protected these children from allergies. That same year, Erika von Mutius, an epidemiologist at Munich University, was looking into the effect of hygiene on asthma. Children from dirtier East Germany, she was shocked to find, had dramatically less asthma than their West German counterparts living in cleaner, more modern circumstances. The East German children had likely been exposed to many more viruses and bacteria. According to the hygiene hypothesis, exposure in early childhood to infectious agents programs the immune system to mount defences against disease-causing viruses, bacteria and parasites. Better sanitary conditions deprive the immune system of this training, so the body fights against harmless particles as if they were deadly threats. The resulting allergic reaction leads to the classic signs of asthma. However, although much data supports the hygiene hypothesis for allergies, the same cannot be said for asthma. Contrary to expectations, asthma rates have increased drastically in urban areas in the US that are not particularly clean.

*** Psychology’s involvement in health dates back to the beginning of the 20th century, but at that time, few psychologists were involved in medicine. The psychosomatic medicine movement sought to bring psychological factors into the understanding of disease, but that view gave way to the biopsychosocial approach to health and disease. By the 1970s, psychologists had begun to develop research and treatment aimed at chronic disease and health promotion. This research and treatment led to the founding of two new fields: behavioural medicine and health psychology. Behavioural medicine is concerned with applying the knowledge and techniques of behavioural research to physical health, including prevention, diagnosis, treatment and rehabilitation. Health psychology strives to enhance health, prevent and treat disease, identify risk factors, improve the health care system and shape public opinion regarding health issues. Health psychology overlaps with behavioural medicine, and the two professions have many common methods of diagnosis and treatment. However, behavioural medicine is an interdisciplinary field, whereas health psychology is a speciality within the field of psychology that is concerned with issues of physical health.

*** “You are what you eat” says an old proverb. Yet, what if it were literally true? What if material from our food actually made its way into the control centres of our cells, taking charge of fundamental gene expression? That is in fact what happens, according to a recent study in China of plant-animal microRNA transfer. MicroRNAs are short sequences of nucleotides – the building blocks of genetic material. For the study, blood samples from 21 volunteers were tested for the presence of microRNAs from crop plants, such as rice, wheat, potatoes and cabbage. The results of the study revealed that the subjects’ bloodstream contained 30 different microRNAs from commonly eaten plants, and it appears that they alter cell function. For example, a specific rice microRNA was shown to inhibit the genetic receptor that controls the removal of cholesterol from the bloodstream. The suggestion that plant microRNAs play a role in controlling human physiology highlights the fact that our bodies are highly integrated ecosystems. These findings may also illuminate our understanding of co-evolution, a process in which genetic changes in one species trigger changes in another. For example, our ability to digest the lactose in milk arose after we domesticated cattle. Could the plants we cultivated have altered us as well?

*** Insulin, a hormone released from the pancreas, controls the amount of sugar in the blood. When people eat or drink, food is broken down into materials, including the simple sugar glucose, that the body needs to function. Sugar is absorbed into the bloodstream and stimulates the pancreas to produce insulin. Insulin allows sugar to move from the blood into the cells. Once inside the cells, it is converted to energy, which is either used immediately or stored as fat or glycogen until it is needed. The levels of sugar in the blood vary normally throughout the day. They rise after a meal and return to normal within about 2 hours after eating. Once the levels of sugar in the blood return to normal, insulin production decreases. The variation in blood sugar levels is usually within a narrow range, about 70 to 110 milligrams per decilitre (mg/dL) of blood. If people eat a large amount of carbohydrates, the levels may increase more. People over 65 tend to have slightly higher levels, especially after eating. If the body does not produce enough insulin to move the sugar into the cells, the resulting high levels of sugar in the blood and the inadequate amount of sugar in the cells together produce the symptoms and complications of diabetes.

***A key strategy in keeping the body free from infection is to prevent the entry of harmful organisms in the first place. Barrier, or passive immunity, acts as a first line of defence against pathogens, providing protection via the physical and chemical barriers presented by the various surfaces of the body. These include both external surfaces such as the skin and mucus-lined internal surfaces like the airways and the gut. Each body surface forms an initial physical barrier to infection, and this is then supplemented by a variety of secreted substances that exhibit antimicrobial properties such as enzymes, which break down bacteria. Additional vital mechanisms function to expel or flush out microbes from the body like coughing, sweating and urination. If barrier immunity is breached, for instance by a skin wound, and pathogens enter the body, the innate immune system then becomes actively involved. Key to this is the activation of an inflammatory response and the deployment of immune cells. Tissue damage results from inflammation, which helps to prevent microbes from spreading. The capillary walls in the affected area become more penetrable, enabling immune cells to access the infected tissue. Damaged cells release chemicals that attract immune cells once they have migrated from the bloodstream.

ÜDS SOSYAL 2012 İLKBAHAR

*** Studies reveal that even subtle, artificial or seemingly unimportant exclusion can lead to strong emotional reactions. A strong reaction makes sense when you are rejected or ignored by your family or close friends, because they are important to you. It is more remarkable that intense feelings of rejection can emerge even when people close to us are not involved. We can feel awful even after people we have never met simply look the other way. This reaction serves an important function. It warns us that something is wrong, that there exists a serious threat to our social and psychological well-being. Psychologists argue that belonging, selfesteem, a sense of control over your life and a belief that existence is meaningful constitute four fundamental psychological needs that we must meet to function as social individuals. Exclusion threatens all these needs. Even in a verbal or physical dispute, individuals are still connected. Total exclusion, however, cuts all bonds. Worse than this, the imposed silence forces us to think about the event in detail, generating self-critical thoughts in our search for an explanation. This forced isolation also makes us feel helpless: You can fight back, but no one will respond. Finally, exclusion makes our very existence feel less meaningful because this type of rejection makes us feel isolated and unimportant.

*** Do people everywhere experience embarrassment in the same way? This is a difficult question to address empirically. Most crosscultural research has focused on people’s self-reports of their experiences, feelings and expressions, which may or may not correspond to what actually happens in social interactions. Another tricky issue is how to translate terms for emotions into different languages. In English, there are separate terms for shame and embarrassment, and research suggests that the two emotions are different. Embarrassment tends to be over less serious errors and almost always happens in the presence of others. Shame seems to be reserved for more serious social violations and can be experienced alone. Smiling, which readily occurs in embarrassment, is unlikely to follow a shameful event. However, the boundary between events that elicit embarrassment or shame is undefined. In many cultures, the same word is used to describe both. For example, in Oriya, a language in India, the word “lajya” refers to a variety of emotional states, including embarrassment as well as shame. Many Asian languages also use one word for the two emotions, although some Asian cultures have multiple words that deal with saving face, shame and so on.

*** A symbol is a term or a picture that may be familiar in daily life, yet possessing specific connotations in addition to its obvious meaning. It implies something vague or hidden from us. Thus, a word or an image is symbolic when it implies something more than its immediate meaning. It has a wider “unconscious” aspect that is never precisely defined or fully explained. As the mind explores the symbol, it is led to ideas that lie beyond the grasp of reason. Since there are many things beyond the range of human understanding, we use symbolic terms to represent concepts that we cannot define or fully comprehend. This is one reason why historically all religions have employed symbolic language or images. However, this conscious use of symbols is only one aspect of a psychological fact of great importance. Man also produces symbols unconsciously and spontaneously, in the form of dreams. It is not easy to grasp this point, but we must if we are to know more about the ways in which the human mind works.

*** Yasuni National Park in Ecuador is considered by many scientists to be the single most biodiverse spot on the planet, but one in danger of being lost. Oil companies have found rich deposits beneath the park’s trees and rivers worth billions of dollars. Ecuador is a small country in which a third of the population lives below the poverty line and petroleum already makes up more than half of its export revenue. It badly needs the money that oil companies and consumers will be only too happy to provide if further drilling is allowed to go forward. If Ecuador follows the usual path of development, that’s exactly what will happen, with disastrous consequences for the park. However, there may be another way. Ecuadorian President Rafael Correa told the international community that his country would be willing to cease drilling and leave Yasuni intact in exchange for donations equal to $3.6 billion over 13 years, or about half the expected market value of the park’s oil deposits. The Yasuni Plan, while conserving the park’s unique biodiversity, would be a first for global environmental policy, recognizing that the international community has a financial responsibility to help developing nations preserve nature.

*** The economic realm of human activity, looked at from the perspective of the entire human experience, can best be seen not simply as a progressive development of freedom, but rather as a series of phases in which new freedoms are made possible only by determining new limits and new responsibilities. So long as humankind respects those new lines of authority, freedom prospers. Without those limits, on the other hand, freedom dies. Legislative bodies around the world that fail to pass environmental laws in the name of freedom have an understanding of freedom that is ultimately self-destructive. We must learn, before it is too late, that the gift of freedom does not mean tearing down walls but relocating them. Just as a line is defined by the empty space that surrounds it, so is freedom defined by the boundaries around its edges. If we fail to appreciate this basic law of nature, if we continue to demand – on the level of the nation-state – freedom without limits and responsibility, we risk punishing our children or our children’s children.

*** China and India need to fix their economies and societies, and to do so, they have to deal with some grim news. Growth is slowing, though in China’s case that helps cool an overheated economy. In both countries, exports are falling, inflation is at painful levels, income inequality is reaching great proportions, and injustices like land grabs are sparking widespread protests. The two countries have lifted countless millions of the unemployed out of poverty, but countless other millions – youths, workers and farmers – remain marginalized and desperate for decent livelihoods. While China does not follow the rules, India has too many rules to follow. China is struggling to contain assets, deal with bad loans and to rebalance its economy away from state-directed investment to consumer-led growth. India’s reputation, meanwhile, has been so damaged by bureaucratic corruption that the country’s top corporations have hired US consultancy firms to make a socalled “credible India” campaign. However, it seems that India needs to do lots of work to overcome its bad reputation and there is no guarantee that it will be able to do so.

ÜDS FEN 2012 SONBAHAR *** We humans long assumed that our visual system stood at the top of evolutionary success. Our knowledge of colour vision was primarily based on what humans see: researchers easily performed experiments on colour perception in humans. Although scientists obtained supporting information from a variety of other species by recording the firing of neurons, we remained unaware until the early 1970s that many vertebrates, mostly animals other than mammals, see colours in a part of the spectrum that is invisible to humans: the ultraviolet. In fact, the discovery of ultraviolet vision began with studies of insects conducted by Sir John Lubbock, who discovered sometime before 1882 that in the presence of ultraviolet light, ants would pick up their young and carry them to dark areas or to areas illuminated by longer wavelengths of light. In the mid-1900s, Karl von Frisch and his students showed that bees and ants not only see ultraviolet light as a distinct colour but use ultraviolet in skylight as a compass. The finding that a great number of insects perceive ultraviolet light misleadingly gave rise to the idea that this spectral region provides a private sensory channel that avian predators like eagles and vultures cannot see. Nothing, however, could have been further from the truth. Subsequent research showed that birds, lizards, turtles and many fish have ultraviolet receptors in their retinas.

*** Astronomers have a reputation for bringing us stunning discoveries: new planets and galaxies on the edge of the known universe. But now they are on the trail of the most notable finding of all: evidence for a whole new universe beyond our own. For millennia, philosophers have insisted everything we see is part of the all-encompassing totality called the universe. And for centuries, astronomers have been mapping its immensity, using more and more powerful telescopes to probe deeper into space. It seemed clear, however, that there was a limit to what they could see, as they were examining what was comprehensible for them. Since the discovery of cosmic expansion, they believed there must be a final frontier, but today, there is mounting excitement that it may be possible to probe beyond this far horizon. According to the latest theories in cosmology, what has been regarded as the universe might be just one of an infinite number making up something far grander – the Multiverse. But the Multiverse model is highly problematic; the biggest difficulty is that the existence of such parallel universes can be neither verified nor falsified. No less amazing, an orbiting observatory, named Planck, has revealed many features about our universe such as its age and size since it was launched in 2009, and astronomers believe it may be able to travel the Multiverse and do even more.

*** Scientists have urged national leaders for years to tackle climate change, based on the assumption that all nations should take steps in harmony for the success of their prevention efforts. But as anyone who has watched the past 15 years of international climate negotiations can attest, most countries are still reluctant to take meaningful steps to lower their production of greenhouse gases, much less address issues such as how to help developing countries protect themselves from the extreme effects of climate change. Mayors and urban managers are taking over as they have a keener sense about how changing weather patterns will affect their cities’ political and economic futures. Indeed, within months after Hurricane Katrina’s landfall, the C40 Cities Climate Leadership Group launched in London in October 2005, and the World Mayors Council on Climate Change (WMCCC) got its start in Kyoto that December. As of June 2011, more than 190 mayors and other local authorities, representing some 300 million people from around the world, have also signed a voluntary pact sponsored by the WMCCC to reduce greenhouse gas emissions. They are tackling climate change, as their cities are suffering from floods, rising sea levels and heat waves. They are innovating ways to reduce carbon dioxide emissions, prevent further changes in weather patterns and benefit from transportation systems that protect the environment.

*** For decades, nets and sprays have been the only effective methods for controlling the mosquitoes that cause malaria. However, Spanish chemist Pilar Mateo thinks she can do better with her invention of embedding pesticides in microcapsules stirred into house paints at her Valencia company. The insecticides are released from the paint slowly, remaining effective for two to four years, while sprays need to be reapplied at least every six months. “The paint acts like a vaccine for houses,” she says. The amounts of pesticides released from the paint are harmless to people but are devastating to insects, according to the tests made by scientists. The paint has already been approved for use in fifteen countries, including China and England. Mateo is seeking approval in the US and a recommendation from the World Health Organization. She says she has received offers to buy her patent but refuses to sell out. Instead, her new venture, another company in Africa, will produce it commercially at a factory in Ghana and employ a great number of workers. “By taking production outside Spain, we can reduce the cost and make it more accessible,” she says. Her idea is to sell the paint as an affordable alternative to sprays. After years of donating paint to poor people in Latin America, Mateo wants to fund her broader humanitarian efforts. “It’s not just the insects that are the problem”, she says, “It’s the poverty.”

*** Forget drilling into the ocean floor to tap into ever-decreasing supplies of oil, because there could soon be a new fuel source beneath the waves – seaweed. A technique has been developed to convert sugars in seaweed into a fuel that can be used to power cars. Biofuels are currently produced from crops such as corn and sugar cane, but these sources are also in demand for use as food, and their production requires large amounts of land, fresh water and fertiliser. Seaweed requires none of these and has the advantage of not containing lignin, a strong strand of sugars that stiffens plant stalks but is difficult to turn into biofuel. Researchers at Bio Architecture Lab in California have been able to produce bioethanol from kombu, an edible brown seaweed. Bioethanol can be blended with petrol and used in engines with little or no modification. Brown seaweed has high sugar content and also grows more quickly than the red or green species. The only potential stumbling block is growing enough of the stuff. Several thousand tonnes are farmed annually for food, but if it is going to be used widely as a fuel, billions of tonnes would be required. But Bio Architecture Lab is still forging ahead, launching a seaweed biofuel pilot project in 2013. It hopes to commercialize seaweed-sourced fuels within three to four years.

*** Typing in passwords could be a thing of the past, thanks to technology that can read your thoughts. A new discovery brings cognitive biometrics very close to reality. In cognitive biometrics, the response of your nervous system to a stimulus is measured, then used to identify you. A series of letters or images are flashed up on a screen and your P300 wave – a type of electrical activity in the brain – is measured using electrodes attached to your head. When a pre-defined word, letter or image is shown, recognition by the user generates their signature, P300 wave. Until now, users have been shown different letters or symbols in different parts of a screen. But tests at the universities of Essex and Wolverhampton have found that the system identifies individuals most accurately if all characters appear in the same location. This also reduces the chances of criminals spotting the brain word by tracking eye movement. As there is no keyboard involved, it is much harder for someone intent on fraud to get access to a password. “You still have to keep your password secure,” says Dr. Palani Ramaswamy. “It’s just that the way of entering the password is more resistant to fraud.” Accuracy still needs to be improved before cognitive biometrics can come into mainstream use. “Once accuracy is close to 100 percent, it can be used for high-security military applications and financial transactions as well,” he maintains.

ÜDS SAĞLIK 2012 SONBAHAR

*** Alzheimer’s disease is a chronic, degenerative condition of the brain cells. Some risk factors can be avoided, but others, such as increasing age and genetic properties, are inevitable. It is now the third most common cause of death in the developed world, with more women than men affected. The first symptom is often an impaired memory for recent events, which can be difficult to distinguish from the normal age-related decline in memory. As the disease progresses, forgetfulness may hinder routine activities such as cooking and household chores. Those affected may be aware of their memory difficulties, so they can deal with the problem by, for example, writing notes or letting someone else manage decisions for them. In the late stages of the disease, the changes in memory and behaviour are marked. Sufferers cannot compensate for their memory lapses and become confused. They may develop paranoid behaviour like jealousy or accusations of theft, and may experience visual hallucinations. People in advanced stages cease to recognize even their family members and close friends. They may also refuse to eat, develop unsteadiness and increasingly lose weight.

*** Vitamin C boosts the immune system by strengthening the action of white blood cells that destroy harmful bacteria and viruses, such as cold and flu viruses. It is also an important antioxidant, protecting the watery internal parts of cells from the damaging effects of excess free radicals, and it works in partnership with vitamin E which protects the fatty outer part of the cells. Moreover, vitamin C can help to prevent cholesterol from becoming oxidised, a phenomenon now thought to be the precursor to the furring of arteries – the reduction of blood flow through arteries. High intakes of vitamin C are also linked with lower levels of stomach cancer. In addition, vitamin C improves the absorption of iron from non-meat sources, so ensuring a regular intake is useful if you are a vegetarian. Also, it has mild antihistamine properties, thus allergy sufferers may find it helpful. Furthermore, when we are under physical or mental stress, vitamin C is depleted from our adrenal glands; it plays a regulatory role in the production of the hormone cortisone that helps us cope with the pressures of life.

*** Environmental health has biologic, chemical, physical and sociological components, including the immediate and future conditions in which people live. In colonial America, little attention was paid to community hygiene and sanitation, and there was almost a complete lack of community organisation for better health services. During this time, epidemics of cholera, smallpox and dysentery continually occurred. Although such epidemics were attributed to environmental health hazards such as overcrowding, inadequate housing and impure water, little was done to improve these conditions. Early attempts to ensure environmental health included a law in 1610 that prohibited the throwing out of water from dirty clothes into the street, and required people to do the necessities of nature outside the town. Those who violated the law were often subjected to strict penalties. Such measures were more concerned with the aesthetics of the environment than with related health consequences, and environmental practices were frequently directed at keeping the environment attractive.

*** Just a few decades ago, doctors would dispense sleeping pills as if they were sweets. Today, sleeping pills are no longer considered to be an appropriate solution to chronic sleeplessness. As they treat only the symptoms of insomnia, any improvement in sleep can only be temporary, thereby perpetuating the cycle of insomnia and drug-induced sleep. Ironically, many people initially turn to sleeping pills, as insomnia has left them helpless and out of control. However, pills can become a trap that escalates feelings of dependency, lowered self-esteem and guilt. Then insomniacs end up having to cope with two stressful problems: insomnia and dependency on sleeping pills. Most patients prefer non-drug approaches like avoiding caffeine and nicotine, but they do not know how best to escape the sleeplessness. However, some doctors guide their patients by prescribing the smallest possible dose, to be used only after two consecutive nights of bad sleep. As an alternative to supplement the pills, professionals provide such sleep aids as antihistamines which produce drowsiness as a side effect. Also, synthetic melatonin has recently been marketed and publicized as a natural sleeping pill. Although several studies found that melatonin was effective for promoting sleep, the studies focused on normal sleepers, not insomniacs. All in all, it seems that careful implementation of new behavioural patterns provides the most promising results to sufferers.

*** Digestible microchips embedded in drugs may soon tell doctors whether a patient is taking their medications as prescribed. These sensors are the first ingestible devices approved by the US Food and Drug Administration (FDA). To some, they signify the beginning of an era in digital medicine. The sandparticle sized sensor consists of a minute silicon chip containing trace amounts of magnesium and copper. When swallowed, it generates a slight voltage in response to digestive juices, which conveys a signal to the surface of a person’s skin where a patch then relays the information to a mobile phone belonging to a health care provider. Currently, the FDA and the analogous regulatory agency in Europe have only approved the device based on studies showing its safety and efficacy when implanted in placebo pills. But scientists hope to have the device approved within other drugs in the near future. Medicines that must be taken for years, such as those for drugresistant diabetes, and for the elderly with chronic diseases, are top candidates. Proponents of digital medical devices predict they will provide alternatives to blood tests, MRIs and CAT scans. Other gadgets in the pipeline include implantable devices that wirelessly inject drug at pre-specified times and sensors that deliver a person’s electrocardiogram to their smartphone.

*** There are several factors that increase the risk of cancer. Mutations that affect genes are believed to contribute to the development of cancer. These genes produce proteins that regulate growth and alter cell division and other basic cell properties. Cancer-causing genetic mutations may result from the damaging effects of drugs and viruses. Additionally, environmental factors such as air pollution and radiation increase the risk of cancer. Some chemicals like pesticides are known to cause cancer, and many others are suspected of doing so, but more study is needed to identify those chemicals that increase the risk. The threat of cancer may also vary according to geographical features. This geographic variation in cancer risk is multifactorial: a combination of genetics, diet and environment. For example, the Japanese, who follow a diet that mainly includes smoked foods, have high rates of colorectal cancer. When they immigrate to the US and eat a Western diet, the risk level declines to that of the US. Age is another risk factor. While some cancers like Vilms’ tumor occur almost exclusively in children, cancers of the lungs and kidneys are more common in older people, probably due to constant exposure to carcinogens and weakening of the body’s immune system. However, not all people who are exposed to carcinogens or who have other risk factors develop cancer.////

ÜDS SOSYAL 2012 SONBAHAR

*** Although Aeschylus is considered to be the first great innovator of Western drama, it is sometimes difficult to judge the full extent of the innovations he introduced, since no plays by his predecessor, Thespis, have been preserved. What we know about Greek drama before Aeschylus suggests that it had developed gradually out of choral lyrics, occasionally interrupted by short dialogues between the chorus and a single speaker or singer representing an individual character. Aeschylus took the decisive step of introducing a second actor, thus enabling for the first time a dialogue or conflict between two individuals to take place on the stage and in front of an audience. The innovations made possible by Aeschylus’s introduction of a second character are very significant. In addition, an actor could leave the stage and reappear in the guise of another character, thus permitting an increase in the overall number of persons represented. In his later plays, Aeschylus used three actors, allowing him to feature a large number of characters, as in The Libation Bearers, the second play of the Oresteia trilogy.

*** A credit rating agency measures credit worthiness of institutions from companies to governments and assesses their ability to pay back a loan. The top three credit rating agencies are Standard and Poor’s (S&P), Fitch Ratings and Moody’s. Each rating agency has developed its own rating system. Fitch Ratings developed its system in 1924, which was later adopted by S&P. Both use a system of letter sliding from the best rating ‘AAA’ to the lowest ‘D’ for borrowers already defaulting on payments. In detail, ‘AAA’ represents the best quality borrowers that are reliable and stable without any foreseeable risk to future payments, while ‘D’ means the institution has defaulted on payment obligations, having failed to pay back the loans – S&P and Fitch Ratings assert it will keep on doing so. Moody’s follows a different rating system. It argues that their ratings have a superior approach that considers not only the likelihood of default, but also the severity of the default. In addition, S&P and Fitch Ratings are only interested in how likely a borrower is to default, whereas Moody’s cares how long the default is likely to last. Most importantly, S&P does not care what the recovery value will be – the amount of money that the lender will end up with after the borrower has defaulted. Moody’s, by contrast, tries to figure out the expected losses, which makes it more preferable.

*** Outside forces have played a major part in the birth and development of Middle Eastern states as well as in shaping the environment in which these states have operated. Since Napoleon’s intervention in Egypt in the late 18th century, European powers have been an important part of the Middle East’s make-up – its politics, socio-economic development and external orientation. It was the European powers who took control of significant areas of the region from the 19th century, and they gave rise to the downfall of the Ottoman Empire and shared its spoils in the early 20th century. It was the same set of European powers that formed new states from territories under their control. But in the second half of the 20th century, the nature of outside intervention changed somewhat. As a penetrated regional system, the Middle East, for all its active internal dynamics (nationalism, the Arab-Israeli War, etc.), was by the 1950s subject to the influence of strategically-driven calculations made by the world’s two superpowers: the US and the USSR. The superpowers’ calculations not only directly affected politics of the region, but also the environment where the local forces were taking shape. For over a generation, the Cold War between superpowers was the framework of the Middle East’s regional system, from North Africa in the west to the borders of the Soviet Caucasus and Central Asia.

*** Today, the European Union is home to more than 20 million immigrants, who represent about 4 percent of the total EU population and make valuable contributions to European society. These new arrivals fill gaps in the labour market that EU workers cannot or do not wish to fill, helping to address the demographic decline in Europe’s working age population. Properly managed, immigration can help contribute to the EU’s long-term economic development and competitiveness. At the same time, ensuring the security and prosperity of the EU population remains vital. The key is to streamline and simplify the legal immigration process, enforce measures against illegal immigrants, secure the external borders and support the Member States’ efforts to promote the integration of immigrants so they become full participants in EU society. One of the EU’s more remarkable achievements is the creation of its single market, where people, goods, services and capital move freely throughout the 27 EU Member States. The flip side of this free movement, however, is that the reduced internal border controls necessitate strengthened external borders. Each border state bears a particular responsibility for defending its portion of the EU’s borders and with it, the security of the entire EU.

*** Recent research suggests that not only can children differentiate between two languages at an early age, but also show cognitive benefits from being exposed to a second language starting as early as infancy. In a study in 2009 of ‘crib bilinguals’, cognitive psychologists Agnes Kovács and Jacques Mehler used a visual test to measure cognitive flexibility in preverbal seven-montholds. Kovács and Mehler wanted to see how quickly the infants could adapt to changing rules. They taught the infants a pattern consisting of speech-like sounds. At the end of the sequence, a visual reward in the form of a puppet would appear in one part of a computer screen. The infants were expected to learn that a given sound pattern predicated the appearance of the puppet in that location. Both bilingual and monolingual infants showed that they associated the sound sequence with the puppet’s location equally well by looking in the right place for the puppet to appear. But when Kovács and Mehler modified the sequence – and moved the puppet – the bilingual infants adjusted, switching their anticipatory gaze to the new location. The monolingual infants, however, continued to look for the puppet in the original location.

*** Humans are preoccupied with vitality; that is, a concern with the generation, transmission, continuation and protection of life itself. The obvious social tie formed around this preoccupation is the family. However, numerous individual families of a nation understand themselves to be just that; thus, the continuation of the nation into the future is regarded as entailing the continuation of the families into the future. Anthropological studies reveal that humans have always formed not only families, but also larger groups of which families are a part. Parents transmit to their own offspring not only their flesh and blood – genetic properties in a broader term – but also their cultural inheritance; the language, traditions, customs and so forth – of the larger group, of the nation. This cultural inheritance is usually viewed by the parents as being quite precious to their existence. This intergenerational transmission of one’s culture may be part of the reason for the tendency to view the nation as a form of kinship, because what is being transmitted is a part of one’s self to one’s descendants.

*** *** *** *** *** *** ***

View more...

Comments

Copyright ©2017 KUPDF Inc.
SUPPORT KUPDF